Baltic Way 팀수학경시대회 1990-2005 pdf 보기

291
엠제곱(M 2 ) Baltic Way 팀 수학경시대회 1990{2005 고봉균 KAIST 수학문제연구회

Transcript of Baltic Way 팀수학경시대회 1990-2005 pdf 보기

Page 1: Baltic Way 팀수학경시대회 1990-2005 pdf 보기

엠제곱(M2)

Baltic Way 팀 수학경시대회1990{2005

고 봉 균

KAIST 수학문제연구회

Page 2: Baltic Way 팀수학경시대회 1990-2005 pdf 보기

2

Page 3: Baltic Way 팀수학경시대회 1990-2005 pdf 보기

차례

머리말 . . . . . . . . . . . . . . . . . . . . . . . . . . . . . . . . . . . . . . . . . . . 5

I 문제편 7

1990년 Baltic Way . . . . . . . . . . . . . . . . . . . . . . . . . . . . . . . . . . . . 9

1991년 Baltic Way . . . . . . . . . . . . . . . . . . . . . . . . . . . . . . . . . . . . 12

1992년 Baltic Way . . . . . . . . . . . . . . . . . . . . . . . . . . . . . . . . . . . . 15

1993년 Baltic Way . . . . . . . . . . . . . . . . . . . . . . . . . . . . . . . . . . . . 18

1994년 Baltic Way . . . . . . . . . . . . . . . . . . . . . . . . . . . . . . . . . . . . 21

1995년 Baltic Way . . . . . . . . . . . . . . . . . . . . . . . . . . . . . . . . . . . . 24

1996년 Baltic Way . . . . . . . . . . . . . . . . . . . . . . . . . . . . . . . . . . . . 27

1997년 Baltic Way . . . . . . . . . . . . . . . . . . . . . . . . . . . . . . . . . . . . 30

1998년 Baltic Way . . . . . . . . . . . . . . . . . . . . . . . . . . . . . . . . . . . . 33

1999년 Baltic Way . . . . . . . . . . . . . . . . . . . . . . . . . . . . . . . . . . . . 36

2000년 Baltic Way . . . . . . . . . . . . . . . . . . . . . . . . . . . . . . . . . . . . 39

2001년 Baltic Way . . . . . . . . . . . . . . . . . . . . . . . . . . . . . . . . . . . . 42

2002년 Baltic Way . . . . . . . . . . . . . . . . . . . . . . . . . . . . . . . . . . . . 45

2003년 Baltic Way . . . . . . . . . . . . . . . . . . . . . . . . . . . . . . . . . . . . 48

2004년 Baltic Way . . . . . . . . . . . . . . . . . . . . . . . . . . . . . . . . . . . . 51

2005년 Baltic Way . . . . . . . . . . . . . . . . . . . . . . . . . . . . . . . . . . . . 54

II 풀이편 57

1990년 Baltic Way 풀이 . . . . . . . . . . . . . . . . . . . . . . . . . . . . . . . . . 59

1991년 Baltic Way 풀이 . . . . . . . . . . . . . . . . . . . . . . . . . . . . . . . . . 70

1992년 Baltic Way 풀이 . . . . . . . . . . . . . . . . . . . . . . . . . . . . . . . . . 83

1993년 Baltic Way 풀이 . . . . . . . . . . . . . . . . . . . . . . . . . . . . . . . . . 93

Page 4: Baltic Way 팀수학경시대회 1990-2005 pdf 보기

4 차례

1994년 Baltic Way 풀이 . . . . . . . . . . . . . . . . . . . . . . . . . . . . . . . . . 105

1995년 Baltic Way 풀이 . . . . . . . . . . . . . . . . . . . . . . . . . . . . . . . . . 120

1996년 Baltic Way 풀이 . . . . . . . . . . . . . . . . . . . . . . . . . . . . . . . . . 133

1997년 Baltic Way 풀이 . . . . . . . . . . . . . . . . . . . . . . . . . . . . . . . . . 148

1998년 Baltic Way 풀이 . . . . . . . . . . . . . . . . . . . . . . . . . . . . . . . . . 164

1999년 Baltic Way 풀이 . . . . . . . . . . . . . . . . . . . . . . . . . . . . . . . . . 179

2000년 Baltic Way 풀이 . . . . . . . . . . . . . . . . . . . . . . . . . . . . . . . . . 193

2001년 Baltic Way 풀이 . . . . . . . . . . . . . . . . . . . . . . . . . . . . . . . . . 207

2002년 Baltic Way 풀이 . . . . . . . . . . . . . . . . . . . . . . . . . . . . . . . . . 219

2003년 Baltic Way 풀이 . . . . . . . . . . . . . . . . . . . . . . . . . . . . . . . . . 234

2004년 Baltic Way 풀이 . . . . . . . . . . . . . . . . . . . . . . . . . . . . . . . . . 249

2005년 Baltic Way 풀이 . . . . . . . . . . . . . . . . . . . . . . . . . . . . . . . . . 264

III 부록 285

대회결과 . . . . . . . . . . . . . . . . . . . . . . . . . . . . . . . . . . . . . . . . . 287

Page 5: Baltic Way 팀수학경시대회 1990-2005 pdf 보기

머리말 5

머리말

발트해 연안의 여러 나라가 참여하는 Baltic Way는 팀으로 치러지는 수학경시대회이다.각

나라마다 5명으로 이루어진 선수팀이 참가하여 4시간30분 동안 20문항을 풀어야 한다. 제

한시간에 비해 문항이 상당히 많기 때문에 팀원별로 분담을 하여 풀고, 어려운 문제는 남은

시간 동안 함께 협력하여 풀어야 할 것이다. 많은 올림피아드 대회들이 한 문항을 7점 만점

으로 평가하는 데 반해, Baltic Way는 5점 만점으로 평가하고, 그래서 총 100점을 만점으로

한다. 대수, 조합, 기하, 수론 등 네 영역에서 각각 5문항씩 출제된다.

이 대회는 1990년에 발트 3국인 라트비아, 리투아니아, 에스토니아가 함께 참여하여 라트

비아의 리가에서 첫 대회가 열렸다. 첫 대회에는 각국에서 2팀씩 참가하였고, 3회 대회 때

부터 주변국을 초청하기 시작하여 현재에는 발트 3국 이외에 러시아 노르웨이 폴란드 독일

핀란드 스웨덴 덴마크 아이슬란드 등 모두 11개국이 고정적으로 참가하고 있다.

11개국 중에 러시아는 발트해에 인접한 도시인 상트 페테르부르크의 대표팀이 참가하고 있

고, 독일은 로스톡과 함부르크의 연합팀이 참가하고 있다. 아이슬란드는 발트해 연안에서

꽤 떨어져있는 나라이지만 발트 3국의 독립을 세계에서 가장 먼저 인정한 나라로서 특별히

대회의 고정 멤버로 초청되고 있다. 매년 나라와 도시를 바꾸어 가며 개최되고 있고, 이스

라엘, 벨로루시, 벨기에 등 해마다 이 11개국 이외의 주변국을 한 나라씩 초대하고 있는 점

도 특이하다.

Baltic Way라는 대회의 명칭은 1989년 8월에 있었던, 발트 3국의 자유와 독립을 위해 에스

토니아의 탈린에서 라트비아의 리가를 거쳐 리투아니아의 빌뉴스까지 총 620 km에 걸쳐

사람들이 손에 손을 잡고 거대한 인간 사슬을 연결했던 시위를 기리기 위한 것이라고 한다.

그래서, 팀으로 협력하며 참가한다는 것이나 주변국들을 한 나라씩 초청하는 것들이 더욱

의미가 있어보인다.

Baltic Way 대회의 문제들은 각국에서 제안한 문제들 중에서 선별하여 출제되는데, 가끔

잘 알려진 문제가 나오기도 하여 아쉽기도 하지만 대체로는 좋은 문제들이라는 인상이다.

문제의 난이도가 일관성이 있기보다는 매우 쉬운 것부터 매우 어려운 것까지 마구잡이로

섞여있는데, 부록으로 각 연도의 국가별/문항별 성적을 실었으니 난이도의 판단에 도움이

될 것이다. 쉬운 문제들을 풀며 자신감을 얻고 또 어려운 문제들에 도전도 함께 해본다고

생각하며 공부하면 좋을 것이다. 매해 수론, 대수, 기하, 조합 분야의 문제가 각각 5개씩 묶

여서 나온다는 점은 자신이 취약한 특정분야의 실전 문제들을 훈련하고자 하는 학생들에

게 상당히 도움이 될 수 있는 부분이라고 여겨진다.

Page 6: Baltic Way 팀수학경시대회 1990-2005 pdf 보기

6 차례

혹시 `왜 하필 Baltic Way라는 대회의 문제집을 내게 되었을까'하고 생각하는 분이 있을

지 모르겠다. 우리 나라가 참가하고 있는 지역국가 대회인 아시아-태평양 수학올림피아

드(APMO)도 Baltic Way와 비슷한 시기(1989년)에 시작되었지만 해마다 5문제뿐이라서

아직 책으로 묶여나오기에는 분량이 적다.국제 수학올림피아드(IMO)는 분량이 너무 많아

서 책을 만들어내려면 시간이 너무 많이 걸릴 것 같았다.각국 대회로는 캐나다 수학올림피

아드 문제 및 풀이집을 만들었던 적이 있으나 캐나다 수학회와의 서신 교환이 여의치가 않

아 저작권 협의가 힘들어 출판을 포기했던 경험이 있다. (캐나다 수학올림피아드 문제 및

풀이는 KAIST수학문제연구회의 홈페이지 http://www.msquare.or.kr 에 온라인으로 완전

공개되어 있으니 원하시는 분은 찾아가서 구해보기 바란다.) 그래서, 저작권 협의가 불필

요한 여러 범국가 대회들 중 분량도 적절하고 대회의 뜻에 감동이 느껴지는 Baltic Way를

먼저 엮어 내게 된 것이니 다른 책을 먼저 기대했던 분들은 널리 양해를 바란다.

이 책에 수록된 풀이는 대부분 KAIST Cyber영재교육 http://talent.kaist.ac.kr 에서 `문제

따먹기'라는 경시대회형 과제를 통해 여러 학생들과 조교분들로부터 접수받은 것이다. 학

생들로부터 접수받은 풀이들이 훌륭하지 못한 경우에는 o±cial solution을 번역하여 실은

경우도 있다. 가능한한 원래 풀이자를 밝히도록 노력하였고, 풀이자가 명시되지 않은 경우

는 대개 여러 학생이 비슷한 풀이를 한 경우이거나 필자의 풀이인 경우이다.

해당 학생들과 조교분들에게 이 지면을 빌어 감사를 드린다.

2006년 11월, KAIST에서

고 봉 균

Page 7: Baltic Way 팀수학경시대회 1990-2005 pdf 보기

제 I 부

문제편

Page 8: Baltic Way 팀수학경시대회 1990-2005 pdf 보기
Page 9: Baltic Way 팀수학경시대회 1990-2005 pdf 보기

1990년 Baltic Way 9

1990년 Baltic Way

라트비아, 리가

1. 한 원의 원주 위에 정수 1; 2; : : : ; n이 적당한 순서로 쓰여져 있다. 이웃한 숫자들의 차

들의 합의 최소값은 얼마인가?

2. 네모난 종이의 네모칸들에 다음과 같이 수를 나열하였다:

1 2 3 4 5 ¢ ¢ ¢ m

1

2

3

4

...

n

1 2

3

4

5

6

7

8

9

10

11

12

13

14

임의의 양의 정수 m;n에 대해 (m;n)의 좌표의 칸에 쓰여진 수를 p(m;n)이라 할 때,

두 변수 m;n으로 된 다항식 p(m;n)을 찾아라.

3. a0 > 0, c > 0 이고,

an+1 =an + c

1¡ anc; n = 0; 1; : : :

라 하자. 처음 1990개의 항 a0; a1; a2; : : : ; a1989이 모두 양수이고 a1990 < 0 이 되는 것

이 가능하겠는가?

4. 임의의 실수 a1; a2; : : : ; an에 대해, 다음 부등식이 성립함을 증명하여라.

nXi;j=1

aiaji+ j ¡ 1 ¸ 0

5. ¤는 연산을 나타내고,각각의 실수쌍 (a; b)에 대해 실수값 a¤b 가 정의된다(예: a¤b =a+ b2 ¡ 17). 연산 ¤이 교환법칙이 성립하거나 결합법칙이 성립하면 (변수들의 가능

한 모든 값에 대해서) 참이 되고, 그렇지 않으면 거짓이 될 수 있는 등식을 하나 찾아

라.

6. ABCD는 jADj = jBCj, \A + \B = 120◦ 인 사각형이고, P는 사각형의 외부에 있

고 DPC가 정삼각형이 되게 하는 점으로 직선 DC에 대해 A와 반대편 영역에 있다.

APB도 정삼각형임을 보여라.

Page 10: Baltic Way 팀수학경시대회 1990-2005 pdf 보기

10

7. 볼록 오각형의 각 변의 중점이 이 변의 양끝점을 제외한 남은 세 꼭지점으로 이루어

진 삼각형의 중선들의 교차점과 선분으로 각각 연결되어 있다. 이러한 다섯 개의 선

분이 모두 한 점에서 만남을 증명하여라.

8. P는 삼각형 ABC의 외접원의 원주 위에 있는 점이다. P에서 직선 AB, BC, CA 위

로 내린 수선의 발이 모두 한 직선(심슨 직선이라 한다) 위에 있음은 알려져 있다. 지

름을 이루는 대칭점 P1, P2의 심슨 직선은 서로 수직임을 증명하여라.

9. 두 개의 똑같은 삼각형이 타원에 내접한다. 그 둘은 반드시 타원의 축이나 중점에 대

해 대칭인가?

10. 직선 t 위에 단위 길이의 선분 AB를 잡자. 이제 이 선분을 t에 평행하도록 유지하면

서 평면 위에서 움직이는데,점 A, B의 자취는 서로 만나지 않으며 마지막에는 t로 돌

아온다. A의 마지막 위치는 처음 위치로부터 얼마나 멀리까지 옮겨질 수 있을까?

11. 정수 계수의 다항식의 정수해의 절대값은 계수의 절대값의 최대값을 넘지 않음을 보

여라.

12. m, n은 양의 정수이다. 만약 3m+ 7n 이 83으로 나누어지는 것과 25m+ 3n 이 83으

로 나누어지는 것이 동치임을 보여라.

13. 방정식 x2 ¡ 7y2 = 1 이 무한히 많은 자연수 해를 가짐을 보여라.

14. 두 개나 그 이상의 수들의 모든 가능한 합이 합성수가 되는 1990개의 서로 소인 수들

이 존재하는가?

15. 다음 수들이 정수의 세제곱이 되는 경우가 없음을 증명하여라.

Fn = 22n + 1; n = 0; 1; 2; : : :

16. 간격이 1인 가로 세로 격자선이 그려진 종이 위에, 이 격자선을 따라 모든 변의 길이

가 홀수인 다각형을 그린다. 이 다각형의 변의 개수가 4의 배수임을 증명하여라.

17. 각각 72개, 30개의 사탕 두 더미가 있다. 두 명의 학생이 번갈아 두 더미 중 하나에서

사탕 몇 개를 가져간다. 매 번 한 더미에서 가져간 사탕의 수는 나머지 더미에 있는

사탕의 수의 배수여야 한다. 둘 중 한 더미에서 마지막 사탕을 가져가는 것이 목표일

때 이기는 사람은 처음 시작한 사람일까, 아니면 그 상대편일까?

18. 양의 정수 1; 2; : : : ; 100; 101을 각각 101번씩 써서 101£ 101 크기의 격자판의 모든 칸

을 채웠다. 최소한 11개의 서로 다른 수들이 쓰여있는 행이나 열이 존재함을 보여라.

Page 11: Baltic Way 팀수학경시대회 1990-2005 pdf 보기

1990년 Baltic Way 11

19. 다음의 성질을 갖는 집합 f1; 2; : : : ; 2n+ 1g의 부분집합들은 최대 몇 개일까?: 임의의

두 부분집합의 교집합은 하나의 원소나 혹은 몇 개의 연속하는 정수들로 이루어진다.

20. 창의력 문제: 독창적인 경시문제를 그것의 풀이와 함께 제안해보아라.

Page 12: Baltic Way 팀수학경시대회 1990-2005 pdf 보기

12

1991년 Baltic Way

에스토니아, 타르투

1. 다음 성질을 갖는 가장 작은 양의 정수 n을 구하여라: n개의 서로 다른 정수를 모은

임의의 집합 fa1; a2; : : : ; ang에 대해 모든 차 ai ¡ aj , i < j 들의 곱이 1991의 배수이

다.

2. 1021991 + 1031991 = nm 을 만족하는 양의 정수 n과 m > 1 은 존재하지 않음을 증명

하여라.

3. 12,000원부터 15,000원의 값이 나가는 20마리의 고양이들과, 100원부터 1,000원까지

에 팔리는 20개의 바구니가 있다(모든 가격은 다르다). 두 명의 소년, 지훈이와 판준

이가 같은 금액을 지불하고 각각 바구니 하나에 고양이 한 마리를 살 수 있음을 증명

하여라. 단, 가격은 10원 단위까지만 붙일 수 있고, 1원 단위 이하는 매기지 않는다.

4. p는 어떤 정수 n에 대해 p(¡n) < p(n) < n 을 만족하는 정수 계수의 다항식이다.

p(¡n) < ¡n 이 성립합을 보여라.

5. 임의의 양의 정수 a, b, c에 대해 다음 부등식이 만족함을 보여라.

1

a+1

b+1

c¸ 2

a+ b+

2

b+ c+

2

c+ a¸ 9

a+ b+ c

6. [x]는 x의 정수부를 나타내고, fxg = x¡ [x] 이다. 다음 방정식을 풀어라.

[x] ¢ fxg = 1991x

7. A, B, C는 예각 삼각형의 세 각이다. 다음 부등식을 증명하여라.

sinA+ sinB > cosA+ cosB + cosC

8. a, b, c, d, e는 서로 다른 실수들이다. 다음 방정식

(x¡ a)(x¡ b)(x¡ c)(x¡ d)

+ (x¡ a)(x¡ b)(x¡ c)(x¡ e)

+ (x¡ a)(x¡ b)(x¡ d)(x¡ e)

+ (x¡ a)(x¡ c)(x¡ d)(x¡ e)

+ (x¡ b)(x¡ c)(x¡ d)(x¡ e) = 0

이 4개의 서로 다른 실수해를 가짐을 보여라.

Page 13: Baltic Way 팀수학경시대회 1990-2005 pdf 보기

1991년 Baltic Way 13

9. 방정식 aex = x3 의 해는 모두 몇 개일까?

10. sin 3◦ 의 값을 무리식으로 나타내어라.

11. 1에서 1 000 000까지의 모든 양의 정수들을 자리수의 합이 홀수이냐 짝수이냐에 따라

두 집합으로 나누자. 두 집합 중 어느 쪽이 더 많은 수를 가질까?

12. 볼록 1991각형의 꼭지점들에 1부터 1991까지의 번호를 붙였다. 그리고, 이 1991각형

의 각 변과 대각선들을 빨간색이나 파란색으로 칠했다.꼭지점들의 번호를 어떻게 다

시 붙이더라도, 원래의 번호에서 k와 l의 꼭지점을 연결한 선의 색깔과 새로 붙인 번

호에서 k와 l의 꼭지점을 연결한 색이 같게 되는, 그런 정수 k; l이 항상 존재함을 보

여라.

13. 정삼각형을 25개의 합동인 삼각형들로 나누고 이들에 1부터 25까지 번호를 붙인다.

공통된 변을 갖는 두 개의 삼각형 중에 번호의 차가 3보다 큰 것이 존재함을 증명하

여라.

14. 어떤 성에 여러 개의 홀과 n개의 문이 있다. 모든 문은 다른 홀로 연결되거나 바깥으

로 연결되어 있다.모든 홀은 2개 이상의 문을 가지고 있다. 한 기사가 성에 들어왔다.

어떤 홀에서나 그는 그 홀에 막 들어올 때 사용한 문을 제외한 임의의 다른 문으로 나

갈 수 있다. 2n개 이상의 홀을 지나기 전에 그 기사가 바깥으로 나올 수 있는 방법을

찾아라. (지나간 홀도 다시 들어갈 때마다 셈한다)

15. 체스판의 각각의 칸에 임의의 정수가 쓰여져 있다. 킹(king)이 체스판 위에서 이동하

기 시작했다. 킹이 이동할 때마다 방문한 칸의 수에 1이 더해진다. 체스판 각 칸의 수

들이 다음을 만족하도록 하는 것이 가능한가?

(a) 모두 짝수

(b) 모두 3의 배수

(c) 모두 같은 수

16. 두 원 C1, C2 (반지름이 각각 r1, r2) 가 서로 외접하고 있고, 이 두 원의 한 공통외접

선을 l이라 하자. 세 번째 원 C3 (반지름 r3 < min(r1; r2)) 이 앞의 두 원에 외접하고,

직선 l에도 접한다. 다음을 증명하여라.

1pr3=

1pr1+

1pr2

Page 14: Baltic Way 팀수학경시대회 1990-2005 pdf 보기

14

17. 3차원 공간의 세 좌표평면(yz-평면, zx-평면, xy-평면)들에 반사하는 성질이 있다고

하자.이 평면 중 하나에 광선 하나를 쏘았다.세 좌표평면들로부터 모두 반사된 후 광

선이 향하는 방향은 처음 쏜 방향과 비교해 어떻게 바뀌는가?

18. 반지름이 1인 구 안에 부피가 12인 두 개의 사면체를 서로 겹치지 않게 넣는 것이 가

능할까?

19. 서로 외접하고 있는 세 원을 조금 팽창시키면 세 쌍의 교차점을 얻는다. 바깥쪽의 교

차점을 A1, B1, C1이라 하고, 이에 대응되는 안쪽 교차점들을 A2, B2, C2라 하자. 다

음 등식이 성립함을 보여라.

jA1B2j ¢ jB1C2j ¢ jC1A2j = jA1C2j ¢ jC1B2j ¢ jB1A2j

20. y = 1x의 그래프 위의 0 < x1 < x2 과 jABj = 2 ¢ jOAj 를 만족하는 두 점 A(x1; y1),

B(x2; y2)를 생각하자(O는 원점,즉 O(0; 0)). 선분 AB의 중점을 C라 하자. x-축과 반

직선 OA 사이의 각이 x-축과 반직선 OC 사이의 각의 3배가 됨을 보여라.

Page 15: Baltic Way 팀수학경시대회 1990-2005 pdf 보기

1992년 Baltic Way 15

1992년 Baltic Way

리투아니아, 빌뉴스

1992년 11월 5{8일

1. p, q를 연속하는 두 홀수인 소수라 하자. p+ q 가 최소 3개 이상의 1보다 큰 (꼭 서로

다를 필요는 없는) 자연수의 곱임을 증명하여라.

2. d(n)을 자연수 n의 모든 양의 약수(1과 n 포함)의 개수라 하자. n=d(n) 이 정수가 되

는 n이 무한히 많이 있음을 보여라.

3. 모든 항이 일정한 상수가 아닌 자연수들의 무한 등차수열 중 각 항이 두 (자연수의)

제곱수의 합도, 두 세제곱수의 합도 아닌 수열을 하나 찾아라.

4. 꼭지점들이 모두 격자점이고 각 변의 길이의 제곱이 연속하는 여섯 개의 양의 정수인

육각형을 그릴 수 있는가?

5. a2+b2+(a+ b)2 = c2+d2+(c+ d)2 이 성립한다. a4+b4+(a+ b)4 = c4+d4+(c+ d)4

임을 증명하여라.

6. 99개의 수k3 ¡ 1k3 + 1

, (k = 2; 3; : : : ; 100) 을 모두 곱한 것이 2=3보다 큼을 증명하여라.

7. a = 1992p1992 라 하자.

aaa¢¢¢ao

1992 와 1992

중 어느 수가 더 큰가?

8. 방정식 2x ¢ (4¡ x) = 2x+ 4 의 모든 정수해를 구하여라.

9. 다항식 f(x) = x3 + ax2 + bx+ c 는 b < 0 와 ab = 9c 를 만족한다. 다항식 f가 서로

다른 세 실근을 가짐을 증명하여라.

10. 다음 네 조건을 모두 만족하는 4차 다항식 p(x)를 모두 찾아라:

(i) p(x) = p(¡x) (모든 x에 대해)

(ii) p(x) ¸ 0 (모든 x에 대해)

(iii) p(0) = 1

(iv) p(x)는 두 극소점 x1, x2를 갖고, jx1 ¡ x2j = 2 이다.

Page 16: Baltic Way 팀수학경시대회 1990-2005 pdf 보기

16

11. Q+를 양의 유리수 전체의 집합이라 하자. 다음 조건을 만족하는 함수 f : Q+ ! Q+

가 유일하게 존재함을 보여라.

(i) 0 < q < 1=2 이면, f(q) = 1 + f(q=(1¡ 2q))(ii) 1 < q · 2 이면, f(q) = 1 + f(q ¡ 1)(iii) f(q) ¢ f(1=q) = 1 (모든 q 2 Q+에 대해)

12. N을 자연수 전체의 집합이라 하자. 함수 Á(x) : N ! N 이 일대일 대응이고 극한값

limn→∞

Á(n)

n= L

이 존재한다. 가능한 L의 값을 모두 구하여라.

13. 임의의 양수 x1; x2; : : : ; xn; y1; y2; : : : ; yn 에 대해 다음 부등식

nXi=1

1

xiyi¸ 4n2Pn

i=1(xi + yi)2

이 성립함을 보여라.

14. 어느 나라에 유한 개의 마을이 있다. 그 마을들은 일방통행로로 연결되어 있다. 어떤

두 마을을 택해도,그 중 한 마을에서 다른 마을로 갈 수 있다.다른 모든 마을로 갈 수

있는 마을이 하나 존재함을 보여라.

15. 노아는 방주에 4개의 우리가 있고 여기에 넣을 8종의 동물들이 있다. 그는 동물들을

종별로 묶어 우리에 넣을 계획을 하고 있다. 각각의 종마다 함께 어울릴 수 없는 다른

종들이 최대 3종 있음을 알아냈다. 이 동물들을 아무 불화없이 우리에 모두 넣을 수

있는 방법이 존재함을 보여라.

문제수정 이 동물들을 아무 불화없이 우리에 각각 2종씩 넣을 수 있는 방법이 존재함

을 보여라.

16. 모든 면이 평행사변형인 어떤 볼록 다면체가 있다. 그 다면체가 정확히 1992개의 면

을 가질 수 있는가?

17. 반지름이 1인 원에 내접하는 사각형 ABCD에서 대각선 AC는 원의 지름이고 또 다른

대각선 BD는 AB와 길이가 같다.두 대각선이 점 P에서 만나고, PC의 길이는 2=5이

다. 변 CD의 길이는 얼마인가?

18. 둔각이 없는 삼각형에서, 삼각형의 둘레의 길이는 항상 외접원의 지름의 2배보다 큼

을 보여라.

Page 17: Baltic Way 팀수학경시대회 1990-2005 pdf 보기

1992년 Baltic Way 17

19. 한 평면 위에 원 C가 주어져 있고, C1과 C2는 원 C와 각각 점 A, B에서 안쪽에서 접

하는 서로 만나지 않는 원이다. C1과 C2의 공통외접선 t는 두 원과 각각 점 D, E에서

접한다. AD와 BE의 교점을 F라 하자. F가 C 위에 있음 보여라.

20. a · b · c 를 직각삼각형의 세 변의 길이라 하고, 2p를 그 둘레의 길이라 하자. S를 삼

각형의 넓이라 할 때, 다음을 보여라.

p(p¡ c) = (p¡ a)(p¡ b) = S

Page 18: Baltic Way 팀수학경시대회 1990-2005 pdf 보기

18

1993년 Baltic Way

라트비아, 리가

1993년 11월 11{15일

1. a1a2a3과 a3a2a1은 세 자리의 두 십진법 수로, a1과 a3은 서로 다른 0이 아닌 수이다.

이 수들의 제곱은 각각 b1b2b3b4b5와 b5b4b3b2b1의 다섯 자리의 수이다.이러한 세 자리

의 수를 모두 구하여라.

2. 각각의 양의 정수 k에 대해서 an+ b가 어떤 양의 정수의 k제곱이 되게 하는 양의 정

수 n이 존재하는 양의 정수 a > b > 1 이 존재하는가?

3. 두 개의 소수의 곱으로 이루어진(두 소수는 서로 같아도 된다) 양의 정수를 재미있는

수라고 하자. 어떤 n개의 연속하는 양의 정수들을 택했더니 모두 재미있는 수들로만

이루어져 있었다. n의 가능한 최대값은 얼마인가?

4. 다음 식이 정수가 되도록 하는 모든 정수 n을 구하여라.s25

2+

r625

4¡ n+

s25

2¡r625

4¡ n

5. 임의의 양의 홀수 n에 대해 n12 ¡ n8 ¡ n4 + 1 이 29의 배수가 됨을 보여라.

6. 2 < x < 4 의 범위에서 정의되는 두 함수 f(x), g(x)가 2 < x < 4 인 임의의 x에 대해

2 < f(x) < 4; 2 < g(x) < 4; f(g(x)) = g(f(x)) = x; f(x) ¢ g(x) = x2

를 만족한다. f(3) = g(3) 임을 증명하여라.

7. 다음 연립방정식의 정수해를 구하여라.8>>>><>>>>:zx = y2x

2z = 4x

x+ y + z = 20

8. 각 자리의 수가 계속 증가하거나 혹은 계속 감소하는 모든 양의 정수들의 합을 구하

여라.

Page 19: Baltic Way 팀수학경시대회 1990-2005 pdf 보기

1993년 Baltic Way 19

9. 다음 연립방정식을 풀어라. 8>>>>>>><>>>>>>>:

x5 = y + y5

y5 = z + z5

z5 = t+ t5

t5 = x+ x5

10. a1; a2; a3; : : : ; an 과 b1; b2; b3; : : : ; bn 은 서로 다른 2n개의 실수로 이루어진 두 개

의 유한 수열이다. 각각의 수열을 증가수열로 재배열 한 것을 a1; a2; a3; : : : ; an 과

b1; b2; b3; : : : ; bn 이라고 하자. 다음을 증명하여라.

max1≤i≤n

jai ¡ bij ¸ max1≤i≤n

jai ¡ bij

11. 어떤 정삼각형이 n2개의 합동인 정삼각형으로 나뉘어져있다. 이 격자의 한 꼭지점에

거미 한 마리가 있고, 또다른 꼭지점에 파리 한 마리가 있다. 이들이 번갈아가면서 이

웃하는 꼭지점으로 이동한다. 거미가 항상 파리를 잡을 수 있음을 보여라.

12. 어떤 왕국에 13개의 도시가 있다. 두 도시들 간에는 양방향의 직행버스나 기차 혹은

비행기 노선을 설치할 수 있다. 이 세 종류의 탈 것 중에서 어느 두 종류를 택해도 나

머지 한 종류의 탈 것을 이용하지 않고도 모든 도시가 다 연결되게 하고 싶다. 최소

몇 개의 노선을 만들면 되겠는가?

13. 정삼각형 ABC가 100개의 합동인 정삼각형으로 나뉘어져있다. 어느 두 점도 삼각형

ABC의 변과 평행한 직선 위에 있지 않도록 작은 삼각형들의 꼭지점들을 택할 때, 최

대 몇 개의 점을 택할 수 있겠는가?

14. 한 정사각형을 16개의 같은 크기의 정사각형으로 나누어 25개의 꼭지점을 얻었다. 여

기서 최소 몇 개의 점을 빼내면, 남아있는 점들 중에 어느 4개의 점을 택해도 처음 정

사각형과 평행한 변을 갖는 정사각형의 꼭지점이 되지 않도록 할 수 있는가?

문제수정 8개의 점을 빼내면 정사각형이 하나도 남지 않도록 할 수 있고, 6개의 점을

빼내는 것으로는 정사각형이 하나도 남지 않도록 할 수 없음을 보여라.

15. 두 개의 주사위의 각각의 면에 양의 정수를 썼다. 두 개의 주사위를 던져 나온 수를

더한다. 이 합이 2, 3, 4, 5, 6, 7, 8, 9, 10, 11, 12, 13이 될 수 있고 각 합이 모두 똑같은

확률로 나오도록 주사위 눈을 정하는 것이 가능한가?

16. 반지름 r을 갖는 두 개의 원이 한 평면 위에 있고 서로 만나지 않는다. 이 평면 위의

한 직선이 한 원과 점 A, B에서 만나고 다른 원과 점 C, D에서 만나는데, jABj =

Page 20: Baltic Way 팀수학경시대회 1990-2005 pdf 보기

20

jBCj = jCDj = 14 cm 이다. 또다른 직선이 두 원과 각각 점 E, F와 점 G, H에서 만

나고, jEF j = jFGj = jGHj = 6 cm 이다. 반지름 r을 구하여라.

17. 한 평면 위의 어느 둘도 서로 평행하지 않은 세 직선을 생각해보자. 각 직선마다 한

점씩 모두 세 개의 점이 서로 다른, 0이 아닌 일정한 속도로 각 직선 위를 움직이고 있

다. (이 움직임은 무한한 시간 동안 계속 되어왔고, 앞으로도 무한히 계속된다고 생각

하자.) 이 점들이 과거, 현재,미래, 어느 때에도 한 직선 위에 절대 있지 않도록, 세 직

선의 위치, 세 점의 속력, 그리고 시점 0에서의 세 점의 위치를 정할 수 있을까?

18. jABj = 15, jBCj = 12, jACj = 13 인 삼각형 ABC가 있다. 중선 AM과 꼭지각의 이

등분선 BK가 점 O에서 만난다(M 2 BC, K 2 AC). OL ? AB, L 2 AB 라 하자.

\OLK = \OLM 임을 보여라.

19. 볼록사각형 ABCD가 중심 O인 원에 내접한다. 네 각 AOB, BOC, COD, DOA는 순

서를 적당히 잡으면 사각형 ABCD의 네 내각과 각각 크기가 같다. ABCD가 정사각

형임을 증명하여라.

20. Q를 단위 정육면체라 하자. 꼭지점들이 모두 Q의 겉표면 위에 있는 정사면체를좋은

사면체라고 부르기로 하자. 좋은 사면체의 가능한 부피들을 모두 구하여라.

Page 21: Baltic Way 팀수학경시대회 1990-2005 pdf 보기

1994년 Baltic Way 21

1994년 Baltic Way

에스토니아, 타르투

1994년 11월 11일

1. a ± b = a+ b¡ ab 라 하자. 다음을 만족하는 모든 정수해 (x; y; z)를 구하여라.

(x ± y) ± z + (y ± z) ± x+ (z ± x) ± y = 0

2. a1; a2; : : : ; a9는 a1 = a9 = 0 인 음이 아닌 실수들이고, 적어도 하나는 0이 아니다.

2 · i · 8인 i 중에 부등식 ai−1 + ai+1 < 2ai 를 만족하는 것이 있음을 보여라. 마지

막 부등식에서 2를 1.9로 바꾼다면 그래도 이 명제가 성립하겠는가?

3. 다음 식의 최대값을 구하여라.

xy + xp1¡ y2 + y

p1¡ x2 ¡

p(1¡ x2)(1¡ y2)

4.pn¡ 1 +pn+ 1 가 유리수가 되도록 하는 정수 n이 존재하는가?

5. p(x)는 정수계수 다항식이고 방정식 p(x) = 1 과 p(x) = 3 은 모두 정수해를 갖는다.

방정식 p(x) = 2 가 서로 다른 두 정수해를 가질 수 있을까?

6. p가 양의 정수이고 q가 양의 홀수인 임의의 기약분수 pq는 적당한 양의 정수 n과 k에

대해서 분수 n2k−1과 같음을 보여라.

7. p > 2 는 소수이고

1 +1

23+1

33+ ¢ ¢ ¢+ 1

(p¡ 1)3 =m

n

에서 m과 n은 서로 소이다. m이 p의 배수임을 보여라.

8. 임의의 정수 a ¸ 5 에 대해, c ¸ b ¸ a 이고 a; b; c가 직각삼각형의 세 변의 길이가 되

는 정수 b, c가 존재함을 보여라.

9. 2a + 3b 이 어떤 정수의 제곱이 되는 양의 정수쌍 (a; b)를 모두 구하여라.

10. 다음 세 조건을 만족하는 양의 정수는 모두 몇 개인가?

(a) 이 수의 각 자리수는 모두 집합 f1; 2; 3; 4; 5g 에서 고른 것이다.

(b) 연속하는 두 개의 자리수의 차는 1이다.

Page 22: Baltic Way 팀수학경시대회 1990-2005 pdf 보기

22

(c) 1994자리의 수이다.

11. NS와 EW는 원 C의 서로 직교하는 두 지름이다. 직선 `이 원 C와 점 S에서 접한다.

A, B는 C 위의 점으로 지름 EW에 대해 서로 대칭이다. `이 두 직선 NA, NB와 각

각 점 A , B 에서 만난다. jSA j ¢ jSB j = jSN j2 임을 보여라.

12. 삼각형 A1A2A3의 내접원이 변 A2A3, A3A1, A1A2와 각각 점 S1, S2, S3에서 접한다.

삼각형 A1S2S3, A2S3S1, A3S1S2의 내접원의 중심을 각각 O1, O2, O3이라고 하자.세

직선 O1S1, O2S2, O3S3가 한 점에서 만남을 보여라.

13. 한 변의 길이가 a인 정사각형이 반지름 1인 다섯 개의 원을 포함하고 이 중 어느 두

원도 서로 겹치지 않는다. a의 최소값을 구하여라.

14. ®, ¯, °는 한 삼각형에서 각각 변 a, b, c에 대응하는 각이다. 다음 부등식을 증명하여

라.

a ¢µ1

¯+1

°

¶+ b ¢

µ1

°+1

®

¶+ c ¢

µ1

®+1

¯

¶¸ 2 ¢

µa

®+

b

¯+

c

°

¶15. 변의 길이와 수선의 길이가 모두 정수이고 둘레의 길이가 1995인 삼각형이 존재하는

가?

16. 원더 아일랜드에는 고슴도치들이 살고있다. 이 고슴도치는 한 점을 공통의 끝점으로

하는 단위 길이의 세 개의 바늘로 구성되어 있고,이들 사이의 각은 모두 120◦이다.모

든 고슴도치들이 이 섬에 평평하게 누워있고, 어느 두 마리도 서로 부딪치지 않는다.

원더 아일랜드에는 유한 마리의 고슴도치들이 있음을 보여라.

17. 어떤 왕국의 왕이 13개의 무인도에 25개의 새로운 마을을 만들기로 결정했다. 각각의

섬에는 마을이 적어도 하나씩 만들어져야 한다. 서로 다른 섬에 있는 두 마을 사이에

는 직행 페리 노선을 설치할 것이다. 설치해야 하는 노선의 최소 개수를 구하여라.

18. n(> 2)개의 직선이 한 평면 위에 있다. 어느 두 직선도 평행하지 않고, 어느 세 직선

도 한 점에서 만나지 않는다. 두 직선이 만나는 모든 교점들에 1과 n¡ 1 사이의 자연

수를 하나씩 매겼다. n이 짝수일 때, 또 그 때만, 각각의 직선이 다른 직선들과 만나

는 n¡ 1개의 교점에 1부터 n¡ 1 까지의 수를 모두 갖도록 수를 매기는 것이 가능함

을 증명하여라.

Page 23: Baltic Way 팀수학경시대회 1990-2005 pdf 보기

1994년 Baltic Way 23

19. 원더 아일랜드의 정보부는 Tartu에 16명의 스파이를 두고 있다. 이들 각각은 그들 동

료 몇 명을 감시하고 있다. 만약 스파이 A가 스파이 B를 감시하고 있다면, B는 A를

감시하고 있지 않다. 또한, 어떤 10명의 스파이를 택해도, 첫 번째 스파이가 두 번째

스파이를 감시하고, 두 번째가 세 번째를, : : : ; 열 번째가 첫 번째를 감시하고 있도록

번호를 붙일 수 있다. 임의의 11명의 스파이도 이와 비슷하게 번호를 붙일 수 있음을

보여라.

20. 한 정삼각형을 각각의 변에 평행한 직선들에 의해 9000000개의 합동인 정삼각형으로

나누었다. 작은 삼각형들의 각 꼭지점은 세 가지 색 중 하나로 색칠된다. 원래 삼각형

의 변에 평행한 세 변을 갖는 삼각형의 꼭지점이 되는 같은 색의 세 점이 존재함을 보

여라.

Page 24: Baltic Way 팀수학경시대회 1990-2005 pdf 보기

24

1995년 Baltic Way

스웨덴, 바스테라스

1995년 11월 12일

1. 다음 연립방정식을 만족하는 모든 양의 정수해 (x; y; z)를 찾아라.8<:x2 = 2(y + z)

x6 = y6 + z6 + 31(y2 + z2)

2. a와 k는 양의 정수이고, a2 + k 가 (a¡ 1)a(a+1) 의 약수가 된다. k ¸ a 임을 보여라.

3. a, b, c는 각각 서로 소인 양의 정수들이다. 또, a와 c는 홀수이고, a2+ b2 = c2 이 성립

한다. b+ c 가 어떤 정수의 제곱임을 보여라.

4. J씨는 M씨보다 나이가 많다. J씨는 자신의 나이(정수)의 두 자리수를 바꾼다면 M씨

의 나이가 된다는 것을 발견했다. 또한, 그들의 나이의 제곱의 차는 어떤 정수의 제곱

이다. M씨와 J씨의 나이는?

5. a < b < c 는 세 개의 양의 정수이다. 연속하는 2c개의 양의 정수 중에는, abc가 xyz를

나누는 서로 다른 세 수 x, y, z가 존재함을 보여라.

6. 양수 a; b; c; d 에 대해 다음이 성립함을 보여라.

a+ c

a+ b+

b+ d

b+ c+

c+ a

c+ d+

d+ b

d+ a¸ 4

7. sin3 18◦ + sin2 18◦ = 1=8 임을 증명하여라.

8. 실수 a, b, c가 jaj ¸ jb+cj, jbj ¸ jc+aj, jcj ¸ ja+bj의부등식들을만족한다. a+b+c = 0

임을 보여라.

9. 다음을 증명하여라.

1995

2¡ 1994

3+1993

4¡ ¢ ¢ ¢ ¡ 2

1995+

1

1996=

1

999+

3

1000+ ¢ ¢ ¢+ 1995

1996

10. 0이 아닌 모든 실수의 집합에 정의되고 실수값을 가지며 다음 조건들을 만족하는 함

수 f를 모두 찾아라.

(i) f(1) = 1

Page 25: Baltic Way 팀수학경시대회 1990-2005 pdf 보기

1995년 Baltic Way 25

(ii) f( 1x+y ) = f( 1x ) + f( 1y ) (0이 아닌 모든 x, y, x+ y에 대해서)

(iii) (x+ y) ¢ f(x+ y) = xy ¢ f(x) ¢ f(y) (0이 아닌 모든 x, y, x+ y에 대해서)

11. 집합 f1; 2; : : : ; 1995g를 공집합이 아닌 세 집합으로 분할하는데, 이들 중 어느 것도 연

속하는 정수 쌍을 포함하지 않도록 하는 방법은 모두 몇 가지인가?

12. 95개의 상자에 19개의 공을 임의로 나누어 넣었다고 하자. 6개의 새로운 공을 가져와

서 6개의 상자를 택해 각각 하나씩 넣는다. 이러한 방법을 여러 번 반복하여, 95개의

상자에 모두 똑같은 개수의 공이 있도록 할 수 있겠는가?

13. 다음 이인용 게임을 생각해보자. 적당량의 공기돌이 탁자 위에 놓여있다. 두 명의 선

수가 교대로, 적당한 양의 정수의 제곱 개의 공기돌을 탁자에서 가져간다. 돌을 더 이

상 가져갈 수 없게 되는 차례의 선수가 게임에서 진다. 첫 번째 선수가 어떻게 경기해

도 두 번째 선수가 항상 이길 수 있는 초기 상황(처음 공기돌의 개수)이 무한히 많이

있음을 보여라.

14. 정삼각형 격자가 그려진 무한히 큰 종이 위에 n마리의 벼룩이 있다. 처음에는 벼룩들

이 n2개의 작은 삼각형들로 이루어진 어떤 큰 정삼각형 안에 있는 서로 다른 삼각형

안에 있다. 벼룩들은 1초에 한 번씩 뛰어, 자기가 있던 삼각형과 한 꼭지점만을 공유

하며 마주보는 이웃한 세 삼각형 중 하나로 각각 옮겨간다. 양의 정수 n이 어떤 수일

때, 유한 번 뛰어서 모든 n마리의 벼룩들이 하나의 작은 삼각형에서 모두 모이게 될

수 있는 초기 배치가 존재하겠는가?

15. 2n + 1개의 꼭지점을 가지고 있는 다각형이 있다. 각 꼭지점과 변의 중점들에 1부터

4n+ 2 까지의 자연수를 하나씩 붙여, 각 변에 붙여진 세 수의 합이 모두 같아지게 할

수 있음을 보여라.

16. 삼각형 ABC에서 C의 외각의 이등분선을 `이라 하자. AB의 중점 O를 지나고 `에 평

행한 직선이 AC와 E에서 만난다. jACj = 7 이고 jCBj = 4 일 때, jCEj의 길이를 구

하여라.

Page 26: Baltic Way 팀수학경시대회 1990-2005 pdf 보기

26

17. 모든 삼각형 ABC에 대해 다음 부등식이 항상 성립한다.

max(hA; hB; hC) · ® ¢min(mA;mB;mC)

단, hA, hB, hC는 세 수선의 길이를 나타내고, mA, mB, mC는 세 중선의 길이를 나

타낸다. 이런 ®가 존재함을 보이고, ®의 최소값을 구하여라.

18. 삼각형 ABC의 변 AC의 중점을 M이라 하고, B에서 AC 위에 내린 수선의 발을 H라

하자. 또, A와 C에서 각 B의 이등분선 위로 내린 수선의 발을 각각 P , Q라 하자. 네

점 H, P , M , Q가 한 원 위에 있음을 보여라.

19. 다음의 방법이 우주비행사 훈련에 사용된다고 한다:

반지름이 2R인 원 C2가 반지름이 nR인 원 C1의 내부를 따라 구른다. n은 2보다 큰

정수이다. 우주비행사는 원 C2의 내부를 따라 구르는 반지름 R인 세 번째 원 C3에 매

달려 있는데, 두 원 C1과 C2가 접하는 점으로부터 두 원 C2와 C3이 접하는 점까지의

거리가 항상 최대가 되도록 유지된다. 원 C2가 원 C1의 내부를 한 번 완전히 도는 동

안 우주비행사는 원 C3과 함께 (지면을 기준으로) 몇 번의 회전을 하겠는가?

20. 모든 꼭지점의 좌표가 모두 정수인 볼록 오각형의 넓이는 52보다 작지 않음을 보여라.

Page 27: Baltic Way 팀수학경시대회 1990-2005 pdf 보기

1996년 Baltic Way 27

1996년 Baltic Way

핀란드, 발키코스키

1996년 11월 3일

1. 정 1996각형의 두 대각선(의 연장선) 사이의 각을 ®라 하고, 또다른 이런 각을 ¯(6=0)라 하자. ®=¯가 유리수임을 보여라.

2. P는 선분 AB 위의 점이고, AB를 지름으로 하는 반원 C1 안에 AP , PB를 지름으로

하는 반원 C2, C3이 있다. P에서 AB에 수직한 직선 PQ를 그린다.원 C는 C1에 내접

하고 C3, PQ에 외접한다. C1의 내부 중에서 C2, C3, C의 외부에 해당하는 영역의 넓

이는 39¼이고, 원 C의 넓이는 9¼이다. 지름 AB의 길이를 구하여라.

3. ABCD는 단위 정사각형이고, P와 Q는 같은 평면 위의 점인데 Q는 삼각형 BPC의

외심이고, D가 삼각형 PQA의 외심이라고 한다. 선분 PQ의 길이로 가능한 값을 모

두 구하여라.

4. ABCD는 AD k BC 인 사다리꼴이다. P는 직선 AB 위의 점 중에서 \CPD가 최대

인 점이고, Q는 직선 CD 위의 점 중에서 \BQA가 최대인 점이다. P가 선분 AB 위

에 있을 때, \CPD = \BQA 임을 보여라.

5. ABCD는 원에 내접하는 볼록 사각형이고, 삼각형 BCD, ACD, ABD, ABC의 내접

원의 반지름을 각각 ra, rb, rc, rd라 하자. ra + rc = rb + rd 임을 보여라.

6. a, b, c, d는 ab = cd 를 만족하는 양의 정수이다. a+ b+ c+ d 가 소수가 아님을 보여

라.

7. 정수 수열 a1; a2; : : : 이 a1 = 1, a2 = 2,

an+2 =

8><>:5an+1 ¡ 3an (an ¢ an+1 이 짝수일 때)

an+1 ¡ an (an ¢ an+1 이 홀수일 때)(n ¸ 1)

로 주어져 있다. 모든 n에 대해서 an 6= 0 임을 보여라.

Page 28: Baltic Way 팀수학경시대회 1990-2005 pdf 보기

28

8. 다음과 같은 수열을 생각하자:

x1 = 19; x2 = 95; xn+2 = lcm(xn+1; xn) + xn (n ¸ 1)

단, lcm(a; b)는 a와 b의 최소공배수를 뜻한다. x1995와 x1996의 최대공약수를 구하여

라.

9. n과 k는 1 · k < n 인 정수이다. 다음 조건을 만족하는 정수 b와 n개의 정수의 집합

A를 찾아라.

(i) A의 서로 다른 k ¡ 1개의 원소들의 곱은 항상 b의 배수가 아니다.

(ii) A의 서로 다른 k개의 원소들의 곱은 항상 b의 배수이다.

(iii) A의 서로 다른 어떤 두 원소도 하나가 다른 것의 배수가 되지 않는다.

10. 양의 정수 n의 서로 다른 양의 약수(1과 n을 포함한)의 개수를 d(n)으로 나타낸다.

a > 1 와 n > 0 은 an + 1 이 소수가 되도록 하는 정수이다. d(an ¡ 1) ¸ n 임을 보여

라.

11. 실수 x1; x2; : : : ; x1996 은 다음 성질을 만족한다: 임의의 2차 다항식 W에 대해 W (x1),

W (x2), : : : ; W (x1996) 중 적어도 3개의 값이 같다. x1; x2; : : : ; x1996 중 적어도 3개의

수가 같음을 보여라.

12. S는 정수의 집합으로 0과 1996을 포함한다. S의 원소를 계수로 갖는 0이 아닌 임의의

다항식의 정수해가 다시 S에 속한다. ¡2가 S에 속함을 보여라.

13. 정수 범위에서 정의되는 함수 f가 임의의 정수 x에 대해

f(x) = f(x2 + x+ 1)

을 만족한다. 이러한 성질을 만족하는 (a) 모든 우함수, (b) 모든 기함수 f를 구하여

라.

14. 함수 f(x) = xn + an−1xn−1 + ¢ ¢ ¢ + a1x + a0 (n > 1) 의 그래프가 직선 y = b 와

점 B1; B2; : : : ; Bn (왼쪽부터 오른쪽으로) 에서 만나고, 직선 y = c (c 6= b) 와는 점

C1; C2; : : : ; Cn (왼쪽부터 오른쪽으로) 에서 만난다. P를 직선 y = c 위의 Cn의 오른

쪽에 있는 점이라고 하자. 다음 합을 구하여라.

cot(\B1C1P ) + ¢ ¢ ¢+ cot(\BnCnP )

Page 29: Baltic Way 팀수학경시대회 1990-2005 pdf 보기

1996년 Baltic Way 29

15. 임의의 정수 n > 2 과 임의의 양의 실수 x1; : : : ; xn 에 대해서 다음 부등식

x1x2 + x2x3 + ¢ ¢ ¢+ xn−1xn + xnx1 ¸ xa1xb2x

a3 + xa2x

b3x

a4 + ¢ ¢ ¢+ xanx

b1x

a2

이 성립한다. 어떤 양의 실수 a, b에 대해 이것이 만족하는가?

16. 무한히 큰 체스판에서 두 명의 선수가 번갈아 표시가 없는 빈 칸에 자기 표시를 한다.

한 명은 X 표시를 사용하고 다른 한 명은 O 표시를 사용한다. 자신의 표시로 2£ 2 정사각형을 먼저 채우는 사람이 이긴다. 먼저 시작한 선수가 항상 이길 수 있는가?

17. 여덟 개의 숫자 1, 3, 4, 5, 6, 7, 8, 9를 정확히 한 번씩만 사용해서, 세 자리의 수

A와 두 자리의 수 B, C (B < C), 그리고 한 자리의 수 D를 만든다. 이 수들은

A+D = B +C = 143 을 만족한다. 이렇게 수를 만드는 방법은 모두 몇 가지인가?

18. 어느 올림피아드 대회의 심사위원은 처음에는 모두 30명이었다. 심사위원들 각각은

이들 동료 중 몇 명은 유능하고 그 외 나머지는 무능하다고 생각한다. 또한 이러한 생

각은 변하지 않는다. 각 세션을 시작할 때마다 투표가 행해지고, 투표자의 과반수가

무능하다고 생각하는 사람은 남은 올림피아드 대회에서 심사위원에서 제외된다. 최

대 15번의 세션이 지난 후에는 더 이상 제외되는 사람이 없음을 보여라. (아무도 자

기 자신이 유능한가에 대한 투표는 할 수 없다)

19. 각각 38, 45, 61, 70개의 성냥개비가 있는 성냥개비 네 더미가 있다.두 명의 선수가 번

갈아, 임의로 두 더미를 선택한 후, 한 더미에서 성냥개비 몇(6= 0) 개를 가져오고, 다

른 한 더미에서도 성냥개비를 몇(6= 0) 개를 가져온다. 더 이상 이렇게 할 수 없는 선

수가 지게 된다. 어느 선수가 필승의 전략을 가질 수 있는가?

20. 모든 양의 정수들을 다음의 성질이 만족되도록 서로 소인 두 집합 A, B로 나누는 것

이 가능한가?

(i) A의 어느 세 개의 수도 등차수열의 연속한 세 항을 이루지 않는다.

(ii) B의 수들만으로 상수수열이 아닌 무한 등차수열을 만들 수 없다.

Page 30: Baltic Way 팀수학경시대회 1990-2005 pdf 보기

30

1997년 Baltic Way

덴마크, 코펜하겐

1997년 11월 9일

1. 모든 실수 x, y에 대해서 f(x)f(y) = f(x¡ y) 를 만족하는, 항등적으로 0은 아닌 함수

f : R! R 을 모두 구하여라.

2. 모든 양의 정수가 정확히 한 번씩 나타나는 양의 정수들의 수열 a1; a2; a3; : : : 이 있

다. 1 < l < m 이고 a1 + am = 2al 인 두 정수 l, m이 존재함을 보여라.

3. x1 = 1 이고 xn+1 = xn +jxnn

k+2 (n = 1; 2; 3; : : : ) 로 주어진 수열에서 x1997의 값을

구하여라. 단, bxc는 x를 넘지 않는 가장 큰 정수를 나타낸다.

4. x1; : : : ; xn의 산술평균 a가 다음을 만족함을 증명하여라.

(x1 ¡ a)2 + ¢ ¢ ¢+ (xn ¡ a)2 · 1

2(jx1 ¡ aj+ ¢ ¢ ¢+ jxn ¡ aj)2

5. 양의 정수들의 수열 u0; u1; : : : 에서, u0은 임의의 수이고,

un+1 =

8><>:12un (un이 짝수일 때)

a+ un (un이 홀수일 때)(n ¸ 0)

이다. 단, a는 주어진 양의 홀수이다. 이 수열이 언젠가부터는 주기적으로 반복됨을

보여라.

6. a ¸ b ¸ c 와 1 ¢ a3 + 9 ¢ b2 + 9 ¢ c+ 7 = 1997 을 만족하는 음이 아닌 정수들의 순서쌍

(a; b; c)를 모두 찾아라.

7. P와 Q는 정수 계수를 갖는 다항식이다.정수 a와 a+1997이 P의 해이고, Q(1998) =

2000 이라 가정하자. 방정식 Q(P (x)) = 1 이 정수해를 갖지 않음을 보여라.

8. 1996과 1997을 더할 때,우리는 먼저 일의 자리의 6과 7을 더한다. 13을 얻으면 아래에

3을 쓰고, 1을 다음 열로 \자리올림"한다. 이러한 식으로 계속하면, 이 덧셈에는 모두

세 번의 자리올림을 하게 됨을 알 수 있다.

3 9 9 3+ 1 9 9 7

1 9 9 61 1 1

Page 31: Baltic Way 팀수학경시대회 1990-2005 pdf 보기

1997년 Baltic Way 31

1996 ¢ k 와 1997 ¢ k 를 더할 때, 전체 계산에서 자리올림이 한 번도 일어나지 않도록

하는 양의 정수 k가 존재하는가?

9. 대세계 속의 모든 세계들에는 1; 2; 3; : : : 의 수가 붙여져 있고, 임의의 정수 n ¸ 1 에

대해, 수 n, 2n, 3n+1의 세계들 사이에는 차원의 문이 연결되어 있어서 마법사 간달

프가 양방향으로 오갈 수 있다. 간달프가 어느 세계에서 여행을 시작해도 항상 다른

어떤 세계에든지 갈 수 있는가?

10. 임의의 연속하는 79개의 양의 정수들 중에는,십진법으로 썼을 때 자리수의 합이 13의

배수가 되는 수가 항상 있음을 증명하여라.

11. 서로 다른 점들 A1; A2; A3; : : : 과 B1; B2; B3; : : : 을 두 개의 평행한 직선 위에 각각

jAiAi+1j = 1, jBiBi+1j = 2 (i = 1; 2; : : : ) 이 되도록 잡았다. \A1A2B1 = ® 일 때, 무

한합 \A1B1A2 +\A2B2A3 +\A3B3A4 + ¢ ¢ ¢ 의 값을 구하여라.

12. 두 원 C1, C2가 두 점 P와 Q에서 만난다. P를 지나는 직선이 C1, C2와 A, B에서 각

각 다시 만나고, X는 AB의 중점이다. Q와 X를 지나는 직선이 C1, C2와 Y , Z에서 각

각 다시 만난다. X가 Y Z의 중점임을 증명하여라.

13. 서로 다른 다섯 점 A, B, C, D, E가 jABj = jBCj = jCDj = jDEj 가 되게 한 직선 위

에 놓여있다. 점 F는 직선의 바깥에 있다. 삼각형 ADF의 외심을 G, 삼각형 BEF의

외심을 H라 하자. 직선 GH와 FC가 서로 수직임을 보여라.

14. 삼각형 ABC에서 jACj2은 jBCj2과 jABj2의 산술평균이다. cot2B ¸ cotA cotC 임을

보여라.

15. 예각삼각형 ABC에서 \A, \B, \C의 이등분선이 외접원과 각각 A1, B1, C1에서 다

시 만난다. AB와 B1C1의 교점을 M , BC와 A1B1의 교점을 N이라 하자. MN이 삼

각형 ABC의 내심을 지남을 증명하여라.

16. 5£ 5 체스판에서 두 명이 다음과 같은 경기를 한다. 첫 번째 선수가 knight를 어떤 칸

위에 놓는다.그럼 두 번째 선수부터 시작하여 번갈아가며 그 knight를 체스의 규칙에

따라 움직인다. 한 번 지나간 칸으로 knight를 옮기는 것은 허용되지 않고, 더 이상 움

직일 수 없는 선수가 진다. 둘 중 누가 승리의 전략을 가지게 되는가?

Page 32: Baltic Way 팀수학경시대회 1990-2005 pdf 보기

32

17. 어떤 직사각형은 n개의 같은 크기의 정사각형들로 나뉘어질 수 있다. 이 직사각형은

n+76개의 같은 크기의 정사각형들로도 나뉘어질 수 있다. 가능한 n의 값을 모두 구

하여라.

18. (a) 임의의 음이 아닌 정수 n이 n = a + b (a 2 A; b 2 B) 꼴로 유일하게 나타내어

지도록 하는, 음이 아닌 정수들로 이루어진|반드시 서로 소일 필요는 없는|두

개의 무한 집합 A, B의 존재성을 보여라.

(b) 위와 같은 각 순서쌍 (A;B)에 대해, A나 B는 어떤 정수 k > 1 의 배수만을 포함

함을 증명하여라.

19. 어느 숲 속에는 n(¸ 3) 마리의 동물들이 자기 굴에서 살고있고, 임의의 두 굴은 각각

정확히 하나의 독립된 통로로 연결되어 있다. 숲의 왕을 뽑기에 앞서 몇 동물들이 선

거 운동을 벌였다. 선거 운동을 하는 각 동물은 다른 굴들을 정확히 한 번씩 방문했

고, 굴과 굴 사이를 이동할 때는 굴통로만을 이용했으며, 한 통로를 지나다 다른 통로

로 옮겨가지도 않았고, 선거 운동의 마지막에는 자신의 굴로 돌아왔다. 또한 어떤 굴

통로도 기껏해야 한 마리의 선거 운동 동물에 의해서만 사용되었다고 한다.

(a) 임의의 소수 n에 대해, 선거 운동을 하는 동물의 수는 최대n¡ 12

마리임을 증명

하여라.

(b) n = 9 일 때, 선거 운동 동물은 최대 몇 마리인가?

20. 12장의 카드가 한 줄로 놓여있다. 카드는 두 면 다 흰 것, 두 면 다 검은 것, 검은 면과

흰 면이 하나씩 있는 것 등 모두 3종류이다. 처음에는 12장 중 9장의 카드가 검은 면

을 보이고 있었다. 1{6번의 카드들을 뒤집었더니, 이제는 12장 중 4장의 카드가 검은

면을 보이게 되었다. 다시 4{9번의 카드들을 뒤집었고, 6장의 카드가 검은 면을 보이

게 되었다. 마지막으로 1{3번의 카드와 10{12번의 카드를 뒤집은 후에는 5장의 카드

가 검은 면을 보였다. 각 종류의 카드가 각각 몇 장씩 있는가?

Page 33: Baltic Way 팀수학경시대회 1990-2005 pdf 보기

1998년 Baltic Way 33

1998년 Baltic Way

폴란드, 바르샤바

1998년 11월 8일

1. Z+ 을 모든 양의 정수의 집합이라 하자. 모든 x; y 2 Z+ 에 대해서 다음 조건을 만족

하는 함수 f : Z+ ! Z+ 을 모두 찾아라.

f(x; x) = x

f(x; y) = f(y; x)

(x+ y)f(x; y) = yf(x; x+ y)

2. 양의 정수 a, b, c를 세 변의 길이로 갖는 삼각형이 존재하고, 변 c의 대각의 크기가

120◦이면 순서쌍 (a; b; c)를 준-피타고라스쌍이라고 부른다. 만약 (a; b; c)가 준-피타고

라스쌍이면 c가 5보다 큰 소인수를 가짐을 보여라.

3. 다음 방정식을 만족하는 양의 정수쌍 (x; y)를 모두 찾아라.

2x2 + 5y2 = 11(xy ¡ 11)

4. P를 정수 계수를 갖는 다항식이라고 하자. n = 1; 2; 3; : : : ; 1998 에 대해서 P (n)이 세

자리의 양의 정수라고 가정하자. 다항식 P가 정수해를 갖지 않음을 보여라.

5. a는 홀수인 한 자리 숫자, b는 짝수인 한 자리 숫자이다. 모든 양의 정수 n에 대해서,

십진법으로 썼을 때 a와 b만을 자리수로 갖는 양의 정수 중에 2n의 배수가 존재함을

보여라.

6. P는 6차 다항식이고, a와 b는 0 < a < b 인 실수이다. P (a) = P (¡a), P (b) = P (¡b),P (0) = 0 이라 할 때, 모든 실수 x에 대해 P (x) = P (¡x) 임을 보여라.

7. R을 모든 실수들의 집합이라고 하자. 모든 x; y 2 R 에 대해 다음 방정식을 만족하는

함수 f : R! R 을 모두 찾아라.

f(x) + f(y) = f(f(x)f(y))

8. Pk(x) = 1 + x+ x2 + ¢ ¢ ¢ + xk−1 라 하자. 모든 실수 x와 모든 양의 정수 n에 대해서

다음 식이 성립함을 보여라.nX

k=1

µn

k

¶Pk(x) = 2

n−1Pn

µ1 + x

2

Page 34: Baltic Way 팀수학경시대회 1990-2005 pdf 보기

34

9. ®, ¯는 0 < ® < ¯ < ¼=2 를 만족하고, °와 ±는 다음과 같이 정의되는 수이다.

(i) 0 < ° < ¼=2 이고, tan °는 tan®와 tan¯의 산술평균이다.

(ii) 0 < ± < ¼=2 이고,1

cos ±는

1

cos®와

1

cos¯의 산술평균이다.

° < ± 임을 보여라.

10. n ¸ 4 은 짝수이다. 정 n각형과 정 (n ¡ 1)각형이 단위원에 내접한다. 이 n각형의 각

각의 꼭지점에 대해서, 원주를 따라 재었을 때 그 점과 가장 가까운 (n¡ 1)각형의 꼭

지점까지의 거리를 생각한다. S를 이 n개의 거리의 합이라 하자. S가 n에 의해서만

결정되고 두 다각형의 상대적 위치와는 상관이 없음을 보여라.

11. a, b, c는 외접원의 반지름이 R인 삼각형의 세 변의 길이이다.

R ¸ a2 + b2

2p2a2 + 2b2 ¡ c2

의 부등식을 증명하여라. 그리고 언제 등호가 성립하겠는가?

12. \BAC = 90◦ 인 삼각형 ABC가 있다. D는 변 BC 위의 점이고, \BDA = 2\BAD

를 만족한다. 다음 식을 증명하여라.

1

jADj =1

2

µ1

jBDj +1

jCDj¶

13. 볼록 오각형 ABCDE에서, 변 AE와 BC가 평행하고 \ADE = \BDC 이다. 대각선

AC와 BE는 점 P에서 만난다. \EAD = \BDP 이고 \CBD = \ADP 임을 보여라.

14. jABj < jACj 인 삼각형 ABC가 주어져있다. B를 지나면서 AC에 평행한 직선이 각

BAC의 외각의 이등분선과 점 D에서 만난다. C를 지나면서 AB에 평행한 직선이 같

은 이등분선과 점 E에서 만난다. F는 변 AC 위의 점이고 jFCj = jABj 를 만족한다.

jDF j = jFEj 임을 보여라.

15. 예각 삼각형 ABC가 주어져있다. A에서 BC에 내린 수선의 발을 D라 하자. E는 선

분 AD 위의 점으로jAEjjEDj =

jCDjjDBj

를 만족한다. D에서 BE에 내린 수선의 발을 F라 하자. \AFC = 90◦ 임을 보여라.

16. 13£ 13 체스판에서 중앙의 한 칸만을 제외하고 4£ 1 크기의 42개의 타일로 전부 까

는 것이 가능하겠는가? (각 타일은 정확히 체스판의 네 칸을 깔고, 타일들은 겹치지

않는 것으로 한다.)

Page 35: Baltic Way 팀수학경시대회 1990-2005 pdf 보기

1998년 Baltic Way 35

17. n과 k는 양의 정수이다. 같은 크기의 nk개의 물건과 각각 n개의 물건을 담을 수 있는

k개의 상자가 있다. 각 물건은 k가지 색 중 어느 하나로 칠해져 있다. 각 상자가 기껏

해야 2가지 색의 물건들을 담도록 물건들을 모두 상자에 담을 수 있음을 보여라.

18. 다음 성질을 갖는 집합 S가 존재하도록 하는 양의 정수 n을 모두 구하여라.

(i) S는 2n−1보다 작은 n개의 양의 정수들로 구성되어 있다.

(ii) S의 임의의 서로 다른 부분집합 A, B에 대해서, A의 원소들의 합은 B의 원소들

의 합과 같지 않다.

19. 각각 1000명의 선수로 이루어져 있는 두 팀 사이의 탁구 시합이 있다. 각 선수들은

상대팀의 모든 선수와 각각 정확히 한 번씩 경기를 하고, 무승무는 없다. 어느 한 팀

에 속한 10명의 선수를 잘 뽑으면, 상대팀의 모든 선수가 이 10명의 선수들 중 누군가

1명에게는 패하였음을 증명하여라.

20. m의 자리수 중에 1, 9, 9, 8이 이 순서로 나타난다면, m이 1998을 덮는다고 한다. (예

를 들어, 215993698은 1998을 덮지만 213326798은 덮지 않는다.) 자리수에 0을 포함

하지 않는 n자리의 양의 정수 중에서 1998을 덮는 것의 개수를 k(n)이라 하자(n ¸ 5).k(n)을 8로 나누었을 때 나머지는 얼마인가?

Page 36: Baltic Way 팀수학경시대회 1990-2005 pdf 보기

36

1999년 Baltic Way

아이슬란드, 레이캬비크

1999년 11월 6일

1. 다음 연립방정식을 만족하는 모든 실수 a, b, c, d를 구하여라.8>>>>>>><>>>>>>>:

abc+ ab+ bc+ ca+ a+ b+ c = 1

bdc+ bc+ cd+ db+ b+ c+ d = 9

cda+ cd+ da+ ac+ c+ d+ a = 9

dab+ da+ ab+ bd+ d+ a+ b = 9

2. n을 십진법으로 쓴 후 마지막 세 자리를 지웠더니 n의 세제곱근이 되었다. 이런 성질

을 갖는 양의 정수 n을 모두 찾아라.

3. a1 + a2 + ¢ ¢ ¢+ an = 0 을 만족하는 모든 실수 a1; a2; : : : ; an에 대해 다음 부등식을 만

족하는 모든 양의 정수 n ¸ 3 을 찾아라.

a1a2 + a2a3 + ¢ ¢ ¢+ an−1an + ana1 · 0

4. 모든 양의 실수 x, y에 대해

f(x; y) = min

µx;

y

x2 + y2

¶가 성립한다. 임의의 양수 x, y에 대해 f(x; y) · f(x0; y0) 을 만족하는 x0, y0이 존재

함을 보여라. 그리고 f(x0; y0)을 구하여라.

5. 점 (a; b)는 원 x2+y2 = 1위에 있다.이 점에서 원에 접하는 접선은 포물선 y = x2+1

과 정확히 한 점에서 만난다. 이러한 점 (a; b)를 모두 찾아라.

6. n£n 체스판에서(n ¸ 4) 한 쪽 구석칸으로부터 대각선으로 맞은 편에 있는 구석칸으

로 knight를 옮기려면 최소 몇 번 이동해야 하는가?

7. 8£ 8 체스판 위에서 공통된 변이나 꼭지점을 가지는 두 칸을 서로 인접하다고 한다.

king이 적당한 칸에서 시작해서 모든 칸을 한 번씩 지나도록 하는데, 맨처음을 제외

한 모든 이동이 이미 지났던 칸들 중 짝수 개가 인접한 칸으로 움직이도록 할 수 있는

가?

Page 37: Baltic Way 팀수학경시대회 1990-2005 pdf 보기

1999년 Baltic Way 37

8. 서로 다른 무게를 갖는 1999개의 동전이 있다. 임의의 세 개의 동전에 대해 그 중 중

간 무게를 갖는 동전을 한 번의 작동으로 골라낼 수 있는 기계가 있다. 이 기계를 사

용하여 1000번째의 무게에 해당되는 동전을 찾아내려고 하는데, 1000000번 이상 작

동시킬 필요가 없음을 증명하여라. 또한 이 동전이 이 기계를 사용하여 무게 순서를

알아낼 수 있는 유일한 동전임을 보여라.

9. 한 변의 길이가 3인 정육면체는 27개의 단위 정육면체로 나뉘어진다. 1, 2, 3, : : : ; 27의

수를 각 단위 정육면체마다 하나씩 배정하여 붙였다. 정육면체의 가로, 세로, 높이 중

어느 한 방향으로 한 줄을 이루는 세 정수의 합을일렬합이라 하자. 그럼 각 방향마다

9개씩 모두 27개의 일렬합이 생긴다. 이 27개의 일렬합 중 홀수인 것은 최대 몇 개인

가?

10. 반지름이 1인 원판의 (원주를 포함한) 점들을 세 부분집합으로 분할하는데, 거리 1만

큼 떨어진 두 점이 한 집합에 포함되지 않도록 할 수 있는가?

11. 어느 세 점도 한 직선 위에 있지 않은, 평면 위의 임의의 네 점이 있다. 이 네 점 중 세

점을 지나고, 네 번째 점 역시 지나거나 혹은 내부에 포함하는 원이 존재함을 보여라.

12. 2jABj = jACj+ jBCj 인 삼각형 ABC가 있다. ABC의 내심, ABC의 외심, AC의 중

점과 BC의 중점이 모두 한 원 위에 있음을 보여라.

13. 삼각형 ABC의 각 A와 각 B의 이등분선이 변 BC, CA와 각각 점 D, E에서 만난다.

jAEj+ jBDj = jABj 라고 할 때, 각 C의 크기를 구하여라.

14. 삼각형 ABC는 jABj = jACj인 이등변삼각형이다.점D와 E는 각각 변 AB와 AC 위

에 있다. B를 지나고 AC에 평행한 직선이 직선 DE와 F에서 만난다.또, C를 지나고

AB에 평행한 직선은 직선 DE와 G에서 만난다. 사변형 PQRS의 넒이를 [PQRS]와

같이 나타내기로 할 때, 다음을 증명하여라.

[DBCG]

[FBCE]=jADjjAEj

15. \C = 60◦ 이고 jACj < jBCj 인 삼각형 ABC가 있다. 변 BC 위에 jBDj = jACj 가되도록 점 D를 잡는다. 또, 변 AC의 연장선 위에 jACj = jCEj 가 되도록 점 E를 잡

는다. jABj = jDEj 임을 증명하여라.

16. 적당한 양의 정수 m, n에 대해 k = 19n ¡ 5m 꼴로 나타내어지는 수들 중 가장 작은

양의 정수 k를 구하여라.

Page 38: Baltic Way 팀수학경시대회 1990-2005 pdf 보기

38

17. 다음을 만족하는 유한한 정수 수열 c1; c2; : : : ; cn이 존재하는가? a+ c1; : : : ; a+ cn 들

이 모두 소수가 되도록 하는 정수 a의 개수가 하나보다 많지만 무한히 많지는 않다.

18. m은 m ´ 2 (mod 4) 인 양의 정수이다. m = ab 와 0 < a¡ b <p5 + 4

p4m+ 1 을 만

족하는 양의 정수쌍 (a; b)는 많아야 하나뿐임을 보여라.

19. 모든 소수 p에 대해 p2 + k 가 합성수가 되는 양의 짝수 k가 무한히 많음을 보여라.

20. a, b, c, d는 a > 3b > 6c > 12d 와 a2 ¡ b2 + c2 ¡ d2 = 1749 를 만족하는 소수들이다.

a2 + b2 + c2 + d2 의 가능한 모든 값을 구하여라.

Page 39: Baltic Way 팀수학경시대회 1990-2005 pdf 보기

2000년 Baltic Way 39

2000년 Baltic Way

노르웨이, 오슬로

2000년 11월 4일

1. K는 삼각형 ABC의 내부의 점이다.직선 AB에 대해 K와 반대쪽에 있는 점 M과, 직

선 BC에 대해 K와 반대쪽에 있는 점 N을 잡자.

\MAB = \MBA = \NBC = \NCB = \KAC = \KCA

일 때, MBNK가 평행사변형임을 보여라.

2. \A = 90◦ 인 이등변 삼각형 ABC가 주어져 있다. M은 AB의 중점이다. A를 지나고

CM에 수직인 직선이 변 BC와 P에서 만난다. \AMC = \BMP 임을 증명하여라.

3. \A = 90◦ 이고 jABj 6= jACj 인 삼각형 ABC가 주어져 있다. 변 BC, CA, AB 위에

각각 점 D, E, F를 AFDE가 정사각형이 되도록 잡는다. 직선 BC와 직선 FE, 그리

고 삼각형 ABC의 외접원의 점 A에서의 접선이 한 점에서 만남을 증명하여라.

4. \A = 120◦ 인 삼각형 ABC가 주어져 있다. K와 L은 각각 변 AB, AC 위의 점이다.

삼각형 ABC의 바깥쪽에 정삼각형 BKP와 CLQ를 그리자. 다음을 증명하여라.

jPQj ¸p3

2¢ (jABj+ jACj)

5. 삼각형 ABC는jBCj

jABj ¡ jBCj =jABj+ jBCj

jACj 를 만족한다. \A : \C 의 비를 구하여

라.

6. Fredek은 호텔을 경영한다. 그는 n ¸ 3명의 손님이 호텔을 찾았을 때면 언제나, 다른

손님들 중 아는 사람의 수가 같고 둘이 다 알거나 둘다 모르는 손님이 있는 두 명의

손님을 고를 수 있다고 주장하였다. Fredek의 주장이 옳은 n은 어떤 값들인가? (알고

모르는 것은 상호 대칭적인 관계이다.)

7. 각각 ON과 OFF두 가지 상태가 있는, 40£50배열로 된 조정 단추들이 있다.한 단추

를 건드리면 그 단추 자신과 그 단추와 같은 행 혹은 같은 열에 있는 모든 단추의 상

태가 뒤바뀐다. 처음에 모두 OFF로 되어있던 조정 단추들을 계속 건드려 언젠가 모

두 ON이 되게 할 수 있음을 보여라. 그리고, 최소 몇 번 건드리면 되는지 구하여라.

Page 40: Baltic Way 팀수학경시대회 1990-2005 pdf 보기

40

8. 파티에서 열 네 명의 친구가 만났다.그들중 한 명인 Fredek은 빨리 자러 가고 싶었다.

그는 친구 중 10명에게 작별인사를 했고,나머지 3명은 깜박 잊어버리고 침실로 갔다.

잠시 후 그가 파티에 다시 돌아와서, 친구 중 10명에게 작별인사를 했고(앞서와 같은

친구에게 할 필요는 없다) 침실로 갔다. 이 후 Fredek은 몇 번 더 파티에 돌아왔고, 그

때마다 정확히 10명에게 작별인사를 하곤 다시 침실로 갔다. 모든 친구에게 각각 최

소 한 번 이상 작별인사를 하고서야 그는 다시 돌아오지 않았다. 아침에 Fredek은 열

세 명의 친구에게 모두 다른 횟수만큼의 작별인사를 했음을 깨달았다! Fredek이 파티

에 돌아온 횟수는 최소 몇 번인가?

9. 단위 정사각형칸으로 구성된 2k £ 2k 의 체스판 위에서 뛰어다니는 개구리가 있다.

개구리는 한 번 뛰어p1 + k2 의 거리를 이동하고, 그것은 반드시 한 칸의 중앙에서

다른 칸의 중앙까지여야 한다. 체스판의 m개의 칸에 X표를 하고, X표 된 칸으로부터

개구리가 뛰어 바로 옮겨갈 수 있는 모든 칸에 O표를 했다(그 칸에 이미 X표가 되어

있으면 O표로 바꾼다). O표가 모두 n개라 할 때, n ¸ m 임을 보여라.

10. 칠판에 두 개의 양의 정수가 쓰여져있다. 처음에는 그 중 하나는 2000이고 다른 하나

는 2000보다 작았다. 두 수의 산술평균 m이 정수이면, 다음 작업을 할 수 있다: 두 수

중 하나를 지우고 m을 대신 쓴다. 이 작업이 11번 이상 수행될 수 없음을 보여라. 그

리고, 이 작업이 10번 수행되는 예를 들어라.

11. 양의 정수들의 수열 a1; a2; : : : 은 임의의 m, n에 대해 다음을 만족한다:만약 m이 n의

약수이고 m < n 이면, am은 an의 약수이고 am < an 이다. 이 때, a2000의 최소값을

구하여라.

12. 양의 정수 x1; x2; : : : ; xn은 이 중 어느 것도 다른 것의 앞부분이 되지 않는 수들이다.

(예를 들어, 12는 12, 125, 12405의 앞부분이다.) 다음을 증명하여라.

1

x1+1

x2+ ¢ ¢ ¢+ 1

xn< 3

13. a1; a2; : : : ; an은 정수로 이루어진 등차수열이고, i = 1; 2; : : : ; n ¡ 1 에 대해서는 항상

i가 ai의 약수이지만, n은 an의 약수가 아니라고 한다. n이 어떤 소수의 거듭제곱임을

보여라.

14. 양의 약수의 개수에 100을 곱했더니 자기 자신이 되는 양의 정수 n을 모두 찾아라.

15. n은 2나 3으로 나누어지지 않는 양의 정수이다.모든 정수 k에 대해, (k+1)n¡kn¡ 1이 k2 + k + 1 로 나누어짐을 보여라.

Page 41: Baltic Way 팀수학경시대회 1990-2005 pdf 보기

2000년 Baltic Way 41

16. 모든 양의 실수 a, b, c에 대해 다음이 성립함을 증명하여라.pa2 ¡ ab+ b2 +

pb2 ¡ bc+ c2 ¸

pa2 + ac+ c2

17. 다음 연립방정식의 모든 실수해를 구하여라.8>>>>>><>>>>>>:

x + y + z + t = 5

xy + yz + zt + tx = 4

xyz + yzt + ztx + txy = 3

xyzt = ¡1

18. 다음 방정식을 만족하는 모든 양의 실수 x, y를 구하여라.

x+ y +1

x+1

y+ 4 = 2 ¢ (p2x+ 1 +p2y + 1 )

19. t ¸ 1

2는 실수이고, n은 양의 정수이다. 다음을 증명하여라.

t2n ¸ (t¡ 1)2n + (2t¡ 1)n

20. 모든 양의 정수 n에 대해, xn을 다음과 같이 정의한다.

xn =(2n+ 1) ¢ (2n+ 3) ¢ ¢ ¢ ¢ ¢ (4n¡ 1) ¢ (4n+ 1)

2n ¢ (2n+ 2) ¢ ¢ ¢ ¢ ¢ (4n¡ 2) ¢ 4n1

4n< xn ¡

p2 <

2

n임을 증명하여라.

Page 42: Baltic Way 팀수학경시대회 1990-2005 pdf 보기

42

2001년 Baltic Way

독일, 함부르크

2001년 11월 4일

1. 어떤 시험에 8문제가 준비되어 있다. 각 학생들에게 이것들 중 3개가 주어진다. 어느

두 학생도 공통적으로 받는 문제는 기껏해야 1개이다. 학생들은 최대 몇 명이 가능하

겠는가?

2. n ¸ 2 는 양의 정수이다. f1; 2; 3; : : : g의 서로 소이고 공집합이 아닌 n개의 부분집합

을 잘 구성하여, 모든 양의 정수가 서로 다른 부분집합에 있는 최대 n개의 정수들의

합으로 유일하게 표현되게 할 수 있는가?

3. 1, 2, : : : ; 49의 수들이 7£ 7로 배열되어 있고, 각 행과 각 열마다 수의 합이 계산되었

다. 이 14개의 합들 중 어떤 것은 홀수이고 어떤 것은 짝수이다. 홀수인 합을 모두 합

한 것을 A, 짝수인 합을 모두 합한 것을 B라 하자. A = B 가 되도록 수를 배열하는

것이 가능한가?

4. p와 q는 서로 다른 두 소수이다. 다음을 증명하여라.jpq

k+j2pq

k+j3pq

k+ ¢ ¢ ¢+

j (q ¡ 1)pq

k=1

2(p¡ 1)(q ¡ 1)

(bxc는 x를 넘지 않는 가장 큰 정수를 나타낸다.)

5. 한 원 위의 2001개의 주어진 점들을 빨강이나 녹색으로 칠했다. 이제 모든 점의 색을

다음의 방법으로 동시에 다시 결정한다: 만약 한 점 P의 바로 이웃한 두 점이 모두

P와 같은 색이었으면 P의 색을 바꾸지 않고 그대로 두지만, 만약 그렇지 않다면 P는

다른 색깔로 바꿔 칠한다. 처음에 원이 채색된 꼴을 F1이라 하고, 위와 같이 다시 칠

한 꼴을 F2,작업을 계속 반복하여 F3, : : : 라 하자. Fn0 = Fn0+2 가 되는 수 n0 · 1000가 있음을 증명하여라. 이 얘기는 1000을 999로 바꾸어도 역시 성립하는가?

6. 한 원 c 위에 점 A, B, C, D, E가 차례로 있고, AB k EC, AC k ED 이다. 원 c의 점

E에서의 접선이 직선 AB와 점 P에서 만난다. 직선 BD와 EC는 점 Q에서 만난다.

jACj = jPQj 임을 증명하여라.

7. 평행사변형 ABCD가 주어져있다. A를 지나는 원이 선분 AB, AC, AD와 각각 선분

내부의 점 M , K, N에서 만난다. 다음을 증명하여라.

jABj ¢ jAM j+ jADj ¢ jAN j = jAKj ¢ jACj

Page 43: Baltic Way 팀수학경시대회 1990-2005 pdf 보기

2001년 Baltic Way 43

8. ABCD는 볼록사변형이고, N은 BC의 중점이다. 또, \AND = 135◦ 라 하자. 다음을

증명하여라.

jABj+ jCDj+ 1p2¢ jBCj ¸ jADj

9. 마름모 ABCD가 주어져있다. \APD + \BPC = 180◦ 를 만족하며 이 마름모 안을

움직이는 점 P의 자취를 구하여라.

10. 삼각형 ABC에서, \BAC의 이등분선이 변 BC와 점 D에서 만난다. jBDj ¢ jCDj =jADj2 과 \ADB = 45◦ 임을 알 때, 삼각형 ABC의 모든 각을 구하여라.

11. 함수 f는 모든 양의 정수에서 정의되고 실수값을 갖는다. 임의의 정수 a > 1, b > 1

에 대해, d = gcd(a; b) 라 할 때,

f(ab) = f(d) ¢µf³ad

´+ f³ bd

´¶의 식이 성립한다. f(2001)의 모든 가능한 값을 구하여라.

12. a1; a2; : : : ; an은

nXi=1

a3i = 3 과

nXi=1

a5i = 5 를 만족하는 양의 실수들이다.nXi=1

ai >3

2임

을 증명하여라.

13. a0; a1; a2; : : : 는 a0 = 1과 an = a 7n=9 + a n=9 (n = 1; 2; : : : ) 을 만족하는 실수 수열

이다. ak <k

2001!가 되는 양의 정수 k가 존재함을 보여라. (bxc는 x를 넘지 않는 가

장 큰 정수를 나타낸다.)

14. 2n개의 카드가 있다.각각의 카드에 구간 [1; 2]의 실수를 하나씩 적었다. 카드를 두 더

미로 나누는데, 두 더미 각각의 합 s1, s2가n

n+ 1· s1

s2· 1을 만족하도록 할 수 있음

을 증명하여라.

15. a0; a1; a2; : : : 는 i ¢ a2i ¸ (i+1) ¢ ai−1ai+1 (i = 1; 2; : : : ) 을 만족하는 양의 실수들의 수

열이다. 또, x, y는 양의 실수이고, bi = xai + yai−1 (i = 1; 2; : : : ) 이라 하자. 모든 정

수 i ¸ 2 에 대해 부등식 i ¢ b2i > (i+ 1) ¢ bi−1bi+1 이 성립함을 보여라.

16. f는 양의 정수들에서 정의되고 실수값을 가지는 함수로, 다음 조건을 만족한다: 모든

n > 1 에 대해

f(n) = f

µn

p

¶¡ f(p)

인 n의 소인수 p가 존재한다. f(2001) = 1 이라 할 때, f(2002)의 값은 얼마인가?

Page 44: Baltic Way 팀수학경시대회 1990-2005 pdf 보기

44

17. n은 양의 정수이다. 집합 f1; 2; 3; : : : ; 2ng에서 다음의 성질을 갖는 2n−1 + n 개 이상

의 수들을 찾을 수 있음을 증명하여라: 임의의 서로 다른 두 수 x, y에 대해, x+ y 는

x ¢ y 의 약수가 아니다.

18. a는 홀수이다. n 6= m 인 모든 양의 정수 n, m에 대해, a2n

+ 22n

과 a2m

+ 22m

은 서

로 소임을 증명하여라.

19. 360의 양의 약수의 개수와 같은 개수의 양의 약수를 갖는 가장 작은 양의 홀수는 무

엇인가?

20. 정수 순서쌍 (a; b; c; d)로부터, 임의의 정수 n에 대해

(c; d; a; b); (b; a; d; c); (a+ nc; b+ nd; c; d); (a+ nb; b; c+ nd; d)

등의 순서쌍들을 바로 얻을 수 있다고 하자. 이 과정을 몇 번 반복하여 (1; 2; 3; 4)로부

터 (3; 4; 5; 7)을 얻을 수 있겠는가?

Page 45: Baltic Way 팀수학경시대회 1990-2005 pdf 보기

2002년 Baltic Way 45

2002년 Baltic Way

에스토니아, 타르투

2002년 11월 2일

1. 다음 연립방정식을 실수 범위에서 풀어라.

a3 + 3ab2 + 3ac2 ¡ 6abc = 1b3 + 3ba2 + 3bc2 ¡ 6abc = 1c3 + 3ca2 + 3cb2 ¡ 6abc = 1

2. a, b, c, d는 다음을 만족하는 실수들이다.

a+ b+ c+ d = ¡2ab+ ac+ ad+ bc+ bd+ cd = 0

네 수 a, b, c, d 중 적어도 하나는 ¡1 이하임을 증명하여라.

3. 임의의 정수 m;n ¸ 0 에 대해 다음 관계식

am2+n2 = a2m + a2n

을 만족하는 실수 수열 a0 · a1 · a2 · ¢ ¢ ¢ 을 모두 찾아라.

4. n은 양의 정수이다. x1 + x2 + ¢ ¢ ¢+ xn = 1 인 임의의 음아닌 실수 x1; x2; : : : ; xn에 대

해nXi=1

xi(1¡ xi)2 ·µ1¡ 1

n

¶2이 성립함을 증명하여라.

5.pa+

pb =p2 +

p3 을 만족하는 모든 양의 유리수해 (a; b)를 찾아라.

6. 단위 정사각형들로 구성된 m£ n 직사각형 판(m;n ¸ 2)에서 다음과 같은 혼자서 하

는 게임을 진행한다. 우선, `차' 하나를 적당한 칸에 둔다. 매 회의 이동에서 이 `차'는

수직이나 수평 방향으로 임의의 칸을 전진한다. 단, 전에 움직여온 방향에서 시계방

향으로 90◦를 꺾은 방향으로 가야한다는 조건을 추가한다. (즉, 왼쪽으로 이동한 후

에는 위쪽으로, 그 다음에는 오른쪽으로, ... 이런 식으로 움직여야 한다.) `차'가 모든

칸을 꼭 한 번씩 들러 원래의 칸으로 돌아올 수 있는 m과 n은 어떤 값이어야 하는가?

Page 46: Baltic Way 팀수학경시대회 1990-2005 pdf 보기

46

(단, `차'가 어떤 칸에 들렀다는 것은 그 칸 위를 스쳐 지나간 경우는 아니고 거기서 멈

춘 경우만을 고려하기로 한다.)

7. 평면 위에 n개의 볼록사각형을 그려 평면을 몇 개의 영역으로 나눈다(한 영역은 무한

영역이다). 이렇게 나뉘는 영역의 개수의 최대값을 구하여라.

8. P는 어느 세 점도 한 직선 위에 있지 않은 평면 위의 n ¸ 3 개의 점들의 집합이다.

P의 점들을 꼭지점으로 하는

µn¡ 12

¶개의 삼각형들을 골라 집합 T라 하는데, T의

각 삼각형은 T의 다른 어떤 삼각형과도 공유하지 않는 변을 적어도 하나씩 가져야 한

다. 이렇게 T를 고르는 방법의 수가 얼마나 될까?

9. 두 명의 마술사가 다음과 같은 마술을 보인다.마술사 A가 방에서 나가고,마술사 B가

1; 2; : : : ; 100의 번호가 각각 적힌 100장의 카드를 들고 세 명의 관객에게 차례로 각각

한 장씩 고르게 한다.마술사 B는 관객들이 고른 카드를 알고 있고,자기가 한 장을 더

골라 4장의 카드를 만든다. 이 4장의 카드를 관객들이 섞은 후, 마술사 A를 불러 그에

게 준다. 마술사 A는 이 4장의 카드를 살펴본 후, 첫 번째 관객이 고른 카드, 두 번째

관객이 고른 카드, 세 번째 관객이 고른 카드를 모두 알아맞춘다. 이런 마술이 가능함

을 증명하여라.

10. N은 양의 정수이다. 두 사람이 다음과 같은 게임을 한다. 첫 번째 사람이 먼저 25 이

하의 양의 정수들|반드시 서로 다를 필요는 없는|의 목록을 적는데 그 합은 최소

200 이상이다. 두 번째 사람이 여기서 몇 개의 수를 고르는데, 그 합 S가 200¡N ·S · 200 +N 의 범위에 있도록 하면 이긴다. 두 번째 사람이 최선을 다하면 항상 이

길 수 있는 N의 최소값은 얼마인가?

11. n은 양의 정수이다.평면 위에 n개의 점이 있는데,어느 세 점도 한 직선 위에 있지 않

고 각 두 점 사이의 거리가 모두 다르다고 한다. 각 점에서부터 가장 가까운 두 점을

찾아 그 두 점까지 선분을 그어 연결한다. 이 선분들은 중복될 수도 있고, 서로 엇갈

릴 수도 있으며, 어느 한 쪽 점의 입장에서만 가장 가까운 두 점에 해당되고 반대쪽

점의 입장에서는 해당되지 않더라도 선분으로 연결해야 한다. 11개보다 더 많은 선분

이 연결된 점이 존재하지 않음을 증명하여라.

12. 평면 위에 서로 다른 네 점이 주어져 있다. 이 중 어느 점을 X로 택해도

XY = XZ +XW

가 성립하도록 나머지 점들을 Y , Z, W로 이름붙일 수 있다고 한다. 이 네 점은 한 직

선 위에 있음을 증명하여라.

Page 47: Baltic Way 팀수학경시대회 1990-2005 pdf 보기

2002년 Baltic Way 47

13. 4ABC는 \BAC > \BCA 를 만족하는 삼각형이고, D는 AB = BD 를 만족하는 변

AC 위의 점이라 하자. 또, F는 4ABC의 외접원 위에 있는 점으로, 직선 FD가 변

BC에 수직이고 두 점 F , B는 직선 AC에 대해 반대편 영역에 놓인다고 하자. 직선

FB가 변 AC에 수직임을 증명하여라.

14. 4ABC에서 세 변 AC, AB, BC 위에 각각 점 L, M , N이 있는데, BL은 \ABC의

이등분선이고 세 선분 AN , BL, CM이 한 점에서 만난다. \ALB = \MNB 이면

\LNM = 90◦ 임을 증명하여라.

15. 한 정육면체의 표면에 거미 한 마리와 파리 한 마리가 앉아있다. 파리는 거미가 이 정

육면체의 표면을 따라 자신에게 다가올 수 있는 최단거리를 가장 멀게 하고 싶다. 정

육면체의 중심에 대해 대칭이 되는 반대편에 앉는 게 언제나 최선이 되는가?

16. 다음을 만족하는 음이 아닌 정수 m을 모두 찾아라: am = (22m+1)2 + 1 을 나누는 서

로 다른 소수가 두 개 이하이다.

17. 수열

µ2002

2002

¶;

µ2003

2002

¶;

µ2004

2002

¶; : : : 의 각 항을 2002로 나눈 나머지는 주기적으로 반

복됨을 보여라.

18. 다음을 만족하는 정수 n > 1을 모두 찾아라: n6¡1의 임의의 소인수는 (n3¡1)(n2¡1)의 약수이다.

19. n은 양의 정수이다. 방정식

x+ y +1

x+1

y= 3n

은 양의 유리해를 가지지 않음을 증명하여라.

20. 모든 항이 ab꼴(a와 b는 양의 정수, b ¸ 2)이고 공차가 0이 아닌 무한등차수열이 존재

하는가?

Page 48: Baltic Way 팀수학경시대회 1990-2005 pdf 보기

48

2003년 Baltic Way

라트비아, 리가

2003년 11월 2일

1. Q+를 양의 유리수 전체의 집합이라고 하자. x 2 Q+에 속하는 모든 x에 대해서 다음

을 만족하는 함수 f : Q+ ! Q+ 를 모두 구하여라.

(1) : f( 1x) = f(x)

(2) : (1 + 1x )f(x) = f(x+ 1)

2. x3 + px+ q = 0 의 모든 실수해가 부등식 4qx · p2 을 만족함을 보여라

3. x, y, z는 xyz = 1 을 만족하는 양의 실수이다. 다음이 성립함을 보여라.

(1 + x)(1 + y)(1 + z) ¸ 2µ1 + 3

ry

x+ 3

rz

y+ 3

rx

z

4. a, b, c를 양의 실수라 하자. 다음이 성립함을 보여라.

2a

a2 + bc+

2b

b2 + ca+

2c

c2 + ab· a

bc+

b

ca+

c

ab

5. 수열 (an)은 다음과 같이 정의된다: a1 =p2, a2 = 2, an+1 = ana

2n−1 (n ¸ 2). 1보다

작지 않은 모든 n에 대해서 다음이 성립함을 보여라.

(1 + a1)(1 + a2) ¢ ¢ ¢ (1 + an) · (2 +p2 )a1a2 ¢ ¢ ¢ an

6. n ¸ 2과 d ¸ 1는 d j n을만족하는 정수들이고, x1; x2; : : : ; xn 은 x1+x2+¢ ¢ ¢+xn = 0

을만족하는실수들이다. xi1+xi2+¢ ¢ ¢+xid ¸ 0을만족하는 1 · i1 < i2 < ¢ ¢ ¢ < id · n

인 d개의 첨자를 고르는 방법이 최소한¡n−1d−1¢가지가 있음을 보여라.

7. X는 다음 성질을 갖는 f1; 2; : : : ; 10000g의 부분집합이다: `a; b 2 X 이고 a 6= b 이면

a ¢ b 62 X 이 된다.' X의 원소의 개수는 최대 몇 개인가?

8. 테이블 위에 2003개의 캔디가 놓여있다. 두 명이 번갈아가며 사탕을 먹는데, 자기 차

례에 하나의 캔디를 먹든가 테이블 위에 있는 캔디의 절반(캔디가 홀수 개라면 하나

를 뺀 절반)을 먹는다. 각 차례마다 최소한 한 개의 캔디는 먹어야한다. 가장 마지막

캔디를 먹는 사람이 이 게임에서 지게 된다. 누구|첫 번째, 혹은 두 번째 사람|에게

필승의 전략이 있는가?

Page 49: Baltic Way 팀수학경시대회 1990-2005 pdf 보기

2003년 Baltic Way 49

9. n은 144 이하의 양의 정수이다. `n이 a보다 작은가?'와 같은 식으로 10번의 질문을 할

수 있다. 답변은 뜸을 들이고 나온다: 즉 i번째 질문의 답은 i+ 1번째 질문을 한 후에

들을 수 있다(i = 1; 2; : : : ; 9). 10번째 질문의 답은 물어본 즉시 나온다. n을 알아낼 수

있는 질문 방법을 찾아보아라.

10. 평면 위에서 각 좌표가 모두 정수인 점을 `격자점(lattice point)'이라고 한다. 점µx1 + x2 + x3 + x4

4;y1 + y2 + y3 + y4

4

¶을 네 점 (xi; yi) (i = 1; 2; 3; 4) 의 `중심(centroid)'이라고 한다. n은 다음 성질을 만족

하는 가장 큰 자연수이다: `임의로 택한 4개의 점의 중심이 항상 격자점이 아닌 점이

되도록 평면 위에 n개의 서로 다른 격자점을 잡을 수 있다.' n = 12 임을 증명하여라.

11. 평면 위에서 1000개의 점을 골라, 최소 6000개의 거리(두 점 사이의 거리)가 모두 같

도록 할 수 있을까?

12. 정사각형 ABCD가 있다. M은 변 BC 내부(선분의 끝점이 아닌 선분의 위)에, N은

변 CD 내부에 있고, \MAN = 45◦이다. AMN의 외심이 변 AC 위에 있음을 증명하

여라.

13. ABCD는 BC = 2 ¢AB 인 직사각형이다. BC의 중점을 E라 하고 P를 변 AD 내부의

임의의 점이라 하자. A에서 변 BP 위에, D에서 변 CP 위에 내린 수선의 발을 각각

F와 G라 한다. 이 때, 점 E, F , P , G가 모두 한 원 위에 있음을 보여라.

14. 임의로주어진삼각형 ABC의바깥에정삼각형 AMB, BNC, CKA를그리자.MN의

중점에서 AC 위로 수선을 내리고, 마찬가지로 NK와 KM의 중점에서 각각 AB와

BC 위로 수선을 내린다. 이 세 개의 수선이 모두 한 점에서 만남을 증명하여라.

15. 원에 내접하는 사각형의 대각선 AC와 BD의 교점을 P라 하자. P를 통과하는 원이

변 CD의 중점 M에서 그 변과 접하고, 대각선 BD, AC와 각각 Q, R에서 만난다.

BS = DQ 가 되도록 BD 위에 점 S를 잡자. S를 지나 AB에 평행한 선이 AC와 T에

서 만난다. AT = RC 임을 증명하여라.

16. a ¡ b는 소수, ab는 완전제곱수(어떤 정수의 제곱이 되는 수)가 되는 양의 정수쌍

(a; b)를 모두 구하여라.

17. 양의 정수 n의 모든 (양의) 약수들이 오름차순으로 배열에 저장되어있다. Mary는 임

의로 선택된 약수 d > 1 에 대해 그것이 소수인지를 판정하는 프로그램을 만들어야

한다. n이 d보다 크지 않은 k개의 약수를 갖는다고 하자. Mary는 n의 약수들 중 처음

Page 50: Baltic Way 팀수학경시대회 1990-2005 pdf 보기

50

dk=2e개에 대해서만 d의 약수가 되는지 확인하면 충분하다고 주장하였다: \만약 그

들 중에서 1보다 큰 d의 약수가 발견된다면 d는 합성수이고, 그렇지 않으면 d는 소수

이다." Mary의 주장이 옳은가?

18. 모든 정수를 파랑, 녹색, 빨강, 노랑 중 하나의 색깔로 칠한다. 만약 a, b, c, d가 모두

0이 아니고 같은 색깔이면 항상 3a¡2b 6= 2c¡3d 이 되도록 색칠하는 것이 가능할까?

19. 양의 정수 a, b가 있다. 만약 a3 + b3 이 어떤 정수의 제곱이라면, a + b 는 서로 다른

두 개의 소수의 곱으로 표현될 수 없음을 보여라.

20. n은 자신을 제외한 모든 (양의) 약수들의 합과 이 약수들의 개수를 더한 값이 n 자신

과 같은 양의 정수이다. n = 2m2꼴임을 증명하여라.

Page 51: Baltic Way 팀수학경시대회 1990-2005 pdf 보기

2004년 Baltic Way 51

2004년 Baltic Way

리투아니아, 빌뉴스

2004년 11월 7일

1. 임의의 첨자 n = 1; 2; : : : 에 대해 다음 조건

(1) an + a2n ¸ 3n(2) an+1 + n · 2pan ¢ (n+ 1)

을 만족하는 음이 아닌 실수들의 수열 a1; a2; a3; : : : 가 주어져 있다.

(a) 임의의 n 2 N 에 대해 부등식 an ¸ n 이 만족됨을 증명하여라.

(b) 이런 수열의 예를 하나 찾아라.

2. P (x)는 음이 아닌 계수들로 된 다항식이다. 부등식 P ( 1x )P (x) ¸ 1 이 x = 1 에서 성

립하면 모든 양수 x에서도 성립함을 증명하여라.

3. p; q; r은 양의 실수들이고 n 2 N 이다. pqr = 1 일 때, 다음을 보여라.

1

pn + qn + 1+

1

qn + rn + 1+

1

rn + pn + 1· 1

4. x1; x2; : : : ; xn은 산술평균이 X인 실수들이다.다음 부분집합들 각각의 산술평균이 모

두 X를 넘지 않도록 하는 양의 정수 K가 존재함을 증명하여라: fx1; x2; : : : ; xKg,fx2; x3; : : : ; xKg, fx3; : : : ; xKg, : : : ; fxK−1; xKg, fxKg

5. 정수 k에 대해 정의되는 다음 함수의 치역을 구하여라.

f(k) = (k)3 + (2k)5 + (3k)7 ¡ 6k

단, (k)2n+1은 2n+ 1의 배수 중에서 k에 가장 가까운 수를 나타낸다.

6. 정육면체의 6개의 면에 양의 정수가 하나씩 쓰여져 있다. 이 정육면체의 각각의 꼭지

점에서는 그 점에 접하는 세 면의 수들의 곱을 계산한다.이 곱들의 합이 1001이다.각

면에 쓰인 6개의 정수의 합은 얼마인가?

7. 양의 정수들의 집합 X는 최소한 2개의 원소를 갖고, n > m 인 임의의 m;n 2 X 에

대해 n = mk2 을 만족하게 하는 k 2 X 가 항상 존재한다. 이런 집합 X를 모두 찾아

라.

Page 52: Baltic Way 팀수학경시대회 1990-2005 pdf 보기

52

8. f(x)는 상수함수가 아닌 정수계수 다항식이다. f(n)이 2004개 이상의 소인수를 갖도

록 하는 정수 n이 존재함을 증명하여라.

9. n¡ 1개의 자연수들의 집합 S가 주어져 있다(n ¸ 3). 이 집합의 어떤 두 원소의 차는

n으로 나누어떨어지지 않는다. S의 공집합이 아닌 부분집합을 잘 골라 그 원소들의

합이 n의 배수가 되게 할 수 있음을 증명하여라.

10. 임의의 n 2 N 에 대해 jpn+1¡ 2pnj = 1인 소수들의 무한 수열 p1; p2; : : : ; pn; pn+1; : : :

이 존재하는가?

11. 각 칸에 +1혹은 ¡1이 쓰여져 있는 m£n 크기의 표가 주어져 있다.맨처음에는 ¡1이하나뿐이고 나머지는 모두 +1이다. 한 번의 움직임은, ¡1인 칸을 하나 택해 그 ¡1을0으로 바꾸고 그 칸과 이웃한 칸의 숫자들에 동시에 ¡1을 곱한다(두 칸이 한 변을 공

유하고 있으면 서로 이웃한다고 한다). 처음 ¡1이 어느 칸에 있든 상관없이 항상 이

런 움직임만을 반복하여 모든 칸을 0으로 만들 수 있는 (m;n)을 모두 찾아라.

12. 서로 다른 2n개의 수들이 일렬로 늘어놓여 있다. 한 번의 움직임에서 우리는 두 수의

위치를 바꾸거나 혹은 세 수를 순환적으로 바꿀 수 있다(세 수 a; b; c를 골라 a를 b의

자리로, b를 c의 자리로, c를 a의 자리로 옮길 수 있다). 항상 이 수들을 증가하는 순서

로 배열할 수 있기에 충분한 회수는 최소 몇 번인가?

13. 유럽 연합의 25개 회원국이 다음 규칙에 따라 한 위원회를 구성하였다: (1) 위원회는

매일 회의를 가져야 한다. (2) 각 회의마다 최소한 한 회원국이 참석해야 한다. (3) 임

의의 서로 다른 두 회의에는, 참석하는 회원국의 집합이 달라야 한다. (4) n번째 회의

에는, 각각의 k < n 에 대해, k번째 회의에 참석했던 회원국 중 적어도 한 나라가 참

석해야 한다. 이 위원회는 회의를 최장 며칠 동안 가질 수 있는가?

14. 땅콩 한 더미란 4개 이상의 땅콩을 말한다. 두 명이 n ¸ 4개의 땅콩 한 더미로 시작하

여 다음과 같은 게임을 한다. 각 차례에서 한 명이 현재 놓인 땅콩 더미들 중에서 한

더미를 택해 그것을 두 집합으로 나눈다(한 쪽이 공집합이 되면 안 되고, 이 두 집합

은 꼭 더미가 될 필요는 없으며, 땅콩 개수는 임의로 나누면 된다). 둘이 번갈아 차례

를 갖다가, 더 이상 자기 차례를 실행할 수 없는 사람이 지게 된다. 먼저 하는 사람이

필승의 전략을 가질 수 있는 n은 어떤 값들인가?

15. 한 원을 13개의 호로 쪼개어 순서대로 1부터 13까지의 번호를 매겼다.다섯 마리의 벼

룩 A, B, C, D, E가 각각 호 1, 2, 3, 4, 5에 앉아있다. 벼룩은 원주의 양쪽 방향으로

다섯 위치 떨어져있는 비어있는 호로 뛸 수 있다. 한 번에 한 마리의 벼룩만 뛸 수 있

Page 53: Baltic Way 팀수학경시대회 1990-2005 pdf 보기

2004년 Baltic Way 53

고, 두 마리의 벼룩이 같은 호에 있을 수는 없다. 몇 번의 뜀 후에 벼룩들은 다시 처음

의 1, 2, 3, 4, 5의 호로 돌아왔는데, 처음과는 앉은 순서가 바뀌었다고 한다. 어떤 순

서가 가능한가?

16. 주어진 원의 한 할선과 한 접선이 원밖의 점 P에서 만난다. 할선은 원과 두 점 A와

B에서 만나고, 접선은 점 C에서 원과 접한다. 그리고, P를 지나는 지름에 대해 이 세

점은 같은 쪽에 있다. C에서 이 지름에 내린 수선의 발을 Q라 하자. QC가 \AQB를

이등분함을 증명하여라.

17. 두 변의 길이가 3과 4인 직사각형을 생각하자. 각 변 위에서 꼭지점이 아닌 한 점씩을

임의로 택해, 이 네 점으로 이어 만든 사각형의 네 변의 길이를 각각 x, y, z, u라 하

자. 25 · x2 + y2 + z2 + u2 · 50 임을 증명하여라.

18. 삼각형 ABC의 꼭지점 A에서 나온 광선이 변 BC와 X에서 만나고 4ABC의 외접원

과 Y에서 만난다. 1AX +

1XY ¸ 4

BC 임을 증명하여라.

19. 주어진 삼각형 ABC의 변 BC의 중점을 D라 하자. \BAM = \DAC 를 만족하는 변

BC 위의 점을 M이라 하자. 삼각형 CAM의 외접원이 변 AB와 만나는 또다른 점을

L이라 하자. 또, 삼각형 BAM의 외접원이 변 AC와 만나는 또다른 점을 K라 하자.

KL k BC 임을 증명하여라.

20. 공통의 양끝점 A, B를 갖는 세 원호 w1; w2; w3이 직선 AB의 한 쪽에 놓여 있다. w2는

w1과 w3 사이에 있다. B에서 나온 두 줄기의 광선이 이 호들과 각각 M1;M2;M3와

K1;K2;K3에서 만난다. M1M2

M2M3= K1K2

K2K3임을 증명하여라.

Page 54: Baltic Way 팀수학경시대회 1990-2005 pdf 보기

54

2005년 Baltic Way

스웨덴, 스톡홀름

2005년 11월 5일

1. a0은 양의 정수이다. 수열 fangn≥0 을 다음과 같이 정의하자: 0 · ci · 9 인 정수 ci들

에 대해

an =

jXi=0

ci10i

로 나타날 때,

an+1 = c20050 + c20051 + ¢ ¢ ¢+ c2005j

이다. 즉, an+1은 an을 십진법으로 전개했을 때의 각 자리수의 2005제곱의 합이다.이

수열의 모든 항이 서로 다르도록 a0을 정하는 것이 가능한가?

2. 0 · ®; ¯; ° < 90◦ 이고 sin®+ sin¯ + sin ° = 1 을 만족하는 세 각 ®, ¯, °가 있다. 다

음을 증명하여라.

tan2 ®+ tan2 ¯ + tan2 ° ¸ 3

8

3. 수열 fakgk≥1 은 다음과 같이 정의된다: a1 = 1, a2 =12 ,

ak+2 = ak +1

2ak+1 +

1

4akak+1(k ¸ 1)

다음을 증명하여라.

1

a1a3+

1

a2a4+

1

a3a5+ ¢ ¢ ¢+ 1

a98a100< 4

4. 모든 실수 x에 대해 P (x2 + 1) = P (x)2 +1 을 만족하는 실계수 다항식 P (x)를 3개만

찾아라.

5. a, b, c는 abc = 1 인 양의 실수들이다. 다음을 증명하여라.

a

a2 + 2+

b

b2 + 2+

c

c2 + 2· 1

6. K와 N은 1 · K · N 인 양의 정수들이다. 서로 다른 N장의 카드 묶음을 섞는데, 묶

음의 제일 위에 있는 K장의 카드를 들어낸 후 그 K장의 순서를 뒤집어서 원래 묶음

의 제일 아래로 옮겨넣는 시행을 반복하여 섞는다. 이 카드 묶음은 4 ¢N2=K2 번 이하

의 시행으로 다시 맨처음의 순서로 돌아옴을 증명하여라.

Page 55: Baltic Way 팀수학경시대회 1990-2005 pdf 보기

2005년 Baltic Way 55

7. 각칸에 0또는 1이적혀있는, n행 6열의숫자배열판이있다.단, n > 2이다.이배열판

의 각 행은 서로 다르다.또,어떤 두 행이 (x1; x2; x3; x4; x5; x6)과 (y1; y2; y3; y4; y5; y6)

이라면 (x1y1; x2y2; x3y3; x4y4; x5y5; x6y6)인 행도 항상 존재한다. 절반 이상이 0으로

되어 있는 열이 있음을 증명하여라.

8. 25 £ 25 개의 단위정사각형칸으로 된 격자판이 있다. 이 격자판의 격자선을 따라 적

당한 크기의 정사각형을 빨간 펜으로 덧그리자.모든 격자선들이 빨간 색으로 덧칠되

기 위해 그려야 하는 빨간 정사각형은 최소 몇 개인가?

9. 어떤 직사각형을 200£ 3 개의 단위정사각형칸으로 구분하였다. 이 직사각형을 1£ 2크기의 작은 직사각형으로 쪼개는 방법의 수는 3으로 나누어떨어짐을 증명하여라.

10. m = 30030 = 2 ¢ 3 ¢ 5 ¢ 7 ¢ 11 ¢ 13 이고, M은 m의 양의 약수들 중 딱 2개의 소인수를 갖

는 것들의 집합이다. 다음 성질을 갖는 최소의 정수 n을 구하여라: M에서 n개의 수

를 어떻게 택해도 그 중 abc = m 이 되는 세 수 a, b, c가 존재한다.

11. 삼각형 ABC에서 변 BC와 AC에서 각각 점 D와 E를 잡아 BD = AE 가 되도록 하

자. 두 삼각형 ADC와 BEC의 외심을 잇는 직선이 두 직선 AC, BC와 각각 K, L에

서 만난다. KC = LC 임을 증명하여라.

12. ABCD는 BC = AD 인 볼록사각형이다. AB와 CD의 중점을 각각 M과 N이라 하

자. 두 직선 AD와 BC는 직선 MN과 각각 P와 Q에서 만난다. CQ = DP 임을 증명

하여라.

13. 다음 크기의 직사각형을 완전히 덮기 위해 필요한 반지름p2의 원판은 최소 몇 개인

가?

(a) 6£ 3 크기의 직사각형

(b) 5£ 3 크기의 직사각형

14. 삼각형 ABC의 중선들이 M에서 만난다. D와 E는 DC = CE = AB 를 만족하는

직선 BC 위의 서로 다른 두 점이고, P와 Q는 각각 선분 BD와 BE 위의 점으로,

2BP = PD 와 2BQ = QE 를 만족한다. \PMQ 의 크기를 구하여라.

15. 두 직선 e와 f가 점 H에서 수직으로 만난다. A와 B는 e 위의 점, C와 D는 f 위의 점

이고, A, B, C, D, H는 모두 서로 다른 점이다. 각각 점 B와 D를 지나고 AC에 수직

인 두 직선 b와 d, 그리고 각각 점 A와 C를 지나고 BD에 수직인 두 직선 a와 c를 생

각하자. a와 b의 교점을 X, c와 d의 교점을 Y라 하자. XY가 H를 지남을 증명하여라.

Page 56: Baltic Way 팀수학경시대회 1990-2005 pdf 보기

56

16. p는 소수이고 n은 양의 정수, 그리고 q는 (n+ 1)p ¡ np 의 한 양의 약수이다. q ¡ 1 이

p로 나누어떨어짐을 보여라.

17. 수열 fxngn≥0 이 다음과 같이 정의된다: x0 = a, x1 = 2 이고, n > 1 에 대해 xn =

2xn−1xn−2¡ xn−1 ¡ xn−2+1 이다. 모든 n ¸ 1 에 대해 2x3n ¡ 1 이 항상 완전제곱수

가 되는 정수 a를 모두 찾아라.

18. x와 y는 양의 정수이고 z = 4xy=(x+ y) 는 홀수라 하자. z는 4n¡ 1꼴의 약수를 가짐

을 증명하여라. 단, n은 양의 정수이다.

19. 2005개의 서로 다른 양의 완전제곱수들의 합이 다시 완전제곱수가 되는 경우를 찾을

수 있는가?

20. n = p1p2 ¢ ¢ ¢ pk 가 (p1 + 1)(p2 + 1) ¢ ¢ ¢ (pk + 1) 을 나누는 양의 정수 n을 모두 구하여

라. 단, p1p2 ¢ ¢ ¢ pk는 n을 소수들의 곱으로 나타낸 것이고, 이 소수들은 서로 다를 필

요는 없다.

Page 57: Baltic Way 팀수학경시대회 1990-2005 pdf 보기

제 II 부

풀이편

Page 58: Baltic Way 팀수학경시대회 1990-2005 pdf 보기
Page 59: Baltic Way 팀수학경시대회 1990-2005 pdf 보기

1990년 Baltic Way 풀이 59

1990년 Baltic Way 풀이

Baltic 1990-1Baltic 1990-1

한 원의 원주 위에 정수 1; 2; : : : ; n이 적당한 순서로 쓰여져 있다. 이웃한 숫자들의 차들의 합의 최

소값은 얼마인가?

풀이 1과 n의 위치에 주목해보자. 1부터 n까지 가는 동안 숫자들이 모두 오름차순으로 되어

있어야만 이웃한 숫자들의 차들의 합이 최소 n¡ 1이 된다.

1부터 n까지 가는 길이 원주를 따라 왼쪽과 오른쪽의 두 방향이 있으므로 이웃한 숫자들의 차

들의 합의 최소값은 2(n¡ 1)이다. }

Baltic 1990-2Baltic 1990-2

네모난 종이의 네모칸들에 다음과 같이 수를 나열하였다:

1 2 3 4 5 ¢ ¢ ¢ m

1

2

3

4

...

n

1 2

3

4

5

6

7

8

9

10

11

12

13

14

임의의 양의 정수 m;n에 대해 (m;n)의 좌표의 칸에 쓰여진 수를 p(m;n)이라 할 때,두 변수 m;n으

로 된 다항식 p(m;n)을 찾아라.

풀이 m+ n = k + 1 인 위치의 칸들을 k번째 줄에 있다고 말하기로 하자. k번째 줄에는 k개

의 칸이 있고 k번째 줄을 다 채워야 그 다음 줄로 넘어가 수를 나열하고 있으므로, k번째 줄의

맨마지막 칸은

p(1; k) = (1 + ¢ ¢ ¢+ k) =k(k + 1)

2

Page 60: Baltic Way 팀수학경시대회 1990-2005 pdf 보기

60

이 된다. (m;n)은 m+n¡ 1번째 줄에 있고, 그 앞줄(m+n¡ 2번째 줄)의 맨마지막 칸에서 n을

더한 값이 되므로

p(m;n) = p(1;m+ n¡ 2) + n =(m+ n¡ 2)(m+ n¡ 1)

2+ n

=m2 + 2mn+ n2 ¡ 3m¡ n+ 2

2

이 된다. }

Baltic 1990-3Baltic 1990-3

a0 > 0, c > 0 이고,

an+1 =an + c

1¡ anc; n = 0; 1; : : :

라 하자. 처음 1990개의 항 a0; a1; a2; : : : ; a1989이 모두 양수이고 a1990 < 0 이 되는 것이 가능하겠는

가?

풀이 (O±cial Solution)

a0 = tan®, c = tan¯ 라 두자. an = tan(®+ n¯) 라 가정하면, tangent 합차공식에 의해

an+1 =an + c

1¡ anc=tan(®+ n¯) + tan ¯

1¡ tan(®+ n¯) tan¯= tan(®+ (n+ 1)¯)

도 성립한다. 즉, 수학적 귀납법에 의해

an = tan(®+ n¯) (n ¸ 0)

이다. 0± < ®; ¯ < 90± 의 각으로,

tan®; tan(®+ ¯); : : : ; tan(®+ 1989¯) > 0 이고 tan(®+ 1990¯) < 0

인 두 각을 잡는 것은 간단한 일이다. }

Baltic 1990-4Baltic 1990-4

임의의 실수 a1; a2; : : : ; an에 대해, 다음 부등식이 성립함을 증명하여라.

nXi;j=1

aiaji+ j ¡ 1 ¸ 0

증명 (O±cial Solution)

f(x) = a1 + a2x+ ¢ ¢ ¢+ anxn¡1 =

nXi=1

aixi¡1

이라 두자. Z1

0

f(x)2dx =

Z1

0

nXi;j=1

aiajxi+j¡2dx =

nXi;j=1

Z1

0

aiajxi+j¡2dx = (준식)

좌변이 0 이상이므로 준식도 0 이상이다. ¤

Page 61: Baltic Way 팀수학경시대회 1990-2005 pdf 보기

1990년 Baltic Way 풀이 61

Baltic 1990-5Baltic 1990-5

¤는 연산을 나타내고, 각각의 실수쌍 (a; b)에 대해 실수값 a¤ b 가 정의된다(예: a ¤ b = a+ b2¡ 17).연산 ¤이 교환법칙이 성립하거나 결합법칙이 성립하면 (변수들의 가능한 모든 값에 대해서) 참이

되고, 그렇지 않으면 거짓이 될 수 있는 등식을 하나 찾아라.

풀이 (경기 과천중 2학년 함태준, 수정됨)

다음이 그런 등식의 예가 된다.

(a ¤ b) ¤ a = a ¤ (b ¤ a)

교환법칙이 성립하면

(a ¤ b) ¤ a = a ¤ (a ¤ b) = a ¤ (b ¤ a)

로 항상 성립하고, 결합법칙이 성립해도 바로 성립한다는 것을 알 수 있다. 교환법칙과 결합법

칙이 성립하지 않을 때는 성립하지 않을 때가 있는데, 예를 들어 a ¤ b = a+ 2b 로 하면

(a ¤ b) ¤ a = (a+ 2b) ¤ a = a+ 2b+ 2a = 3a+ 2b

a ¤ (b ¤ a) = a ¤ (b+ 2a) = a+ 2(b+ 2a) = 5a+ 2b

로 두 식의 값이 다르게 된다. }

Baltic 1990-6Baltic 1990-6

ABCD는 jADj = jBCj, \A+\B = 120± 인 사각형이고, P는 사각형의 외부에 있고 DPC가 정삼

각형이 되게 하는 점으로 직선 DC에 대해 A와 반대편 영역에 있다. APB도 정삼각형임을 보여라.

증명 (대전 어은중 3학년 김두리)

먼저 4ADP와 4BCP가 합동임을 확인하자.

\ADC + \BCD = 360± ¡ (\A+ \B) = 240±, \CDP + \DCP = 120± 이므로 오각형 안쪽

으로의 각으로는 \ADP + \BCP = 360± 이 된다. 두 삼각형 ADP와 BCP의 내각으로는 두

각 중 하나가 오각형의 바깥쪽 각이 되어 \ADP = \BCP 가 된다. AD = BC, DP = CP 임

은 쉬우므로 이제

4ADP ´ 4BCP

임이 확인되었다. 이로부터, AP = BP 이고 \APB = \DPC = 60± 가 된다. 따라서, APB는

정삼각형이다. ¤

Page 62: Baltic Way 팀수학경시대회 1990-2005 pdf 보기

62

Baltic 1990-7Baltic 1990-7

볼록 오각형의 각 변의 중점이 이 변의 양끝점을 제외한 남은 세 꼭지점으로 이루어진 삼각형의 중

선들의 교차점과 선분으로 각각 연결되어 있다. 이러한 다섯 개의 선분이 모두 한 점에서 만남을 증

명하여라.

증명 다섯 개의 선분은 모두 주어진 볼록오각형의 다섯 꼭지점의 좌표평균점을 지난다. 즉,

이 오각형의 임의의 한 변을 AB라 하고, 나머지 세 점에 의한 삼각형을 CDE라 하면, AB의

중점과 CDE의 무게중심은 각각 좌표 혹은 벡터로

M =A+B

2; N =

C +D + E

3

이고, 선분 MN을 3 : 2 으로 내분하는 점은

2M + 3N

5=

A+B + C +D + E

5= G

와 같이 주어진 볼록 오각형의 꼭지점들의 좌표평균점이다. ¤

Baltic 1990-8Baltic 1990-8

P는 삼각형 ABC의 외접원의 원주 위에 있는 점이다. P에서 직선 AB, BC, CA 위로 내린 수선의

발이 모두 한 직선(심슨 직선이라 한다) 위에 있음은 알려져 있다. 지름을 이루는 대칭점 P1, P2의

심슨 직선은 서로 수직임을 증명하여라.

증명 (대전 전민중 2학년 김충은/정명진)

(1) 원주 위의 임의의 점 R에 대해 R에서 BC에 내린 수선이 다시 원과 만나는 점을 R0이라

하면, R에 대한 심슨 직선은 AR0과 평행함을 먼저 확인하자.

R에서 변 BC, CA, AB 위로 내린 수선의 발을 각각 D, E, F라 하면 ¤REDC는 원에 내

접하는 사각형이므로,

\DEC = \DRC = \R0RC = \R0AC

따라서, 심슨 직선 DEF와 AR0이 평행함을 알 수 있다.

Page 63: Baltic Way 팀수학경시대회 1990-2005 pdf 보기

1990년 Baltic Way 풀이 63

(2) 원주 위의 임의의 두 점을 P , Q라 하면, 두 점에 대한 심슨 직선이 만나는 각은 현 PQ의

원주각과 같음을 확인하자. (1)에 의해 두 심슨 직선이 이루는 각은 AP 0과 AQ0이 이루는

각, 즉 P 0Q0의 원주각과 같다. PP 0과 QQ0은 평행하므로 PQ = P 0Q0, 따라서 두 심슨 직

선이 이루는 각은 PQ의 원주각과 같다.

(1), (2)에 의해, P , Q가 원의 지름을 이루는 대칭점이면 원주각이 90±이므로 심슨 직선은 서로

수직이 된다. ¤

별증 (대전과학고 1학년 김진현, 조금 수정)

그림에서

\L3L2B = \L3P2B = 90± ¡ \P2BA = \P1BH2 = \P1H3H2

즉, 두 심슨 직선 L2L3, H2H3은 서로 수직인 두 직선 BC, PH3으로부터 각각 같은 크기의 각

만큼 기울어져있으므로 역시 서로 수직이다. ¤

Baltic 1990-9Baltic 1990-9

두 개의 똑같은 삼각형이 타원에 내접한다. 그 둘은 반드시 타원의 축이나 중점에 대해 대칭인가?

풀이 합동인 타원 둘을 비스듬히 겹쳐 4개의 교점이 생기도록 하자.

이 4개의 교점 중에서 3개를 이어 삼각형을 만들자. 그럼 이 삼각형은 하나의 타원에서 타원의

축이나 중점에 대칭이 아니도록 내접한 두 개의 합동인 삼각형이 된다. 따라서, 문제의 명제는

거짓이다. }

Page 64: Baltic Way 팀수학경시대회 1990-2005 pdf 보기

64

Baltic 1990-10Baltic 1990-10

직선 t 위에 단위 길이의 선분 AB를 잡자. 이제 이 선분을 t에 평행하도록 유지하면서 평면 위에서

움직이는데, 점 A, B의 자취는 서로 만나지 않으며 마지막에는 t로 돌아온다. A는 처음 위치로부터

얼마나 멀리까지 옮겨질 수 있을까?

풀이 다음의 그림과 같이 해보자.

처음에 가고자 하는 만큼 미리 d를 얼마든지 작게 잡을 수 있으므로 A는 얼마든지 멀리 옮겨질

수 있다. }

주 A를 직전에 B가 있던 바로 아래의 위치까지 옮기는 것을 한 단계라고 할 때, 매 단계마

다 d를 12씩 줄이며 옮기는 것을 생각한다면 아예 한 번 출발하여 끝없이 무한히 멀리 옮겨가는

것도 가능하다.

Baltic 1990-11Baltic 1990-11

정수 계수의 다항식의 정수해의 절대값은 계수의 절대값의 최대값을 넘지 않음을 보여라.

증명 (대전 대덕중 2학년 고기혁, 수정됨)

영다항식이 아닌 다항식 P (x) = anxn + ¢ ¢ ¢+ a1x+ a0 만을 생각해야 문제가 성립한다. 0이 아

닌 계수 중에서 가장 낮은 차수의 것을 ak라 하자. 즉, P (x) = anxn + ¢ ¢ ¢ + akx

k. 그리고, 이

다항식의 임의의 정수해를 m이라 하자. m = 0 이면 문제가 당연히 성립하므로, m 6= 0 일 때

만 보자.

anmn + ¢ ¢ ¢+ akm

k = 0

에서 mk으로 나누고 ak를 제외한 모든 항을 이항하면

ak = ¡(anmn¡k + ¢ ¢ ¢+ ak+1m)

우변은 m의 배수이므로 m j ak. ak 6= 0 이므로 이로부터 jmj · jakj 이다. 따라서, 문제가 성립

한다. ¤

증명 (한국과학영재학교 1학년 김대식)

임의의 정수 계수 다항식 p(x) = a0xn + ¢ ¢ ¢+ an¡1x+ an 의 한 정수근을 ®라 하고, 귀류법으

로 j®j > ja0j; : : : ; janj 이라고 하자.

p(®) = ®(a0®n¡1 + ¢ ¢ ¢+ an¡1| {z }

p1(®)

) + an = 0

Page 65: Baltic Way 팀수학경시대회 1990-2005 pdf 보기

1990년 Baltic Way 풀이 65

에서 p1(®) 6= 0 이면 j®p1(®)j ¸ j®j > janj 이어서 ®p1(®) + an = 0 이 될 수 없다. 따라서,

p1(®) = 0 이고 an = 0. 같은 방식으로 반복하여, p2(®) = a0®n¡2 + ¢ ¢ ¢+ an¡2 = 0, an¡1 = 0

이 되고, 귀납적으로 a0 = ¢ ¢ ¢ = an = 0 임을 알 수 있다. 그러므로, p가 영다항식일 때만 반례

가 되고, 영다항식이 아니면 문제가 성립한다. ¤

Baltic 1990-12Baltic 1990-12

m, n은 양의 정수이다. 만약 3m+ 7n 이 83으로 나누어지는 것과 25m+ 3n 이 83으로 나누어지는

것이 동치임을 보여라.

증명 (대전 전민중 2학년 김충은)

25(3m+ 7n) + (¡3)(25m+ 3n) = 166n

으로 83의 배수이다. 25와 3은 83과 서로 소이므로, 3m + 7n과 25m + 3n 중 어느 한 쪽이

83의 배수이면 나머지 한쪽도 83의 배수여야 함을 바로 알 수 있다. 즉, 83 j 3m + 7n 이면

83 j (¡3)(25m+ 3n) 이고, 그럼 83 j 25m+ 3n 이 된다. 반대방향도 비슷하다. ¤

별증 (경기 과천중 2학년 함태준)

3m+ 7n ´ 0 (mod 83) () 28(3m+ 7n) ´ 0 (mod 83)

() 84m+ 196n ´ m+ 30n ´ 0 (mod 83)

() 25(m+ 30n) ´ 0 (mod 83)

() 25m+ 3n ´ 0 (mod 83)

28과 25는 83과 서로 소이므로 위의 과정이 양방향으로 다 잘 된다. ¤

Baltic 1990-13Baltic 1990-13

방정식 x2 ¡ 7y2 = 1 이 무한히 많은 자연수 해를 가짐을 보여라.

증명 (대전 전민중 2학년 정명진)

(x2 ¡ 7y2)2 = (x2 + 7y2)2 ¡ 7(2xy)2

임으로부터, (x; y)가 하나의 해이면 (x2 + 7y2; 2xy)도 해가 됨을 알 수 있다. 이것은 x, y가 점

점 증가하는 점화식이고, (8; 3)이 하나의 해가 되므로, 여기서부터 무한히 많은 해가 계속 만들

어짐을 알 수 있다. ¤

별증 (전남과학고 1학년 정한주)

우선 (x; y) = (8; 3) 이 해가 됨을 알 수 있다. x2 ¡ 7y2 = (x +p7 y)(x ¡ p7 y) = 1 이면

(x2 ¡ 7y2)n = (x+p7 y)n(x¡p7 y)n = 1 이 됨을 이용하자.

an +p7 bn = (x+

p7 y)n (an; bn 2 N)

Page 66: Baltic Way 팀수학경시대회 1990-2005 pdf 보기

66

라 하자.p7이 남는 항들은

p7 y가 홀수 번 곱해진 항들이므로

(x¡p7 y)n = an ¡

p7 bn

이 된다. 따라서,

a2n ¡ 7b2n = (an +p7 bn)(an ¡

p7 bn)

= (x+p7 y)n(x¡p7 y)n

= (x2 ¡ 7y2)n = 1

이 된다. 즉, (an; bn)이 모두 해가 되므로 자연수 해는 무한히 많다. ¤

Baltic 1990-14Baltic 1990-14

두 개나 그 이상의 수들의 모든 가능한 합이 합성수가 되는 1990개의 서로 소인 수들이 존재하는가?

풀이 2 · m · 1990 에 대해 항상 m j M 이 되는 자연수 M을 하나 택하자. 예를 들어

2; 3; : : : ; 1990 의 최소공배수나 1990! 등을 M으로 택할 수 있다. 그럼 Mk + 1꼴의 수들(k =

1; 2; 3; : : : ) 중에서 n개(2 · n · 1990)를 골라 합하면 그것은 MK + n꼴이므로 n의 배수(즉,

합성수)가 된다는 것을 알 수 있다. 이제 Mk + 1꼴의 수들 중에서 서로 소인 1990개를 골라낼

수 있는가 하는 것이 관건이다. a1 =M + 1 로 두고

an+1 = a1 ¢ ¢ ¢ anM + 1 (n ¸ 1)

의 점화식으로 a1; : : : ; a1990을 정의하면 어떤 두 항도 서로 소임을 알 수 있다. }

별해 (한국과학영재학교 2학년 이태희)

최초의 1990개의 소수를 p1; p2; : : : ; p1990이라 하자.

ai =

Ã1990Xk=1

pk

!! + pi (1 · i · 1990)

로 1990개의 수 a1; : : : ; a1990을 정의하자. 이들 중에서 임의의 몇 항 ai1 ; : : : ; ain을 합하면 pi1 +

¢ ¢ ¢ + pin j (P pk)!, 즉 pi1 + ¢ ¢ ¢ + pin j ai1 + ¢ ¢ ¢ + ain 으로 합성수가 된다. 이제 이들이

서로 소라는 것만 보이면 된다. 임의의 두 항 ai와 aj의 임의의 공통소인수를 p라 하자. 그럼

p j (ai ¡ aj) = (pi ¡ pj) j (P pk)! 이므로 p j pi, p j pj 가 되고, 이것은 모순. 따라서, 임의의 두

항 ai와 aj는 항상 서로 소이다. }

Baltic 1990-15Baltic 1990-15

다음 수들이 정수의 세제곱이 되는 경우가 없음을 증명하여라.

Fn = 22n + 1; n = 0; 1; 2; : : :

Page 67: Baltic Way 팀수학경시대회 1990-2005 pdf 보기

1990년 Baltic Way 풀이 67

증명 (서울 성재중 3학년 예창완)

Fn이 정수의 세제곱이 되는 경우가 있다고 가정하자.

Fn = 22n + 1 = k3

22n

= k3 ¡ 1 = (k ¡ 1)(k2 + k + 1)

k(k+1)은 연속하는 두 수의 곱이므로 짝수이고, 따라서 우변의 k2 + k+1 은 3 이상의 홀수이

다. 좌변의 22n

은 소인수가 2뿐이므로 이것은 모순이고, 따라서 귀류법에 의해 문제는 참임을

알 수 있다. ¤

별증 (경기 과천중 2학년 함태준)

세제곱수를 7로 나눈 나머지는 0, 1, 6만 가능하다.

23 ´ 1 (mod 7) 이고 2n 6́ 0 (mod 3)

이므로,

22n

= 23k+(1 또는 2) ´ 2 또는 22 (mod 7)

으로 세제곱수가 될 수 없음을 알 수 있다. ¤

Baltic 1990-16Baltic 1990-16

간격이 1인 가로 세로 격자선이 그려진 종이 위에, 이 격자선을 따라 모든 변의 길이가 홀수인 다각

형을 그린다. 이 다각형의 변의 개수가 4의 배수임을 증명하여라.

증명 (대전 전민중 2학년 김충은)

다각형의 한 점에서 출발하여 변을 따라 움직인 후 다시 돌아오는 것을 생각하자. 세로변을 따

라 올라간 길이를 차례대로 a1; a2; : : : ; an 이라 하면(내려가면 이 값이 음수가 된다),

a1 + a2 + ¢ ¢ ¢+ an = 0

an들이 모두 홀수이므로 n은 짝수이다. 세로변 다음에는 항상 가로변이 이어지므로 다각형의

전체 변의 개수는 2n이고, 이것은 4의 배수이다. ¤

Baltic 1990-17Baltic 1990-17

각각 72개, 30개의 사탕 두 더미가 있다. 두 명의 학생이 번갈아 두 더미 중 하나에서 사탕 몇 개를

가져간다. 매 번 한 더미에서 가져간 사탕의 수는 나머지 더미에 있는 사탕의 수의 배수여야 한다.

둘 중 한 더미에서 마지막 사탕을 가져가는 것이 목표일 때 이기는 사람은 처음 시작한 사람일까,

아니면 그 상대편일까?

Page 68: Baltic Way 팀수학경시대회 1990-2005 pdf 보기

68

풀이 (전남과학고 1학년 정한주)

72 = 30£ 2 + 1230 = 12£ 2 + 612 = 6£ 2

먼저 하는 A는 72개와 30개가 있는 경우나 12개와 30개가 남아있는 경우에서 위와 같이 몫이

2이므로 B의 수를 하나로 제한할 수 있다.

처음 A가 30개를 가져가면, 42개와 30개가 남아 B는 30개를 가져갈 수밖에 없다. 다음 12개와

30개가 있는 상황에서 A가 12개를 가져가면, 12개와 18개가 남아 B는 12개를 가져갈 수 밖에

없다. 이렇게 되면 12개와 6개가 남고, A가 12개를 가져가면 이긴다. }

Baltic 1990-18Baltic 1990-18

양의 정수 1; 2; : : : ; 100; 101을 각각 101번씩 써서 101£ 101 크기의 격자판의 모든 칸을 채웠다. 최

소한 11개의 서로 다른 수들이 쓰여있는 행이나 열이 존재함을 보여라.

증명 각각의 i = 1; 2; : : : ; 101 에 대해, i가 쓰여있는 행의 개수를 ai, 열의 개수를 bi라고 하

자. 이들 행과 열이 만나는 곳에만 i가 쓰여질 수 있으므로, i가 쓰여진 개수에 관해 aibi ¸ 101이라는 부등식이 성립한다. 그럼 A.M ¸ G.M 에 의해

ai + bi ¸ 2paibi > 20 (¤)

이 된다. 이제 문제의 결론 `최소한 11개의 서로 다른 수들이 쓰여있는 행이나 열이 존재함'을

부정해보자. i와 그것을 포함하는 행 혹은 열 ®의 짝 (i; ®)를 모두 센 것을 xi라 하자. 각각의

i에 대해서는 xi = ai + bi 로 세어진다. 또, 각각의 행과 열에 대해서는 각각 최대 10번씩 세어

지므로 Xi

(ai + bi) =Xi

xi · 10 ¢ 101 + 10 ¢ 101 = 20 ¢ 101

이 된다. 이것은 (¤)과 명백히 모순이고, 따라서 문제가 증명되었다. ¤

Baltic 1990-19Baltic 1990-19

다음의 성질을 갖는 집합 f1; 2; : : : ; 2n + 1g의 부분집합들은 최대 몇 개일까?: 임의의 두 부분집합

의 교집합은 하나의 원소나 혹은 몇 개의 연속하는 정수들로 이루어진다.

풀이 a에서 b까지의 모든 정수의 집합을 구간 [a; b]라고 말하기로 하자. 그리고, 임의의 부분

집합 A에 대해 A를 포함하는 가장 작은 구간(A의 가장 작은 수에서 가장 큰 수까지의 구간)을

I(A)로 나타내기로 하자.

(1) A1; : : : ; Ak가 문제의 조건을 만족하는 서로 다른 k개의 부분집합들이면, 그것을 키운

I(A1); : : : ; I(Ak)도 역시 문제의 조건을 만족하는 서로 다른 k개의 부분집합들이다.

Page 69: Baltic Way 팀수학경시대회 1990-2005 pdf 보기

1990년 Baltic Way 풀이 69

(2) 모든 부분집합이 구간일 경우만 생각하면 충분하다.

(3) 모든 구간에 속하는 공통원소가 하나 존재한다.

(4) 공통원소가 n+ 1 일 때 구간의 개수가 최대 (n+ 1)2이 될 수 있다.

위의 사실들을 확인해보자.

(1)의 증명 I(Ai) = I(Aj) 라면 Ai와 Aj는 가장 작은 원소와 가장 큰 원소가 모두

같다. 그럼 Ai \Aj 에도 그 두 원소가 있으므로, 문제의 조건에 의해 이 교집합은 구

간이어야 하므로 Ai\Aj ¾ I(Ai)가 된다.이것을 만족하는 경우는 Ai = Aj = I(Ai)

일 때뿐이고, 따라서, Ai 6= Aj 이면 I(Ai) 6= I(Aj) 이다. 즉, I(A1); : : : ; I(Ak)들도

모두 다르다.

(2)의 증명 (1)에 의해, 문제의 조건을 만족하는 부분집합들의 최대 개수는 문제의

조건을 만족하는 부분구간들의 최대 개수와 같다.

(3)의 증명 부분구간 I1; : : : ; Ik에 대해, li의 하한과 상한을 각각 mi, Mi라 하자.

즉, Ii = [mi;Mi] 이다. 그리고,

m = maxi

mi; M = mini

Mi

로 두자. m = ma, M =Mb 인 구간 Ia, Ib가 존재하므로, m > M 이면 Ia \ Ib = ?

이 되어 모순. 즉, I1 \ ¢ ¢ ¢ \ Ik = [m;M ] 이므로 m ·M 이어야 하고, 이 때 m은 모

든 I1; : : : ; Ik들의 공통원소이다.

(4)의 증명 모든 부분구간에 속하는 원소 k가 있으면 문제의 조건은 자동적으로

만족되므로, k를 갖는 모든 구간의 개수를 구하면 그것이 최대개수가 된다. k를 갖

는 구간은 1; : : : ; k 중에서 하한을 고르고 k; : : : ; 2n + 1 중에서 상한을 고르면 되므

로 모두 k(2n+ 2¡ k)개 있다. G.M · A.M 에서

k(2n+ 2¡ k) ·µk + (2n+ 2¡ k)

2

¶2= (n+ 1)2

이므로, k = n+ 1 일 때 구간의 개수는 최대 (n+ 1)2개이다.

이상에서 부분집합들의 최대개수는 ¢ ¢ ¢ 답 (n+ 1)2 }

주 (3)은 Helly의 정리의 1차원 버전 `유한 개의 구간이 있는데 이들 중 어느 둘도 공통원소

를 갖는다면 모든 구간에 속하는 공통원소가 있다'의 특수한 경우이다. MathLetter 2006년

1월호(통권 제160호)를 참조하기 바란다.

Page 70: Baltic Way 팀수학경시대회 1990-2005 pdf 보기

70

1991년 Baltic Way 풀이

Baltic 1991-1Baltic 1991-1

다음 성질을 갖는 가장 작은 양의 정수 n을 구하여라: n개의 서로 다른 정수를 모은 임의의 집합

fa1; a2; : : : ; ang에 대해 모든 차 ai ¡ aj , i < j 들의 곱이 1991의 배수이다.

풀이 1991 = 11 ¢181 로 소인수분해된다. 182개 이상의 정수가 있으면 그 중 181로 나눈 나머

지가 같은 두 수가 항상 있으므로, 이 두 수의 차가 181의 배수여서 문제의 곱 P는 181의 배수

가 된다. 또 그럼 12개 이상의 정수가 있는 것이므로 그 중 11로 나눈 나머지가 같은 두 수가 있

어 P는 11의 배수이기도 하다. 따라서, n ¸ 182 이면 1991 j P 이다. 한편, n · 181 이면 181로

나눈 나머지가 모두 다르도록 n개의 정수를 고를 수 있고, 그럼 어느 두 수의 차도 181의 배수

가 아니므로 181 - P , 즉 1991 - P 이다. 그러므로, 구하는 n의 최소값은 182 ¢ ¢ ¢ 답 }

Baltic 1991-2Baltic 1991-2

1021991 + 1031991 = nm 을 만족하는 양의 정수 n과 m > 1 은 존재하지 않음을 증명하여라.

증명 (과천중 2학년 함태준)

우선

1021991 + 1031991 ´ 21991 + (¡2)1991 = 0 (mod 5)

이다. 페르마-오일러의 정리에 의해 220 ´ 320 ´ 1 (mod 25) 이므로,

1021991 + 1031991 ´ 21991 + 31991 ´ 211 + 311 ´ 23 + 22 ´ 20 (mod 25)

따라서, 이 수는 5의 배수이면서 25의 배수는 아니다. 즉, 5 j n 이지만 m · 1 이므로 원하는 n,

m은 존재하지 않는다. ¤

별증 (대전 전민중 2학년 김충은, 수정됨)

인수분해하면 다음과 같다.

1021991 + 1031991 = (102 + 103| {z }A

)(1021990 ¡ 10219891031 + 10219881032 ¡ ¢ ¢ ¢+ 1031990| {z }B

)

5 j A 이므로 5 j nm, 즉 5 j n. 문제에서 m > 1 이므로 52 j AB, 여기서 52 - A 이므로 5 j B 가

되어야 한다. 그런데,

B ´ 21990 ¡ 21989 ¢ (¡2)1 + 21988 ¢ (¡2)2 ¡ ¢ ¢ ¢+ (¡2)1990

´ 21990 + 21990 + 21990 + ¢ ¢ ¢+ 21990

= 1991 ¢ 21990 6́ 0 (mod 5)

로 5 - B 이 되어 모순. 따라서, 만족하는 해는 없다. ¤

Page 71: Baltic Way 팀수학경시대회 1990-2005 pdf 보기

1991년 Baltic Way 풀이 71

Baltic 1991-3Baltic 1991-3

12,000원부터 15,000원의 값이 나가는 20마리의 고양이들과, 100원부터 1,000원까지에 팔리는 20개

의 바구니가 있다(모든 가격은 다르다). 두 명의 소년, 지훈이와 판준이가 같은 금액을 지불하고 각

각 바구니 하나에 고양이 한 마리를 살 수 있음을 증명하여라. 단, 가격은 10원 단위까지만 붙일 수

있고, 1원 단위 이하는 매기지 않는다.

증명 (대전 어은중 3학년 김두리)

바구니 하나에 고양이 한 마리의 가격은 12,100원부터 16,000원까지 가능하고, 10원 단위이므

로 모두 391가지가 가능하다. (바구니,고양이) 쌍은 모두 20 ¢ 20 = 400가지로 가능한 가격의 가

지수 391보다 많으므로, 비둘기집의 원리에 의해 같은 가격의 (바구니,고양이) 쌍이 둘 존재한

다. 이 두 쌍에서 바구니가 겹친다면 고양이가 서로 다르므로 가격의 합이 같을 수 없고, 고양

이가 겹치는 경우도 마찬가지. 따라서, 서로 다른 바구니와 서로 다른 고양이로 된 두 쌍이고,

지훈이와 판준이가 한 쌍씩 사면 된다. ¤

Baltic 1991-4Baltic 1991-4

p는 어떤 정수 n에 대해 p(¡n) < p(n) < n 을 만족하는 정수 계수의 다항식이다. p(¡n) < ¡n 이

성립합을 보여라.

증명 (성남 수내중 3학년 임동혁)

인수정리에 의해 일반적으로 (x ¡ y) j p(x) ¡ p(y) 가 성립하므로, 2n j p(n) ¡ p(¡n) 이다. 따

라서, p(¡n) · p(n)¡ 2n < ¡n 이 된다. ¤

Baltic 1991-5Baltic 1991-5

임의의 양의 정수 a, b, c에 대해 다음 부등식이 만족함을 보여라.

1

a+1

b+1

c¸ 2

a+ b+

2

b+ c+

2

c+ a¸ 9

a+ b+ c

증명 (대원외국어고 2학년 최일규)

산술-조화 평균 부등식에 의해1a +

1b

2¸ 2

a+ b1b +

1c

2¸ 2

b+ c1c +

1a

2¸ 2

c+ a

이 세 식을 변변 더하면 왼쪽 부등식을 얻는다. 오른쪽 부등식도 산술-조화 평균에 의해2

a+b +2

b+c +2

c+a

3¸ 3

a+b2+ b+c

2+ c+a

2

로 확인된다. ¤

Page 72: Baltic Way 팀수학경시대회 1990-2005 pdf 보기

72

Baltic 1991-6Baltic 1991-6

[x]는 x의 정수부를 나타내고, fxg = x¡ [x] 이다. 다음 방정식을 풀어라.

[x] ¢ fxg = 1991x

풀이 x > 0 이면 [x] ¢ fxg · [x] · x < 1991x 이므로 해가 없다. x = 0 일 때는 해가 된다. 이

제 x < 0 일 때만 보면 된다. x · ¡1 일 때는 [x] ¢ fxg > [x] > x¡ 1 ¸ 2x > 1991x 이므로 해

가 없다. 따라서, ¡1 < x < 0, 즉 [x] = ¡1 이고, 이 때 ¡(x + 1) = 1991x 를 풀면 x = ¡ 11992 .

¢ ¢ ¢ 답 0, ¡ 11992 }

별해 (광주과학고 2학년 김대영, 수정됨)

우선 x가 정수인 경우, 즉 fxg = 0 인 경우는 x = 0 만 답이 된다. 이제 fxg 6= 0 인 경우를 풀

자. x = [x] + fxg 이므로 [x]fxg ¡ 1991[x]¡ 1991fxg = 0, 따라서

([x]¡ 1991)(fxg ¡ 1991) = 19912

그런데 0 < fxg < 1 이므로 ¡1991 < fxg ¡ 1991 < ¡1990 이다. 따라서,

¡1993 < ¡19912

1990< [x]¡ 1991 < ¡1991; 즉 ¡ 2 < [x] < 0

따라서, [x] = ¡1. 이 때, 대입하여 풀면 x = ¡ 11992 이 된다. }

Baltic 1991-7Baltic 1991-7

A, B, C는 예각 삼각형의 세 각이다. 다음 부등식을 증명하여라.

sinA+ sinB > cosA+ cosB + cosC

증명 (대전 전민중 2학년 김충은)

cosC = cos(180± ¡A¡B) = ¡ cos(A+B) = ¡ cosA cosB + sinA sinB 임을 이용하면

(준식) () sinA+ sinB > cosA+ cosB ¡ cosA cosB + sinA sinB() (1¡ cosA)(1¡ cosB) > (1¡ sinA)(1¡ sinB) (¤)

예각삼각형이므로 A;B < 90± < A+B, 즉 0± < 90±¡B < A, 0± < 90±¡A < B 이고, 따라서

cosA = sin(90± ¡A) < sinB; cosB = sin(90± ¡B) < sinA

가 성립하고, 이로부터 (¤)이 성립함을 알 수 있다. ¤

Page 73: Baltic Way 팀수학경시대회 1990-2005 pdf 보기

1991년 Baltic Way 풀이 73

Baltic 1991-8Baltic 1991-8

a, b, c, d, e는 서로 다른 실수들이다. 다음 방정식

(x¡ a)(x¡ b)(x¡ c)(x¡ d)

+ (x¡ a)(x¡ b)(x¡ c)(x¡ e)

+ (x¡ a)(x¡ b)(x¡ d)(x¡ e)

+ (x¡ a)(x¡ c)(x¡ d)(x¡ e)

+ (x¡ b)(x¡ c)(x¡ d)(x¡ e) = 0

이 4개의 서로 다른 실수해를 가짐을 보여라.

증명 문제의 식을 f(x)라 하고, 일반성일 잃지 않고 a < b < c < d < e 라 하자. 그럼

f(a) = (a ¡ b)(a ¡ c)(a ¡ d)(a ¡ e) > 0, f(b) = (b ¡ a)(b ¡ c)(b ¡ d)(b ¡ e) < 0, 비슷하게

f(c) > 0, f(d) < 0, f(e) > 0 으로 계속 부호가 바뀐다. 따라서, 중간값 정리에 의해 구간 (a; b),

(b; c), (c; d), (d; e)마다 f(x) = 0 의 근이 적어도 하나씩 있다. 4차 방정식이므로 근은 최대 4개

이고, 따라서 각 구간마다 근이 꼭 하나씩 있어 서로 다른 4개의 실근이 된다. ¤

별증 (광주과학고 2학년 김대영)

F (x) = (x¡ a)(x¡ b)(x¡ c)(x¡ d)(x¡ e) 라 두면 F (x) = 0 이 5개의 서로 다른 실근을 가지

므로 y = F (x) 의 그래프는 극대점과 극소점을 2개씩 모두 4개를 갖는다. 따라서, F 0(x) = 0 이

4개의 서로 다른 실수해를 갖는다. ¤

Baltic 1991-9Baltic 1991-9

방정식 aex = x3 의 해는 모두 몇 개일까?

풀이 (전남과학고 1학년 정한주)

우선 a < 0 일 때에는 y = aex 과 y = x3 의 그래프가 아래와 같으므로 교점은 항상 1개, 즉 항

상 1개의 해를 갖는다.

a = 0 일 때에도 x = 0 의 1개의 해를 갖는다. 이제 a > 0 일 때만 보면 된다. 준식의 양변에

ln을 취하면

x+ ln a = 3 ln x

Page 74: Baltic Way 팀수학경시대회 1990-2005 pdf 보기

74

가 된다. y = x+ ln a 와 y = 3 lnx 의 그래프를 그리면 아래와 같다.

따라서, 접할 때를 찾으면 되겠다. 접할 때는 y = 3 ln x 의 기울기가 1이 될 때이므로, 3x = 1,

x = 3, 즉 a = 33

e3 이다.

그러므로, 지금까지의 결과를 종합하면, 이 방정식의 해의 개수 r은

a ¢ ¢ ¢ 0 ¢ ¢ ¢ 33

e3¢ ¢ ¢

r 1 1 2 1 0

이 된다. }

별해 (광주과학고 2학년 김대영)

f(x) = x3

ex 라 두자. 그럼 f(x) = a 의 해의 개수를 구하는 문제가 된다.

f 0(x) =3x2ex ¡ x3ex

e2x=

x2(3¡ x)

ex

이고, x!1;¡1 일 때 f(x)! 0;¡1 이므로, 증감을 조사해보면

x ¡1 ¢ ¢ ¢ 0 ¢ ¢ ¢ 3 ¢ ¢ ¢ 1f 0(x) + 0 + 0 ¡f(x) ¡1 % 0 % 33

e3& 0

따라서, 이것으로 그래프의 개요를 떠올리면, f(x) = a 의 해의 개수는 a에 따라 다음과 같이

변한다:

¢ ¢ ¢ 답 a · 0 이거나 a = r 이면 1개, 0 < a < r 이면 2개, a > r 이면 0개. (r = 33

e3) }

Baltic 1991-10Baltic 1991-10

sin 3± 의 값을 무리식으로 나타내어라.

풀이 (창원남고 2학년 조지환)

먼저 15±에 대한 삼각비를 구하자.

Page 75: Baltic Way 팀수학경시대회 1990-2005 pdf 보기

1991년 Baltic Way 풀이 75

sin 15± = sin(60± ¡ 45±)= sin 60± cos 45± ¡ cos 60± sin 45±

=

p3

2¢ 1p2¡ 12¢ 1p2=

p3¡ 12p2

cos 15± = cos(60± ¡ 45±)= cos 60± cos 45± + sin 60± sin 45±

=1

2¢ 1p2¡p3

2¢ 1p2=

p3 + 1

2p2

다음 µ = 18± 에 대한 삼각비를 구하자. sin 2µ = cos 3µ 가 되므로, cos 3µ = 4 cos3 µ ¡ 3 cos µ,sin 2µ = 2 sin µ cos µ 의 공식을 이용하면

4 cos3 µ ¡ 3 cos µ = 2 sin µ cos µ4(1¡ sin2 µ)¡ 3 = 2 sin µ

4 sin2 µ + 2 sin µ ¡ 1 = 0

의 2차 방정식을 얻는다. sin µ > 0 이므로, 근의 공식으로 풀면

sin 18± =

p5¡ 14

; 또 cos 18± =p1¡ sin2 18± = 5 +

p5

2p2

이 된다. 이제 sin 3± 의 값을 구하자.

sin 3± = sin(18± ¡ 15±)= sin 18± cos 15± ¡ cos 18± sin 15±

=

p5¡ 14

¢p3 + 1

2p2

¡ 5 +p5

2p2

¢p3¡ 12p2

무리식으로 나타내라고 했으므로 이것으로 충분하다. }

Baltic 1991-11Baltic 1991-11

1에서 1 000 000까지의 모든 양의 정수들을 자리수의 합이 홀수이냐 짝수이냐에 따라 두 집합으로

나누자. 두 집합 중 어느 쪽이 더 많은 수를 가질까?

풀이 (전남과학고 1학년 정한주)

십의 자리 이상이 같은 것끼리 10k+0; : : : ; 10k+9 까지를 하나의 그룹으로 생각하자. 각 그룹

에는 홀수와 짝수가 5개씩 같은 개수만큼 있으므로, 꽉 채워지지 않은 그룹만 생각하면 된다.

1; : : : ; 9 에는 홀수가 1개 많고, 1; 000; 000 에 의해서도 홀수가 1개 추가되므로, 홀수가 2개 더

많다. }

Page 76: Baltic Way 팀수학경시대회 1990-2005 pdf 보기

76

Baltic 1991-12Baltic 1991-12

볼록 1991각형의 꼭지점들에 1부터 1991까지의 번호를 붙였다. 그리고, 이 1991각형의 각 변과 대

각선들을 빨간색이나 파란색으로 칠했다. 꼭지점들의 번호를 어떻게 다시 붙이더라도, 원래의 번호

에서 k와 l의 꼭지점을 연결한 선의 색깔과 새로 붙인 번호에서 k와 l의 꼭지점을 연결한 색이 같게

되는, 그런 정수 k; l이 항상 존재함을 보여라.

증명 (대전 어은중 3학년 김두리)

이런 k; l이 존재하지 않는다고 하자. 그럼 처음의 선과 새로 옮겨진 선의 색이 다 달라야 하므

로, 빨간색 선은 파란색 선으로, 파란색 선은 빨간색 선으로 일대일 대응해야 해서 빨간 선과 파

란 선의 개수가 같아야 한다. 그럼 전체 선의 개수가 짝수가 되어야 한다. 그런데, 볼록 1991각

형의 변과 대각선의 개수는 모두 1991¢19902 = 1991 ¢ 995 로 홀수이므로 모순. 따라서, 그런 k; l이

항상 존재한다. ¤

Baltic 1991-13Baltic 1991-13

정삼각형을 25개의 합동인 삼각형들로 나누고 이들에 1부터 25까지 번호를 붙인다. 공통된 변을 갖

는 두 개의 삼각형 중에 번호의 차가 3보다 큰 것이 존재함을 증명하여라.

증명 1의 칸에서 25의 칸까지 가려면 1 ! 4 ! 7! ¢ ¢ ¢ ! 22! 25 로 최소한 8번 이동해야

한다. 즉, 1의 칸과 25의 칸은 적어도 8의 거리(이웃한 칸을 통해 옮겨갈 수 있는 횟수)만큼 떨

어져있어야 한다. 25개의 칸으로 나뉜 정삼각형에는 9 이상의 거리에 있는 두 칸은 없고, 8의 거

리에 있는 경우는 한 칸이 코너에 있고 다른 한 칸이 그에 대응되는 변에 있는 경우뿐이다. 따

라서, 1과 25 둘 중 하나는 코너의 칸에 있어야 한다. 이것은 (1,25)뿐만 아니라 (1,24), (2,25),

(2,24)의 경우도 마찬가지이다. 코너의 칸은 3개뿐이므로 1, 2, 24, 25 중 어느 하나는 코너가 아

닌 변에 있어야 하고, 일반성을 잃지 않고 24나 25가 코너 아닌 변에 있다면, 1과 2가 그 변에

대응되는 같은 코너에 있어야 해서 모순. 따라서, 이웃한 칸의 차가 모두 3 이하가 되도록 할

수는 없다. ¤

Baltic 1991-14Baltic 1991-14

어떤 성에 여러 개의 홀과 n개의 문이 있다. 모든 문은 다른 홀로 연결되거나 바깥으로 연결되어 있

다. 모든 홀은 2개 이상의 문을 가지고 있다. 한 기사가 성에 들어왔다. 어떤 홀에서나 그는 그 홀에

막 들어올 때 사용한 문을 제외한 임의의 다른 문으로 나갈 수 있다. 2n개 이상의 홀을 지나기 전에

그 기사가 바깥으로 나올 수 있는 방법을 찾아라. (지나간 홀도 다시 들어갈 때마다 셈한다)

풀이 각 홀에서 나갈 수 있는 문을 아무렇게나 선택하여 진행하기로 하자. 그러다가 이미 지

났던 홀을 처음으로 다시 들렀을 때 일단 멈춘다. 그 홀을 A라 하자. 문의 수가 유한하므로 유

한 번 안에 이렇게 멈추거나 아니면 그 전에 우연히 성밖으로 나가게 된다. 우연히 성밖으로 나

Page 77: Baltic Way 팀수학경시대회 1990-2005 pdf 보기

1991년 Baltic Way 풀이 77

가게 된 경우에는 한번 지났던 문을 다시 지난 경우가 없으므로, 사용한 문은 기껏해야 n개, 즉

지나간 홀은 기껏해야 n¡ 1 개이므로 만족한다.

홀 A에 재방문했을 때부터는 처음 성에 들어와 A까지 왔던 그 경로로 다시 성밖으로 나가는 것

으로 한다. (A에 첫방문했을 때 사용한 문을 다시 이용해 A에 재방문했다면 그 직전의 홀 B가

A보다 먼저 재방문한 홀이 되므로, A에 재방문했을 때 사용한 문은 첫방문했을 때 사용한 문

과 같을 수 없다.) 홀 A에 첫방문했을 때까지 사용한 문의 개수를 a, 그때부터 홀 A에 재방문했

을 때까지 사용한 문의 개수를 b라 하자. a와 b에는 같은 문이 중복되어 사용되었을 수 없으므

로, a + b · n. 그럼 문을 지난 횟수는 총 2a + b번이고, 지난 홀의 수는 2a + b¡ 1개이므로 이

것은 2a+ b¡ 1 · 2a+ 2b · 2n 임을 알 수 있다. }

Baltic 1991-15Baltic 1991-15

체스판의 각각의 칸에 임의의 정수가 쓰여져 있다. 킹(king)이 체스판 위에서 이동하기 시작했다.

킹이 이동할 때마다 방문한 칸의 수에 1이 더해진다. 체스판 각 칸의 수들이 다음을 만족하도록 하

는 것이 가능한가?

(a) 모두 짝수

(b) 모두 3의 배수

(c) 모두 같은 수

풀이 (수원 대평고 1학년 오정진, 수정됨)

아래 그림들과 같이 체스판의 모든 칸을 정확히 한 번씩 지나 원래의 위치로 돌아오는 길을 찾

을 수 있으므로, (c)만 증명하면 이 길을 이용해 모든 칸에 계속 1씩 더할 수 있어 (a)와 (b)는

저절로 보인 것이 된다. 왼쪽 그림은 빗꼴돌기, 오른쪽 그림은 2차 빗꼴돌기라고 부르기로 하

자. (빗꼴돌기는 가로 세로가 짝수인 2m£ 2n 크기의 판에서 언제가 가능하며, 2차 빗꼴돌기는

가로 세로가 모두 4의 배수인 4m£ 4n 크기의 판을 2m£ 2n 개의 2£ 2 구역들로 나누어 구성

하는 것으로 역시 항상 가능하다.)

이제 (c)를 증명하자. 체스판을 2£ 2의 구역 4£ 4개로 나누어 생각하자. 그리고, 이 작은 구역

들을 2차 빗꼴돌기 혹은 4£ 4 빗꼴돌기로 또 차례로 돌면서, 각 구역의 모든 칸을 같은 수로 만

들도록 해보자. 2£ 2 구역에서는 모든 칸들이 킹의 움직임에 의해 서로 바로 이웃해있다. 네 칸

의 수가 a ¸ b ¸ c ¸ d 라고 하자. 먼저 d의 칸으로 가서 a의 칸과 계속 둘만 왔다갔다 하며 d가

c와 같아질 때까지 a와 d를 함께 증가시킨다. 다음은 c = d 둘만 계속 왔다갔다 하며 b = c = d

Page 78: Baltic Way 팀수학경시대회 1990-2005 pdf 보기

78

가 될 때까지 증가시킨다. 마지막으로 b = c = d 셋을 계속 돌며 a = b = c = d 가 되도록 증가

시킨다. 예를 들어, 다음과 같다:

7 (9)

(1) 2! 7 10

(2) (2)! (7) 10

(7) (7)! 10 10

10 10

이제 다음 구역으로 넘어가야 하는데, 이 작업이 다음 구역으로 넘어가는 출구칸에서 끝나지

않은 경우가 있다. 예를 들어 a칸이 현재 킹이 있는 칸이고 b칸이 출구칸이라 여기로 옮겨가야

한다면, ! d ! a ! c! b 와 같이 옮겨주면 여전히 모든 칸의 수가 같도록 하며 다음 구역으

로 넘어갈 수 있다. 이렇게 모든 구역을 차례로 돌며 각 구역의 수가 같도록 만드는데 성공했으

면, 이제 나타난 수의 최대값을 M이라 할 때, 다시 2차 빗꼴돌기로 각 구역을 차례로 돌며 도

중 한 구역 안을 몇 바퀴 뱅뱅 돌고 다음 구역으로 넘어가는 식으로 하면, 모든 칸이 M + 1이

되도록 할 수 있다. }

별해 (O±cial Solution)

다음과 같은 쌍빗돌기를 생각하자.

임의의 칸 a에 대해, 위의 길을 조금만 수정하여 a칸을 두 번 방문하고 나머지 칸은 모두 한 번

씩 돌고오는 길을 언제나 쉽게 만들 수 있다. 어느 칸 a에 대해서도 그 칸의 바로 옆을 지나는

변(길이 1인 경로의 일부) b! c 가 있어서, 그 변을 b ! a ! c 로 수정해주면 되기 때문이다.

최소값인 칸을 a로 택하여 도는 방법을 반복하다보면 최소값과 최대값의 간격이 차츰 줄어들

어 유한번만에 모든 칸의 수가 같게 된다. }

Baltic 1991-16Baltic 1991-16

두 원 C1, C2 (반지름이 각각 r1, r2) 가 서로 외접하고 있고, 이 두 원의 한 공통외접선을 l이라 하

자. 세 번째 원 C3 (반지름 r3 < min(r1; r2)) 이 앞의 두 원에 외접하고, 직선 l에도 접한다. 다음을

증명하여라.

1pr3=

1pr1+

1pr2

증명 (마산 가포고 1학년 신상)

Page 79: Baltic Way 팀수학경시대회 1990-2005 pdf 보기

1991년 Baltic Way 풀이 79

다음 그림에서 세 개의 직각삼각형을 관찰할 수 있다.

그럼 l의 접점 사이의 거리는 피타고라스의 정리에 의해p(r1 + r2)2 ¡ (r1 ¡ r2)2 =

p(r1 + r3)2 ¡ (r1 ¡ r3)2 +

p(r2 + r3)2 ¡ (r2 ¡ r3)2

즉, 2pr1r2 = 2

pr1r3 + 2

pr2r3 이 성립한다. 양변을 2

pr1r2r3로 나누면 문제의 식을 얻어 증

명이 끝난다. ¤

Baltic 1991-17Baltic 1991-17

3차원 공간의 세 좌표평면(yz-평면, zx-평면, xy-평면)들에 반사하는 성질이 있다고 하자. 이 평면

중 하나에 광선 하나를 쏘았다. 세 좌표평면들로부터 모두 반사된 후 광선이 향하는 방향은 처음 쏜

방향과 비교해 어떻게 바뀌는가?

풀이 yz-평면에 반사된 광선은 반사하지 않고 통과한 경우와 비교하면 yz-평면에 대칭이 된

다. 즉, 입사한 광선의 방향이 (a; b; c)라면 반사된 광선은 x-방향의 부호가 바뀌어 (¡a; b; c)의방향을 갖는다. 다른 평면에 대해서도 비슷하게 되고, 따라서 세 평면에 모두 반사된 후 광선이

향하는 방향은 (¡a;¡b;¡c)로 처음 쏜 방향의 반대방향이다. }

Baltic 1991-18Baltic 1991-18

반지름이 1인 구 안에 부피가 12인 두 개의 사면체를 서로 겹치지 않게 넣는 것이 가능할까?

풀이 가능하다고 가정하고 다음의 단계를 따라가보자.

(1) 사면체는 볼록한 도형이므로, 두 사면체 사이를 지나는(두 사면체를 서로 반대편 영역에

놓이게 하는) 평면이 있다. 이 평면에 의해 구가 두 부분으로 나뉘고, 그 중 작은 부분은

어떤 반구에 포함되므로, 반구 안에도 하나의 사면체를 넣을 수 있어야 한다.

(2) 각 사면체의 꼭지점을 차례로 그 대면으로부터 더 멀어지는 방향으로(높이가 길어지는,

즉 부피가 늘어나는 방향으로) 최대한 움직이면, 각 꼭지점이 반구의 표면 위에 있고 부

피가 12 이상인 사면체를 얻을 수 있다. 또한, 반구의 밑면 ®의 내부에 꼭지점이 있을 때는

그 꼭지점을 대면에서 더 멀어지도록(대면과 반구의 밑면이 평행하다면 적어도 대면에서

더 가까워지지는 않도록) 밑면 ® 위에서 미끄러뜨려서 ®의 원주 P 위로 옮길 수 있다(여

전히 부피가 12 이상이 되도록). 즉, 반구의 구면 위에 네 꼭지점이 있는 부피가 1

2 이상인

사면체 T가 있다.

Page 80: Baltic Way 팀수학경시대회 1990-2005 pdf 보기

80

(3) 이제 ® 위에서 반구의 중심 O를 지나는 한 직선을 축으로 하여 사면체 T를 회전시키면

T의 어느 한 꼭지점 A가 원주 P 위에 놓이도록 할 수 있다. 다시 OA를 축(O = A 라면

적당히 아무 축)으로 하여 사면체 T를 회전시키면 T의 또다른 꼭지점 B가 P 위에 놓이

도록 할 수 있다. 마지막으로 AB를 축으로 하여 적당히 돌리면 T의 한 면 ABC가 ®에 접

하도록 할 수 있다.

(4) 단위원 P의 내부에 놓이는 삼각형 ABC의 넓이는 정삼각형일 때 최대 3p34 이고(이것은

잘 알려진 사실이고, Jensen 부등식 등으로 쉽게 증명할 수 있다) 이것을 밑면으로 하는

높이는 최대 1이므로, T의 부피는 기껏해야 13 ¢ 3

p34 ¢ 1 =

p34 이다. 그런데 이것은 1

2보다

작으므로 모순.

따라서, 귀류법에 의해 불가능함이 증명되었다. }

별해 구 안에 2개의 사면체가 있으면 적어도 하나는 구의 중심을 포함하지 않는다. 그 사면

체의 네 면 중에 사면체와 구의 중심을 서로 반대쪽 영역에 놓이게 하는 면이 적어도 하나 있다.

(다음을 생각하면 이것은 자명하다: 사면체의 각 면을 연장시킨 평면으로 공간을 잘라, 사면체

가 놓이지 않은 영역을 모두 버리면 그 공간에서 사면체만 남는다.) 사면체를 그 면 ®에 수직한

방향으로 평행이동시켜 ®가 구의 중심을 포함하는 면이 되도록 하자. 그럼 어떤 반구에 포함되

고 한 면 ®가 반구의 밑면에 포함되는 사면체를 얻었다. 이 사면체의 밑면의 넓이를 S, 높이를

h라 하면, h · r = 1 이고 밑면은 단위원에 놓인 삼각형이므로 넓이가 최대인 경우는 내접하는

정삼각형일 때, 즉 S · 3p34 r2 = 3

p34 이다. 따라서, 이 사면체의 부피는

V =1

3Sh · 1

3¢ 3p3

4¢ 1 =

p3

4<1

2

이다. 즉, 두 개의 사면체 중에 적어도 하나는 부피가 12보다 작다. 따라서, 불가능하다. }

Baltic 1991-19Baltic 1991-19

서로 외접하고 있는 세 원을 조금 팽창시키면 세 쌍의 교차점을 얻는다. 바깥쪽의 교차점을 A1, B1,

C1이라 하고, 이에 대응되는 안쪽 교차점들을 A2, B2, C2라 하자. 다음 등식이 성립함을 보여라.

jA1B2j ¢ jB1C2j ¢ jC1A2j = jA1C2j ¢ jC1B2j ¢ jB1A2j

증명 세 공통현 A1A2, B1B2, C1C2는 한 점 P에서 만난다(¤).

Page 81: Baltic Way 팀수학경시대회 1990-2005 pdf 보기

1991년 Baltic Way 풀이 81

4PA1B2 » 4PB1A2 이므로

A1B2B1A2

=PA1PB1

; 마찬가지로B1C2C1B2

=PB1PC1

;C1A2A1C2

=PC1PA1

이 세 식을 변변 곱하면 원하는 결과를 얻는다. ¤

(¤)의 증명의 개요 A1A2와 B1B2의 교점을 P라 하고 C1P의 연장이 두 원과 만나는 점을 각

각 C02, C002이라 하면,

PC1 ¢ PC02 = PA1 ¢ PA2 = PB1 ¢ PB2 = PC1 ¢ PC002

임에서 C02과 C002이 같은 점임을 알 수 있다.

주 반전 기하를 이용해 세 원 중 두 원을 직선으로 옮겨놓고 풀 수도 있다.

Baltic 1991-20Baltic 1991-20

y = 1x의 그래프 위의 0 < x1 < x2 과 jABj = 2 ¢ jOAj 를 만족하는 두 점 A(x1; y1), B(x2; y2) 를 생

각하자(O는 원점, 즉 O(0; 0)). 선분 AB의 중점을 C라 하자. x-축과 반직선 OA 사이의 각이 x-축

과 반직선 OC 사이의 각의 3배가 됨을 보여라.

증명 (전남과학고 1학년 정한주)

AB를 연장하여 x축과 만나는 점을 D라 하자.

\AOD = 3\COD 라는 것은 \AOC = 2\COD 가 되는 것과 동치이다. OA = AC 이므로

4AOC는 이등변삼각형이고, 그럼 이 조건은 \ACO = 2\COD 와 동치가 된다. 이것은 또

\ACO가 4COD의 외각임에서 \COD = \CDO, 즉 4COD가 이등변삼각형이라는 것과 동

치이다. 이것을 증명하기로 하자. C의 x-좌표는 x1+x22 이므로, D의 x좌표가 x1 + x2 임을 증명

하면 된다. 직선 AB의 식은

y ¡ 1

x1=

1x2¡ 1

x1

x2 ¡ x1(x¡ x1) = ¡ 1

x1x2(x¡ x1)

x-절편 D를 구하기 위해 y = 0 을 대입하면 1x1= 1

x1x2(x ¡ x1), x2 = x¡ x1, 즉 x = x1 + x2

임이 확인된다. ¤

Page 82: Baltic Way 팀수학경시대회 1990-2005 pdf 보기

82

별증 A, B, C의 좌표는 각각 A(x1;1x1), B(x2;

1x2), C(x1+x2

2;1x1+ 1x2

2) 이다. 직선 OC와 직

선 AB의 기울기 m, m0을 각각 구하면

m =

1x1+ 1

x2

x1 + x2=

1

x1x2

m0 =1x2¡ 1

x1

x2 ¡ x1= ¡ 1

x1x2

으로 서로 부호만 다른 기울기이다. 따라서, \ACO = 2arg¡!OC 가 된다. OA = AC 로 이등변

삼각형이므로 \AOC = \ACO 이고, 이로부터 arg¡!OA = \AOC + arg

¡!OC = 3arg

¡!OC 임이

증명되었다. ¤

Page 83: Baltic Way 팀수학경시대회 1990-2005 pdf 보기

1992년 Baltic Way 풀이 83

1992년 Baltic Way 풀이

Baltic 1992-1Baltic 1992-1

p, q를 연속하는 두 홀수인 소수라 하자. p + q 가 최소 3개 이상의 1보다 큰 (꼭 서로 다를 필요는

없는) 자연수의 곱임을 증명하여라.

증명 (마산 해운중 3학년 신훈)

p+q2 는 p와 q 사이의 정수이다. p와 q 사이에는 더 이상 다른 소수가 없으므로 이것은 합성수,

즉 2개 이상의 1보다 큰 자연수의 곱이다. 따라서, p+ q는 3개 이상의 1보다 큰 자연수의 곱이

다. ¤

Baltic 1992-2Baltic 1992-2

d(n)을 자연수 n의 모든 양의 약수(1과 n 포함)의 개수라 하자. n=d(n) 이 정수가 되는 n이 무한히

많이 있음을 보여라.

증명 (수원 대평고 1학년 오정진)

임의의 소수 p에 대해 n = pp¡1 로 두면 d(n) = p 이므로 n=d(n) = pp¡2 는 정수가 된다. ¤

Baltic 1992-3Baltic 1992-3

모든 항이 일정한 상수가 아닌 자연수들의 무한 등차수열 중 각 항이 두 (자연수의) 제곱수의 합도,

두 세제곱수의 합도 아닌 수열을 하나 찾아라.

풀이1 (대전 전민중 2학년 정명진)

x를 임의의 정수라 할 때 x2 ´ 0; 1; 4; 7 (mod 9), x3 ´ 0;§1 (mod 9) 이다. 따라서, 등차수열

9n+ 3 (n = 1; 2; : : : ) 은 어떤 항도 두 제곱수의 합이나 두 세제곱수의 합이 되지 않는다. }

풀이2 (수원 대평고 1학년 오정진)

x2 ´ 0; 1 (mod 4) 로부터 4m+ 3꼴이면 두 제곱수의 합이 될 수 없고, x3 ´ 0;§1 (mod 7) 로부터 7k+3꼴이면 두 세제곱수의 합이 될 수 없음을 알 수 있다. 따라서, 둘다 만족하는 등차수

열 28n+ 3 (n = 1; 2; : : : ) 을 잡으면 된다. }

Baltic 1992-4Baltic 1992-4

꼭지점들이 모두 격자점이고 각 변의 길이의 제곱이 연속하는 여섯 개의 양의 정수인 육각형을 그

릴 수 있는가?

Page 84: Baltic Way 팀수학경시대회 1990-2005 pdf 보기

84

풀이1 (수원 대평고 1학년 오정진)

모든 꼭지점이 격자점이므로 각 변의 길이의 제곱은 a2 + b2꼴이다. 연속하는 여섯 개의 양의

정수 중에는 4k+3 꼴의 수가 적어도 하나 있는데, 두 제곱수의 합은 mod 4로 0, 1, 2만 가능하

므로 이것은 모순. 따라서, 이런 육각형은 그릴 수 없다. }

주 위의 풀이는 n ¸ 4 인 모든 n각형에 대해서도 적용된다.

풀이2 (대전 전민중 2학년 정명진)

여섯 꼭지점을 (x1; y1), (x2; y2), : : : ; (x6; y6)라 하자. 그럼 각 변의 길이의 제곱의 합 P는

P = (x1 ¡ x2)2 + (y1 ¡ y2)

2 + ¢ ¢ ¢+ (x6 ¡ x1)2 + (y6 ¡ y1)

2

= 2(x21 + ¢ ¢ ¢+ x26 + y21 + ¢ ¢ ¢+ y26)¡ 2(x1x2 + ¢ ¢ ¢+ x6x1 + y1y2 + ¢ ¢ ¢+ y6y1)

로 짝수인데, 연속한 여섯 정수의 합은 m+ (m+ 1) + ¢ ¢ ¢+ (m+ 5) = 6m+ 15 로 홀수이므로

같을 수 없다. 따라서, 그런 육각형은 그릴 수 없다. }

주 위의 풀이는 n = 4k + 2꼴인 모든 n각형에 대해서 적용된다.

Baltic 1992-5Baltic 1992-5

a2 + b2+ (a+ b)2 = c2+ d2 + (c+ d)2 이 성립한다. a4 + b4 + (a+ b)4 = c4 + d4 + (c+ d)4 임을 증

명하여라.

증명 (x2 + y2 + (x + y)2)2 = 2(x4 + y4 + (x + y)4) 의 항등식이 성립함을 전개하여 확인할

수 있다. 이것을 이용하면 문제는 당연해진다. ¤

Baltic 1992-6Baltic 1992-6

99개의 수k3 ¡ 1k3 + 1

, (k = 2; 3; : : : ; 100) 을 모두 곱한 것이 2=3보다 큼을 증명하여라.

증명 인수분해하면

k3 ¡ 1k3 + 1

=(k ¡ 1)(k2 + k + 1)

(k + 1)(k2 ¡ k + 1)=

k ¡ 1k + 1

¢ k2 + k + 1

(k ¡ 1)2 + (k ¡ 1) + 1

이므로, telescoping의 방법으로

100Yk=2

k3 ¡ 1k3 + 1

=

100Yk=2

k ¡ 1k + 1

¢100Yk=2

k2 + k + 1

(k ¡ 1)2 + (k ¡ 1) + 1

=1 ¢ 2 ¢ ¢ ¢ ¢ ¢ 993 ¢ 4 ¢ ¢ ¢ ¢ ¢ 101 ¢

(22 + 2 + 1)(32 + 3 + 1) ¢ ¢ ¢ (1002 + 100 + 1)(12 + 1 + 1)(22 + 2 + 1) ¢ ¢ ¢ (992 + 99 + 1)

=1 ¢ 2

100 ¢ 101 ¢1002 + 100 + 1

12 + 1 + 1=2

3¢ 1010110100

와 같이 계산된다. 따라서, 2=3보다 큼을 확인할 수 있다. ¤

Page 85: Baltic Way 팀수학경시대회 1990-2005 pdf 보기

1992년 Baltic Way 풀이 85

Baltic 1992-7Baltic 1992-7

a = 1992p1992 라 하자.

aaa¢¢¢a o

1992 와 1992

중 어느 수가 더 큰가?

풀이1 a1992 = 1992 이다. 그럼 aa1992

= a1992 = 1992 이기도 하고, 비슷하게 하면 항상

aa¢¢¢a1992

= 1992 가 된다. 1 < a < 1992 이므로

aa¢¢¢aa

< aa¢¢¢a1992

= 1992

따라서, 1992가 더 크다. }

풀이2 1 < b < 1992 이면 1 < ab(= 1992b=1992) < 1992 이다. 이것을 이용하면

1 < a < 1992 =) 1 < aa < 1992

=) 1 < aaa

< 1992

...

=) 1 < aa¢¢¢aa

< 1992

이다. 즉, 1992가 크다. }

Baltic 1992-8Baltic 1992-8

방정식 2x ¢ (4¡ x) = 2x+ 4 의 모든 정수해를 구하여라.

풀이 2x > 0 이므로 4¡ x 와 2x+ 4 가 부호가 같아야 한다. 따라서, ¡2 < x < 4. 이 범위에

서 정수인 x는 ¡1; 0; 1; 2; 3 뿐이고, 각각 대입하여 확인해주면 x = 0; 1; 2 만 답이 된다. }

Baltic 1992-9Baltic 1992-9

다항식 f(x) = x3 + ax2 + bx + c 는 b < 0 와 ab = 9c 를 만족한다. 다항식 f가 서로 다른 세 실근

을 가짐을 증명하여라.

증명1 (광주과학고 1학년 문현식)

도함수 f 0(x) = 3x2 + 2ax+ b 은 판별식이 D0 = a2 ¡ 3b > 0 이므로 두 실근 ®, ¯를 갖는다. 따

라서, 3차함수 f는 극소점과 극대점을 모두 갖는다. 3차함수의 그래프를 생각할 때, 극소점과

극대점에서의 부호가 다르면 f가 틀림없이 서로 다른 세 실근을 갖게 된다. f(x)를 f 0(x)로 나

눈 나머지는 ab = 9c 를 적용하면 r(x) = ( 23b¡ 2

9a2)x 이므로,

f(®)f(¯) = r(®)r(¯) =22

92(3b¡ a2)2®¯

3b ¡ a2 < 0 이므로 (3b ¡ a2)2 > 0 이고, 근과 계수와의 관계에서 ®¯ = b3 < 0 이므로

f(®)f(¯) < 0 임을 확인하였다. ¤

Page 86: Baltic Way 팀수학경시대회 1990-2005 pdf 보기

86

증명2 (대전과학고 1학년 김병수)

x = 0;§p¡b 를 대입하면 f(0) = c, f(§p¡b ) = ¡ab+ c = ¡8c. 따라서, c = 0 일 때는 이 세

점이 세 실근이다. c 6= 0 이면 중간값 정리에 의해 (¡p¡b; 0), (0;p¡b ) 에 각각 근이 하나씩

있고, 3차함수의 그래프를 생각할 때 c의 부호에 따라 (¡1;¡p¡b ) 또는 (p¡b;1)에 근이 하

나 더 있다. ¤

증명3

f(¡a) = ¡ab+ c = ¡8c

c > 0 이면 a < 0, ¡a > 0 인데, 이 때

x ¡1 0 ¡a 1f(x) ¡1 + ¡ 1

이므로 세 실근을 갖는다. c < 0 이면 a > 0, ¡a < 0 인데, 이 때

x ¡1 ¡a 0 1f(x) ¡1 + ¡ 1

이므로 세 실근을 갖는다. c = 0 이면 a = 0, 즉 f(x) = x3 + bx = x(x2 + b) 이고 이것은 b < 0

일 때 틀림없이 세 실근을 갖는다. ¤

Baltic 1992-10Baltic 1992-10

다음 네 조건을 모두 만족하는 4차 다항식 p(x)를 모두 찾아라:

(i) p(x) = p(¡x) (모든 x에 대해)

(ii) p(x) ¸ 0 (모든 x에 대해)

(iii) p(0) = 1

(iv) p(x)는 두 극소점 x1, x2를 갖고, jx1 ¡ x2j = 2 이다.

풀이 (i)에서 우함수이므로 짝수 차수의 항만 있음을 알 수 있고, (iii)에서 상수항이 1이다.

따라서,

p(x) = ax4 + bx2 + 1

로 쓸 수 있다. 또, y축에 대칭이므로 (iv)에서 두 극소점은 ¡1과 1이다. 즉, 도함수 p0(x) =

4ax3 + 2bx 에서 p0(1) = 4a+ 2b = 0, 따라서

p(x) = ax4 ¡ 2ax2 + 1

이 된다. 이제 (ii)의 조건을 만족하려면, 우선 최고차항의 계수 a > 0 이어야 하고, 또, 함수의

그래프를 생각할 때 극소점에서의 함수값 p(1) = ¡a+ 1 ¸ 0 이어야 한다.

¢ ¢ ¢ 답 p(x) = ax4 ¡ 2ax2 + 1 (0 < a · 1) }

Page 87: Baltic Way 팀수학경시대회 1990-2005 pdf 보기

1992년 Baltic Way 풀이 87

Baltic 1992-11Baltic 1992-11

Q+를 양의 유리수 전체의 집합이라 하자. 다음 조건을 만족하는 함수 f : Q+ ! Q+ 가 유일하게

존재함을 보여라.

(i) 0 < q < 1=2 이면, f(q) = 1 + f(q=(1¡ 2q))

(ii) 1 < q · 2 이면, f(q) = 1 + f(q ¡ 1)

(iii) f(q) ¢ f(1=q) = 1 (모든 q 2 Q+에 대해)

증명 (KAIST 수학과 03학번 김린기)

우선 (iii)에서 f(1)2 = 1 이므로 f(1) = 1 이다. 임의의 양의 유리수 q에 대해 그 함수값이 유일

하게 결정됨을 보이면 된다. 만약 12 · q < 1 이거나, q > 2 라면, (iii)에 의해 그 역수 1=q의 함

수값을 구해 역수값을 취해주면 되므로

0 < q <1

2or 1 < q · 2

로 놓아도 무방하다. q = ab 라 하자(a, b는 서로 소).

(1) 0 < ab < 1

2 인 경우: 첫 번째 식에 대입하면

f³ab

´= 1 + f

µa

b¡ 2a¶

이므로, f( ab¡2a )의 값에 의해 결정된다. 여기서 a+ (b¡ 2a) < a+ b 에 주목하자.

(2) 1 < ab · 2 인 경우: 두 번째 식에 대입하면

f³ab

´= 1 + f

µa¡ b

b

이므로, f( a¡bb )의 값에 의해 결정된다. 여기서 (a¡ b) + b < a+ b 에 주목하자.

위에서 임의의 유리수 ab의 함수값을 알기 위해 필요한 유리수를 찾았다. 그런데 필요한 유리수

의 분모와 분자의 합은 원래 알고자 했던 유리수의 분모와 분자의 합 a+ b 보다 작다. 분모, 분

자의 합이 무한히 작아질 수 없으므로 언젠가는 분모, 분자의 합이 2가 되고, 그 때 a = b = 1

이 된다. 그 때 f(1) = 1 로 함수값이 유일하게 결정되므로 f(q)의 값도 유일하게 결정된다. ¤

Baltic 1992-12Baltic 1992-12

N을 자연수 전체의 집합이라 하자. 함수 Á(x) : N ! N 이 일대일 대응이고 극한값

limn!1

Á(n)

n= L

이 존재한다. 가능한 L의 값을 모두 구하여라.

Page 88: Baltic Way 팀수학경시대회 1990-2005 pdf 보기

88

풀이 (한국과학영재학교 1학년 권혁준)

(i) L < 1 일 때: limn!1

Á(n)n < 1 이므로, 충분히 큰 자연수 N이 존재하여 n > N 인 모든 n에

대해Á(n)n < 1, 즉 Á(n) < n 이다. N 이상의 수 중에서 Á(1); Á(2); : : : ; Á(N) 보다 더 큰 자

연수 N 0을 잡자. 그럼

Á(1); Á(2); : : : ; Á(N 0) < N 0

이 되고, N 0보다 작은 자연수는 N 0개보다 적으므로 이것은 Á(1); : : : ; Á(N 0)이 모두 다를

수 없어서 모순이다.

(ii) L > 1 일 때: 비슷하게, 충분히 큰 자연수 N이 존재하여 n > N 인 모든 n에 대해

Á(n)n > 1, 즉 Á(n) > n 이다. N개보다 더 많은 수 1; 2; : : : ; N + 1 중에는 N개의 수

Á(1); Á(2); : : : ; Á(N)에 해당되지 않는 것이 있다. 그 수를 m이라 하면, Á는 일대일 대응

이므로 Á(n) = m 인 n > N 이 있다. 그럼

N + 1 · n < Á(n) = m · N + 1

이므로 모순이 생긴다.

(iii) L = 1 일 때: Á(x) = x 인 항등함수를 생각하면 가능함을 알 수 있다.

따라서, L = 1 만 가능하다. }

Baltic 1992-13Baltic 1992-13

임의의 양수 x1; x2; : : : ; xn; y1; y2; : : : ; yn 에 대해 다음 부등식

nXi=1

1

xiyi¸ 4n2Pn

i=1(xi + yi)2

이 성립함을 보여라.

증명 산술-조화 평균 부등식에 의해Pni=1

1xiyi

n¸ nPn

i=1 xiyi

또 (xi + yi)2 ¸ 4xiyi 임에서

1Pni=1 xiyi

¸ 4Pni=1(xi + yi)2

이 두 부등식을 연결하면 문제의 부등식이 얻어진다. ¤

Baltic 1992-14Baltic 1992-14

어느 나라에 유한 개의 마을이 있다. 그 마을들은 일방통행로로 연결되어 있다. 어떤 두 마을을 택

해도, 그 중 한 마을에서 다른 마을로 갈 수 있다. 다른 모든 마을로 갈 수 있는 마을이 하나 존재함

을 보여라.

Page 89: Baltic Way 팀수학경시대회 1990-2005 pdf 보기

1992년 Baltic Way 풀이 89

증명1 가장 많은 수의 마을로 갈 수 있는 마을을 A라 하자. 만일 A에서 갈 수 없는 마을 B가

있다면, B에서 A로 갈 수 있어야 한다. 그럼 B는 A에도 갈 수 있고 A로부터 갈 수 있는 모든

다른 마을에도 갈 수 있으므로 A보다 더 많은 수의 마을로 갈 수 있다. 이것은 모순. 따라서,

A는 다른 모든 마을로 갈 수 있다. ¤

증명2 수학적 귀납법으로 증명하자. 마을이 2개일 때는 자명하다. 마을이 n개일 때 성립한다

고 가정하자. 마을이 n+ 1개일 때는 그 중 하나의 마을 B를 제외하고 나머지 n개의 마을에 대

해서만 먼저 생각하자. 그럼 그 n개의 마을 중에는 다른 모든 마을로 갈 수 있는 마을 A가 있다.

만일 A와 B 사이에 일방통행로가 A ! B 로 되어 있다면 A가 우리가 찾는 마을이다. B ! A

라면 B가 우리가 찾는 마을이다. 따라서, n+ 1일 때에도 성립함을 알 수 있다. 이로써 마을의

수가 몇 개이든 항상 성립함이 증명되었다. ¤

Baltic 1992-15Baltic 1992-15

노아는 방주에 4개의 우리가 있고 여기에 넣을 8종의 동물들이 있다. 그는 동물들을 종별로 묶어 우

리에 넣을 계획을 하고 있다. 각각의 종마다 함께 어울릴 수 없는 다른 종들이 최대 3종 있음을 알

아냈다. 이 동물들을 아무 불화없이 우리에 모두 넣을 수 있는 방법이 존재함을 보여라.

문제수정 이 동물들을 아무 불화없이 우리에 각각 2종씩 넣을 수 있는 방법이 존재함을 보여라.

단계1 (수원 대평고 1학년 오정진)

우선 `2종씩'은 무시하고 문제를 풀자. 8종의 동물을 아무렇게나 순서를 정해 차례로 우리에 넣

는 것을 생각한다. 각 동물은 함께 어울릴 수 없는 다른 종들이 많아야 3종이므로, 함께 어울릴

수 없는 종이 이미 들어있는 우리는 많아야 3개이다. 따라서, 함께 어울릴 수 있는 종들만 있는

우리가 적어도 하나 있고, 거기다 그 동물을 넣으면 된다. ¤

주 원래 출제진은 4개의 우리에 각각 2종씩 넣는 문제를 생각하였는데, 문제지 편집과정에

서 `2종씩'을 빠뜨리는 실수가 있었다. 그래서 문제가 위와 같이 간단해져버렸는데, 원래 원하

던 문제의 풀이는 다음과 같이 계속된다.

단계2 4종 이상의 동물이 있는 우리 A가 있으면 1종 이하의 동물이 있는 우리 B가 반드시 있

으므로(모두 2종 이상 있으면 전체 10종 이상이 되어 모순), 우리 A의 동물들 중 우리 B의 동물

과 어울릴 수 있는 종이 있으므로 그 종을 우리 B로 옮긴다. 그리고 빈 우리에는 가장 많은 동

물이 있는 우리에서 1종을 옮겨온다. 이렇게 하면 모든 우리에 각각 1종 이상, 3종 이하의 동물

이 있다. 이미 (2; 2; 2; 2)라면 문제가 풀린 것이고, 그렇지 않은 경우는 (3; 2; 2; 1)과 (3; 3; 1; 1)의

경우뿐이다. (3; 3; 1; 1)인 경우에는 처음 두 우리의 6종의 동물들 중 세 번째 우리의 동물과 어

일릴 수 있는 종을 거기로 옮겨 (3; 2; 2; 1)로 만든다. 이제 (3; 2; 2; 1)만 풀면 된다. 순서대로 각

각 우리 A, B, C, D라 하자. 우리 A에서 우리 D로 1종을 옮길 수 없는 경우는 D의 종과 어울

릴 수 없는 3종이 모두 A에 있을 경우뿐이다. 우리 A의 3종과 모두 어울릴 수 없는 종은 기껏

Page 90: Baltic Way 팀수학경시대회 1990-2005 pdf 보기

90

해야 3종뿐이므로, 우리 B와 C의 동물 중에 그렇지 않은 종이 있고, 일반성을 잃지 않고 우리

B의 동물 b가 우리 A의 동물 a와 어울릴 수 있다면, 우리 B의 b 이외의 동물을 우리 D로 옮기

고 a를 B로 옮기면 된다. ¤

Baltic 1992-16Baltic 1992-16

모든 면이 평행사변형인 어떤 볼록 다면체가 있다. 그 다면체가 정확히 1992개의 면을 가질 수 있

는가?

풀이 이 다면체의 한 면을 택하고, 그 면에서 한 변 L1을 택하자. 그 모서리에 인접한 옆면을

생각하면, 그것도 평행사변형이므로, L1에 평행하며 길이도 같은 변 L2가 있다. 비슷하게 L2를

평행이동시킨 변 L3을 찾을 수 있고, 변의 수는 유한하므로 이렇게 계속하면 언젠가

L1 k L2 k ¢ ¢ ¢ k Lk k L1

의 루프를 이룬다. 즉, 이 루프는 평행사변형의 면들이 둥글게 이어진 띠이다. 각 면은 정확히

2개의 띠에 속하고, 임의의 2개의 띠는 정확히 2개의 면에서 만난다. (각 띠의 평행변을 연장하

여 만든 볼록기둥을 생각하면, 그런 두 기둥은 모두 원래의 볼록다면체에 원래의 띠에서 외접

하고, 두 기둥이 만나는 영역이 어떻게 생겼는지 생각하면 이것을 어렵지 않게 알 수 있다.) 즉,

띠가 모두 m개 있다고 하면 이 다면체에는

2 ¢Ãm

2

!= m(m¡ 1)

개의 면이 있다. 1992는 연속한 두 자연수의 곱으로 나타나지 않으므로 이 다면체는 1992개의

면을 가질 수 없다. }

Baltic 1992-17Baltic 1992-17

반지름이 1인 원에 내접하는 사각형 ABCD에서 대각선 AC는 원의 지름이고 또 다른 대각선 BD는

AB와 길이가 같다. 두 대각선이 점 P에서 만나고, PC의 길이는 2=5이다. 변 CD의 길이는 얼마인

가?

풀이 (대전 전민중 2학년 정명진)

ABD가 AB = BD 인 이등변삼각형임에서 OB ? AD 임을 알 수 있다.

Page 91: Baltic Way 팀수학경시대회 1990-2005 pdf 보기

1992년 Baltic Way 풀이 91

또, AC가 지름이므로 지름에 대한 원주각으로부터 CD ? AD 이다. 따라서, OB k CD 이고

이로부터

4POB » 4PCD

가 된다. 그럼 CD : OB = PC : PO, 즉 CD : 1 = 25: 35 이므로, CD = 2

3 이다. }

Baltic 1992-18Baltic 1992-18

둔각이 없는 삼각형에서, 삼각형의 둘레의 길이는 항상 외접원의 지름의 2배보다 큼을 보여라.

증명1 (O±cial Solution)

둔각이 없는 삼각형 ABC의 세 변 AB, BC, CA의 중점을 각각 K, L, M이라 하자. ABC의 외

심 O는 ABC와 닮아서 둔각이 없는 삼각형 KLM의 수심이기도 하므로, O는 KLM의 내부(둘

레 포함)에 있다. 같은 길을 더 멀리 돌아가는 상황을 생각하면

AK +KL+ LC > AO +OC

임을 알 수 있고, 양변을 2배 하면 AB + AC + BC > 2d 의 원하는 부등식이 된다. 단, d는 외

접원의 지름이다. ¤

증명2 삼각형 ABC를 둔각이 없는 삼각형이라 하고, 둘레의 길이를 l, 외접원의 반지름을

R이라 하자. 싸인 법칙에 의해 l = 2R(sinA+sinB+sinC)이므로 증명하고자 하는 식 l > 4R

t := sinA+ sinB + sinC > 2

와 동치이다. C = 180± ¡ (A+B) 임과 삼각함수의 공식에 의해

t = sinA+ sinB + sin(A+B)

= 2 sinA+B

2cos

A¡B

2+ 2 sin

A+B

2cos

A+B

2

= 2 sinA+B

2

µcos

A¡B

2+ cos

A+B

2

¶여기서 sin A+B

2; cos A+B

2> 0 이고 cos A¡B

2¸ 0 이므로, A + B가 고정되어 있다고 볼 때(즉,

C가 고정되어 있다고 볼 때)이것은 cos A¡B2 가 최소일 때 최소가 된다.즉, jA¡Bj가 최대일 때

만 보면 되고, 그것은 A, B 둘 중 하나가 직각일 때이다. ABC가 직각삼각형이면 빗변은 2R과

같고 나머지 두 변의 길이의 합은 빗변보다 크므로 l > 4R 이 틀림없이 성립한다. ¤

Baltic 1992-19Baltic 1992-19

한 평면 위에 원 C가 주어져 있고, C1과 C2는 원 C와 각각 점 A, B에서 안쪽에서 접하는 서로 만

나지 않는 원이다. C1과 C2의 공통외접선 t는 두 원과 각각 점 D, E에서 접한다. AD와 BE의 교점

을 F라 하자. F가 C 위에 있음 보여라.

Page 92: Baltic Way 팀수학경시대회 1990-2005 pdf 보기

92

증명 (서울 구정중 3학년 박상욱, 조금 수정)

A;C1; C와 B;C2; C는 각각 일직선을 이룬다.

\CAF = a, \CBF = b 라 두자. 접선과 현이 이루는 각에서

\AFB = 180± ¡ (\FDE + \FED)

= (90± ¡ \FDE) + (90± ¡ \FED)

= \GDE +\HED = a+ b

그리고 ¤CAFB에서

\ACB(바깥각) = 360± ¡\ACB(안쪽각) = \CAF +\AFB + \CBF = 2(a+ b)

로 \AFB의 2배이므로 원주각과 중심각의 관계로부터 F는 원 C 위에 있다. ¤

Baltic 1992-20Baltic 1992-20

a · b · c 를 직각삼각형의 세 변의 길이라 하고, 2p를 그 둘레의 길이라 하자. S를 삼각형의 넓이

라 할 때, 다음을 보여라.

p(p¡ c) = (p¡ a)(p¡ b) = S

증명 다음과 같이 직접 계산하면 확인된다.

p(p¡ c) =a+ b+ c

2¢ a+ b¡ c

2=(a+ b)2 ¡ c2

4=

ab

2= S

(p¡ a)(p¡ b) =¡a+ b+ c

2¢ a¡ b+ c

2=

c2 ¡ (a¡ b)2

4=

ab

2= S

중간에 피타고라스의 정리 a2 + b2 = c2 가 이용되었다. ¤

Page 93: Baltic Way 팀수학경시대회 1990-2005 pdf 보기

1993년 Baltic Way 풀이 93

1993년 Baltic Way 풀이

Baltic 1993-1Baltic 1993-1

a1a2a3과 a3a2a1은 세 자리의 두 십진법 수로, a1과 a3은 서로 다른 0이 아닌 수이다. 이 수들의 제

곱은 각각 b1b2b3b4b5와 b5b4b3b2b1의 다섯 자리의 수이다. 이러한 세 자리의 수를 모두 구하여라.

풀이 (논산중 3학년 백승호)

abc2 ¸ 10000a2 이 다섯 자리 수를 넘지 않으려면 a · 3 이다. 따라서, a1; a3 2 f1; 2; 3g. 그럼

fa1; a3g = f1; 2g, f1; 3g, f2; 3g뿐이다.

(1) fa1; a3g = f1; 2g 일 때: 1a222은 일의 자리가 4이므로 2a21

2= 40000 + 4000a2 + ¢ ¢ ¢ 의

만의 자리가 4가 되려면 a2 · 2 라야 한다. a2 = 0; 1; 2 일 때를 각각 계산해보면 10404와

40401, 12544와 44521, 14884와 48841로 잘 만족한다.

(2) fa1; a3g = f1; 3g 일 때: 1a232은 일의 자리가 9이므로 3a21

2= 90000 + 6000a2 + ¢ ¢ ¢ 에

서 a2 · 1 이다. a2 = 0; 1 일 때를 각각 계산하면 10609와 90601, 12769와 96721로 잘 만

족한다.

(3) fa1; a3g = f2; 3g 일 때: 같은 방법으로 a2 · 1. a2 = 0; 1 을 대입하면 41209와 91204,

45369와 97344로 만족하는 것이 없다.

이상에서 a1a2a3 = 102; 103; 112; 113; 122; 201; 211; 221; 301; 311 이 답이 된다. }

Baltic 1993-2Baltic 1993-2

각각의 양의 정수 k에 대해서 an+ b가 어떤 양의 정수의 k제곱이 되게 하는 양의 정수 n이 존재하

는 양의 정수 a > b > 1 이 존재하는가?

풀이 존재한다. a = 10, b = 5 로 놓으면 k ¸ 2 에 대해

5k ¡ 5 = 5(5k¡1 ¡ 1)

은 10의 배수이므로 5k = 10n+ 5 가 되는 자연수 n이 항상 존재한다. 또한, k = 1 일 때는 자

명하다. }

주 이밖에도 6n+ 3, 20n+ 5, 14n+ 7 등 여러 가지 예가 있다.

Baltic 1993-3Baltic 1993-3

두 개의 소수의 곱으로 이루어진(두 소수는 서로 같아도 된다) 양의 정수를 재미있는 수라고 하자.

어떤 n개의 연속하는 양의 정수들을 택했더니 모두 재미있는 수들로만 이루어져 있었다. n의 가능

한 최대값은 얼마인가?

Page 94: Baltic Way 팀수학경시대회 1990-2005 pdf 보기

94

풀이 최대값은 n = 3 이고, 그러한 예는 33, 34, 35가 있다. n ¸ 4 일 수 없음을 보이자. 연속

한 4개의 양의 정수 중에는 4의 배수 4k가 꼭 하나 있다. k = 1 일 때는 4k 양쪽의 3과 5 모두

재미있는 수가 아니므로 곤란하고, k > 1 일 때는 4k가 재미있는 수가 아니라서 곤란하다. }

Baltic 1993-4Baltic 1993-4

다음 식이 정수가 되도록 하는 모든 정수 n을 구하여라.s25

2+

r625

4¡ n+

s25

2¡r625

4¡ n

풀이 우선 근호 안이 0 이상이어야 함에서 0 · n · 6254 . 준식을 k(> 0) 라 하면

k2 = 25 + 2pn

로 정리된다. 여기서 2pn 이 정수이므로 4n이 완전제곱수, 즉 n은 완전제곱수이다. n = m2 으

로 두면 0 · m2 · (252)2 이므로 0 · m · 12. 그럼 25 · (k2 =)25 + 2m · 49 이고, 이 범위에

서 홀수이면서 완전제곱수인 것은 25와 49뿐이다. 즉, n = 0; 12 만이 해가 되고, 이 때 준식의

값은 각각 k = 5; 7 이다. }

Baltic 1993-5Baltic 1993-5

임의의 양의 홀수 n에 대해 n12 ¡ n8 ¡ n4 + 1 이 29의 배수가 됨을 보여라.

증명 n12 ¡ n8 ¡ n4 + 1 = (n4 ¡ 1)(n8 ¡ 1) = (n4 + 1)(n2 + 1)2(n+ 1)2(n¡ 1)2 으로 인수분

해된다. n이 홀수이면 n4 + 1, n2 + 1, n+ 1, n¡ 1 모두 짝수이므로 여기서 27의 배수이고, 특

히 n+ 1과 n¡ 1 둘 중 하나는 4의 배수이므로 22이 더 있어서, 이 식은 29의 배수가 된다. ¤

Baltic 1993-6Baltic 1993-6

2 < x < 4 의 범위에서 정의되는 두 함수 f(x), g(x)가 2 < x < 4 인 임의의 x에 대해

2 < f(x) < 4; 2 < g(x) < 4; f(g(x)) = g(f(x)) = x; f(x) ¢ g(x) = x2

를 만족한다. f(3) = g(3) 임을 증명하여라.

증명 (성남 수내중 2학년 임동혁)

h(x) = f(x)=x 로 두자. 그럼 f(x) ¢ g(x) = x2 임에서 g(x)=x = 1=h(x).

h(f(x)) =f(x)

g(f(x))=

f(x)

x= h(x)

이므로, 귀납적으로 항상 h(f (k)(x)) = h(x) 임을 알 수 있다. 그럼

f (k+1)(x)

f (k)(x)=

f(f (k)(x))

f (k)(x)= h(f (k)(x)) = h(x)

Page 95: Baltic Way 팀수학경시대회 1990-2005 pdf 보기

1993년 Baltic Way 풀이 95

이므로,

h(x)k =f (k)(x)

f (k¡1)(x)£ ¢ ¢ ¢ £ f(x)

x=

f (k)(x)

x2 (1

2 ; 2)

이것이 임의의 자연수 k에 대해 성립해야 하므로, h(x)는 항등적으로 1일 수 밖에 없다. 따라

서, h(3) = 1 이고 f(3) = g(3) = 3 이다. ¤

주 비슷하게, f (k)(x)=g(k)(x) = (f(x)=g(x))k 임을 보여 해결할 수도 있다. (대전과학고 1학

년 이정국 학생이 이런 방법으로 풀었다.)

별증 F (x) = 13f(3x), G(x) =

13g(3x) 로 두자. 그럼 F;G : ( 23 ;

43 )! ( 23 ;

43 ) 이고,

F (G(x)) = G(F (x)) = x; F (x) ¢G(x) = x2

이 여전히 잘 성립한다. 또한, F (1) = m 이라 할 때, F (m) = m2=G(m) = m2, 그리고 이로부

터 귀납적으로

F (k+1)(1) = F (F (k)(1)) =(F (k)(1))2

G(F (k)(1))=

m2k

F (k¡1)(1)= mk+1

이 얻어진다. 이 값이 항상 ( 23 ;43 )의 범위에 있어야 하므로 m = 1. 따라서, F (1) = G(1) = 1 이

다. ¤

Baltic 1993-7Baltic 1993-7

다음 연립방정식의 정수해를 구하여라. 8>>>><>>>>:zx = y2x

2z = 4x

x+ y + z = 20

풀이 (청주 대성중 3학년 강진현)

x = 0 이라면 두 번째 식에서 z = 0, 그럼 첫 번째 식에서 00이 나타나므로 이것은 곤란. 따라

서 x 6= 0 이고, 그럼 첫 번째 식에서 z = §y2, 그리고 두 번째 식에서 z = 2x 이다.

(1) z = y2 일 때: x = 12y2 이므로 세 번째 식에서 3

2y2 + y = 20, 즉 3y2 + 2y ¡ 40 = 0,

(y + 4)(3y ¡ 10) = 0. 따라서, y = ¡4, z = 16, x = 8.

(2) z = ¡y2 일 때: x = ¡ 12y2 이므로 세 번째 식에서 ¡ 32y2 + y = 20, 즉 3y2 ¡ 2y + 40 = 0,

이 때에는 판별식 D=4 = 1¡ 120 < 0 으로 해가 없다.

따라서, (x; y; z) = (8;¡4; 16) 이 유일한 해이다. }

Baltic 1993-8Baltic 1993-8

각 자리의 수가 계속 증가하거나 혹은 계속 감소하는 모든 양의 정수들의 합을 구하여라.

Page 96: Baltic Way 팀수학경시대회 1990-2005 pdf 보기

96

풀이 (부산 건국고 1학년 신승현, 수정됨)

자리수가 계속 증가하는 자연수의 집합을 A라 하고, 계속 감소하는 집합을 B라 하자. 자리수가

계속 증가하는 임의의 k자리의 자연수 a = a1a2 ¢ ¢ ¢ ak 를 생각하자. 각각의 i에 대해 bi = 9¡ai

라 하면,

b = b1b2 ¢ ¢ ¢ bk 2 B 이고; b0 = 9b1b2 ¢ ¢ ¢ bk 2 B

이다. 또한 B의 임의의 원소는 그 첫자리가 9이면 b0의 꼴로, 9가 아니면 b의 꼴로 A의 원소

로부터 만들어진다. (즉, A와 B는 1 : 2로 대응한다. 그리고, 0은 합에 영향을 미치지 않으므

로 B에 9 ¡ 9 = 0 이 들어있다고 생각해도 된다.) (a + b) + (a + b0) = 99 ¢ ¢ ¢ 9 + 999 ¢ ¢ ¢ 9 =(10k ¡ 1) + (10k+1 ¡ 1) 이고, 이런 k자리의 수 a는

¡9k

¢개 있으며, 1 · k · 9 이므로, 다음의 식

이 성립한다.

2®+ ¯ =

9Xk=1

Ã9

k

!(11 ¢ 10k ¡ 2) =: X

단, ®와 ¯는 각각 집합 A와 B의 원소의 합이다. 이번에는 ci = 10¡ ai 로 해보자. 그럼

c = c1c2 ¢ ¢ ¢ ck 와 c0 = c1c2 ¢ ¢ ¢ ck0

이 둘다 B의 원소가 되고 또 B의 전부가 된다.이로부터 또 같은 방식으로 하면, (a+c)+(10a+

c0) = 11 ¢ ¢ ¢ 10 + 11 ¢ ¢ ¢ 100 = 109(10k ¡ 1) + 100

9(10k ¡ 1) 이므로,

11®+ ¯ =9X

k=1

Ã9

k

!110

9(10k ¡ 1) =: Y

그럼 두 식으로부터 연립방정식을 풀어 ®와 ¯를 계산할 수 있다. 그런데 특히 한 자리의 수는

A와 B에 중복되어 포함되므로 구하는 합은 다음과 같다.

S = ®+ ¯ ¡ 45 = 10

9X ¡ 1

9Y ¡ 45

=

9Xk=1

Ã9

k

!µ990¡ 110

9210k ¡ 90 ¢ 2¡ 110

92

¶¡ 45

=880

92(119 ¡ 1)¡ 70

92(29 ¡ 1)¡ 45 = 80

811110 ¡ 35

81210 ¡ 55

굳이 계산한다면 25617208985 ¢ ¢ ¢ 답 }

Baltic 1993-9Baltic 1993-9

다음 연립방정식을 풀어라. 8>>>>>>><>>>>>>>:

x5 = y + y5

y5 = z + z5

z5 = t+ t5

t5 = x+ x5

Page 97: Baltic Way 팀수학경시대회 1990-2005 pdf 보기

1993년 Baltic Way 풀이 97

풀이 x = 0 이면 마지막 식에서 t = 0, 다시 차례로 z = 0, y = 0 이 된다. x > 0 이면 마지

막 식에서 t5 = x + x5 > x5 이므로 t > x, 다시 차례로 z > t, y > z, x > y 가 된다. 이것은

x > y > z > t > x 이므로 모순이다. x < 0 일 때에도 비슷하게 x < y < z < t < x 로 역시 성

립하지 않는다. 따라서, 해는 x = y = z = t = 0 이 유일하다. }

별해 네 식을 모두 더하면 x+ y + z + t = 0 을 얻는다. 네 식 각각을 다시 관찰하면, x, y, z,

t는 모두 동시에 양수이거나, 모두 동시에 음수이거나, 모두 동시에 0이어야 함을 알 수 있다.

따라서, 가능한 경우는 x = y = z = t = 0 뿐이다. }

Baltic 1993-10Baltic 1993-10

a1; a2; a3; : : : ; an 과 b1; b2; b3; : : : ; bn 은 서로 다른 2n개의 실수로 이루어진 두 개의 유한 수열이다.

각각의 수열을 증가수열로 재배열 한 것을 a01; a02; a

03; : : : ; a

0n 과 b01; b

02; b

03; : : : ; b

0n 이라고 하자. 다음

을 증명하여라.

max1·i·n

jai ¡ bij ¸ max1·i·n

ja0i ¡ b0ij

증명 (성남 수내중 2학년 임동혁, 조금 수정됨)

귀류법으로 풀자. max jai ¡ bij < ja0j ¡ b0j j 이라고 하자. 일반성을 잃지 않고 a0j < b0j 이라고 하

자. 그런데,

a0j > a0j¡1 > a0j¡2 > ¢ ¢ ¢ > a02 > a01

b0j < b0j+1 < b0j+2 < ¢ ¢ ¢ < b0n¡1 < b0n

이다. 여기서 보면, (비둘기집의 원리에 의해, 비둘기는 n + 1개인데 집은 n개이므로) 아무렇

게나 재배열했을 때 a01부터 a0j까지 중의 하나와 b0j부터 b0n까지 중의 하나는 쌍을 이뤄야 한

다. 그 어느 둘을 am, bm 이라 하자. 그렇다면 am · a0j 일 것이고, bm ¸ b0j 일 것이다. 이 때

bm ¡ am ¸ b0j ¡ a0j > max jai ¡ bij 으로 모순. 따라서, 본 문제가 성립한다. ¤

Baltic 1993-11Baltic 1993-11

어떤 정삼각형이 n2개의 합동인 정삼각형으로 나뉘어져있다. 이 격자의 한 꼭지점에 거미 한 마리

가 있고, 또다른 꼭지점에 파리 한 마리가 있다. 이들이 번갈아가면서 이웃하는 꼭지점으로 이동한

다. 거미가 항상 파리를 잡을 수 있음을 보여라.

증명 거미는 우선 파리와 같은 높이로 이동하기로 한다. 높이는 아래로도 위로도 한계가 있

으므로 유한 번(10번) 안에 파리는 거미에게 같은 높이로 따라잡힌다. 일단 같은 높이가 되었

으면, 이후부터는 거미는 늘 파리와 같은 높이를 유지하면서 쫓을 수 있다. 즉, 같은 높이가 되

었을 때 일반성을 잃지 않고 파리가 거미보다 오른쪽에 있다고 하면, 파리가 좌상이나 우상으

로 이동하면 거미는 우상으로 이동하고, 파리가 좌하나 우하로 이동하면 거미는 우하로 이동하

고, 파리가 좌나 우로 이동하면 거미는 우로 이동한다. 오른쪽으로 가는 것에도 한계가 있으므

로, 다시 유한 번(10번) 안에 파리는 거미에게 좌우로도 따라잡히게 된다. ¤

Page 98: Baltic Way 팀수학경시대회 1990-2005 pdf 보기

98

Baltic 1993-12Baltic 1993-12

어떤 왕국에 13개의 도시가 있다. 어떤 두 도시들 간에는 양방향의 직행버스나 기차 혹은 비행기 노

선이 있다. 임의의 두 도시 사이를 이동할 때 항상 이 세 종류의 탈 것 중에서 어느 두 종류만을 이

용하여 갈 수 있도록 하려면 최소 몇 개의 노선을 만들면 되겠는가?

풀이 (과천중 2학년 함태준, 조금 수정)

직행버스, 기차, 비행기 노선의 수를 각각 a, b, c라 하자. 일반적으로 n개의 점이 모두 연결되

기 위해서는 적어도 n¡ 1개의 변이 필요하다. 즉, 어느 두 종류만으로도 모든 도시가 연결되어

야 하므로,

a+ b ¸ 12b+ c ¸ 12c+ a ¸ 12

이 세 식을 모두 더하면 a+ b+ c ¸ 18 이 된다.

위의 그림과 같이 하면 18개의 노선만으로 조건에 맞게 구성할 수 있다. }

Baltic 1993-13Baltic 1993-13

정삼각형 ABC가 100개의 합동인 정삼각형으로 나뉘어져있다. 어느 두 점도 삼각형 ABC 의 변과

평행한 직선 위에 있지 않도록 작은 삼각형들의 꼭지점들을 택할 때, 최대 몇 개의 점을 택할 수 있

겠는가?

풀이 (성남 수내중 2학년 임동혁)

문제를 만족하는 점들의 수를 n개라 하자. 이 때, 각 점 Xi에서 변 BC, CA, AB까지의 거리(격

자선을 따라 이르는 최단거리)를 각각 pi, qi, ri라 하자(1 · i · n). 그럼

pi + qi + ri = 10 (1 · i · n)

이고, 따라서nXi=1

(pi + qi + ri) = 10n

이다. 한편, 문제의 조건에서 각각의 pi들은 서로 다른 0 이상의 정수이다. 따라서,

nXi=1

pi ¸ 0 + 1 + ¢ ¢ ¢+ (n¡ 1) = 1

2n(n¡ 1)

이다. 이것은 qi, ri들도 마찬가지이므로,

10n ¸ 3

2n(n¡ 1)

Page 99: Baltic Way 팀수학경시대회 1990-2005 pdf 보기

1993년 Baltic Way 풀이 99

이어야 한다. 이것을 풀면 32 (n ¡ 1) · 10, n¡ 1 · 20

3 < 7, 즉 n < 8 이다. n = 7 일 때는 다음

과 같이 성립하는 경우가 있다.

따라서, 점은 최대 7개까지만 택할 수 있다. }

Baltic 1993-14Baltic 1993-14

한 정사각형을 16개의 같은 크기의 정사각형으로 나누어 25개의 꼭지점을 얻었다. 여기서 최소 몇

개의 점을 빼내면, 남아있는 점들 중에 어느 4개의 점을 택해도 처음 정사각형과 평행한 변을 갖는

정사각형의 꼭지점이 되지 않도록 할 수 있는가?

문제수정 8개의 점을 빼내면 정사각형이 하나도 남지 않도록 할 수 있고, 6개의 점을 빼내는 것으

로는 정사각형이 하나도 남지 않도록 할 수 없음을 보여라.

풀이 우선 다음과 같이 8개의 점을 지우면 정사각형이 하나도 남지 않게 된다.

²²²²²

²² ²

6개로 가능하다고 가정하자. 우선 한 변의 길이가 4인 정사각형이 제거되어야 하므로 네 코너

의 점 중 하나는 제거해야 한다. 그 점을 A1이라 하자. 이제 그림 (a)를 생각하자. 2£ 2 판에사

는 최소 점 2개를 빼내야 정사각형이 하나도 남지 않으므로, 점 B2를 제거하지 않고서는 굵은

정사각형을 모두 지우기 위해 필요한 점의 수를 따지면 6개, 즉 A1을 포함하여 7개가 필요하므

로 모순. 따라서, 점 B2를 제거해야 한다.

E

D

C

B

A

1 2 3 4 5² ²

²²²²

²²²²

²²²²²

²

(a) (b) (c) (d) (e)

이제 그림 (b)를 생각하자. 굵은 정사각형을 모두 지우려면 비슷하게 5개의 점이 추가로 필요

하므로 중복된 C3를 반드시 제거해야 한다. 계속하여 그림 (c)를 생각하면 D4를, 그림 (d)에서

는 B4와 D2를 제거해야 하고, 이렇게 6개의 점을 제거해도 그림 (e)에서 남은 정사각형이 있

음을 확인할 수 있다. 이것은 모순이고, 따라서 6개로는 불가능하다. }

Page 100: Baltic Way 팀수학경시대회 1990-2005 pdf 보기

100

주 이 문제의 O±cial Solution에는 답이 7개로 되어있었고 그것은 틀렸다. 7개로는 불가능하

다는 것은 매우 심한 막노동으로 따져볼 수는 있는데, 그 풀이를 써놓는 것은 거의 의미가 없는

일이라 여겨지므로 생략한다.

Baltic 1993-15Baltic 1993-15

두 개의 주사위의 각각의 면에 양의 정수를 썼다. 두 개의 주사위를 던져 나온 수를 더한다. 이 합이

2, 3, 4, 5, 6, 7, 8, 9, 10, 11, 12, 13이 될 수 있고 각 합이 모두 똑같은 확률로 나오도록 주사위 눈을

정하는 것이 가능한가?

풀이 가능하다. (1,1,1,2,2,2)와 (1,3,5,7,9,11)이면 된다. }

주 또는 (1,1,1,7,7,7)과 (1,2,3,4,5,6)으로 해도 된다.

Baltic 1993-16Baltic 1993-16

반지름 r을 갖는 두 개의 원이 한 평면 위에 있고 서로 만나지 않는다. 이 평면 위의 한 직선이 한 원

과 점 A, B에서 만나고 다른 원과 점 C, D에서 만나는데, jABj = jBCj = jCDj = 14 cm 이다. 또

다른 직선이 두 원과 각각 점 E, F와 점 G, H에서 만나고, jEF j = jFGj = jGHj = 6 cm 이다. 반

지름 r을 구하여라.

풀이 (서울 목일중 3학년 김글빛, 조금 수정)

두 원을 각각 O1, O2라 하자. O1에서 AB, EF에 내린 수선의 발을 각각 I, J이라 하고, O2에

서 CD, GH에 내린 수선의 발을 각각 L, K라 하자.

그림에서 JK < O1O2 = IL 이므로 ABCD와 EFGH가 서로 역할이 바뀔 수 없음을 알 수 있

다. O1O2 = IL = 7+14+7 = 28 이고 JK = 3+6+3 = 12 이다. 그리고 직각삼각형 O1JF와

O1JM에서 피타고라스의 정리를 쓰면(M은 O1O2의 중점)

O1M2 ¡ JM2 = O1J

2 = O1F2 ¡ JF 2

즉, 142 ¡ 62 = r2 ¡ 32 이므로, 이것을 풀면 r2 = 169, r = 13 cm. }

Page 101: Baltic Way 팀수학경시대회 1990-2005 pdf 보기

1993년 Baltic Way 풀이 101

Baltic 1993-17Baltic 1993-17

한 평면 위의 어느 둘도 서로 평행하지 않은 세 직선을 생각해보자. 각 직선마다 한 점씩 모두 세 개

의 점이 서로 다른, 0이 아닌 일정한 속도로 각 직선 위를 움직이고 있다. (이 움직임은 무한한 시간

동안 계속 되어왔고, 앞으로도 무한히 계속된다고 생각하자.) 이 점들이 과거, 현재, 미래, 어느 때

에도 한 직선 위에 절대 있지 않도록, 세 직선의 위치, 세 점의 속력, 그리고 시점 0에서의 세 점의

위치를 정할 수 있을까?

풀이 (광주 동성고 1학년 방재혁, 서울 목일중 3학년 김글빛)

정삼각형 ABC를 그리고, 세 직선 AB, BC, CA를 생각하자.

X는 A점에서 출발하여 B의 방향으로, Y는 B점에서 출발하여 C의 방향으로, Z는 C점에서 출

발하여 A의 방향으로, 모두 같은 속도로 움직이는 점들이다. 그럼 XY Z는 항상 정삼각형을 이

루므로 이 세 점은 한 직선 위에 있는 경우가 없다. }

별해 세 점 중 한 점을 항상 원점으로 두는 변환을 생각하자. 즉, 이 한 점의 관점에서 나머

지 두 점의 상대적인 위치를 관찰하는 것이다. 그럼 원점 O와 등속운동을 하는 두 동점 P , Q에

관한 문제로 바뀐다.

P는 (1; 0)에서 출발하여 y-축의 양의 방향으로, Q는 (0; 1)에서 출발하여 x-축의 음의 방향으로

같은 속도로 이동한다고 해보자. 그럼 항상 Q는 P로부터 90± 회전된 위치이므로 POQ는 직각

이등변삼각형이고, 따라서 이 세 점이 한 직선 위에 있는 경우는 없다. }

Baltic 1993-18Baltic 1993-18

jABj = 15, jBCj = 12, jACj = 13 인 삼각형 ABC가 있다. 중선 AM과 꼭지각의 이등분선 BK가

점 O에서 만난다(M 2 BC, K 2 AC). OL ? AB, L 2 AB 라 하자. \OLK = \OLM 임을 보여

라.

Page 102: Baltic Way 팀수학경시대회 1990-2005 pdf 보기

102

증명 A(0; 0), B(15; 0)로 두고, 점 F의 좌표를 (xF ; yF )와 같은 식으로 쓰기로 하자(yC > 0).

cosine 제2법칙에서

xC = 13 cosA = 13 ¢ 132 + 152 ¡ 1222 ¢ 13 ¢ 15 =

250

2 ¢ 15 =25

3

C를 지나 AB에 평행한 직선 `을 그리자. 직선 AM , BK, LM , LK가 `과 만나는 점을 각각 P ,

Q, X, Y라 하자. 4CBQ는 이등변삼각형이므로 CQ = CB = 12, 또 4MAB ´ 4MPC 이므

로 PC = AB = 15 이다. 그럼 4OAB » 4OPQ 이므로

xO = xP ¢ OA

OA+OP= (xC + 15) ¢ AB

AB + PQ=70

3¢ 1542=25

3= xC

따라서, O, C, L은 한 직선 위에 있다. ` k AB 에 의한 삼각형의 닮음으로부터

CY=AL(= CK=AK) = CQ=AB

CX=BL(= CM=BM) = CP=AB

CP=AL(= OC=OL) = CQ=BL

첫째 식의 분자 분모를 뒤집은 후 이 세 식을 변변 곱하면 CX = CY 가 얻어진다. 따라서, 직

선 OL은 이등변삼각형 XLY의 꼭지각의 이등분선이다. ¤

별증 직선 CO가 AB와 만나는 점을 P라 하자.

AM이 중선이므로 PK k BC. 이것과 각의 이등분선 정리에 의해 AP : PB = AK : KC =

AB : BC = 5 : 4 가 된다.

AC2 ¡BC2 = 25 = AP 2 ¡BP 2

이므로, P는 C에서 AB에 내린 수선의 발이 된다. 따라서 P와 L은 같은 점이다. M이 직각삼

각형 BPC의 외심이므로, \OPK = \OCM = \OPM 이 성립한다. ¤

Page 103: Baltic Way 팀수학경시대회 1990-2005 pdf 보기

1993년 Baltic Way 풀이 103

Baltic 1993-19Baltic 1993-19

볼록사각형 ABCD가 중심 O인 원에 내접한다. 네 각 AOB, BOC, COD, DOA는 순서를 적당히

잡으면 사각형 ABCD의 네 내각과 각각 크기가 같다. ABCD가 정사각형임을 증명하여라.

증명 네 각 \A, \B, \C, \D의 크기를 각각 a, b, c, d라 하고, 각 변을 바라보는 중심각의

크기를 그림과 같이 x, y, z, w라 하자.

일반성을 잃지 않고, a, b, c, d 중 가장 큰 것을 a라 하자. 그럼 z +w · 2a 이고, 원주각과 중심

각의 관계에서 등호가 성립할 때가 되므로 z = w = a 이다. 또, a+ c = b+ d = 180± 이므로 a,

b, c, d 중에서 c가 최소이고, 그럼 마찬가지로 x+ y ¸ 2c 로부터 x = y = c 라야 한다. 즉,

fx; y; z; wg = fa; a; c; cg = fa; b; c; dg

그럼 b도 최대 혹은 최소의 각이므로 x = w, 따라서 x = y = z = w = 90± 이고 이것은 정사각

형이다. ¤

Baltic 1993-20Baltic 1993-20

Q를 단위 정육면체라 하자. 꼭지점들이 모두 Q의 겉표면 위에 있는 정사면체를 좋은 사면체라고

부르기로 하자. 좋은 사면체의 가능한 부피들을 모두 구하여라.

풀이 단위 정육면체에 내접하는 사면체의 최대 부피는 13이다(¤). 따라서, 좋은 사면체의 부

피도 13 이하이다.

그리고, 위의 그림과 같은 좋은 사면체는 크기를 0부터 13까지 연속적으로 변화시킬 수 있으므

로, 13 이하의 값은 모두 좋은 사면체의 부피가 될 수 있다. }

(¤)의 증명 먼저 이런 사면체의 부피의 최대값은 사면체의 네 꼭지점이 모두 주어진 단위 정

육면체의 꼭지점들일 때만 살피면 된다는 것을 보이자. 사면체의 임의의 한 꼭지점 A를 택하

Page 104: Baltic Way 팀수학경시대회 1990-2005 pdf 보기

104

고, 나머지 세 꼭지점 BCD로 결정되는 평면을 밑면 ®라 하자. A를 지나고 ®와 평행한 평면을

¯라 하면, ®와 ¯ 사이의 거리 h가 이 사면체의 높이가 된다. 따라서, 고정된 BCD에 대해 A를

움직이는 것을 생각하면흔들기 ¯가 정육면체와 접할 때 h가 최대가 되어 사면체의 부피가 가

장 크고, 즉 A가 정육면체의 꼭지점일 때만 생각해도 그 중에 최대값이 있다. 이것은 차례로 B,

C, D에 대해서도 얘기할 수 있고, 결국 사면체의 네 꼭지점이 모두 정육면체의 꼭지점일 때 중

에 부피가 최대인 경우가 있다.

사면체의 네 꼭지점이 모두 정육면체의 꼭지점일 때 중에서 부피가 최대인 경우를 찾아보자.

정육면체의 한 면에 사면체의 네 꼭지점이 모두 있으면경우 나누기 사면체가 아니다. 정육면체

의 한 면에 사면체의 네 꼭지점 중 세 꼭지점이 있으면 그때의 부피는 16이 된다. 정육면체의 모

든 면에 사면체의 꼭지점이 2개 이하 있을 때는 다음 그림과 같은 경우뿐이고,

이 때 사면체 BDEG의 부피는 정육면체 전체에서 사면체 ABDE와 같은 사면체를 4개 빼어

준 것이므로 1¡ 4 ¢ 16 = 13 이다. 이것이 위에 고려한 것 중 가장 크므로, 이 때가 최대이고 부피

의 최대값은 13이다. ¤

Page 105: Baltic Way 팀수학경시대회 1990-2005 pdf 보기

1994년 Baltic Way 풀이 105

1994년 Baltic Way 풀이

Baltic 1994-1Baltic 1994-1

a ± b = a+ b¡ ab 라 하자. 다음을 만족하는 모든 정수해 (x; y; z)를 구하여라.

(x ± y) ± z + (y ± z) ± x+ (z ± x) ± y = 0

풀이 (a±b)±c = (a+b¡ab)+c¡(a+b¡ab)c = a+b+c¡ab¡bc¡ca+abc = (a¡1)(b¡1)(c¡1)+1이므로, 준식은 3(x¡ 1)(y ¡ 1)(z ¡ 1) + 3 = 0, 즉

(x¡ 1)(y ¡ 1)(z ¡ 1) = ¡1

이 된다. 그럼 x¡ 1, y ¡ 1, z ¡ 1 중에 ¡1이 홀수 개이고 그외에는 모두 1. (x; y; z) = (0; 0; 0),

(2; 2; 0), (2; 0; 2), (0; 2; 2) ¢ ¢ ¢ 답 }

주 (광주 동성고 2학년 이영곤)

1¡ (a ± b) = (1¡ a)(1¡ b)

임을 관찰하면

1¡ (a ± b ± c) = (1¡ a)(1¡ b)(1¡ c)

임을 전개해보지 않아도 쉽게 알 수 있다.

Baltic 1994-2Baltic 1994-2

a1; a2; : : : ; a9는 a1 = a9 = 0 인 음이 아닌 정수들이고, 적어도 하나는 0이 아니다. 2 · i · 8인 i 중

에 부등식 ai¡1 + ai+1 < 2ai 를 만족하는 것이 있음을 보여라. 마지막 부등식에서 2를 1.9로 바꾼

다면 그래도 이 명제가 성립하겠는가?

증명 (서울 중앙중 1학년 이수홍)

귀류법으로 풀자. i = 1; 2; : : : ; 8 에 대해 ai¡1 + ai+1 ¸ 2ai 의 부등식을 모두 변변 합하여 정

리하면 a1 + a9 ¸ a2 + a8 이 된다. 즉, a2 = a8 = 0 이다. 비슷하게 반복하면 a3 = a7 = 0,

a4 = a6 = 0, a5 = 0 도 차례로 얻는다. 그런데 적어도 하나는 0이 아니라고 했으므로 이것은

모순. 따라서, 문제가 성립한다.

2를 1.9로 바꾸어도 여전히 성립한다.귀류법으로,모든 2 · i · 8에 대해서 ai¡1+ai+1 ¸ 1:9ai

Page 106: Baltic Way 팀수학경시대회 1990-2005 pdf 보기

106

라 하자.

i = 2 =) 19

10a2 · a3

i = 3 =) 19

10a3 · a2 + a4 · 10

19a3 + a4 =) 261

190a3 · a4

i = 4 =) 19

10a4 · a3 + a5 · 190

261a4 + a5 =) 2059

2610a4 · a5

i = 5 =) 32021

30590a5 · a6

i = 6 =) 302999

320210a6 · a7

i = 7 =) 2545381

3024990a7 · a8

i = 8 =) 18112339

25453810a8 · a9 = 0

마지막 단계에서 a8 · 0, 따라서 a8 = 0 이고, 마찬가지로 a7 = 0, a6 = 0, : : : , 이렇게

a1; : : : ; a9가 모두 0이 되므로 모순. 따라서 ai¡1 + ai+1 < 1:9ai 인 i가 존재한다. ¤

Baltic 1994-3Baltic 1994-3

다음 식의 최대값을 구하여라.

xy + xp1¡ y2 + y

p1¡ x2 ¡

p(1¡ x2)(1¡ y2)

풀이 (성남 수내중 3학년 임동혁)

근호 안이 음수가 될 수 없으므로, jxj; jyj · 1. 그리고, 문제의 식을 잘 관찰하면 x, y 중에 음수

가 있다면 둘다 부호만 양으로 바꿔줬을 때 식의 값이 더 커지므로, x; y ¸ 0 일 때만 살피면 충

분하다. x = cosA, y = cosB 라 하자(0 · A;B;· ¼2). 삼각함수의 합차공식에 의해

(준식) = cosA cosB + cosA sinB + cosB sinA¡ sinA sinB= cos(A+B) + sin(A+B)

=p2 sin

³A+B +

¼

4

´· p2

따라서, A+B = ¼4 , A;B ¸ 0 일 때 최대값

p2 를 갖는다. }

별해 z = cosA+ i sinA, w = cosB + i sinB 라 두고 복소평면에서 생각하자. 0 · A;B · ¼

의 단위(반)원 위에서 준식은 zw의 실수부와 허수부의 합, 즉 x-좌표와 y-좌표의 합이다. 단

위(반)원과 직선 x+ y = k 가 만날 때 k의 최대값을 구하는 것이므로p2가 최대값임이 분명하

다. }

Baltic 1994-4Baltic 1994-4

pn¡ 1 +pn+ 1 가 유리수가 되도록 하는 정수 n이 존재하는가?

Page 107: Baltic Way 팀수학경시대회 1990-2005 pdf 보기

1994년 Baltic Way 풀이 107

풀이 (대전 전민중 3학년 김충은, 조금 수정됨)pn¡ 1 +pn+ 1 =

2pn+ 1¡pn¡ 1 = k(> 0) 라 하자.

pn+ 1 +

pn¡ 1 = k

pn+ 1¡pn¡ 1 = 2

k

이므로

pn+ 1 =

1

2

µk +

2

k

¶pn¡ 1 = 1

2

µk ¡ 2

k

¶k = p

q 의 유리수라 하면(p; q는 서로 소)

n =k2

4+1

k2=

p2

4q2+

q2

p2=

p4 + 4q4

4p2q2

이것이 정수가 된다면 q2 j p4, 그럼 서로 소임에서 q = 1, 즉 k = p 이어야 한다. 그럼 n = p4+44p2

.

다시 이것이 정수이려면 p2 j 4, 즉 p = 1 또는 2이다. 이 중에 어느 것을 대입해도 n이 정수가

되지 않으므로 모순. 따라서, 그런 정수 n은 존재하지 않는다. }

Baltic 1994-5Baltic 1994-5

p(x)는 정수계수 다항식이고 방정식 p(x) = 1과 p(x) = 3은 모두 정수해를 갖는다.방정식 p(x) = 2

가 서로 다른 두 정수해를 가질 수 있을까?

풀이 p(a) = 1, p(b) = 3, p(x) = 2 라 하자. 인수정리에 의해

(x¡ a) j p(x)¡ p(a) = 1; (x¡ b) j p(x)¡ p(b) = ¡1

이므로, jx¡ aj = jx¡ bj = 1 이다. a와 b는 서로 다른 정수이므로 a; b는 x보다 1만큼 크거나 작

은 두 수이고, 이 두 수 사이의 정수 x는 유일하므로 2개일 수 없다. }

Baltic 1994-6Baltic 1994-6

p가 양의 정수이고 q가 양의 홀수인 임의의 기약분수 pq는 적당한 양의 정수 n과 k에 대해서 분수

n2k¡1과 같음을 보여라.

증명 20; 21; 22; 23; : : : 를 q로 나눈 나머지는 같은 것이 있을 수 밖에 없으므로 언젠가부터 순

환하게 된다. 특히 2i+1 ´ 2j+1 (mod q) 이라면 2와 q가 서로 소인 것 때문에 2i ´ 2j (mod q)

이기도 하므로 순환은 맨처음부터 이루어져야 한다. 즉, 2k ´ 20 (mod q) 인 k가 존재하고, 그

럼 q j 2k ¡ 1, 즉2k ¡ 1 = qb

꼴이 된다. 그럼p

q=

pb

2k ¡ 1 이므로 증명 끝. ¤

주 오일러-페르마의 정리 2Á(q) ´ 1 (mod q) 를 안다면 더 간단히 정리할 수 있다.

Page 108: Baltic Way 팀수학경시대회 1990-2005 pdf 보기

108

Baltic 1994-7Baltic 1994-7

p > 2 는 소수이고

1 +1

23+1

33+ ¢ ¢ ¢+ 1

(p¡ 1)3 =m

n

에서 m과 n은 서로 소이다. m이 p의 배수임을 보여라.

증명 두 항씩 묶은 꼴을 생각하면

1

i3+

1

(p¡ i)3=(p¡ i)3 + i3

i3(p¡ i)3=

p(p2 ¡ 3pi+ 3i2)i3(p¡ i)3

로 항상 분자에는 p가 있고 분모에는 p가 없다. 준식은 이런 꼴들 여러 개를 합한 식이므로, 그

것을 통분하면 분모에는 여전히 p가 없고 분자는 p로 묶이므로 p를 인수로 갖는다. 따라서, 기

약분수 mn 으로 나타내더라도 분자 m은 p를 인수로 갖는다. ¤

Baltic 1994-8Baltic 1994-8

임의의 정수 a ¸ 5 에 대해, c ¸ b ¸ a 이고 a; b; c가 직각삼각형의 세 변의 길이가 되는 정수 b, c가

존재함을 보여라.

증명 a가 홀수(¸ 5)일 때는 b = a2¡12, c = a2+1

2 로 두면 c ¸ b ¸ a 와 a2+ b2 = c2 을 모두 잘

만족한다. a가 짝수(¸ 6)일 때는 b = a2

4¡ 1, c = a2

4+1 로 두면 역시 c ¸ b ¸ a 와 a2+ b2 = c2

을 모두 잘 만족한다. ¤

별증 a가 홀수(¸ 3)일 때는 앞의 증명에서처럼 b = a2¡12 , c = a2+1

2 로 두면 된다. 3 이상의

어떤 홀수를 약수로 갖는 짝수 a는 앞에서 구한 홀수일 때의 (a; b; c)에 상수배를 해주면 된다.

그럼 2의 거듭제곱들만 남는데, (8; 15; 17)이 해가 되므로 a가 8보다 큰 2의 거듭제곱이면 저기

에 상수배를 해주면 되므로, a ¸ 5 가 모두 해결되었다. ¤

Baltic 1994-9Baltic 1994-9

2a + 3b 이 어떤 정수의 제곱이 되는 양의 정수쌍 (a; b)를 모두 구하여라.

풀이 (대전 전민중 3학년 정명진, 수정됨)

a가 홀수이면 2a + 3b ´ 2 (mod 3) 이므로 완전제곱수가 될 수 없다(완전제곱수는 mod 3으로

0 또는 1). a는 짝수이므로 a = 2c 로 두자. b가 홀수이면 4c + 3b ´ 3 (mod 4) 이므로 완전제

곱수가 될 수 없다(완전제곱수는 mod 4로 0 또는 1). 따라서, b도 짝수이고, b = 2d 로 두자. 이

제 (2c)2 + (3d)2 = k2, 즉

4c = (k ¡ 3d)(k + 3d)

의 자연수해를 구하면 된다. k ¡ 3d = 2i, k + 3d = 2j , i+ j = 2c, i < j 라 둘 수 있다. 여기서

2 ¢ 3d = 2j ¡ 2i = 2i(2j¡i ¡ 1) 이 되므로 양변의 소인수 2의 개수를 생각하면 i = 1 이다. 즉,

Page 109: Baltic Way 팀수학경시대회 1990-2005 pdf 보기

1994년 Baltic Way 풀이 109

j = 2c¡ 1. 그럼 3d = 22c¡2 ¡ 1 = (2c¡1 ¡ 1)(2c¡1 + 1) 이 되고, 차이가 2인 두 수 2c¡1 ¡ 1 과

2c¡1 + 1 이 모두 3의 거듭제곱이므로 두 수는 1과 3이 될 수밖에 없다. 즉, c = 2, d = 1 이다.

즉, 만족하는 (a; b)는 유일하게 (4; 2) ¢ ¢ ¢ 답 }

Baltic 1994-10Baltic 1994-10

다음 세 조건을 만족하는 양의 정수는 모두 몇 개인가?

(a) 이 수의 각 자리수는 모두 집합 f1; 2; 3; 4; 5g 에서 고른 것이다.

(b) 연속하는 두 개의 자리수의 차는 1이다.

(c) 1994자리의 수이다.

풀이 (논산중 3학년 백승호, 조금 수정)

(a), (b)의 조건을 만족하는 n자리수 중에서 숫자 a로 시작하는 것의 개수를 fa(n)으로 쓰기로

하자. 우선 각 숫자에 대하여 다음에 올 수 있는 숫자를 살펴보자.

1! 2; 2! f1; 3g; 3! f2; 4g; 4! f3; 5g; 5! 4

1과 5, 그리고 2와 4 등은 대칭적인 성질이 있음을 알 수 있다. 즉, f5(n) = f1(n), f4(n) = f2(n)

이다. 따라서, 위의 대응에서 5와 4를 각각 1과 2로 대체하여 생각해도 되겠다.

1! 2; 2! f1; 3g; 3! f2; 2g

이제 2로 시작하는 수들을 모두 세어보자.

2 1

3

2

2

2

1

3

1

3

1

3

¢ ¢ ¢

반복적인 형태가 발견됨을 알 수 있다. 따라서,

f2(1 + 2m) = 3m; f2(2 + 2m) = 2 ¢ 3m (m = 0; 1; : : : )

이다. 그리고 앞의 대응으로부터

f1(n) = f2(n¡ 1); f3(n) = 2f2(n¡ 1) (n = 1; 2; : : : )

가 된다. 따라서, 우리가 구하는 개수는

f(1994) = f1(1994) + f2(1994) + f3(1994) + f4(1994) + f5(1994)

= 2f1(1994) + 2f2(1994) + f3(1994)

= 2f2(1994) + 4f2(1993)

= 4 ¢ 3996 + 4 ¢ 3996 = 8 ¢ 3996 ¢ ¢ ¢ 답

Page 110: Baltic Way 팀수학경시대회 1990-2005 pdf 보기

110

이 된다. }

별해 fa(n)을 앞의 풀이에서처럼 정의하자. f5 = f1, f4 = f2 이므로 f5와 f4는 대체가능하

고, 또 f3(n) = 2f2(n¡ 1) = 2f1(n) 이므로 f3도 대체가능하다. 그럼

f1(n) = f2(n¡ 1); f2(n) = f1(n¡ 1) + f3(n¡ 1) = 3f1(n¡ 1)

의 점화식이면 충분하고,

f(n) = f1(n) + f2(n) + f3(n) + f4(n) + f5(n) = 4f1(n) + 2f2(n)

임을 알 수 있다. 다시 앞의 점화식을 이용하면

f(n) = 4f1(n) + 2f2(n)

= 4f2(n¡ 1) + 6f1(n¡ 1)= 12f1(n¡ 2) + 6f2(n¡ 2)= 3f(n¡ 2)

를 얻을 수 있다.따라서, f(r+2m) = 3mf(r)이 되고,구하는 답은 f(1994) = 3996f(2) = 3996 ¢8¢ ¢ ¢ 답 }

Baltic 1994-11Baltic 1994-11

NS와 EW는 원 C의 서로 직교하는 두 지름이다. 직선 `이 원 C와 점 S에서 접한다. A, B는 C 위

의 점으로 지름 EW에 대해 서로 대칭이다. `이 두 직선 NA, NB와 각각 점 A0, B0에서 만난다.

jSA0j ¢ jSB0j = jSN j2 임을 보여라.

증명 (광주 동성고 2학년 방재혁, 부산 장전중 2학년 안현태)

S에 대한 B0의 대칭점 B00이라 하고, NB00과 원의 또 다른 교점을 X라 하자.

점 A를 FW에 대해 대칭시킨 후(점 B) 다시 NS에 대해 대칭시킨 점이 X이므로 A와 X는 원

의 대칭점이고 AX는 지름이다. 따라서, \A0NB00는 직각. 그럼 직각삼각형으로부터 jSA0j ¢jSB00j = jSN j2 이 되고, 이것은 준식과 동치이므로 성립한다. ¤

Page 111: Baltic Way 팀수학경시대회 1990-2005 pdf 보기

1994년 Baltic Way 풀이 111

Baltic 1994-12Baltic 1994-12

삼각형 A1A2A3의 내접원이 변 A2A3, A3A1, A1A2와 각각 점 S1, S2, S3에서 접한다. 삼각형

A1S2S3, A2S3S1, A3S1S2의 내접원의 중심을 각각 O1, O2, O3이라고 하자. 세 직선 O1S1, O2S2,

O3S3가 한 점에서 만남을 보여라.

증명 (광주 동성고 2학년 방재혁, 대전 탄방중 3학년 김민기, 수정됨)

4A1A2A3의 내심을 I라 하자. 먼저 O1, O2, O3가 내접원 I 위에 있음을 보이자.

IS1, IS3는 각각 A2A3, A1A2에 수직이므로 IS3A2S1은 원에 내접하는 사각형이다. 마찬가지

로 IS1A3S2도 원에 내접. 그럼

\S3S1S2 = \IS1S3 + \IS1S2

= \IA2S3 + \IA3S2

=\A2 + \A3

2=

¼ ¡ \A12

이다. 한편

\S3O1S2 = ¼ ¡ \O1S3S2 ¡ \O1S2S3

= ¼ ¡ \A1S3S2 + \A1S2S32

= ¼ ¡ ¼ ¡ \A12

=¼ +\A1

2

이다. 이로부터 \S3S1S2 + \S3O1S2 = ¼, 즉 O1이 내접원 I 위에 있음을 확인할 수 있다. O2,

O3도 마찬가지.

다음으로, O1S1, O2S2, O3S3가 4S1S2S3의 세 내각의 이등분선임을 보이자. 그럼 이 세 직선

은 4S1S2S3의 내심에서 만나게 된다. A1S3와 A1S2는 내접원 I에 접하고 A1O1은 각의 이등

분선이므로 A1O1에 대한 좌우대칭으로부터 호 O1S3와 O1S2는 길이가 같다. 따라서, 그 원주

각도 \O1S1S3 = \O1S1S2 로 같다. 따라서, O1S1은 \S3S1S2이 이등분선. O2S2, O3S3도 마

찬가지이고, 따라서 증명이 끝났다. ¤

Page 112: Baltic Way 팀수학경시대회 1990-2005 pdf 보기

112

Baltic 1994-13Baltic 1994-13

한 변의 길이가 a인 정사각형이 반지름 1인 다섯 개의 원을 포함하고 이 중 어느 두 원도 서로 겹치

지 않는다. a의 최소값을 구하여라.

풀이 a = 2 + 2p2 일 때 다음과 같이 가능하다.

이것이 최소임을 보이자. 이 정사각형 A의 각 변에서 거리 1만큼 안쪽으로 들어온 곳에 변을 갖

는 한 변의 길이가 a ¡ 2인 정사각형 B를 생각하자. 그럼 각 원의 중심은 모두 B의 내부(경계

포함)에 있다. 그리고, 그 다섯 중심은 서로 거리 2 이상 떨어져있다. 정사각형 B를 한 변의 길

이가 a¡22 인 작은 정사각형들로 4등분하자. 비둘기집의 원리에 의해 다섯 중심 중에서 어느 두

점은 하나의 작은 정사각형에 포함되고, 그 두 중심 사이의 거리는 a¡2p2

이하이다. 즉, 다음 부

등식을 얻는다.

a¡ 2p2¸ 2

따라서, a ¸ 2 + 2p2 가 된다. }

Baltic 1994-14Baltic 1994-14

®, ¯, °는 한 삼각형에서 각각 변 a, b, c에 대응하는 각이다. 다음 부등식을 증명하여라.

a ¢µ1

¯+1

°

¶+ b ¢

µ1

°+1

®

¶+ c ¢

µ1

®+1

¯

¶¸ 2 ¢

µa

®+

b

¯+

c

°

증명 대칭식이므로 a ¸ b ¸ c 라 해도 일반성을 잃지 않고, 그럼 ® ¸ ¯ ¸ ° 가 된다. 그럼 재

배열 부등식에서

a

¯+

b

®¸ a

®+

b

¯;

b

°+

c

¯¸ b

¯+

c

°;

c

®+

a

°¸ c

°+

a

®

이고, 이 세 부등식을 변변 합하면 문제의 부등식이 된다. ¤

Baltic 1994-15Baltic 1994-15

변의 길이와 수선의 길이가 모두 정수이고 둘레의 길이가 1995인 삼각형이 존재하는가?

Page 113: Baltic Way 팀수학경시대회 1990-2005 pdf 보기

1994년 Baltic Way 풀이 113

풀이 그런 삼각형 ABC가 있다고 가정하자. BC = a, CA = b, AB = c 라 하고(a+ b + c =

1995), A로부터의 수선의 길이를 h라 하자. 밑변과 높이에 의한, 그리고 헤론의 공식에 의한 삼

각형의 넓이에서

S =1

2af =

s1995

2

µ1995

2¡ a

¶µ1995

2¡ b

¶µ1995

2¡ c

¶양변에 4를 곱하고 제곱하면

1995(1995¡ 2a)(1995¡ 2b)(1995¡ 2c) = 4(af)2

이 된다. 여기서 좌변은 홀수, 우변은 짝수이므로 모순. 따라서, 그런 삼각형은 존재하지 않는

다. }

주 특히 문제의 조건 중에서 수선의 길이가 모두 정수일 필요도 없고, 세 수선 중 하나의 길

이가 정수라는 조건만으로도 이런 삼각형이 존재하지 않음을 알 수 있다.

Baltic 1994-16Baltic 1994-16

원더 아일랜드에는 고슴도치들이 살고있다. 이 고슴도치는 한 점을 공통의 끝점으로 하는 단위 길

이의 세 개의 바늘로 구성되어 있고, 이들 사이의 각은 모두 120±이다. 모든 고슴도치들이 이 섬에

평평하게 누워있고, 어느 두 마리도 서로 부딪치지 않는다. 원더 아일랜드에는 유한 마리의 고슴도

치들이 있음을 보여라.

증명 (수원 대평고 2학년 오정진, 수정됨)

적당한 양수 a에 대해, 두 고슴도치의 중심 사이의 거리는 항상 2a 이상임을 보이자. 한 고슴도

치 A의 각 바늘 위에 한 점씩 잡아 한 변의 길이가 1인 정삼각형 XY Z를 다음과 같이 그려보

자.

만일 다른 고슴도치의 중심 B가 이 정삼각형 내부에 들어온다면,일반성을 잃지 않고4AXY의

내부에 들어온다면, 고슴도치 B의 세 바늘 중 적어도 하나는 변 XY로부터 멀어지는(안쪽을

향하는) 방향에 있게 되므로, 이 바늘이 고슴도치 A의 바늘과 부딪히지 않으려면 4AXY 내부

에 완전히 포함되어야 한다. 그런데 4AXY 내부에 포함될 수 있는 선분은 길이가 1보다 작음

이 분명하므로 이것은 모순. 따라서, 다른 고슴도치의 중심은 4XY Z 내부에 있을 수 없다.

Page 114: Baltic Way 팀수학경시대회 1990-2005 pdf 보기

114

A를 중심으로 하고 4XY Z에 포함되는 반지름 2a인 원을 생각하면, 어떤 두 마리 고슴도치의

중심 사이의 거리도 2a 이상이 된다는 것이 되고, 이것은 다시, 각 고슴도치의 중심에 반지름

a인 원을 그리면 이런 원은 어떤 두 개도 서로 겹치지 않게 된다. 즉, 모든 고슴도치는 다른 고

슴도치와 겹치지 않는 일정한 넓이의 영역을 가지므로, 유한한 넓이의 섬에 무한 마리의 고슴

도치가 있을 수는 없다. ¤

Baltic 1994-17Baltic 1994-17

어떤 왕국의 왕이 13개의 무인도에 25개의 새로운 마을을 만들기로 결정했다. 각각의 섬에는 마을

이 적어도 하나씩 만들어져야 한다. 서로 다른 섬에 있는 두 마을 사이에는 직행 페리 노선을 설치

할 것이다. 설치해야 하는 노선의 최소 개수를 구하여라.

풀이 (대전 전민중 3학년 김충은)

각 섬에 만들어질 마을의 수를 a1; a2; : : : ; a13이라 하자. 같은 섬에 있는 두 마을 사이에만 노선

을 설치하지 않으므로 설치해야할 전체 노선의 수는¡252

¢¡P¡ai2 ¢ 가 된다. 이것의 최소값을 구

하는 것이므로P¡ai

2

¢를 최대로 해야 한다.

XÃai2

!=1

2

Xai(ai ¡ 1)

=1

2

Xa2i ¡ 1

2

Xai

=1

2(a21 + a22 + ¢ ¢ ¢+ a213)¡ 252

· 1

2(132 + 12 + ¢ ¢ ¢+ 12)¡ 25

2(¤)

=1

2(169 + 12¡ 25) = 78

이 된다. 여기서 부등식 (¤)은, a ¸ b ¸ 1 일 때 (a+ 1)2 + (b ¡ 1)2 ¸ a2 + b2 이므로, a+ b 가

일정할 때 a와 b의 차가 클수록 a2 + b2 이 더 커짐으로부터 얻었다. 따라서, 최소의 노선 수는¡252

¢¡ 78 = 300¡ 78 = 222개 ¢ ¢ ¢ 답 }

Baltic 1994-18Baltic 1994-18

n(> 2)개의 직선이 한 평면 위에 있다. 어느 두 직선도 평행하지 않고, 어느 세 직선도 한 점에서 만

나지 않는다. 두 직선이 만나는 모든 교점들에 1과 n¡ 1 사이의 자연수를 하나씩 매겼다. n이 짝수

일 때, 또 그 때만, 각각의 직선이 다른 직선들과 만나는 n¡ 1개의 교점에 1부터 n¡ 1 까지의 수를

모두 갖도록 수를 매기는 것이 가능함을 증명하여라.

증명 (한국과학영재학교 1학년 김건희)

1인 교점의 개수를 m이라 하면, 하나의 1은 두 직선에 나타나고 n개의 직선이 각각 하나의 1을

갖고있으므로 2m = n,즉짝수라야한다.직선들에 1부터 n까지번호를매기자. 1 · i; j · n¡1,i 6= j 인 두 직선 i와 j 사이의 교점에는 i + j mod n¡ 1 의 자연수를 매긴다(이 값이 0이면 0

Page 115: Baltic Way 팀수학경시대회 1990-2005 pdf 보기

1994년 Baltic Way 풀이 115

대신 n ¡ 1 을 매긴다). 그리고, 두 직선 i(· n ¡ 1) 와 n 사이의 교점에는 (위와 같이 했을 때

i와 i 사이에 매겨졌어야 했을) 2i mod n¡ 1 의 자연수를 매긴다. 그럼 n이 짝수일 때 각 직선

위의 교점들에 매겨지는 수가 1; 2; : : : ; n¡ 1 이고 모두 다르다는 것은 바로 확인된다. ¤

별증 직선을 vertex(꼭지점)로 보고, 두 직선의 교점을 두 vertex를 잇는 edge(변)로 보는

graph를 생각하기로 하자. 그럼 문제는 다음과 같은 것이 된다:

`완전그래프 Kn의 각 변에 1부터 n¡ 1 까지의 수를 하나씩 매기는데, 각각의 점마

다 그 점에 연결된 n ¡ 1개의 변에 1부터 n ¡ 1까지의 수가 꼭 하나씩 있도록 하는

것은 n이 짝수일 때만 가능하다.'

또, 각각의 i = 1; 2; : : : ; n¡ 1 에 대해, i를 매긴 변만 모두 모으면 모든 점을 둘씩 짝짓기한 집

합이 되므로,

`Kn의 모든 변을 n¡ 1가지의 색으로 칠하는데, 각각의 색의 변을 모두 모으면 모든

점을 둘씩 짝짓기한 것이 되도록 하자.'

는 문제로 볼 수도 있다. 이런 문제로 생각하기로 하자. 이상의 고찰에서 n이 짝수여야 한다는

것은 분명하다(다시 검토하자면, 1을 매긴 변이 k개라면, 각 점마다 k인 변이 꼭 하나씩 연결되

어 있고 각 변마다 두 점이 연결되어 있으므로 점은 모두 2k개가 된다). 이제 n이 짝수이면 항

상 가능함을 보이자. 먼저 다음의 주장을 확인하자.

Claim 1 n일 때 가능하면 2n일 때도 가능하다.

Claim 1의 증명 2n개의 꼭지점을 임의로 둘씩 묶어 n개의 조 A1; : : : ; An을 만들자. 점

이 n개일 때 가능하므로 우선 Ai들을 점으로 보고 조건을 만족하는 그래프 G를 그리

자. 이로부터 2n개의 꼭지점에 해당하는 그래프 G0을 만드는데, 다음과 같이 하면 된다:

Ai = fai; big 라 하자(i = 1; : : : ; n). G의 각 색깔 c마다 G0에는 2가지 색깔 c=, c£를 부

여하는데, G에서 변 AiAj가 c의 색깔이라면 G0에서 변 aiaj , bibj는 c=의 색깔로 하고 변

aibj , biaj는 c£의 색깔로 한다.

그리고, 각각의 Ai마다 변 aibi에는 2n¡1번째 색깔인 d를 칠한다.그럼 이렇게 만든 G0도

문제의 조건을 잘 만족함을 쉽게 알 수 있다. ¤

Claim 1에 의해, 모든 짝수 n에 대해 성립함을 보이는 대신, 다음만 보이면 충분하다.

Claim 2 n = 2k (k는 홀수) 꼴일 때 항상 가능하다.

Page 116: Baltic Way 팀수학경시대회 1990-2005 pdf 보기

116

증명 n개의 점을 한 원의 둘레 위에 일정한 간격이 되도록 배열하고, 서로 옆으로 i번

째에 있는 두 점을 간격 i에 있다고 말하자(i · k). 즉, 이웃한 두 점은 1인 간격에 있다.

먼저 각각의 주대각선(가장 긴, 즉 간격이 k인 대각선) AB에 대해 그것과 수직인 변을 모

두 모아 한 가지 색깔로 칠하자.

AB를 택하는 방법이 k가지이므로 여기에 k가지 색깔이 쓰이게 되고, 주대각선 외에는 간

격이 짝수인 변들만 모두 색칠이 되었다. 이제 간격이 (k 이외의) 홀수인 변들만 색칠하

면 된다. 임의의 홀수 j(< k) 에 대해, 간격이 j인 변을 모두 모으면 2n개이고 이들은 하

나 이상의 별꼴정다각형을 구성한다.

한 점에서 출발하여 같은 방향으로 계속 간격 j로 움직이다보면 언젠가 원래의 점에 도착

하는데, b번만에 도착했다면 그동안 움직인 거리(간격)가 jb이고 이것은 n의 배수가 되어

야 하므로 n j jb, 여기서 n은 짝수, j는 홀수이므로 b는 반드시 짝수이고, 따라서 이 정다

각형은 짝수각형이다. 그럼 이 정다각형들을 이웃하는 변이 서로 다른 색깔이 되도록 두

가지 색으로 칠할 수 있다. 이렇게 각각의 j마다 2가지 색을 사용하므로 간격이 (k 이외

의) 홀수인 변들에 k ¡ 1가지 색깔이 쓰이게 되고, 각 색깔의 변은 모든 점을 둘씩 짝지은

것이 틀림없다. 이렇게 모두 2k ¡ 1 = n¡ 1가지 색깔을 사용하여 잘 색칠하는 것이 가능

함을 알 수 있다. ¤

모든 짝수는 2t ¢ k (k는 홀수, t ¸ 1) 꼴이고, Claim 2에서 2k가 가능하므로 다시 Claim 1에 의

해 22k; 23k; : : : 들도 모두 가능하게 된다. 따라서, 모든 짝수 n에 대해 가능하다. ¤

Baltic 1994-19Baltic 1994-19

원더 아일랜드의 정보부는 Tartu에 16명의 스파이를 두고 있다. 이들 각각은 그들 동료 몇 명을 감

시하고 있다. 만약 스파이 A가 스파이 B를 감시하고 있다면, B는 A를 감시하고 있지 않다. 또한,

어떤 10명의 스파이를 택해도, 첫 번째 스파이가 두 번째 스파이를 감시하고, 두 번째가 세 번째를,

: : : ; 열 번째가 첫 번째를 감시하고 있도록 번호를 붙일 수 있다. 임의의 11명의 스파이도 이와 비슷

하게 번호를 붙일 수 있음을 보여라.

Page 117: Baltic Way 팀수학경시대회 1990-2005 pdf 보기

1994년 Baltic Way 풀이 117

증명 (대전과학고 1학년 진태진)

16명 중에 임의로 한 명을 골라 a라 하자. a 외에 9명을 더 뽑아 10명의 그룹을 만들면(현재 6명

이 남아있다) 그중에 a가 감시하는 스파이 x1이 있다. x1을 그룹에서 제외시키고 남은 5명 중

한 명을 그룹에 새로 포함하면 그중에 a가 감시하는 또다른 스파이 x2가 있다. 이런 식으로 계

속하면 a가 감시하는 스파이가 적어도 7명 있음을 알 수 있다. 비슷한 방법으로 a를 감시하는

스파이도 7명 이상 있다. a는 임의로 선택했던 것이므로, 사실은 모든 스파이에게 그가 감시하

는 스파이와 그를 감시하는 스파이가 각각 7명 이상 있다. 두 스파이가 서로를 감시할 수는 없

으므로, 임의의 스파이 a에 대해 a가 감시하는 7명과 a를 감시하는 7명, 그리고 아직은 a와의

관계가 있는지 어떤지 잘 파악이 안 되는 1명이 있다고 생각할 수 있다.

이제 임의의 스파이 11명에 대해서 생각하자. 그중 임의로 한 명을 택해 a라 하고, a를 제외한

10명으로 x1 ! x2 ! ¢ ¢ ¢ ! x10 ! x1 의 그룹을 만들자.

(1) a가 이 10명 모두와 관계가 있는(감시하거나 감시당하거나) 경우:

a가 10명을 모두 감시할 수도 없고 10명이 모두 a를 감시할 수도 없다. 그럼 일반성을 잃

지 않고 x1이 a를 감시하는 스파이라고 하고 x2를 검사하자. x2가 a가 감시하는 스파이라

면 x1과 x2 사이에 a를 끼워넣으면 11명의 그룹이 된다. x2가 a를 감시하는 스파이라면

다음으로 x3을 검사하자. x3이 a가 감시하는 스파이라면 x2와 x3 사이에 a를 끼워넣으면

된다. x3이 a를 감시하는 스파이더라도 이런 식으로 계속하면 언젠가 a가 감시하는 첫 번

째 스파이 xi가 발견될 수밖에 없고, 그럼 xi의 앞에 a를 끼워넣으면 된다.

(2) 이 10명 중에 a와 관계가 없는 자가 있는 경우:

굳이 처음 택한 a가 아니더라도 11명 중에 나머지 10명과 모두 관계가 있는 스파이가 누

군가 있으면 그를 a로 하고 (1)처럼 하면 된다. 따라서, 11명 모두가 나머지 10명 중 정확

히 9명과 관계가 있는 경우만 살피면 된다. 그럼, 이 11명 중에서 관계가 있는 두 명의 쌍

의 수는 11¢92 가 되어야 하는데 이것은 정수가 아니므로 모순.

따라서, 임의의 11명의 스파이도 항상 차례로 감시하는 그룹으로 만들 수 있다. ¤

Baltic 1994-20Baltic 1994-20

한 정삼각형을 각각의 변에 평행한 직선들에 의해 9000000개의 합동인 정삼각형으로 나누었다. 작

은 삼각형들의 각 꼭지점은 세 가지 색 중 하나로 색칠된다. 원래 삼각형의 변에 평행한 변을 갖는

삼각형의 꼭지점이 되는 같은 색의 세 점이 존재함을 보여라.

증명 (서울 당산서중 2학년 박민재)

문제의 조건을 만족하는 세 점이 존재하지 않는다고 가정하자. 처음 정삼각형 XY Z의 한 변

XY를 살펴보자.이 변은 3000개의 선분들로 나뉘어져 있으므로 XY 위에는 3001개의 꼭지점이

있다. 비둘기집의 원리에 의해 이 3001개의 꼭지점 중에는 어떤 같은 색의 꼭지점들이 1001개

Page 118: Baltic Way 팀수학경시대회 1990-2005 pdf 보기

118

이상 있다.

이러한 꼭지점들의 집합을 A0이라고 하고 a라는 색으로 칠했다고 하자.

jA0j ¸ 1001

이제 A0에 포함되는 두 점을 꼭지점으로 하는 정삼각형을 생각하면 그런 정삼각형은¡10012

¢개

이상 있다. 이런 식으로 만들어진 작은 정삼각형의 꼭지점 중 A0에 포함되지 않는 점들의 집합

을 B라고 하자. B의 원소의 개수 역시

jBj ¸Ã1001

2

!= 500500

이다. 그리고, B의 점들은 색이 a일 수 없다. 비둘기집의 원리에 의해서, XY에 평행한 어떤 직

선 L이 존재해서 직선 L 위에는 B에 속하는 꼭지점의 개수가 최소한 d 5005003000 e = 167개 있을 것

이다. 이 점들은 a가 아닌 나머지 두 가지 색으로 칠해져야만 하므로, 다시 비둘기집의 원리에

의해서 직선 L 위에 최소한 d 1672 e = 84개의 점이 같은 색(b라 하자)일 것이다. 이 점들의 집합

을 B0이라고 하자.

jB0j ¸ 84

B0에 포함되는 두 점을 꼭지점으로 하는 바른(역삼각형이 아닌) 정삼각형은 최소한¡842

¢=

3486개 만들 수 있다. 이 정삼각형들의 B0에 포함되지 않는 꼭지점들의 집합을 C라고 하자.

여기서 C가 B에 포함됨은 자명하다. (C에서 B0의 두 원소를 이은 직선을 연장하면 XY 위의

A0의 두 꼭지점과 만날 것이므로) 그리고

jCj ¸Ã84

2

!= 3486

이다. 이것은 3000보다 크므로, 이제 비둘기집의 원리에 의해서, XY와 평행하고 C의 점을 2개

이상 포함하는 어떤 직선 L0이 존재할 것이다. 이 두 점은 색이 a나 b일 수 없다. 그러면 마지막

남은 색(c라 하자)이어야 한다. 그리고 다시 이 두 점을 꼭지점으로 하는 바른 정삼각형을 만들

고, 나머지 한 꼭지점을 D라 하자. 그럼 D는 a, b, c 중 어느 색도 될 수 없다. 이는 모순이다.

Page 119: Baltic Way 팀수학경시대회 1990-2005 pdf 보기

1994년 Baltic Way 풀이 119

즉, 어떤 식으로든 세 꼭지점의 색이 같고 원래 정삼각형과 평행인 작은 정삼각형이 존재한다.

특히, 바른 정삼각형 중에서만 찾아도 존재한다. ¤

Page 120: Baltic Way 팀수학경시대회 1990-2005 pdf 보기

120

1995년 Baltic Way 풀이

Baltic 1995-1Baltic 1995-1

다음 연립방정식을 만족하는 모든 양의 정수해 (x; y; z)를 찾아라.8><>:x2 = 2(y + z)

x6 = y6 + z6 + 31(y2 + z2)

풀이 (고양 백신고 2학년 신동주 등)

뒷식에서 x > y; z. 앞식에서 x는 짝수이고 x2 = 2(y + z) < 4x, 즉 x < 4. 따라서 x = 2 뿐이

다. 앞식에서 y + z = 2 이므로 y = z = 1 뿐이다. (x; y; z) = (2; 1; 1) 을 뒷식에 대입했을 때 잘

만족하므로 이것이 답이 된다. }

Baltic 1995-2Baltic 1995-2

a와 k는 양의 정수이고, a2 + k 가 (a¡ 1)a(a+ 1) 의 약수가 된다. k ¸ a 임을 보여라.

증명 (서울 환일고 2학년 김태우)

(a¡ 1)a(a+ 1) = a(a2 + k)¡ (k + 1)a

이므로 a2 + k j (a ¡ 1)a(a + 1) 은 a2 + k j (k + 1)a 와 동치이다. (k + 1)a는 양수이므로

a2 + k · (k + 1)a 라야 한다. 이것은 (a¡ 1)(a¡ k) · 0 으로 인수분해되므로 1 · a · k 이다.

¤

증명 (대전 전민중 3학년 정명진)

a = 1 일 때는 당연하므로 a > 1 일 때만 보자. 귀류법으로 k < a 라 하자. 그럼

a2 ¡ 1 < a2 + k < a2 + a

이고, 적당한 정수 x에 대해

(a2 ¡ 1)£ a = (a2 + k)£ x = (a2 + a)£ (a¡ 1) = (a¡ 1)a(a+ 1)

이 된다. 여기서 모든 항이 다 양수이므로 a¡ 1 < x < a 가 되어야 하는데 그럼 x가 정수일 수

없으므로 모순. 따라서 k ¸ a 이다. ¤

Baltic 1995-3Baltic 1995-3

a, b, c는 각각 서로 소인 양의 정수들이다. 또, a와 c는 홀수이고, a2+ b2 = c2 이 성립한다. b+ c 가

어떤 정수의 제곱임을 보여라.

Page 121: Baltic Way 팀수학경시대회 1990-2005 pdf 보기

1995년 Baltic Way 풀이 121

증명 (부산 여명중 3학년 박성민 등)

a, c가 홀수이므로 b는 짝수이다.

a2 = c2 ¡ b2 = (c+ b)(c¡ b)

에서 c+ b = p 와 c¡ b = q 가 서로 소이면 b+ c 는 완전제곱수가 된다. 두 홀수 p, q의 최대공

약수를 G라 하자. 그럼 G도 홀수.

G j p+ q = 2c; G j p¡ q = 2b

에서 G는 홀수이므로 G j c, G j b 이다. b와 c는 서로 소라고 했으므로 G = 1, 따라서 p와 q도

서로 소이고, b+ c 는 완전제곱수이다. ¤

Baltic 1995-4Baltic 1995-4

J씨는 M씨보다 나이가 많다. J씨는 자신의 나이(정수)의 두 자리수를 바꾼다면 M씨의 나이가 된다

는 것을 발견했다. 또한, 그들의 나이의 제곱의 차는 어떤 정수의 제곱이다. M씨와 J씨의 나이는?

풀이 (북서울중 3학년 류종하 등)

J씨의 나이를 ab라 하자. M씨의 나이는 ba. 그리고

(10a+ b)2 ¡ (10b+ a)2 = 99(a2 ¡ b2) = c2

이다. 즉, a2 ¡ b2 = 11k2 꼴이다. 11 j (a+ b)(a¡ b) 에서 a, b는 1에서 9 사이의 한 자리 수이고

a > b 이므로 a+ b = 11 인 경우뿐이다. (a; b) = (9; 2), (8; 3), (7; 4), (6; 5) 중에서 a¡ b = k2 꼴

인 것은 (6; 5)뿐이다. 따라서, M씨와 J씨는 각각 56, 65세 ¢ ¢ ¢ 답 }

Baltic 1995-5Baltic 1995-5

a < b < c 는 세 개의 양의 정수이다. 연속하는 2c개의 양의 정수 중에는, abc가 xyz를 나누는 서로

다른 세 수 x, y, z가 존재함을 보여라.

증명 먼저 연속한 b개의 정수 중에서는 ab j xy 인 서로 다른 두 수 x, y를 항상 찾을 수 있음

을 보이자. 연속한 b개의 정수 중에는 b j x, a j y0 인 x, y0이 틀림없이 있다. x 6= y0 이면 y = y0

으로 택하면 끝이다. x = y0 일 때를 보자. m = lcm(a; b) 라 하면 m j x 이다. d = gcd(a; b) 라

하면 a < b 이므로 d · 12b 이다. 따라서, 연속한 b개의 정수 중에 d j y 인 y 6= x 가 존재한다.

그럼 ab = md j xy 이다.

연속한 2c개의 자연수를 연속한 c개씩 두 그룹으로 나누자. 그럼 먼저 증명했던 사실에 따라,

한 그룹에서 ab j xy 인 서로 다른 두 수 x, y를 찾을 수 있다. 또한, 나머지 한 그룹에서 c j z 인

z를 찾을 수 있다. 따라서, abc j xyz 인 x, y, z를 찾을 수 있다. ¤

Page 122: Baltic Way 팀수학경시대회 1990-2005 pdf 보기

122

Baltic 1995-6Baltic 1995-6

양수 a; b; c; d 에 대해 다음이 성립함을 보여라.

a+ c

a+ b+

b+ d

b+ c+

c+ a

c+ d+

d+ b

d+ a¸ 4

증명 (성남 수내중 3학년 임동혁)

산술-조화 평균 부등식에서

a+ c

a+ b+

a+ c

c+ d¸ 4 ¢ a+ c

a+ b+ c+ db+ d

b+ c+

b+ d

a+ d¸ 4 ¢ b+ d

a+ b+ c+ d

두 식을 변변 더하면 본제가 성립. ¤

Baltic 1995-7Baltic 1995-7

sin3 18± + sin2 18± = 1=8 임을 증명하여라.

증명1 (광주 동성고 2학년 방재혁)

그림과 같이 꼭지각 \A = \CBD = 36± 인 두 이등변삼각형 ABC와 BCD를 그리고, B가

EC의 중점이 되도록 E를 잡자.

AB = 1, AD = x 라 두면, x = AD = BD = BC = EB 이고 AB : BC = BC : CD 로부터

CD = x2 이다. AC = AD +DC 로부터

x2 + x = 1

\CDE = 72± + 18± = \R 이므로 k = sin 18± = sin\E = CDEC =

x2

2x =x2 . 이로부터

sin3 18± + sin2 18± = k3 + k2 =x3

8+

x2

4=

x(x2 + x) + x2

8=

x+ x2

8=1

8

임을 얻을 수 있다. ¤

Page 123: Baltic Way 팀수학경시대회 1990-2005 pdf 보기

1995년 Baltic Way 풀이 123

증명2 (부산 건국고 2학년 신승현)

S = sin3 18± + sin2 18± 로 두자. 2배각 공식에 의해 S ¢ cos2 18± = 14 sin

2 36±(1 + sin 18±).

S =sin2 36±(1 + sin 18±)

4(1¡ sin 18±)(1 + sin 18±) =1¡ cos 72±8(1¡ sin 18±) =

1

8

이 된다. ¤

Baltic 1995-8Baltic 1995-8

실수 a, b, c가 jaj ¸ jb + cj, jbj ¸ jc+ aj, jcj ¸ ja + bj 의 부등식들을 만족한다. a + b + c = 0 임을

보여라.

증명 (대전 전민중 3학년 김충은 등)

각 부등식을 제곱하면

a2 ¸ b2 + 2bc+ c2

b2 ¸ c2 + 2ca+ a2

c2 ¸ a2 + 2ab+ b2

변변 더하고 a2 + b2 + c2 을 하나씩 소거하면

0 ¸ a2 + b2 + c2 + 2bc+ 2ca+ 2ab = (a+ b+ c)2

따라서, a+ b+ c = 0 이다. ¤

Baltic 1995-9Baltic 1995-9

다음을 증명하여라.

1995

2¡ 1994

3+1993

4¡ ¢ ¢ ¢ ¡ 2

1995+

1

1996=

1

999+

3

1000+ ¢ ¢ ¢+ 1995

1996

증명 (대전 탄방중 3학년 김민기, 수정됨)

1997¡ k

k¡ 1997¡ (k + 1)

k + 1= 1997

µ1

k¡ 1

k + 1

¶1997¡ 2k1997¡ k

= 2¡ 1997

1997¡ k

Page 124: Baltic Way 팀수학경시대회 1990-2005 pdf 보기

124

임을 이용해 양변을 간단히 해보자.

우변 = 2 ¢ 998¡ 1997 ¢µ1

999+

1

1000+ ¢ ¢ ¢+ 1

1996

¶= 1997¡ 1997 ¢

µ1

999+ ¢ ¢ ¢+ 1

1996+

1

1997

¶좌변 = 1997

µ1

2¡ 13+1

4¡ 15+ ¢ ¢ ¢+ 1

1996¡ 1

1997

¶= 1997

µ2 ¢µ1

2+1

4+ ¢ ¢ ¢+ 1

1996

¶¡µ1

2+1

3+ ¢ ¢ ¢+ 1

1997

¶¶= 1997

µµ1 +

1

2+ ¢ ¢ ¢+ 1

998

¶¡µ1

2+1

3+ ¢ ¢ ¢+ 1

1997

¶¶= 1997¡ 1997 ¢

µ1

999+ ¢ ¢ ¢+ 1

1997

¶로 양변이 같음을 알 수 있다. ¤

Baltic 1995-10Baltic 1995-10

0이 아닌 모든 실수의 집합에 정의되고 실수값을 가지며 다음 조건들을 만족하는 함수 f를 모두 찾

아라.

(i) f(1) = 1

(ii) f( 1x+y ) = f( 1x ) + f( 1y ) (0이 아닌 모든 x, y, x+ y에 대해서)

(iii) (x+ y) ¢ f(x+ y) = xy ¢ f(x) ¢ f(y) (0이 아닌 모든 x, y, x+ y에 대해서)

풀이 (수원 대평고 2학년 오정진, 수정됨)

(ii)에 x, y 대신 1x 를 대입하면

f³x2

´= 2f(x) (1)

(iii)에 x, y 대신 x2 를 대입하면

xf(x) =x2

4f³x2

´2(2)

(1)을 (2)에 대입하면 xf(x) = x2f(x)2. x 6= 0 이므로 f(x) = xf(x)2, 즉

f(x) = 0 또는1

x

이다. 만일 f(a) = 0 인 a가 있으면, (iii)에서 (x; y) = (a; 1 ¡ a) 를 대입하면 f(1) = 0 이 되어

(1)에 모순. 따라서,

f(x) ´ 1

x

뿐이다. 그리고 이것은 (1){(3)을 모두 잘 만족함을 확인할 수 있다. }

Baltic 1995-11Baltic 1995-11

집합 f1; 2; : : : ; 1995g를 공집합이 아닌 세 집합으로 분할하는데, 이들 중 어느 것도 연속하는 정수

쌍을 포함하지 않도록 하는 방법은 모두 몇 가지인가?

Page 125: Baltic Way 팀수학경시대회 1990-2005 pdf 보기

1995년 Baltic Way 풀이 125

풀이 1과 2는 같은 집합에 속할 수 없으므로, 세 집합은 항상 1을 포함하는 집합(A), 2를 포

함하는 집합(B), 나머지 한 집합(C)로 구분할 수 있다. 3은 B에만 넣을 수 없고 A나 C에 넣

을 수 있다. 그 다음 4는 3을 넣은 집합을 제외한 나머지 두 집합에 넣을 수 있다. 그 다음 5는

4를 넣은 집합을 제외한 나머지 두 집합에 넣을 수 있다. 이렇게 항상 두 가지 경우씩 생기면

서 1995까지 계속되므로, 모두 21993가지의 경우가 있다. 단, C가 공집합이 되는 유일한 경우

A = f1; 3; : : : ; 1995g, B = f2; 4; : : : ; 1994g 일 때는 제외해야 하므로 구하는 방법은 모두 ¢ ¢ ¢답 21993 ¡ 1 가지 }

Baltic 1995-12Baltic 1995-12

95개의 상자에 19개의 공을 임의로 나누어 넣었다고 하자. 6개의 새로운 공을 가져와서 6개의 상자

를 택해 각각 하나씩 넣는다. 이러한 방법을 여러 번 반복하여, 95개의 상자에 모두 똑같은 개수의

공이 있도록 할 수 있겠는가?

풀이1 (부산 건국고 2학년 신승현)

16번의 작업이면 96개의 공을 넣게 되므로, 95개의 상자 중 한 곳에만 2개를 넣고 나머지 모든

상자에는 1개씩 넣을 수 있다. 즉, 16번씩의 작업으로 어떤 상자의 공의 개수를 다른 상자의 공

의 개수보다 증가량이 꼭 1이 더 많도록 할 수 있다. 공의 개수가 최대가 아닌 상자를 이런 작

업으로 다른 상자보다 하나씩 공의 개수가 더 증가되도록 하면, 유한 번 안에 모든 상자의 공의

개수가 똑같아진다. }

풀이2 (수원 대평고 2학년 오정진)

공이 들어있는 상자는 많아야 19개이다. 비어있는 상자 5개(A그룹)를 선택한다.

[작업1] 6개의 새로운 공을 가져와 넣을 때 5개는 이 선택된 상자에 하나씩 넣고 나머지 하나는

나머지 90개의 상자(B그룹)중 하나에 넣는데, 이 90개의 상자의 공의 개수가 모두 똑같아질 때

까지 계속한다. 이 작업을 완료했을 때 이 90개의 상자에 k개씩의 공이 있게 된다고 하자. 그럼

B그룹 전체에는 90k개의 공이 있는 것이고, 맨처음에는 B그룹에 19개의 공이 있었고 매 작업

마다 B그룹에 1개씩 그리고 A그룹의 각 상자에 1개씩 공이 추가되었으므로, 이 작업을 완료했

을 때 A그룹의 상자에는 각각 90k ¡ 19개씩의 공이 있게 된다.

[작업2] 이제 6개의 공을 15번 가져오면 90개의 상자에 공을 모두 1개씩 추가할 수 있다. 이 작

업을 B그룹의 90개의 상자 모두 90k ¡ 19개의 공을 갖게 될 때까지 계속하면 된다. }

Baltic 1995-13Baltic 1995-13

다음 이인용 게임을 생각해보자. 적당량의 공기돌이 탁자 위에 놓여있다. 두 명의 선수가 교대로,

적당한 양의 정수의 제곱 개의 공기돌을 탁자에서 가져간다. 돌을 더 이상 가져갈 수 없게 되는 차

례의 선수가 게임에서 진다. 첫 번째 선수가 어떻게 경기해도 두 번째 선수가 항상 이길 수 있는 초

기 상황(처음 공기돌의 개수)이 무한히 많이 있음을 보여라.

Page 126: Baltic Way 팀수학경시대회 1990-2005 pdf 보기

126

증명 (성남 수내중 3학년 임동혁)

만약 유한하다고 가정하면, 어떤 n보다 큰 수에 대해선 모두 선수 필승. 그 n을 잡고, n2 + 2n

을 보자. 이 수는 (n+ 1)2보다 작다. 선수가 아무리 많이 가져가봐야 n2개이고, 그럼 후수의 차

례에 2n개 이상의 돌이 테이블이 남게 된다. 가정에서 n보다 큰 수에 대해서는 모두 선수 필승

이므로, 이 상태에서 시작하는 것은 후수의 사람이므로 후수가 이기게 된다. 따라서 모순. ¤

Baltic 1995-14Baltic 1995-14

정삼각형 격자가 그려진 무한히 큰 종이 위에 n마리의 벼룩이 있다. 처음에는 벼룩들이 n2개의 작

은 삼각형들로 이루어진 어떤 큰 정삼각형 안에 있는 서로 다른 삼각형 안에 있다. 벼룩들은 1초에

한 번씩 뛰어, 자기가 있던 삼각형과 한 꼭지점만을 공유하며 마주보는 이웃한 세 삼각형 중 하나

로 각각 옮겨간다. 양의 정수 n이 어떤 수일 때, 유한 번 뛰어서 모든 n마리의 벼룩들이 하나의 작

은 삼각형에서 모두 모이게 될 수 있는 초기 배치가 존재하겠는가?

풀이 한 변의 길이가 3인 아래의 삼각형에서 °표된 세 위치에 있는 벼룩들은 두 번 뛰어 이

중 한 위치에 모두 모일 수 있다.

이것을 응용하면 다음과 같은 위치에 있는 벼룩들은 유한 번 뛰어서 모두 이 중 한 위치로 모두

모일 수 있음을 알 수 있다.

이런 위치의 개수를 p(n)이라 하면, p(n) ¸ n 일 때는 이런 위치 중에서 n곳을 골라 벼룩을 배

치하면 된다. n이 홀수일 때

p(n) = 1 + 2 + ¢ ¢ ¢+ n+ 1

2=(n+ 1)(n+ 3)

8

이고, 따라서 n이 5 이상의 홀수이면 p(n) ¸ n + 1 ¸ n 이다. n이 6 이상의 짝수일 때도

p(n) = p(n¡ 1) ¸ n 으로 성립한다. n = 3 일 때도 p(3) = 3 으로 성립하고, n = 1 일 때는 성

Page 127: Baltic Way 팀수학경시대회 1990-2005 pdf 보기

1995년 Baltic Way 풀이 127

립함이 자명하다. 따라서, n = 2; 4 일 때만 살피면 되는데, 이 때에는 불가능함을 증명하겠다.

정삼각형 격자의 모든 위치를 그림과 같이 분류하자. A에서 한 번 뛰면 항상 A0의 위치로 옮겨

가고, A0에서 한 번 뛰면 항상 A의 위치로 옮겨간다. (B;B0), (C;C0), (D;D0)들도 마찬가지로

그렇게 서로 한 번 뛰어 옮겨가게 되는 짝이다. 따라서, 이 8종류의 위치 중 서로 다른 위치에

있던 벼룩은 아무리 뛰어도 같은 위치로 모일 수가 없다. 즉, 유한 번 안에 한 칸에 모두 모이려

면 처음에 모두 같은 종류의 같에 놓여있어야 한다. n = 2 일 때와 n = 4 일 때의 큰 정삼각형

의 각 칸들은 이 8가지로 분류하면 다음과 같다.

n = 2 일 때는 모든 칸이 모두 다른 종류이므로 같은 종류의 칸에 벼룩을 2마리 놓을 수 없고,

n = 4 일 때에도 같은 종류의 칸이 최대 3개씩뿐이므로 4마리의 벼룩을 같은 종류의 칸에 놓을

수 없다. 따라서, n = 2; 4 일 때는 불가능하다. }

Baltic 1995-15Baltic 1995-15

2n+ 1개의 꼭지점을 가지고 있는 다각형이 있다. 각 꼭지점과 변의 중점들에 1부터 4n+ 2 까지의

자연수를 하나씩 붙여, 각 변에 붙여진 세 수의 합이 모두 같아지게 할 수 있음을 보여라.

증명1 (대전 전민중 3학년 정명진)

다각형의 꼭지점에 한 점씩 건너뛰며 1부터 차례로 번호를 붙이면 그림과 같이 된다.

Page 128: Baltic Way 팀수학경시대회 1990-2005 pdf 보기

128

여기까지에서 각 변에 붙여진 두 수의 합은

(n+ 1) + 1 < 1 + (n+ 2) < (n+ 2) + 2 < ¢ ¢ ¢ < (2n+ 1) + (n+ 1)

로 차례로 꼭 1씩 커지는 것을 관찰할 수 있다. (n+ 1 의 점을 제외한 각 점은 그 오른쪽 점에

왼쪽 점보다 1이 큰 수가 배정되어 있으므로 이것은 쉽게 확인된다.) 따라서, 이 변들의 중점에

차례로

4n+ 2 > 4n+ 1 > 4n > ¢ ¢ ¢ > 2n+ 2

을 붙이면 각 변의 세 수의 합이 모두 5n+ 4 로 똑같아진다. ¤

주 방재혁 학생도 이와 거의 비슷한데 숫자가 조금 다른 방법을 사용했다.

먼저 각 꼭지점에 한 점씩 건너뛰며 1; 2; 3; : : : ; 2n+1 대신 1; 3; 5; : : : ; 4n+2 의 번호를 붙인다.

그럼 여기까지에서 각 변에 붙여진 두 수의 합은 꼭 2씩 커진다. 이제 나머지 짝수들을 각 변의

중점에 2씩 짝아지는 순서로 차례로 붙이면 된다.

Baltic 1995-16Baltic 1995-16

삼각형 ABC에서 C의 외각의 이등분선을 `이라 하자. AB의 중점 O를 지나고 `에 평행한 직선이

AC와 E에서 만난다. jACj = 7 이고 jCBj = 4 일 때, jCEj의 길이를 구하여라.

풀이1 (마산 가포고 2학년 신상 등)

O를 지나고 `에 평행한 직선이 BC의 연장선과 만나는 점을 P라 하고, O에서 BC에 평행한 선

을 그어 AC와 만나는 점을 D라 하자.

C의 외각의 절반을 ®라 할 때 \CEP = \EPC = \EOD = ®. 그럼 ED = OD = 12BC = 2.

따라서, CE = CD +DE = 12AC +DE = 7

2+ 2 = 11

2¢ ¢ ¢ 답 }

풀이2 (고양 백석고 2학년 조상영)

C의 외각의 절반을 a라 하자. B를 지나고 `에 평행한 직선 k를 그리고, k와 AC가 만나는 점을

Page 129: Baltic Way 팀수학경시대회 1990-2005 pdf 보기

1995년 Baltic Way 풀이 129

X라 하자. 그리고, C에서 k에 내린 수선의 발을 Y라 하면 CY는 \C의 이등분선이다.

그럼 CX = BC = 4 이고 이로부터 AX = 7 ¡ CX = 3 이 된다. E는 AX의 중점이므로

XE = 12AX = 3

2 . 따라서, CE = CX +XE = 4 + 32 =

112 ¢ ¢ ¢ 답 }

Baltic 1995-17Baltic 1995-17

모든 삼각형 ABC에 대해 다음 부등식이 항상 성립한다.

max(hA; hB ; hC) · ® ¢min(mA;mB ;mC)

단, hA, hB , hC는 세 수선의 길이를 나타내고, mA, mB, mC는 세 중선의 길이를 나타낸다. 이런

®가 존재함을 보이고, ®의 최소값을 구하여라.

풀이 (서울 당산서중 2학년 박민재, 수정됨)

일반성을 잃지 않고 a ¸ b ¸ c 라고 하자(a; b; c는 각각 A;B;C의 대변의 길이). 그러면

2S = ahA = bhB = chC 에서 max(hA; hB ; hC) = hC 가 되고 Pappus의 중선정리에 의해

서 min(mA;mB ;mC) = mA 가 된다. 즉, 문제는hCmA

· ® 인 ®를 찾는 것이다.

위의 그림과 같이, BC의 중점을 M이라 하고, C, M에서 AB에 내린 수선의 발을 각각 D, E라

하자. 그럼 mA ¸ME 이다. 그리고 삼각형 BDC와 삼각형 BEM은 2:1의 비로 닮은 삼각형이

다. 결국hCmA

· hCME

=hC12hC

= 2

가 된다. 따라서, 문제를 만족시키는 ® = 2 가 존재한다. 또, E와 A가 일치할 때 등호가 성립하

여hCmA

= 2 가 되므로, ®는 2보다 작아질 수 없고 따라서 최소값은 2이다. }

Page 130: Baltic Way 팀수학경시대회 1990-2005 pdf 보기

130

Baltic 1995-18Baltic 1995-18

삼각형 ABC의 변 AC의 중점을 M이라 하고, B에서 AC 위에 내린 수선의 발을 H라 하자.또, A와

C에서 각 B의 이등분선 위로 내린 수선의 발을 각각 P , Q라 하자. 네 점 H, P , M , Q가 한 원 위

에 있음을 보여라.

증명 (대전 대덕중 3학년 고기혁 등)

BP와 AC의 교점을 D라 하고, AP와 BC의 교점을 E라 하자.

4BPA ´ 4BPE (SAS합동)이므로 P는 AE의 중점. 따라서,중점연결정리에의해 PM k BC

이다. 그럼 \AMP = \C. 한편, \BHC = \R = \BQC 이므로 BHQC는 원에 내접하는 사

각형이고, \HQB = \C. 즉,

\HMP = \HQP (= \C)

로 같으므로 원주각의 성질에 의해 HQMP는 원에 내접하는 사각형이다. ¤

Baltic 1995-19Baltic 1995-19

다음의 방법이 우주비행사 훈련에 사용된다고 한다:

반지름이 2R인 원 C2가 반지름이 nR인 원 C1의 내부를 따라 구른다. n은 2보다 큰 정수이다. 우

주비행사는 원 C2의 내부를 따라 구르는 반지름 R인 세 번째 원 C3에 매달려 있는데, 두 원 C1과

C2가 접하는 점으로부터 두 원 C2와 C3이 접하는 점까지의 거리가 항상 최대가 되도록 유지된다.

원 C2가 원 C1의 내부를 한 번 완전히 도는 동안 우주비행사는 원 C3과 함께 (지면을 기준으로) 몇

번의 회전을 하겠는가?

Page 131: Baltic Way 팀수학경시대회 1990-2005 pdf 보기

1995년 Baltic Way 풀이 131

풀이 C2가 구르는 곡선 C1을 C2의 윗쪽에 접하는 직선 C01으로 쭈욱 펼쳐서 생각해보자. 반

지름의 비가 n : 2 이므로 둘레의 길이의 비도 n : 2 이고, 따라서 C2는 C01을 따라 한 바퀴 구르

는 동안 자신은 n2바퀴 구른다. 또한 C3은 C2가

n2바퀴 구르는 동안 n바퀴 구른다. C2가 C1을

따라 구를 때와 C 01을 따라 구를 때를 비교해보자. 우선 C1과 C2의 접점 A는 어느 경우에나 동

일하므로, C2 위의 A에서 가장 멀리떨어진 점 B도 두 경우 모두 동일한 점이다. 이것이 바로

C2와 C3의 접점이므로, 결국 C01에서 구를 때도 C3은 같은 방식으로 구른다는 것을 알 수 있다.

(이것은 C1에서 중심을 향하는 방향과 C01에서 지면을 향하는 방향이 같다는 것으로부터도 알

수 있다.) 즉, C01에서 구른 경우를 조사하여 다시 C1과 같이 도로 둥글게 말아주는 것을 생각하

면 된다. C3은 C01에서는 시계방향으로 n바퀴를 도는데, C01을 C1으로 둥글게 마는 것은 시계반

대방향으로 한 바퀴 돌려주는 효과를 갖는다. 따라서, C3은 지면을 기준으로 n ¡ 1 바퀴를 돈

다. }

Baltic 1995-20Baltic 1995-20

모든 꼭지점의 좌표가 모두 정수인 볼록 오각형의 넓이는 52보다 작지 않음을 보여라.

증명 좌표가 모두 정수인 점을 격자점이라 하고, 격자점만을 꼭지점으로 갖는 다각형을 격

자다각형이라 하자. 우선 다음의 보조정리를 확인해두자.

보조정리 평면 위의 격자다각형의 넓이는 항상 m2 꼴이다. (m은 양의 정수)

볼록한 격자오각형 중에서 최소의 넓이를 갖는 것을 하나 택해 ABCDE라 하자. 만일 변 EA

내부에 다른 격자점 X가 존재한다면 XBCDE는 더 작은 넓이를 갖는 볼록한 격자오각형이므

로 최소성에 모순된다. 따라서, 오각형 ABCDE의 각 변에는 더 이상 격자점이 존재하지 않는

다.

이제 또하나의 유명한 비둘기집의 원리 문제를 인용하자. 모든 격자점은 각 좌표가 홀수냐 짝

수냐에 따라 4종류로 구분되고, 각 좌표의 홀짝이 같은 두 점은 그 중점도 격자점이 된다. 따라

서, ABCDE의 어느 두 꼭지점의 중점 P는 다시 격자점이고, 이 점이 변 위에 있을 수 없으므

로 P는 이 오각형의 내부의 점이다. 그럼 PAB, PBC, PCD, PDE, PEA는 각각 격자삼각형

이므로 넓이가 12 이상이고, ABCDE의 넓이는 이들의 합으로 5

2 이상임을 알 수 있다. ¤

보조정리의 설명 위의 보조정리는 유명한 Pick의 정리

A = I +1

2B ¡ 1

로부터도 확인할 수 있고(A는 격자다각형의 넓이, I는 이 도형 내부에 완전히 포함되는 격자

점의 개수, B는 이 도형의 경계에 놓인 격자점의 개수), 혹은 그보다 쉽게는 격자삼각형들의

합과 차로 모든 다각형을 표현할 수 있음을 말한 후(오목다각형의 경우 격자삼각형을 붙여 먼

저 볼록다각형으로 만들고 이 볼록다각형은 한 꼭지점에서 시작되는 모든 대각선들에 의해 격

자삼각형으로 쪼개는 것을 생각), 격자삼각형의 넓이는 세 꼭지점의 좌표가 (x1; y1), (x2; y2),

Page 132: Baltic Way 팀수학경시대회 1990-2005 pdf 보기

132

(x3; y3) 일 때

A =1

2

°°°°°°x1 x2 x3 x1

y1 y2 y3 y1

°°°°°°와 같이 행렬식꼴로 나타난다는 것을 이용할 수도 있다.

주 이 문제는 미국 대학생들의 수학경시대회인 William Lowell Putnam 수학경시대회에

1990년에 출제된 문제와 같다.

Page 133: Baltic Way 팀수학경시대회 1990-2005 pdf 보기

1996년 Baltic Way 풀이 133

1996년 Baltic Way 풀이

Baltic 1996-1Baltic 1996-1

정 1996각형의 두 대각선(의 연장선)사이의 각을 ®라 하고,또다른 이런 각을 ¯(6= 0)라 하자. ®=¯가

유리수임을 보여라.

증명 (광주 동성고 2학년 방재혁, 수정됨)

1996은 별로 의미가 없고 정다각형이면 충분하다. 정다각형은 원에 내접하고, 한 변에 대한 중

심각의 크기가 ¼n로 일정하므로 한 대각선(현)에 대한 중심각의 크기도 ¼의 유리수배이고, 그에

대한 원주각도 ¼의 유리수배이다. 두 대각선이 만나는 각은 그림에서 보듯이 두 원주각의 합이

므로 역시 ¼의 유리수배이다.

따라서, 그런 두 각의 비 ®=¯는 유리수이다. ¤

Baltic 1996-2Baltic 1996-2

P는 선분 AB 위의 점이고, AB를 지름으로 하는 반원 C1 안에 AP , PB를 지름으로 하는 반원 C2,

C3이 있다. P에서 AB에 수직한 직선 PQ를 그린다. 원 C는 C1에 내접하고 C3, PQ에 외접한다.

C1의 내부 중에서 C2, C3, C의 외부에 해당하는 영역의 넓이는 39¼이고, 원 C의 넓이는 9¼이다.

지름 AB의 길이를 구하여라.

풀이 (광주 광덕고 2학년 임익주 등)

반원 C1, C2, C3, C의 반지름을 각각 r1, r2, r3, r이라 하자(r = 3). 문제에 제시된 넓이 조건으

로부터1

2¼r21 ¡ 1

2¼r22 ¡ 1

2¼r23 ¡ 9¼ = 39¼

Page 134: Baltic Way 팀수학경시대회 1990-2005 pdf 보기

134

즉, r21 = r22 + r23 + 96 이다. r1 = r2 + r3 이므로, 다시 정리하면 r2r3 = 48 이다. C에서 AB에

내린 수선의 발을 H라 하자. 4CC1C3에서 CH ? C1C3 임을 이용해 피타고라스의 정리를 적

용하면

(CC1)2 ¡ (CC3)

2 = (C1H)2 ¡ (HC3)

2

(r1 ¡ r)2 ¡ (r + r3)2 = (C1C3 ¡HC3)

2 ¡ (HC3)2

(r1 ¡ 3)2 ¡ (3 + r3)2 = C1C3(C1C3 ¡ 2HC3)

r21 ¡ 6r1 ¡ r23 ¡ 6r3 = (r1 ¡ r3)((r1 ¡ r3)¡ 2(r3 ¡ r))

= (r1 ¡ r3)(r1 ¡ 3r3 + 6)= r21 ¡ 4r1r3 + 3r23 + 6r1 ¡ 6r3

4r1r3 = 12r1 + 4r23

4(r2 + r3)r3 = 12(r2 + r3) + 4r23

r2r3 = 3(r2 + r3)

따라서, r1 = r2 + r3 = 16 이고, AB = 2r1 = 32 ¢ ¢ ¢ 답 }

Baltic 1996-3Baltic 1996-3

ABCD는 단위 정사각형이고, P와 Q는 같은 평면 위의 점인데 Q는 삼각형 BPC의 외심이고, D가

삼각형 PQA의 외심이라고 한다. 선분 PQ의 길이로 가능한 값을 모두 구하여라.

풀이 (마산 가포고 2학년 신상 등)

먼저 Q는 BC의 수직이등분선 l 위에 있어야 한다. 또한 P와 Q는 D를 중심으로 하고 A를 지

나는 원 위에 있다. l와 원 D가 만나는 두 점을 Q1, Q2라 하면 Q의 후보는 이 두 점뿐이다.

B;C를 지나는 원 Q1과 원 Q2를 그리면 원 D와 C 이외에 각각 한 점 P1, P2에서 만나게 된다.

Page 135: Baltic Way 팀수학경시대회 1990-2005 pdf 보기

1996년 Baltic Way 풀이 135

P1Q1 = BQ1 =q

BM2 +MQ21 =

sµ1

2

¶2+

µ1 +

p3

2

¶2=1

2

q1 + (2 +

p3 )2 =

1

2

q8 + 4

p3 =

p6 +

p2

2

P2Q2 = BQ2 =q

BM2 +MQ22 =

sµ1

2

¶2+

µ1¡

p3

2

¶2= ¢ ¢ ¢ =

p6¡p22

따라서 PQ로 가능한 값은 ¢ ¢ ¢ 답p6§p22

}

Baltic 1996-4Baltic 1996-4

ABCD는 AD k BC 인 사다리꼴이다. P는 직선 AB 위의 점 중에서 \CPD가 최대인 점이고, Q는

직선 CD 위의 점 중에서 \BQA가 최대인 점이다. P가 선분 AB 위에 있을 때, \CPD = \BQA

임을 보여라.

증명 \CPD가 최대인 점 P는 C와 D를 지나고 직선 AB에 접하는 원의 접점이다. 마찬가

지로 A와 B를 지나고 직선 CD에 접하는 원의 접점이 Q가 된다.

AB에 대해 점 Q를 결정하는 것이나 CD에 대해 점 P를 결정하는 것은 대칭적인 작업(두 직선

AB와 CD가 만나는 각의 이등분선에 대해 대칭시킨 후 두 직선의 교점에 대해 적당히 확대축

소하면 CPD와 BQA가 정확히 일치함)이므로 \CPD = \BQA 임은 분명하다. P가 선분 AB

위에 있다는 조건은 불필요하다. ¤

Baltic 1996-5Baltic 1996-5

ABCD는 원에 내접하는 볼록 사각형이고, 삼각형 BCD, ACD, ABD, ABC의 내접원의 반지름을

각각 ra, rb, rc, rd라 하자. ra + rc = rb + rd 임을 보여라.

증명1 (서울 당산서중 1학년 박민재)

Carnot의 정리를 이용하여 풀자.

Page 136: Baltic Way 팀수학경시대회 1990-2005 pdf 보기

136

Carnot의 정리 임의의 삼각형 ABC에서 외심으로부터 각 변까지의 거리를 ra, rb, rc라

하고, 외접원의 반지름을 R, 내접원의 반지름을 r이라 하면

ra + rb + rc = R+ r

이 성립한다. (단, 둔각삼각형의 경우 밖으로 나간 길이는 음의 길이로 쳐준다.)

Carnot의 정리의 증명 Steiner의 정리를 이용하여 증명할 수도 있지만 여기서는 Ptole-

my의 정리를 이용하여 쉽게 증명하기로 한다.

(1) 예각삼각형일 때:

그림에서 중선연결정리에 의해 FD = a, DE = b, EF = c 이고, ¤ADOF , ¤BEOD,

¤CFOE 등은 원에 내접하는 사각형이므로 Prolemy의 정리에 의해

b ¢ rc + c ¢ rb = R ¢ ac ¢ ra + a ¢ rc = R ¢ ba ¢ rb + b ¢ ra = R ¢ c

이 된다. 또, 삼각형의 넓이에서

a ¢ ra + b ¢ rb + c ¢ rc = r(a+ b+ c)

이다. 이 네 개의 식을 변변 더하면

(a+ b+ c)(ra + rb + rc) = (R+ r)(a+ b+ c)

가 되고, 양변에서 a+ b+ c 를 소거하면 원하는 등식이 된다.

Page 137: Baltic Way 팀수학경시대회 1990-2005 pdf 보기

1996년 Baltic Way 풀이 137

(2) 둔각삼각형일 때:

앞에서와 비슷하게 하면 된다.

b ¢ rc + c ¢ rb = R ¢ aa ¢ rc = c ¢ ra +R ¢ ba ¢ rb = b ¢ ra +R ¢ c

¡a ¢ ra + b ¢ rb + c ¢ rc = r(a+ b+ c)

이 네 식을 변변 더하면

(a+ b+ c)(¡ra + rb + rc) = (R+ r)(a+ b+ c)

가 되고, 양변에서 a+ b+ c 를 소거하면 원하는 등식이 된다. ¤

이제 원래 문제를 증명하자.

4BCD와 4ABD에 Carnot의 정리를 쓰면,

OF +OG¡OJ = R+ ra

OJ +OH +OE = R+ rc

이고, 변변 더하면

OE +OF +OG+OH = ra + rc + 2R

이 된다. 비슷하게 4ACD와 4ABC에서도 Carnot의 정리에 의해

OG+OH ¡OI = R+ rb

OE +OF +OI = R+ rd

Page 138: Baltic Way 팀수학경시대회 1990-2005 pdf 보기

138

따라서,

OE +OF +OG+OH = rb + rd + 2R

가 된다. 이 두 결과를 비교하면 ra + rc = rb + rd 임을 알 수 있다. ¤

증명2 (KAIST 수학과 03학번 윤태동)

삼각형의 넓이에 관한 두 가지 식

S =1

2ab sinC = 2R2 sinA sinB sinC

S =1

2r(a+ b+ c) = Rr(sinA+ sinB + sinC)

의 관계로부터

r = 2R ¢ sinA sinB sinC

sinA+ sinB + sinC

의 식이 나온다. 이것을 삼각함수의 합차공식과 2배각공식 등을 이용하여 좀더 변형하면(A +

B + C = ¼ 임을 상기하자)

r = 2R ¢ sinA sinB sin(A+B)

sinA+ sinB + sin(A+B)

= 2R ¢ sin(A+B) sinA sinB

2 sin A+B2 cos A¡B2 + 2 sin A+B

2 cos A+B2

= 2R ¢ sin(A+B) sinA sinB

2 sin A+B2 (cos A¡B2 + cos A+B2 )

= 2R ¢ sin(A+B) sinA sinB

2 sin A+B2 ¢ 2 cos A2 cos B2

= 4R ¢ cos A+B

2sin

A

2sin

B

2

= 4R ¢µcos

A

2cos

B

2¡ sin A

2sin

B

2

¶sin

A

2sin

B

2

= R(sinA sinB ¡ (1¡ cosA)(1¡ cosB))= R(cosA+ cosB ¡ cos(A+B)¡ 1) = R(cosA+ cosB + cosC ¡ 1)

과 같은 공식을 얻을 수 있다(이 결과는 사실 앞의 Carnot의 정리와 같은 것이다). 이것을 이용

하기로 하자.

Page 139: Baltic Way 팀수학경시대회 1990-2005 pdf 보기

1996년 Baltic Way 풀이 139

원호 AB, BC, CD, DA에 대응되는 원주각의 크기를 각각 ®, ¯, °, ±라 하자. rb와 rd를 위의

공식을 이용하여 나타내어보면(a+ b+ c+ d = ¼),

rb + rd = R(cos®+ cos¯ ¡ cos(®+ ¯)¡ 1) +R(cos ° + cos ± ¡ cos(° + ±)¡ 1)= R(cos®+ cos¯ + cos ° + cos ± ¡ 2)

이 식은 a, b, c, d에 대해 대칭식이므로 ra + rc 를 같은 방법으로 구하면 위와 똑같이 값이 나

오게 된다. ¤

증명3 (대전 탄방중 3학년 김민기)

4BCD, 4ACD, 4ABD, 4ABC의 내심을 각각 I, J , K, L이라 하자.

CI, CJ는 각각 \BCD, \ACD의 이등분선이므로

\JCI =1

2\ACB 마찬가지로 \JDI =

1

2\ADB

이다. 여기서 \ADB = \ACB 이므로 \JCI = \JDI 가 된다. 따라서, ¤ICDJ는 원에 내접

하는 사각형이다. 마찬가지로 ¤ICBL도 원에 내접하는 사각형이다. 그럼 두 사각형의 내대각

의 합에서

\LIJ = \JDC + \LBC =1

2(\B +\D) = 90±

이다. 대칭적으로 ¤IJKL의 다른 내각에 대해서도 마찬가지이고, 따라서 ¤IJKL은 직사각형

이다. I, J , K, L에서 BD, AC, BD, AC에 내린 수선의 발을 각각 P , Q, R, S라 하자.

IK2 = PR2 + (IP +KR)2 = PR2 + (ra + rc)2

JL2 = SQ2 + (JQ+ LS)2 = SQ2 + (rb + rd)2

인데, ¤IJKL이 직사각형이므로 IK = JL 이다. 즉,

ra + rc = rb + rd =) PR = SQ

Page 140: Baltic Way 팀수학경시대회 1990-2005 pdf 보기

140

이므로 PR = SQ 임을 증명하면 된다. 그리고 이것은

PR = BD ¡ PD ¡BR

= BD ¡ BD + CD ¡BC

2¡ BD +AB ¡AD

2

=BC +AD ¡AB ¡ CD

2

SQ = AC ¡AS ¡QC

= AC ¡ AC +AB ¡BC

2¡ AC + CD ¡AD

2

=BC +AD ¡AB ¡ CD

2

이므로 성립한다. ¤

증명4 (대전과학고 2학년 송지용)

삼각형 ABC의 내접원과 외접원의 반지름 r과 R 사이에 다음과 같은 관계식이 성립한다는 것

이 잘 알려져있다.

r = 4R sinA

2sin

B

2sin

C

2

현 AB, BC, CD, DA에 대한 원주각의 크기를 각각 ®, ¯, °, ±라 하자. 그럼 위의 관계식에 따

ra + rc

4R sin ®2 sin

¯2 sin

°2 sin

±2

=sin ¯+°

2

sin ¯2 sin

°2

+sin ±+®

2

sin ±2 sin

®2

=sin ¯

2cos °

2+ cos ¯

2sin °

2

sin ¯2sin °

2

+sin ±

2cos ®

2+ cos ±

2sin ®

2

sin ±2 sin

®2

= cot°

2+ cot

¯

2+ cot

®

2+ cot

±

2

대칭적으로rb + rd

4R sin ®2 sin

¯2 sin

°2 sin

±2

= cot°

2+ cot

¯

2+ cot

®

2+ cot

±

2

이고, 따라서 ra + rc = rb + rd 이다. ¤

Baltic 1996-6Baltic 1996-6

a, b, c, d는 ab = cd 를 만족하는 양의 정수이다. a+ b+ c+ d 가 소수가 아님을 보여라.

증명 ab = cd 로부터

a = pq; b = rs; c = pr; d = qs

꼴이 된다(p = gcd(a; c), s = gcd(b; d) 등). 그럼

a+ b+ c+ d = (p+ s)(q + r)

이므로 합성수이다. ¤

Page 141: Baltic Way 팀수학경시대회 1990-2005 pdf 보기

1996년 Baltic Way 풀이 141

Baltic 1996-7Baltic 1996-7

정수 수열 a1; a2; : : : 이 a1 = 1, a2 = 2,

an+2 =

8><>:5an+1 ¡ 3an (an ¢ an+1 이 짝수일 때)

an+1 ¡ an (an ¢ an+1 이 홀수일 때)

(n ¸ 1)

로 주어져 있다. 모든 n에 대해서 an 6= 0 임을 보여라.

증명 an+2 ´ an+1 + an (mod 2) 임으로부터 수열 fang은 홀, 짝, 홀, 홀, 짝, 홀, : : : 을 반복

함을 알 수 있다. 즉, 2 - anan+1 () 3 j n 이다. 따라서

an+2 ´

8><>:¡an+1 3 - n

an+1 ¡ an 3 j n(mod 3)

이 되고, a1 ´ ¡a2 6́ 0 (mod 3) 으로부터 어느 경우에나 an+2 ´ ¡an+1 (mod 3), 즉 수열

fang은 mod 3으로 1;¡1; 1;¡1; : : : 을 반복함을 알 수 있다. 따라서 3 - an. ¤

Baltic 1996-8Baltic 1996-8

다음과 같은 수열을 생각하자:

x1 = 19; x2 = 95; xn+2 = lcm(xn+1; xn) + xn (n ¸ 1)

단, lcm(a; b)는 a와 b의 최소공배수를 뜻한다. x1995와 x1996의 최대공약수를 구하여라.

풀이 (서울 환일고 2학년 김태우)

적당한 정수 k에 대해 xn+2 = kxn+1 + xn 꼴이므로, 유클리드 호제법의 원리에 의해

gcd(xn+2; xn+1) = gcd(xn+1; xn)

이다. 따라서, gcd(x1996; x1995) = ¢ ¢ ¢ = gcd(x2; x1) = 19 ¢ ¢ ¢ 답 }

Baltic 1996-9Baltic 1996-9

n과 k는 1 · k < n 인 정수이다. 다음 조건을 만족하는 정수 b와 n개의 정수의 집합 A를 찾아라.

(i) A의 서로 다른 k ¡ 1개의 원소들의 곱은 항상 b의 배수가 아니다.

(ii) A의 서로 다른 k개의 원소들의 곱은 항상 b의 배수이다.

(iii) A의 서로 다른 어떤 두 원소도 하나가 다른 것의 배수가 되지 않는다.

풀이 (KAIST 수학과 03학번 윤태동)

홀수인 소수들을 작은 것부터 늘어놓은 것을 p1; p2; p3; : : : 이라 하자.

A = f2p1; 2p2; : : : ; 2png; b = 2k

라 놓으면 주어진 조건을 만족한다는 것을 쉽게 확인할 수 있다. }

Page 142: Baltic Way 팀수학경시대회 1990-2005 pdf 보기

142

Baltic 1996-10Baltic 1996-10

양의 정수 n의 서로 다른 양의 약수(1과 n을 포함한)의 개수를 d(n)으로 나타낸다. a > 1 와 n > 0

은 an + 1 가 소수가 되도록 하는 정수이다. d(an ¡ 1) ¸ n 임을 보여라.

증명 (과천고 1학년 홍성준)

만일 n이 어떤 홀수 m(¸ 3)의 배수라면 n = mk 를 대입하면,

an + 1 = (ak + 1)((ak)m¡1 ¡ (ak)m¡2 + (ak)m¡3 ¡ ¢ ¢ ¢+ 1)

이 되어 ak +1 이 an +1 을 나누게 되므로 모순이다. 따라서 n은 3 이상의 홀수인 약수를 가질

수 없고, 즉 n = 2q꼴이다. Fm := a2m+ 1(> 0) 이라 정의하자. 그러면,

an ¡ 1 = a2q ¡ 1 = Fq¡1(a

2q¡1 ¡ 1) = ¢ ¢ ¢ = Fq¡1Fq¡2 ¢ ¢ ¢F1F0(a¡ 1)

여기서 Fm들은 모두 서로 소이다. (* 위의 식은 일반적으로 Fm ¡ 2 = Fm¡1 ¢ ¢ ¢F0(F0 ¡ 2) 이기도 하므로, 임의의 a < b 에 대해 Fa j Fb ¡ 2, 즉 gcd(Fa; Fb) j 2, 두 수가 모두 홀수이므로

Fa와 Fb는 서로 소이다.) 따라서, Fq¡1; : : : ; F0 에서 몇 개를 뽑아 곱한 수들은 모두 an ¡ 1 의

서로 다른 양의 약수들이다. 이런 곱이 2q개(부분집합의 개수) 있으므로 d(an ¡ 1) ¸= 2q = n

이다.

등호는 a = 2 이고 Fq¡1; : : : ; F0 이 모두 소수일 때만 성립한다. ¤

Baltic 1996-11Baltic 1996-11

실수 x1; x2; : : : ; x1996 은 다음 성질을 만족한다: 임의의 2차 다항식 W에 대해 W (x1), W (x2), : : : ;

W (x1996) 중 적어도 3개의 값이 같다. x1; x2; : : : ; x1996 중 적어도 3개의 수가 같음을 보여라.

증명 (대전과학고 2학년 송지용)

x1; x2; : : : ; x1996을 재배열 한 것을 y1 · y2 · ¢ ¢ ¢ · y1996 이라고 하자. y0 < y1 인 임의의 실수

y0을 잡자. W (x) = (x¡ y0)2 인 W (x)를 잡으면, 0 < y1 ¡ y0 · y2 ¡ y0 · ¢ ¢ ¢ · y1996 ¡ y0 이

므로, W (x)는 정의역 y1; : : : ; y1996 에 대해 단조증가함수이다. W (x) = (x¡ y0)2 에 대해서도

적어도 세 개의 실수 ya; yb; yc (a 6= b 6= c 6= a) 에 대해 W (ya) = W (yb) = W (yc) 이어야 하므

로 ya = yb = yc, 즉 적어도 세 개의 수가 같다. 따라서, x1; : : : ; x1996 중 적어도 세 개의 수가 같

다. ¤

별증 (부산 건국고 2학년 신승현)

x1; x2; : : : ; x1996에서 3개를 택하는 방법의 수는 유한하다. A = fx2+rx j r 2 Rg이라 하면 A는

무한집합이고 A의 이차식 각각에 대해 함수값이 같은 3개의 xi가 있으므로, 비둘기집의 원리

에 의해 그 3개의 xi (a, b, c라 하자)가 똑같은 두 이차식 f; g 2 A 가 있다. f(a) = f(b) = f(c),

g(a) = g(b) = g(c) 이므로 h(x) = f(x)¡ g(x) 라 할 때

h(a) = h(b) = h(c)

Page 143: Baltic Way 팀수학경시대회 1990-2005 pdf 보기

1996년 Baltic Way 풀이 143

이다. h는 일차항의 계수가 0이 아닌 일차함수이므로 일대일대응이고, 따라서 a = b = c 이다.

¤

Baltic 1996-12Baltic 1996-12

S는 정수의 집합으로 0과 1996을 포함한다. S의 원소를 계수로 갖는 0이 아닌 임의의 다항식의 정

수해가 다시 S에 속한다. ¡2가 S에 속함을 보여라.

증명 (북서울중 3학년 류종하)

우선 1996x+ 1996 = 0 에서 ¡1 2 S 이다. 1996의 2진법 전개를 생각해 보면

1996 = 210 + 29 + 28 + 27 + 26 + 23 + 22

이 되므로,

¡x10 ¡ x9 ¡ x8 ¡ x7 ¡ x6 ¡ x3 ¡ x2 + 1996 = 0

에서 2 2 S 이다. 마지막으로 ¡x2 ¡ x+ 2 = 0 에서 1;¡2 2 S 임을 알 수 있다. ¤

주 ¡x1996¡x1995¡¢ ¢ ¢¡x+1996 = 0에서 1 2 S 를 먼저 얘기해놓고 x+2 = 0에서 ¡2 2 S

를 보일 수도 있다.

Baltic 1996-13Baltic 1996-13

정수 범위에서 정의되는 함수 f가 임의의 정수 x에 대해

f(x) = f(x2 + x+ 1)

을 만족한다. 이러한 성질을 만족하는 (a) 모든 우함수, (b) 모든 기함수 f를 구하여라.

풀이 (KAIST 수학과 03학번 윤태동)

조건식에서

f(¡x) = f(x2 ¡ x+ 1) = f((x¡ 1)2 + (x¡ 1) + 1) = f(x¡ 1)

임을 먼저 확인해두자.

(a) 우함수이면 f(x) = f(¡x) 이므로 f(x) = f(x¡ 1). 따라서, 정수 범위에서 정의되는 함수

f는 상수함수이다. 그리고, 상수함수는 모두 조건식을 만족하며 우함수이므로 답이 된다.

(b) 기함수이면 f(x) = ¡f(¡x) 이므로 f(x) = ¡f(x ¡ 1). 따라서, jf j는 상수함수이다. 또,

기함수임에서 f(0) = 0 이므로 jf j ´ 0, 즉 f는 영함수이다.

¢ ¢ ¢ 답 (a) 모든 상수함수 (b) 영함수 }

Page 144: Baltic Way 팀수학경시대회 1990-2005 pdf 보기

144

Baltic 1996-14Baltic 1996-14

함수 f(x) = xn + an¡1xn¡1 + ¢ ¢ ¢+ a1x+ a0 (n > 1) 의 그래프가 직선 y = b 와 점 B1; B2; : : : ; Bn

(왼쪽부터 오른쪽으로) 에서 만나고, 직선 y = c (c 6= b) 와는 점 C1; C2; : : : ; Cn (왼쪽부터 오른쪽

으로) 에서 만난다. P를 직선 y = c 위의 Cn의 오른쪽에 있는 점이라고 하자. 다음 합을 구하여라.

cot(\B1C1P ) + ¢ ¢ ¢+ cot(\BnCnP )

풀이 (KAIST 수학과 03학번 윤태동)

점 Ci의 x-좌표를 ci라 하고, 점 Bi의 x-좌표를 bi라 하자(i = 1; 2; : : : ; n). 그러면 fcigi=1;:::;n는 f(x) ¡ c = 0 의 해집합이며 fbigi=1;¢¢¢ ;n 는 f(x) ¡ b = 0 의 해집합이 된다. 이제 기울기와

tan 값 사이의 관계를 이용하면,

cot\BiCiP =1

tan\BiCiP=

1b¡cbi¡ci

=bi ¡ cib¡ c

이제 위 식과 근과 계수와의 관계를 이용하면,

nXi=1

cot\BiCiP =1

b¡ c

ÃnXi=1

bi ¡nXi=1

ci

!=

1

b¡ c(¡an¡1 ¡ (¡an¡1)) = 0

¢ ¢ ¢ 답 0 }

Baltic 1996-15Baltic 1996-15

임의의 정수 n > 2 과 임의의 양의 실수 x1; : : : ; xn 에 대해서 다음 부등식

x1x2 + x2x3 + ¢ ¢ ¢+ xn¡1xn + xnx1 ¸ xa1xb2x

a3 + xa2x

b3x

a4 + ¢ ¢ ¢+ xanx

b1x

a2

이 성립한다. 어떤 양의 실수 a, b에 대해 이것이 만족하는가?

풀이 x1 = x2 = ¢ ¢ ¢ = xn = x 를 대입하면

nx2 ¸ nx2a+b; 즉 x2 ¸ x2a+b

이 모든 양의 실수 x에 대해 성립해야 한다. x > 1 이면 2 ¸ 2a + b 여야 하고 x < 1 이면

2 · 2a+ b 여야 하므로

2a+ b = 2

이다. 또, n = 4 일 때, (x1; x2; x3; x4) = (x; 1; x; 1) 을 대입하자. 그럼

4x ¸ 2x2a + 2xb; 즉 2x ¸ x2a + xb

이 모든 양의 실수 x에 대해 성립해야 한다. 그런데, 산술-기하 평균부등식에서 x2a + xb ¸2px2axb = 2x. 따라서, 등호가 성립해야 하고, 그 경우 x2a = xb, 즉

2a = b = 1

Page 145: Baltic Way 팀수학경시대회 1990-2005 pdf 보기

1996년 Baltic Way 풀이 145

이다. 또, 2a = b = 1 일 때는 역시 산술-기하 평균부등식에서

x1x2 + x2x3 + ¢ ¢ ¢+ xnx1 ¸ x1x2 + x2x32

+ ¢ ¢ ¢+ xnx1 + x1x22

¸ (우변)

으로 항상 성립한다.

¢ ¢ ¢ 답 (a; b) = ( 12 ; 1) }

Baltic 1996-16Baltic 1996-16

무한히 큰 체스판에서 두 명의 선수가 번갈아 표시가 없는 빈 칸에 자기 표시를 한다. 한 명은 X 표

시를 사용하고 다른 한 명은 O 표시를 사용한다. 자신의 표시로 2£ 2 정사각형을 먼저 채우는 사람

이 이긴다. 먼저 시작한 선수가 항상 이길 수 있는가?

풀이 (부산 건국고 2학년 신승현, 수정됨)

무한 체스판을 그림과 같이 벽돌을 쌓듯이 2£ 1의 블럭들로 분할하자.

한 블럭 안에 있는 두 칸을 서로 짝이 되는 칸이라고 부르기로 하자. 이제, 나중에 하는 사람은

먼저 시작한 사람이 어디에 두든 그 칸의 짝이 되는 칸에 두기로 하자. 그럼 각각의 블럭에는

항상 O와 X가 1개씩 들어가게 된다. 2£ 2 정사각형은 항상 1개의 블럭을 완전히 포함하고, 먼

저 하는 사람이 하나의 블럭을 모두 표시할 수는 없으므로, 먼저 하는 사람은 2£ 2 정사각형을

채울 수 없다. 따라서, 먼저 하는 사람에게 필승의 전략을 가질 수 없다. }

Baltic 1996-17Baltic 1996-17

여덟 개의 숫자 1, 3, 4, 5, 6, 7, 8, 9를 정확히 한 번씩만 사용해서, 세 자리의 수 A와 두 자리의 수

B, C (B < C), 그리고 한 자리의 수 D를 만든다. 이 수들은 A +D = B + C = 143 을 만족한다.

이렇게 수를 만드는 방법은 모두 몇 가지인가?

풀이 우선 A의 백의 자리는 1이다(2 이상이면 D와의 합이 200을 넘어 곤란). 또, A의 십의

자리는 3이다(2 이하이면 D와의 합이 140이 안 되고, 4 이상이면 14a+D = 143꼴이어야 하는

데 a나 D에 1, 2를 사용할 수 없어 곤란). 그러면 이제

A = 13a; D = d; B = be; C = cf

라 하자. 우선 a + d = 13 이 되어야 함을 알 수 있다. 또한 e나 f에 1, 2를 쓸 수 없으므로

e+ f 6= 3, 즉 e+ f = 13 이 되고 자리올림에 의하여 십의 자리의 합도 b+ c = 13 이 되어야 한

다. 따라서 4; 5; 6; 7; 8; 9를 a; b; c; d; e; f에

a+ d = e+ f = b+ c = 13; b < c

Page 146: Baltic Way 팀수학경시대회 1990-2005 pdf 보기

146

를 만족하도록 배정하는 방법의 수이다. 따라서, 3!£ 23=2 = 24 가지. ¢ ¢ ¢ 답 }

Baltic 1996-18Baltic 1996-18

어느 올림피아드 대회의 심사위원은 처음에는 모두 30명이었다. 심사위원들 각각은 이들 동료 중

몇 명은 유능하고 그 외 나머지는 무능하다고 생각한다. 또한 이러한 생각은 변하지 않는다. 각 세

션을 시작할 때마다 투표가 행해지고, 투표자의 과반수가 무능하다고 생각하는 사람은 남은 올림피

아드 대회에서 심사위원에서 제외된다. 최대 15번의 세션이 지난 후에는 더 이상 제외되는 사람이

없음을 보여라. (아무도 자기 자신이 유능한가에 대한 투표는 할 수 없다)

증명 (서울 경희중 3학년 송은호, 수정됨)

한 번의 세션에서 제외되는 사람이 없으면 계속되는 세션에서도 제외되는 사람이 없게 되므로

투표가 종료된 때로 간주하자.

(1) 짝수명인 경우에 1명만 제외되면 투표가 종료된다.

2k명의 사람이 있을 때 1명이 제외되어 2k ¡ 1명이 되었다면, 2k명이었을 때도 2k ¡ 1명이었

을 때도 과반수는 k명(자기 자신을 제외한 인원의 과반수)으로 같으므로, 2k¡ 1명일 때 제외되

는 사람은 이미 앞선 2k명일 때에도 과반수가 무능하다고 투표할 것이므로 제외되었어야 한다.

즉, 2k ! 2k ¡ 1명이 되면 더 이상 제외되는 사람은 없고, (1)이 확인되었다. 따라서, 짝수명일

때 1명만 제외되는 경우보다는 2명 이상 제외되는 경우가 투표를 더 길게 가져갈 수 있으므로

(2) 짝수명일 때는 2명 이상 제외되는 경우만 생각해도 된다.

세션이 진행됨에 따른 인원의 변화를

a0 ! a1 ! a2 ! ¢ ¢ ¢ ! an

이라 하자. 이것은 감소하는 수열이고, (2)에 의해

f30; 29g; f28; 27g; f26; 25g; : : : ; f2; 1g; f0g

의 집합에서 각각 많아야 1개의 수만 나타날 수 있다. 모두 16개의 집합이므로 최대 16개의 수

가 나타날 수 있고, 그럼 n · 15 이다. n은 열린 세션의 수이므로, 세션은 15번을 넘을 수 없음

을 알 수 있다. ¤

Baltic 1996-19Baltic 1996-19

각각 38, 45, 61, 70개의 성냥개비가 있는 성냥개비 네 더미가 있다. 두 명의 선수가 번갈아, 임의로

두 더미를 선택한 후, 한 더미에서 성냥개비 몇(6= 0) 개를 가져오고, 다른 한 더미에서도 성냥개비

를 몇(6= 0) 개를 가져온다. 더 이상 이렇게 할 수 없는 선수가 지게 된다. 어느 선수가 필승의 전략

을 가질 수 있는가?

Page 147: Baltic Way 팀수학경시대회 1990-2005 pdf 보기

1996년 Baltic Way 풀이 147

풀이 Losing state(먼저 시작하면 지는 경우)는 a, a, a, b (a · b) 꼴일 때이다(최소값이 3곳

이상일 때).

² Losing state를 만들어 넘겨주면, 나중에 하는 사람은 3곳 모두를 더 줄일 수는 없고 3곳

중 적어도 한 곳은 줄여야 하므로, 어떻게 해도 최소값인 곳이 2곳 이하가 된다.

² Winning state가 되어 넘겨 받으면(최소값이 2곳 이하이면), 먼저 하는 사람은 최소값이

아닌 두 곳을 최소값과 같게 줄여주면 항상 최소값이 3곳 이상이 되도록 만들 수 있다.

따라서, 먼저 하는 사람이 38, 38, 38, 70을 만든 후 위와 같이 계속 Losing state를 만들어 넘겨

주면 이긴다. }

Baltic 1996-20Baltic 1996-20

모든 양의 정수들을 다음의 성질이 만족되도록 서로 소인 두 집합 A, B로 나누는 것이 가능한가?

(i) A의 어느 세 개의 수도 등차수열의 연속한 세 항을 이루지 않는다.

(ii) B의 수들만으로 상수수열이 아닌 무한 등차수열을 만들 수 없다.

풀이 (과천고 2학년 홍성준, 조금 수정)

결론적으로, 이렇게 나누는 것이 가능하다. 자연수 전체의 집합 N에서 몇 개의 수들을 뽑아서

A에 넣고, 남은 것을 B로 할 것이다. (ii)로부터 N에서 취할 수 있는 모든 무한 등차수열에서

적어도 하나의 항은 A에 들어가 있어야 함을 알 수 있다. N에 속하는 어떤 무한 등차수열을

m;m+ k;m+ 2k; : : : 라고 놓자(m, k는 자연수). 이런 수열은 순서쌍 (m;k)들과 1 : 1로 대응

하고, 따라서 이런 수열은 무한히 많다. 이제 이 순서쌍들을 m+ k가 작은 것부터 다음과 같이

차례로 늘어놓을 수 있다(즉 이러한 순서쌍들의 집합은 가산무한집합이다. Cantor의 방법).

(1; 1) (1; 2) (2; 1) (1; 3) (2; 2) (3; 1) (1; 4) (2; 3) ¢ ¢ ¢ (1)

이제 여기서 늘어놓은 순서대로 각 순서쌍에 해당하는 등차수열에서 하나씩 수를 뽑아 A에 넣

는데, 먼저 자연수 1을 A에 넣고, 다음부터 뽑는 수는 바로 이전에 뽑은 수의 2배보다 큰 것으

로 뽑는다. 그럼 A에서 임의의 세 항 Ai; Aj ; Ak (i < j < k) 를 뽑으면 Ak¡Aj > Aj > Aj ¡Ai

이므로 등차수열의 세 항이 될 수 없고, 따라서 A는 (i)을 만족한다. 그리고, 모든 무한 등차수

열에서도 각각 항이 적어도 하나씩 빠져있으므로 B도 (ii)를 만족한다. }

Page 148: Baltic Way 팀수학경시대회 1990-2005 pdf 보기

148

1997년 Baltic Way 풀이

Baltic 1997-1Baltic 1997-1

모든 실수 x, y에 대해서 f(x)f(y) = f(x ¡ y) 를 만족하는, 항등적으로 0은 아닌 함수 f : R ! R을 모두 구하여라.

풀이 (대구 지산중 2학년 김규완)

y에 0을 대입하면, f(x)f(0) = f(x) 이고, f(x) 6= 0 인 x가 존재하므로

f(0) = 1

y에 x를 대입하면, f(x)2 = f(0) = 1. 따라서, 모든 x에 대하여,

f(x) = §1 6= 0

마지막으로 y = x2 를 대입하면, f(x)f(x2 ) = f(x2 ). 여기서 f(x2 ) 6= 0 이므로, f(x) = 1. 즉, 구

하는 함수는 상수함수 f(x) ´ 1 뿐이다. }

별해 (대전과학고 1학년 고기혁)

f(x)f(0) = f(x), f(0)f(x) = f(¡x) 이므로

f(x) = f(¡x)

의 우함수이다. 그럼 f(x)f(y) = f(x¡ y), f(x)f(¡y) = f(x+ y) 에서

f(x+ y) = f(x¡ y)

임을 알 수 있고, 그럼 f(x+ x) = f(0) 으로 상수함수이다. f(x) = c 로 두고 대입하면 c2 = c.

c 6= 0 이므로 c = 1. 따라서, f(x) = 1 의 상수함수뿐이다. }

Baltic 1997-2Baltic 1997-2

모든 양의 정수가 정확히 한 번씩 나타나는 양의 정수들의 수열 a1; a2; a3; : : : 이 있다. 1 < l < m

이고 a1 + am = 2al 인 두 정수 l, m이 존재함을 보여라.

증명 (분당중앙고 1학년 임동혁)

모든 양의 정수를 적당한 순서로 늘어놓은 수열로 생각하면 된다. 귀류법을 쓰자. 그럼 a1 = k

라 할 때, k; x; y가 등차수열을 이루면 k; y; x의 순서로 나타나야 한다. k의 오른쪽에 2k가 있

을텐데, k; 2k; 3k 에서 3k는 2k보다 왼쪽에 나타나야 한다. 또, 5k는 3k보다 왼쪽에 나타나고,

9k는 5k보다 왼쪽에 나타나고, 이런 식으로 2k의 왼쪽에 무한히 많은 항이 들어가야 한다. 그

러나, 2k의 왼쪽에는 항이 유한개밖에 있을 수 없으므로 모순. 따라서, 제 순서로 나타나는 것

이 반드시 있다. ¤

Page 149: Baltic Way 팀수학경시대회 1990-2005 pdf 보기

1997년 Baltic Way 풀이 149

별증 (제주 대기고 1학년 오재성)

al > a1 인 가장 작은 l을 잡자. 그럼 am = 2al¡ a1 > a1 이므로 l의 최소성에서 m > l 이다. 따

라서, 원하는 m, l을 찾았다. ¤

주 원래 제출된 문제는 ak + am = 2al (k < l < m) 로, a1의 첨자를 일반화한 것이었다. 문

제 선정 위원회에서는 이런 일반화가 핵심적인 차이를 만드는 것이 없다고 판단하여 위와 같이

문제를 수정하였다. 이 문제를 `모든 자연수를 적당한 순서로 늘어놓은 수열에서 3항짜리 부분

등차수열이 항상 존재하는가' 하는 것으로 본다면, `s항짜리 부분 등차수열이 존재하는가' 하는

문제로의 일반화를 생각해볼 수 있다. s = 3 일 때까지는 위의 문제에서처럼 성립하고, s = 5

일 때에는 반례가 있다는 것을 아래 논문에서 보였다.

J. A. Davis, R. C. Entringer, R. L. Graham and G. J. Simmons, On Permutations

containing no long arithmatic progressions , Acta Arithmetica 1(1977), pp. 81{90

그리고, s = 4 일 때에 대해서는 여전히 미해결 문제로 남아있다.

Baltic 1997-3Baltic 1997-3

x1 = 1 이고 xn+1 = xn +jxnn

k+ 2 (n = 1; 2; 3; : : : ) 로 주어진 수열에서 x1997의 값을 구하여라.

단, bxc는 x를 넘지 않는 가장 큰 정수를 나타낸다.

풀이 xn = an+ b (0 · b < n) 으로 나타낸다면

xn+1 = a(n+ 1) + (b+ 2)

가 된다. 따라서, xn = an꼴의 n에서 출발한다면 xn+i = a(n+ i) + 2i (i = 1; 2; : : : ; n¡ 1) 이다가(¤)

x2n = (a+ 1)(2n)

으로 a가 1 증가하게 된다. 즉, x1 = a ¢ 1 이라면, x2 = (a+ 1) ¢ 2, x4 = (a+ 2) ¢ 4, 이렇게 계속

하여 일반적으로

x2k = (a+ k) ¢ 2k

이다. 또한 (¤)에 의해

xn = (a+ k)n+ 2m (n = 2k +m; 0 · m < 2k)

이다. 1997 = 210 + 973 이고 또 a = 1 인 상황이므로, a1997 = (1 + 10) ¢ 1997 + 2 ¢ 973 = 23913¢ ¢ ¢ 답 }

Baltic 1997-4Baltic 1997-4

x1; : : : ; xn의 산술평균 a가 다음을 만족함을 증명하여라.

(x1 ¡ a)2 + ¢ ¢ ¢+ (xn ¡ a)2 · 1

2(jx1 ¡ aj+ ¢ ¢ ¢+ jxn ¡ aj)2

Page 150: Baltic Way 팀수학경시대회 1990-2005 pdf 보기

150

증명1 (KAIST 수학과 03학번 윤태동)

zi := xi ¡ a (i = 1; 2; : : : ; n) 로 치환하자. 그러면

z1 + z2 + ¢ ¢ ¢+ zn = 0

이제 준 부등식은 다음과 같이 나타낼 수 있다.

z21 + z22 + ¢ ¢ ¢+ z2n · 1

2(jz1j+ jz2j+ ¢ ¢ ¢+ jznj)2

양변에 2를 곱하고 소거하여 동치변형하면

z21 + ¢ ¢ ¢+ z2n · 2 ¢Xi<j

jzizj j

그런데, 0 = (z1 + ¢ ¢ ¢+ zn)2 = z21 + ¢ ¢ ¢+ z2n + 2

Pi<j zizj 임에서

2 ¢Xi<j

(¡zizj) · 2 ¢Xi<j

jzizj j

와 같은 식이 된다. ¡zizj · jzizj j 는 당연하므로 이것은 잘 성립하는 식이고, 따라서 준 부등

식이 성립함이 증명되었다. 등호는 모든 zi의 부호가 서로 다를 때, 즉 n = 2 일 때 또는 zi가 모

두 0일 때 성립한다. ¤

증명2 (한국과학영재학교 1학년 김민기)

역시 위의 치환을 이용하자.

z21 + ¢ ¢ ¢+ z2n · 2 ¢Xi<j

jzizj j

을 다시 쓰면

z21 + ¢ ¢ ¢+ z2n ·Xcyc.

jz1j(jz2j+ ¢ ¢ ¢+ jznj)

즉, Xcyc.

jz1j(jz2j+ ¢ ¢ ¢+ jznj ¡ jz1j) ¸ 0

이 된다. 그런데 z1 = ¡(z2 + ¢ ¢ ¢+ zn) 등이므로

jz2j+ ¢ ¢ ¢+ jznj ¡ jz1j = jz2j+ ¢ ¢ ¢+ jznj ¡ jz2 + ¢ ¢ ¢+ znj ¸ 0

의 삼각부등식으로 위의 식은 성립한다. 문제의 부등식은 이것과 동치이므로 역시 성립한다. ¤

증명3 같은 치환에서, 일반성을 잃지 않고 z1; : : : ; zp ¸ 0 이고 zp+1; : : : ; zn · 0 이라 하자.

z1 + ¢ ¢ ¢+ zp + zp+1 + ¢ ¢ ¢+ zn = 0 이므로,

z1 + ¢ ¢ ¢+ zp = a 라 하면 ¡ (zp+1 + ¢ ¢ ¢+ zn) = a

Page 151: Baltic Way 팀수학경시대회 1990-2005 pdf 보기

1997년 Baltic Way 풀이 151

이다. 이제

z21 + ¢ ¢ ¢+ z2n = (z21 + ¢ ¢ ¢+ z2p) + (z

2p+1 + ¢ ¢ ¢+ z2n)

· (z1 + ¢ ¢ ¢+ zp)2 + (zp+1 + ¢ ¢ ¢+ zn)

2

= a2 + a2 =1

2(a+ a)2

=1

2(jz1j+ ¢ ¢ ¢+ jznj)2

으로 부등식이 증명되었다. ¤

증명4 (O±cial Solution)

한 변의 길이가 jz1j+ ¢ ¢ ¢+ jznj 인 큰 정사각형 A 안에, 한 변의 길이가 jz1j; : : : ; jznj인 작은 정

사각형 Bi들이 차례로 A의 한 변에 내접하는 그림을 생각하자. 그럼 좌변은 Bi들의 넓이의 합

이고 우변은 A의 넓이의 절반이다.

A

B1 B2 ¢ ¢ ¢ Bn

jz1j = jz2 + ¢ ¢ ¢+ znj · jz2j+ ¢ ¢ ¢+ jznj 이므로 jz1j은 A의 한 변의 길이의 절반을 넘지 않고, 다

른 jzij들도 마찬가지이다. 즉, 모든 Bi들이 그림과 같이 A의 반쪽 영역에 모두 포함된다. 따라

서, 부등식이 성립한다. ¤

Baltic 1997-5Baltic 1997-5

양의 정수들의 수열 u0; u1; : : : 에서, u0은 임의의 수이고,

un+1 =

8><>:12un (un이 짝수일 때)

a+ un (un이 홀수일 때)

(n ¸ 0)

이다. 단, a는 주어진 양의 홀수이다. 이 수열이 언젠가부터는 주기적으로 반복됨을 보여라.

증명 (KAIST 수학과 03학번 윤태동)

우선 다음의 사실을 증명하자.

모든 n에 대하여 un · 2 ¢max(a; u0) (¤)

Page 152: Baltic Way 팀수학경시대회 1990-2005 pdf 보기

152

n에 대한 수학적 귀납법을 사용하자. n = 0; 1일 때 성립함은 쉽게 확인된다. n · k (k ¸ 1) 인모든 n에 대하여 준 명제가 성립함을 가정하고 n = k + 1 일 때 성립함을 보이자.

(i) 만일 uk가 짝수라면,

uk+1 =1

2uk · max(a; u0) · 2 ¢max(a; u0)

가 되므로 성립한다.

(ii) 만일 uk가 홀수라면, uk¡1은 짝수일 수밖에 없으므로,

uk+1 = a+ uk = a+1

2uk¡1 · a+max(a; u0) · 2 ¢max(a; u0)

가 되어 역시 성립한다.

따라서 (¤)이 증명되었다. 이제 (¤)에 의하여 수열 fung이 가질 수 있는 가능한 값은 1부터

2 ¢max(a; u0)까지의 정수들, 즉 유한개라는 것을 알 수 있다. 따라서 비둘기집의 정리에 의해

ui = uj 를 만족하는 i, j가 존재해야만 하고, ui부터 시작하는 수열과 uj부터 시작하는 수열이

동일하므로 수열은 순환할 수밖에 없다. ¤

별해 (O±cial Solution)

un > a 라 가정하자. 그럼, 만일 un이 짝수이면 un+1 =12un < un 이 되고, un이 홀수이면

un+1 = a+ un < 2un; un+2 =1

2un+1 < un

이 된다. 즉, 어떤 경우에나 un보다 작은 항이 반드시 나타나고, 이것을 계속 반복하면 언젠

가(유한 번 안에) un · a 인 항이 나타난다. 따라서, 어떤 m ¸ 0 에 대해서도 un · a 인 n > m

을 반드시 찾을 수 있고, 따라서 un · a 을 만족하는 n은 무한히 많다. a 이하의 자연수는 유한

개뿐이므로, 비둘기집의 원리에 의해 un · a 인 n들 중에 un = um 인 n, m이 반드시 존재한

다. 그럼 un으로부터 시작하여 jn¡mj을 주기로 갖는 수열이 된다. ¤

Baltic 1997-6Baltic 1997-6

a ¸ b ¸ c 와 1 ¢ a3 + 9 ¢ b2 + 9 ¢ c+ 7 = 1997 을 만족하는 음이 아닌 정수들의 순서쌍 (a; b; c)를 모

두 찾아라.

풀이 (KAIST 수학과 03학번 윤태동)

우선 주어진 조건에 의해 다음의 부등식이 만족해야 한다.

a3 + 7 · a3 + 9b2 + 9c+ 7 (= 1997) · a3 + 9a2 + 9a+ 7

양변은 a에 대한 증가함수이므로, 여기서 10 · a · 12 을 금방 알 수 있다. 또, 문제의 식에서

a3 + 7 ´ 8 (mod 9), 즉a3 ´ 1 (mod 9)

Page 153: Baltic Way 팀수학경시대회 1990-2005 pdf 보기

1997년 Baltic Way 풀이 153

인데, a = 10; 11; 12 중에서 이것을 만족하는 것은 a = 10뿐이다. 그럼

997 = 9b2 + 9c+ 7 · 9a2 + 9a+ 7 = 997

이 되고, 등호가 성립할 때이므로 b = c = a = 10 이다. ¢ ¢ ¢ 답 (10,10,10) }

Baltic 1997-7Baltic 1997-7

P와 Q는 정수 계수를 갖는 다항식이다. 정수 a와 a+1997 이 P의 해이고, Q(1998) = 2000 이라 가

정하자. 방정식 Q(P (x)) = 1 이 정수해를 갖지 않음을 보여라.

증명 (KAIST 수학과 03학번 윤태동)

a와 a + 1997 은 서로 부호가 다르므로 둘 중 하나는 짝수, 하나는 홀수이다. 이 두 수가 모두

P의 해가 되므로,

P (짝수) ´ P (홀수) ´ 0 (mod 2)

임을 알 수 있다. 즉, 모든 정수 x에 대하여 P (x)는 짝수. 한편 Q(1998) = 2000 이므로, Q(짝

수) ´ 0 (mod 2). 그러면 모든 정수 x에 대하여,

Q(P (x)) ´ Q(짝수) ´ 0 (mod 2)

따라서, Q(P (x))는 홀수인 1이 될 수 없다. ¤

Baltic 1997-8Baltic 1997-8

1996과 1997을 더할 때, 우리는 먼저 일의 자리의 6과 7을 더한다. 13을 얻으면 아래에 3을 쓰고,

1을 다음 열로 \자리올림"한다. 이러한 식으로 계속하면, 이 덧셈에는 모두 세 번의 자리올림을 하

게 됨을 알 수 있다.

3 9 9 3

+ 1 9 9 71 9 9 61 1 1

1996 ¢ k 와 1997 ¢ k 를 더할 때, 전체 계산에서 자리올림이 한 번도 일어나지 않도록 하는 양의 정수

k가 존재하는가?

풀이 (대전과학고 1학년 고기혁)

1996k = 2k ¢ 1000¡ 4k1997k = 2k ¢ 1000¡ 3k

이므로 3k가 1000의 배수에 가깝고 2k가 작은 자리수들로 구성되면 찾기가 쉬울 것 같다. 실제

로, k = 666 으로 하면

1996£ 666 = 1329336+ 1997£ 666 = 1330002

2659338

의 덧셈에서 자리올림이 발생하지 않는다. }

Page 154: Baltic Way 팀수학경시대회 1990-2005 pdf 보기

154

별해 두 수를 더했을 때 999 ¢ ¢ ¢ 9꼴이 된다면 그 덧셈에는 (일의 자리부터 차근차근 관찰하

면) 자리올림이 있을 수 없다. 따라서, 1996k + 1997k = 3993k = 999 ¢ ¢ ¢ 9꼴이 되는 경우가 있

다는 것만 보이면 된다. 이것은

10n ´ 1 (mod 3993)

의 자연수해 n이 있느냐를 묻는 것과 같다. 10; 102; 103; : : : 은 무한히 많은 항을 가지므로

10i ´ 10j (mod 3993)인 i > j 가 있을 수밖에 없고,그럼 10과 3993은 서로 소이므로 10i¡j ´ 1(mod 3993)이 된다.혹은오일러-페르마의 정리로부터 n = Á(3993) = Á(3¢113) = (3¡1)(113¡112) = 2420 으로 직접 제시할 수도 있다. }

주 컴퓨터 프로그램으로 찾은 자리올림이 없는 가장 작은 k는 162였다.

1996£ 162 = 323352+ 1997£ 162 = 323514

646866

Baltic 1997-9Baltic 1997-9

대세계 속의 모든 세계들에는 1; 2; 3; : : : 의 수가 붙여져 있고, 임의의 정수 n ¸ 1 에 대해, 수 n, 2n,

3n+ 1 의 세계들 사이에는 차원의 문이 연결되어 있어서 마법사 간달프가 양방향으로 오갈 수 있

다. 간달프가 어느 세계에서 여행을 시작해도 항상 다른 어떤 세계에든지 갈 수 있는가?

풀이 (KAIST 수학과 03학번 윤태동)

그렇다. 모든 세계가 1과 오갈 수 있다는 것을 보이자.

3n+ 1 » n (n ¸ 1)

3n+ 2 » 6n+ 4 » 2n+ 1 (n ¸ 0)

3n » 9n+ 1 » 36n+ 4 » 12n+ 1 » 4n » 2n (n ¸ 1)

위와 같이 1보다 큰 어떠한 세계에서도 자신보다 번호가 작은 어떤 세계와 오갈 수 있다. 이러

한 과정을 계속하다보면 자연수가 무한히 작아질 수는 없으므로 언젠가는 자신보다 작은 번호

가 없는 세계에 이르게 되며 그것이 바로 번호 1인 세계이다. 따라서 모든 세계에서 1로 오갈

수 있으므로, m » 1 » n 과 같이 임의의 두 세계 m과 n도 서로 오갈 수 있다. }

Baltic 1997-10Baltic 1997-10

임의의 연속하는 79개의 양의 정수들 중에는, 십진법으로 썼을 때 자리수의 합이 13의 배수가 되는

수가 항상 있음을 증명하여라.

Page 155: Baltic Way 팀수학경시대회 1990-2005 pdf 보기

1997년 Baltic Way 풀이 155

증명 십의 자리 이상은 모두 같고 일의 자리가 0부터 9까지 변하는 연속하는 10개의 자연수

들의 묶음을 한 `마디'라고 부르기로 하자. 임의의 연속하는 79개의 양의 정수들 중에는 항상

7개의 완전한 마디가 포함된다. 또, (마디와 마디의 사이에서) 백의 자리가 바뀌는 경우는 기껏

해야 1번뿐이므로, 7개의 마디 중에는 반드시 백의 자리 이상이 모두 같은 연속한 4개의 마디

가 있다. 귀류법으로, 자리수의 합이 13의 배수가 되는 수가 하나도 없다면, 각각의 마디는 자

리수의 합을 13으로 나눈 나머지가

(1; 2; : : : ; 10); (2; 3; : : : ; 11); (3; 4; : : : ; 12)

등의 세 가지 종류 중 하나여야 한다. 그런데, 백의 자리가 바뀌지 않으면서 다음 마디로 넘어

갈 때는 일의 자리가 9만큼 감소하고 십의 자리가 1만큼 증가하므로, 자리수의 합이 8만큼 감

소하게 된다. 즉,

(0; 1; : : : ; 9)! (1; 2; : : : ; 10)! (2; 3; : : : ; 11)! (3; 4; : : : ; 12)! (3; 4; : : : ; 0)

으로 바뀌어가게 되므로, 연속한 4개의 마디가 모두 앞의 세 가지 종류일 수는 없다. 따라서, 연

속한 79개의 자연수들 중에는 13의 배수가 항상 있다. ¤

Baltic 1997-11Baltic 1997-11

서로 다른 점들 A1; A2; A3; : : : 과 B1; B2; B3; : : : 을 두 개의 평행한 직선 위에 각각 jAiAi+1j = 1,jBiBi+1j = 2 (i = 1; 2; : : : )이 되도록 잡았다. \A1A2B1 = ® 일 때,무한합 \A1B1A2+\A2B2A3+\A3B3A4 + ¢ ¢ ¢ 의 값을 구하여라.

풀이 (KAIST 수학과 03학번 윤태동)

점 A1; A2; : : :의 반대쪽으로 1의 간격으로 좌표처럼 A0; A¡1; A¡2; : : : 의 점들을 잡자.

그러면 n ¸ 1인 n들에 대해서 AnBn k A2¡nB1 이 되므로,

\AnBnAn+1 = \A3¡nB1A2¡n

Page 156: Baltic Way 팀수학경시대회 1990-2005 pdf 보기

156

임을 쉽게 확인할 수 있다. 따라서,

\A1B1A2 + \A2B2A3 + \A3B3A4 + ¢ ¢ ¢= \A2B1A1 + \A1B1A0 +\A0B1A¡1 + ¢ ¢ ¢= \A2B1B0

이것은 \A1A2B1과 보각이므로 ¼ ¡ ® 가 된다. }

Baltic 1997-12Baltic 1997-12

두 원 C1, C2가 두 점 P와 Q에서 만난다. P를 지나는 직선이 C1, C2와 A, B에서 각각 다시 만나고,

X는 AB의 중점이다. Q와 X를 지나는 직선이 C1, C2와 Y , Z에서 각각 다시 만난다. X가 Y Z의

중점임을 증명하여라.

증명 A, B가 P에 대해 서로 반대쪽에 있을 때를 보자. A가 선분 PA 위에 있다고 해도 일반

성을 잃지 않고, 그럼 다음 그림과 같다.

문제의 조건에서 XA = XB, 그리고 원주각에 의해 \Y AX = \XQP = \ZBX, 또 X에서의

맞꼭지각까지 하여

4Y AX ´ 4ZBX

가 된다(ASA합동). 따라서, Y X = XZ 이므로 X는 Y Z의 중점이다.

A, B가 P에 대해 같은 쪽에 있을 때도 비슷한 과정으로(위의 그림에서 P;A;B와 Q;Y;Z를 서

로 바꾸어놓고 보면 된다) 성립한다. ¤

별증 (KAIST 수학과 03학번 윤태동)

원과 두 직선에 의한 비례관계에 의하여,

XY ¢XQ = XA ¢XP

XZ ¢XQ = XB ¢XP

XA = XB 이므로 XY = XZ. ¤

Page 157: Baltic Way 팀수학경시대회 1990-2005 pdf 보기

1997년 Baltic Way 풀이 157

Baltic 1997-13Baltic 1997-13

서로 다른 다섯 점 A, B, C, D, E가 jABj = jBCj = jCDj = jDEj 가 되게 한 직선 위에 놓여있

다. 점 F는 직선의 바깥에 있다. 삼각형 ADF의 외심을 G, 삼각형 BEF의 외심을 H라 하자. 직선

GH와 FC가 서로 수직임을 보여라.

증명 FC의 연장선이 4ADF , 4BEF의 외접원들과 만나는 점을 각각 X, Y라 하자.

그럼 원과 두 직선의 비례관계에 의해

FC ¢ CX = AC ¢ CD

FC ¢ CY = BC ¢ CE

AC = CE, BC = CD 이므로 이 두 식은 같고, 따라서 CX = CY , 즉 X = Y 이고 FC는 두 원

의 공통현과 같은 직선을 이룬다. 두 원의 중심을 잇는 직선과 공통현은 직교하므로, GH ? FC

이다. ¤

주 좌표기하로 놓고 풀 수도 있다. 막노동이라 별로 좋지 않은 방법이긴 하지만.

Baltic 1997-14Baltic 1997-14

삼각형 ABC에서 jACj2은 jBCj2과 jABj2의 산술평균이다. cot2B ¸ cotA cotC 임을 보여라.

증명 b = AC, a = BC, c = AB 라 하면 주어진 조건은 a2 + c2 = 2b2 이 된다. cot = cossin 이

므로 주어진 부등식은cos2B

sin2 B¸ cosA cosC

sinA sinC

와 같고, 여기에 싸인 법칙 sinA = a2R 등과 코싸인 제2법칙 cosA = b2+c2¡a2

2bc 등을 대입하자.

그럼 (2R)2은 모두 소거되고

(a2 + c2 ¡ b2)2

(2ac)2 ¢ b2 ¸ (b2 + c2 ¡ a2)(a2 + b2 ¡ c2)

2bc ¢ 2ab ¢ ac(a2 + c2 ¡ b2)2 ¸ (b2 + c2 ¡ a2)(a2 + b2 ¡ c2)

Page 158: Baltic Way 팀수학경시대회 1990-2005 pdf 보기

158

이 된다. 산술-기하 평균부등식에서

(b2 + c2 ¡ a2)(a2 + b2 ¡ c2) ·µ(b2 + c2 ¡ a2) + (a2 + b2 ¡ c2)

2

¶2= b4

이고, a2 + c2 = 2b2 이므로 b4 = (a2 + c2 ¡ b2)2 이 되어 주어진 부등식은 성립한다. ¤

증명 (서울 서초고 1학년 정명진)

2b2 = a2 + c2 에 코싸인 제2법칙을 대입, 변형하여 다음을 얻을 수 있다.

2(a2 + c2 ¡ 2ac cosB) = (b2 + c2 ¡ 2bc cosA) + (a2 + b2 ¡ 2ab cosC)2ac cosB = bc cosA+ ab cosC

2cosB

b=cosA

a+cosC

c

2cosB

sinB=cosA

sinA+cosC

sinC(* 싸인 법칙)

cotB =cotA+ cotC

2¸pcotA cotC (산술-기하)

따라서, 문제의 부등식이 성립한다. ¤

Baltic 1997-15Baltic 1997-15

예각삼각형 ABC에서 \A, \B, \C의 이등분선이 외접원과 각각 A1, B1, C1에서 다시 만난다.

AB와 B1C1의 교점을 M , BC와 A1B1의 교점을 N이라 하자. MN이 삼각형 ABC의 내심을 지남

을 증명하여라.

증명 (대전과학고 1학년 고기혁)

내심을 I라 하자.

이등분각과 원주각에서

\MBI = \NBI ´ \CBB1 = \MC1I

이므로 IBC1M은 원에 내접하는 사각형이다. 마찬가지로, IBA1N도 원에 내접한다. 따라서,

\IMB = \IC1B ´ \CC1B = \A

같이 하여 \INB = \C

Page 159: Baltic Way 팀수학경시대회 1990-2005 pdf 보기

1997년 Baltic Way 풀이 159

이고, 이로부터

IM k CA k IN

임을 알 수 있다. 따라서, M , I, N은 한 직선 위에 있다. ¤

별증 (KAIST 수학과 03학번 윤태동)

위의 풀이처럼 IBC1M , IBA1N이 원에 내접함을 확인했으면,

\MIB = 180± ¡\MC1B ´ 180± ¡ \B1C1B = \B1A1B ´ \NA1B = 180± ¡ \NIB

로 \MIB + \NIB = 180±, 즉 MIB가 일직선을 이룸을 확인할 수 있다. ¤

주 C1A = C1I = C1B, B1C1 ? AA1 등이 잘 알려져 있으므로 B1C1은 AI를 수직이등분한

다. 따라서, B1C1 \ AC = K 라 하면 AMIK 등이 마름모가 됨을 확인할 수도 있고, 이것으로

도 증명이 된다.

Baltic 1997-16Baltic 1997-16

5£ 5 체스판에서 두 명이 다음과 같은 경기를 한다. 첫 번째 선수가 knight를 어떤 칸 위에 놓는다.

그럼 두 번째 선수부터 시작하여 번갈아가며 그 knight를 체스의 규칙에 따라 움직인다. 한 번 지나

간 칸으로 knight를 옮기는 것은 허용되지 않고, 더 이상 움직일 수 없는 선수가 진다. 둘 중 누가 승

리의 전략을 가지게 되는가?

풀이 (KAIST 수학과 03학번 윤태동)

첫 번째 사람이 승리의 전략을 갖는다. 우선 처음에 체스판의 한쪽 모서리에 knight를 놓은 다

음 아래의 표대로 말을 움직이면 된다.

07 12 17 22 05

18 23 06 11 16

13 08 25 04 21

24 19 02 15 10

01 14 09 20 03

01! 02 로 가는 부분과 09! 10 으로 가는 부분은 두 번째 사람이 두 가지의 선택을 할 수 있

지만 두 가지의 선택이 대칭적이므로 이 표를 대칭적으로 변형시키면 마찬가지의 승리의 전략

을 얻을 수 있다. }

별해 (O±cial Solution)

첫 번째 사람이 승리의 전략을 갖는다. 아래의 그림은 X를 제외한 각각의 칸을, 서로 직접 오갈

Page 160: Baltic Way 팀수학경시대회 1990-2005 pdf 보기

160

수 있는 위치끼리 둘씩 묶어 같은 수를 배정한 것이다.

X 12 8 3 11

5 3 11 1 7

12 8 6 10 4

2 5 9 7 1

9 6 2 4 10

첫 번째 사람은 처음에 X의 위치에 knight를 놓고, 그 다음부터는 두 번째 사람이 어느 칸으로

움직이든 항상 그 칸과 같은 수가 적힌 칸으로 움직이면 된다. 그럼 두 번째 사람이 움직일 수

있으면 첫 번째 사람도 항상 움직일 수 있으므로, 더이상 못 움직이는 때가 온다면 그것은 두

번째 사람의 차례인 때일 수밖에 없다. }

Baltic 1997-17Baltic 1997-17

어떤 직사각형은 n개의 같은 크기의 정사각형들로 나뉘어질 수 있다. 이 직사각형은 n+76 개의 같

은 크기의 정사각형들로도 나뉘어질 수 있다. 가능한 n의 값을 모두 구하여라.

풀이 n개의 직사각형으로 나누었을 때 가로 방향의 개수를 a, 세로 방향의 개수를 b라 하

자(a ¸ b, 즉 긴 쪽을 가로 방향으로 하자). 즉, n = ab 이고, 마찬가지로 n+ 76 = cd 라 하자.

a : b = c : d 이므로,

a

b=

c

d=

p

q

인 기약분수 pq가 존재한다. 그럼

a = px; b = qx; c = py; d = qy

라 둘 수 있다. 이것을 cd¡ ab = 76 에 대입하면

pq(y2 ¡ x2) = 76

pq(y + x)(y ¡ x) = 22 ¢ 19

y + x 와 y ¡ x 는 홀짝이 같고 y + x > y ¡ x 이므로

(y + x; y ¡ x) = (19; 1) 또는 (2 ¢ 19; 2 ¢ 1)

이다. 후자의 경우는 전자의 경우보다 x, y가 2배씩인데 pq는 14배이므로, 어느 경우나 n = ab =

pqx2 = 4 ¢ 92 = 324 로 같다. 실제로 구해보면 각각 (a; b; c; d) = (36; 9; 40; 10) 과 (18; 18; 20; 20)

이 된다. ¢ ¢ ¢ 답 324 }

Page 161: Baltic Way 팀수학경시대회 1990-2005 pdf 보기

1997년 Baltic Way 풀이 161

Baltic 1997-18Baltic 1997-18

(a) 임의의 음이 아닌 정수 n이 n = a+ b (a 2 A; b 2 B) 꼴로 유일하게 나타내어지도록 하는, 음

이 아닌 정수들로 이루어진|반드시 서로 소일 필요는 없는|두 개의 무한 집합 A, B의 존재

성을 보여라.

(b) 위와 같은 각 순서쌍 (A;B)에 대해, A나 B는 어떤 정수 k > 1 의 배수만을 포함함을 증명하

여라.

증명 (a) 음 아닌 정수들을 k진법으로 나타냈을 때, 홀수번째 자리수(k2r자리의 수)가 모두

0인 수들의 집합을 A, 짝수번째 자리수가 모두 0인 수들의 집합을 B라 하자. 그럼 이 두 집합

이 조건을 만족한다는 것은 아주 당연하다.

(b) 0은 0 + 0으로만 표현되므로 A, B 모두 0을 포함한다. n ¸ 1 은 항상 A, B 중이 A, B 양쪽

에 모두 들어있으면 n = n + 0 = 0 + n 으로 유일성을 만족하지 않으므로 곤란하다. 1을 포함

하는 것을 A라 하고, A에 들어있지 않은 가장 작은 자연수를 k ¸ 2 라 하자. 그럼

0; 1; : : : ; k ¡ 1 2 A; k =2 A; 1; : : : ; k ¡ 1 =2 B

이므로, k = a+ b꼴로 표현하는 방법은 k = 0 + k 만 가능하다. 그러므로,

k 2 B

이다. 이제 문제에서 증명하고자 하는 것보다 좀더 강력한 다음 claim을 수학적 귀납법으로 보

이자.

Claim B는 k의 배수만을 갖는다. 그리고, A가 k의 배수를 가지면, 그것을 tk라 하면,

tk; tk + 1; : : : ; tk + (k ¡ 1) 2 A 이다.

Claim의 증명 tk 이전까지 성립한다고 가정하자. t = 1 일 때는 이미 제시되었다. tk +

1; : : : ; tk + (k ¡ 1) 이 하나도 B에 포함되지 않고, 또 만일 하나라도 A에 포함되는 것이

있다면 모두다(tk까지) A에 포함됨을 보이면 된다.

(1) tk 2 A 인 경우: 0 < r < k 에 대해 tk + r 2 B 라면

(t+ 1)k = (k ¡ r) + (tk + r) = tk + k

로 유일성에 모순. 따라서, B에 대해서는 확인되었다. tk + r =2 A 라면 tk + r =

(uk+r)+vk 꼴로 표현되어야 하는데,그럼 귀납법의 가정에 의해 uk; uk+1; : : : ; uk+

(k ¡ 1) 2 A 이므로,

tk = uk + vk = tk + 0

으로 유일성에 모순. 따라서, A에 대해서도 확인되었다.

Page 162: Baltic Way 팀수학경시대회 1990-2005 pdf 보기

162

(2) tk 2 B 인 경우:

(tk + r) = r + tk = (tk + r) + 0 = 0 + (tk + r)

이므로 tk + r =2 A;B 임을 알 수 있다.

(3) tk =2 A[B 인 경우: tk = uk+ vk꼴로 유일하게 표현되어야 하고, 그럼 귀납법의 가

정에 의해 uk; uk + 1; : : : ; uk + (k ¡ 1) 2 A 이다. 그럼

(tk + r) = (uk + r) + vk = (tk + r) + 0 = 0 + (tk + r)

이므로 역시 tk + r =2 A;B 임을 알 수 있다.

(1){(3)에 의해 claim이 증명되었다. ¤

따라서, claim에 의해, B는 k의 배수만을 원소로 갖는다. ¤

Baltic 1997-19Baltic 1997-19

어느 숲 속에는 n(¸ 3) 마리의 동물들이 자기 굴에서 살고있고, 임의의 두 굴은 각각 정확히 하나

의 독립된 통로로 연결되어 있다. 숲의 왕을 뽑기에 앞서 몇 동물들이 선거 운동을 벌였다. 선거 운

동을 하는 각 동물은 다른 굴들을 정확히 한 번씩 방문했고, 굴과 굴 사이를 이동할 때는 굴통로만

을 이용했으며, 한 통로를 지나다 다른 통로로 옮겨가지도 않았고, 선거 운동의 마지막에는 자신의

굴로 돌아왔다. 또한 어떤 굴통로도 기껏해야 한 마리의 선거 운동 동물에 의해서만 사용되었다고

한다.

(a) 임의의 소수 n에 대해, 선거 운동을 하는 동물의 수는 최대n¡ 12

마리임을 증명하여라.

(b) n = 9 일 때, 선거 운동 동물은 최대 몇 마리인가?

풀이 (대전과학고 1학년 맹진현)

입후보자수를 k마리라 하자. 각자 n개을 굴을 들러 돌아와야 하므로 n개씩의 통로를 지나고,

통로는 임의의 두 굴마다 하나씩 있으므로, 전체 입후보자들이 들르는 통로의 수는

kn ·Ãn

2

!=

n(n¡ 1)2

따라서, 양변을 n으로 나누면 입후보자는 언제나 n¡12 마리 이하이다.

(a) n이 소수이면 m = n¡12 이 정확한 최대값임을 보이자. 각각의 동물마다 0; 1; : : : ; n¡ 1의 번

호를 붙이고, 각 후보자를 a1; : : : ; am 이라 하자. 그럼 후보자 ai는 현재 들른 굴이 k번이면 다

음은 k + i번(mod n) 굴로 옮겨가는 것으로 하면 된다.

(b) 4마리이다. n = 9 일 때 n¡12= 4 이므로 4마리보다 많을 수는 없고, 4마리는 다음과 같이

Page 163: Baltic Way 팀수학경시대회 1990-2005 pdf 보기

1997년 Baltic Way 풀이 163

하면 실제로 가능하다.

각각 점선과 실선에 한 마리씩 해당하는, 두 마리씩을 그린 그림이다. }

Baltic 1997-20Baltic 1997-20

12장의 카드가 한 줄로 놓여있다. 카드는 두 면 다 흰 것, 두 면 다 검은 것, 검은 면과 흰 면이 하나

씩 있는 것 등 모두 3종류이다. 처음에는 12장 중 9장의 카드가 검은 면을 보이고 있었다. 1{6번의

카드들을 뒤집었더니, 이제는 12장 중 4장의 카드가 검은 면을 보이게 되었다. 다시 4{9번의 카드들

을 뒤집었고, 6장의 카드가 검은 면을 보이게 되었다. 마지막으로 1{3번의 카드와 10{12번의 카드

를 뒤집은 후에는 5장의 카드가 검은 면을 보였다. 각 종류의 카드가 각각 몇 장씩 있는가?

풀이 9장, 4장, 6장, 5장의 검은 면을 보인 때를 차례로 1단게, 2단게, 3단계, 4단계라고 하자.

위의 시행을 검토하면 2단계와 4단계는 모든 카드가 서로 완전히 뒤집혀져있음을 알 수 있다.

따라서, 모든 카드의 양면을 모두 세었을 때 검은 면은 모두 9면이다. 그럼 1단계에서 모든 검

은 면이 다 보여지고 있으므로 1단계에서 가려진 뒷면은 모두 흰 면이고, 따라서, 검은 면과 흰

면이 하나씩 있는 것이 9장, 두 면 다 흰 것이 3장 있고, 두 면 다 검은 것은 한 장도 없다. }

Page 164: Baltic Way 팀수학경시대회 1990-2005 pdf 보기

164

1998년 Baltic Way 풀이

Baltic 1998-1Baltic 1998-1

Z+ 을 모든 양의 정수의 집합이라 하자. 모든 x; y 2 Z+ 에 대해서 다음 조건을 만족하는 함수

f : Z+ ! Z+ 을 모두 찾아라.

f(x; x) = x

f(x; y) = f(y; x)

(x+ y)f(x; y) = yf(x; x+ y)

풀이 먼저 lcm(x; y)가 f의 조건을 모두 만족시킨다는 것을 눈치챌 수 있다. 처음 두 조건은

자명하고, 마지막 조건은 잘 알려진 AB = GL, 즉

x(x+ y) = gcd(x; x+ y) ¢ lcm(x; x+ y)

xy = gcd(x; y) ¢ lcm(x; y)

의 식에서 gcd(x; x+ y) = gcd(x; y) 임에 주목하면 역시 성립함을 확인할 수 있다.

이제 이 lcm함수가 문제의 조건을 만족하는 유일한 f임을 보이자. 주어진 조건식에 의해 이 함

수가 유일하게 결정된다는 것만 확인하면 충분하다. x+ y 의 값에 대한 수학적 귀납법으로 보

이기로 하겠다. 먼저 x + y = 2 일 때는 x = y = 1 일 때뿐이고, 그럼 첫 번째 조건에 의해

f(x; y) = f(1; 1) = 1 로 함수값이 결정된다. 이제 x + y < n 일 때 함수값이 유일하게 결정됨

을 가정하고, x+ y = n 인 (x; y)들에 대해서도 함수값이 유일하게 결정됨을 보이자. x = y 인

경우는 첫 번째 조건에 의해 자명하고, x > y 인 경우는 두 번째 조건에 의해 x < y 인 경우로

바꾸어줄 수 있으므로 x < y 인 경우만 확인하면 된다. 이 때, y ¡ x = y0 이라 하면

x+ y0 < n (* y < x+ y)

이므로 귀납법의 가정에 의해 f(x; y0)의 값은 결정되어 있고,그럼 세 번째 조건에 의해 f(x; x+

y0), 즉 f(x; y)의 값도 유일하게 결정된다.

따라서, f가 유일함을 증명하였고, 그럼 f = lcm 일 수밖에 없다. }

Baltic 1998-2Baltic 1998-2

양의 정수 a, b, c를 세 변의 길이로 갖는 삼각형이 존재하고, 변 c의 대각의 크기가 120±이면 순서

쌍 (a; b; c)를 준-피타고라스쌍이라고 부른다. 만약 (a; b; c)가 준-피타고라스쌍이면 c가 5보다 큰 소

인수를 가짐을 보여라.

Page 165: Baltic Way 팀수학경시대회 1990-2005 pdf 보기

1998년 Baltic Way 풀이 165

증명 (O±cial Solution)

코사인 제 2법칙을 사용하면

c2 = a2 + b2 + ab

이다. d = gcd(a; b; c), (a; b; c) = (dx; dy; dz) 라 하면 z2 = x2 + y2 + xy 이고, z의 소인수는 c의

소인수이기도 하므로, d = 1 일 때만 생각해도 된다. c가 5보다 큰 소인수를 갖지 않는다면, c는

2, 3, 5만을 소인수로 갖는다(¤).

(1) 2 j c 라면: a, b, c가 서로 소임에서 a, b 중 적어도 하나는 홀수이다. 둘 중 하나만 홀수일

때나 둘다 홀수일 때나 a2+ b2+ ab 는 항상 홀수이다. 이것은 c가 짝수임에 모순. 따라서,

c는 2를 소인수로 갖지 않는다.

(2) 3 j c 라면:

4c2 = 4a2 + 4ab+ 4b2 = 3a2 + (a+ 2b)2

에서 3 j 4c2, 3a2 이므로 3 j (a + 2b)2 이고 그럼 제곱수임에서 9 j (a + 2b)2 이다. 다시

9 j 4c2, (a + 2b)2 이므로 9 j 3a2 이고 그럼 3 j a. 이 때 또 3 j b 가 되어 a, b, c가 서로 소

임에 모순. 따라서, c는 3도 소인수로 갖지 않는다.

(3) 5 j c 라면: 다시 5 j 4c2 = 3a2 + (a + 2b)2 의 식에서, a, b, c가 서로 소이므로 5 - 3a2,

(a+ 2b)2. 그런데, 제곱수는 5로 나눈 나머지가 0 이외에 §1뿐이므로, 3a2 + (a+ 2b)2 ´§3§ 1 6= 0 (mod 5) 이 되어 모순이다. 즉, c는 5도 소인수로 갖지 않는다.

(¤), (1), (2), (3)에서 c는어떤소인수도가질수없으니 c = 1이어야하는데 c2 = a2+b2+ab > 1

이므로 모순. 따라서, (¤)의 가정이 틀렸고, c는 5보다 큰 소인수를 갖는다. ¤

주 (서울 경희고 1학년 이승후)

이차잉여를 사용하면 c의 소인수에 대해 보다 더 강한 사실을 얻을 수도 있다. p j c 인 임의의

홀수인 소수 p에 대해 생각하자. p j c2 이므로

a2 + ab+ b2 ´ 0 (mod p)

(2a+ b)2 + 3b2 ´ 0 (mod p)

(2a+ b)2 ´ ¡3b2 (mod p)

따라서,

1 =

µ(2a+ b)2

p

¶=

µ¡3b2p

¶=

µ¡3p

¶이다. 단, (np )는 르장드르 기호. 잘 알려진 르장드르 기호의 성질들로부터

1 =

µ¡3p

¶=

µ¡1p

¶µ3

p

¶= (¡1) p¡12

³p3

´(¡1)p¡12 =

³p3

´따라서, p는 3k+1꼴의 소수이고,특히 홀수이므로 6k+1꼴의 소수이다.즉, c는 gcd(a; b; c) = 1

일 때 6k + 1꼴의 소인수만을 갖는다.

Page 166: Baltic Way 팀수학경시대회 1990-2005 pdf 보기

166

Baltic 1998-3Baltic 1998-3

다음 방정식을 만족하는 양의 정수쌍 (x; y)를 모두 찾아라.

2x2 + 5y2 = 11(xy ¡ 11)

풀이 (KAIST 수학과 03학번 김재훈)

준식을 다음과 같이 변형할 수 있다.

(2x¡ y)(x¡ 5y) = ¡121

좌변의 두수는 모두 정수이므로

(2x¡ y; x¡ 5y) = (§1;¨121); (§11;¨11)

일 경우만 조사해보면 된다(복호동순).

x =5(2x¡ y)¡ (x¡ 5y)

9; y =

(2x¡ y)¡ 2(x¡ 5y)9

로 각각 풀면 (x; y) = (§14;§27), (§ 223;§ 11

3) 이므로, 정수해는 (14; 27), (¡14;¡27)가 전부임

을 알 수 있다. }

Baltic 1998-4Baltic 1998-4

P를 정수 계수를 갖는 다항식이라고 하자. n = 1; 2; 3; : : : ; 1998 에 대해서 P (n)이 세 자리의 양의

정수라고 가정하자. 다항식 P가 정수해를 갖지 않음을 보여라.

증명1 (서울 경희고 1학년 이승후)

P (x) = 0 인 정수해 x가 존재한다고 하자. 세 자리의 수는 0이 아니므로 1 · x · 1998 이 아님

은 자명하고, 그럼 x · 0 또는 x ¸ 1999 이다.

(1) x · 0 일 때: 인수 정리에 의해

(1998¡ x) j P (1998)¡ P (x) = P (1998)

그런데, 1998¡ x는 1998 이상의 수이고, P (1998)은 세 자리의 수이므로 배수가 될 수 없

다. 따라서, 모순.

(2) x ¸ 1999 일 때:

(x¡ 1) j P (x)¡ P (1) = ¡P (1)

을 생각하면 마찬가지로 모순이 나온다.

따라서, P는 정수해를 가질 수 없다. ¤

Page 167: Baltic Way 팀수학경시대회 1990-2005 pdf 보기

1998년 Baltic Way 풀이 167

증명2 (수원 산남중 3학년 문지훈)

i; j 2 f1; 2; : : : ; 1998g 에 대해 jP (i)¡ P (j)j · 899 이고, 인수 정리에 의해

ji¡ jj j jP (i)¡ P (j)j · 899

이므로 ji ¡ jj ¸ 900 이면 P (i) = P (j) 일 수밖에 없다. 즉, P (1) = P (901) = P (902) = ¢ ¢ ¢ =P (1998) 이고 P (1998) = P (1098) = P (1097) = ¢ ¢ ¢ = P (1) 이므로

P (1) = P (2) = ¢ ¢ ¢ = P (1998) = k (상수)

가 된다. 따라서, 인수 정리에 의해

P (x)¡ k = (x¡ 1)(x¡ 2) ¢ ¢ ¢ (x¡ 1998)Q(x)

인 정수 계수 다항식 Q(x)가 존재한다. 만일 P (n) = 0 이라면 ¡k = (n ¡ 1)(n ¡ 2) ¢ ¢ ¢ (n ¡1998)Q(n) 인데, 우변은 연속한 1998개의 정수가 곱해져있으므로 0이거나 1998!보다 절대값이

큰데 좌변은 세 자리의 수이므로 등호가 성립할 수 없다. 따라서, 이런 n은 존재하지 않는다. ¤

증명3 a ´ b (mod m) 이면 P (a) ´ P (b) (mod m) 이다. 따라서, 임의의 정수 n에 대해

P (n) ´ P (a) (mod 1998)

인 a 2 f1; 2; : : : ; 1998g 가 항상 하나씩 존재한다. 이 때 P (a)는 세 자리의 수로 1998의 배수가

될 수 없으므로 P (n)도 1998의 배수가 될 수 없다. 따라서, P (n) = 0 일 수 없다. ¤

Baltic 1998-5Baltic 1998-5

a는 홀수인 한 자리 숫자, b는 짝수인 한 자리 숫자이다. 모든 양의 정수 n에 대해서, 십진법으로 썼

을 때 a와 b만을 자리수로 갖는 양의 정수 중에 2n의 배수가 존재함을 보여라.

증명 (KAIST 수학과 03학번 김재훈)

좀더 강한 명제로, a와 b만을 자리수로 갖는 n자리의 수 중에 2n의 배수가 존재함을 보이자. 귀

납법을 사용하자. 일단 n = 1 일 때는 b이면 가능하다. n = k 일 때 a와 b로만 구성된 k자리의

수 A가 2k의 배수라고 하자. 그럼 A ´ 0 or 2k (mod 2k+1) 이다.

(i) A ´ 0 (mod 2k+1) 일 때:

B = b ¢ 10k +A = bA

가 a와 b로만 구성된 k + 1자리의 2k+1의 배수가 된다. b가 짝수이므로 b ¢ 10k 도 2k+1의

배수임을 알 수 있다.

(ii) A ´ 2k (mod 2k+1) 일 때:

B = a ¢ 10k +A = aA

가 a와 b로만 구성된 k+1자리의 2k+1의 배수가 된다. a¢10k = (홀수)¢2k ´ 2k (mod 2k+1)이기 때문이다.

Page 168: Baltic Way 팀수학경시대회 1990-2005 pdf 보기

168

따라서 n = k 일 때 성립하면 n = k + 1 일 때도 성립하고, 수학적 귀납법에 의해서 모든 정수

n에 대해서 성립한다. ¤

Baltic 1998-6Baltic 1998-6

P는 6차 다항식이고, a와 b는 0 < a < b 인 실수이다. P (a) = P (¡a), P (b) = P (¡b), P 0(0) = 0 이

라 할 때, 모든 실수 x에 대해 P (x) = P (¡x) 임을 보여라.

증명 (KAIST 수학과 03학번 김재훈)

P (x) = Ax6 + Bx5 + Cx4 +Dx3 + Ex2 + Fx + G 라고 하자. P 0(0) = 0 에서 F = 0 임을 알

수 있고 P (a)¡ P (¡a) = 0, P (b)¡ P (¡b) = 0 에서

2(Ba5 +Da3) = 2(Bb5 +Db3) = 0

임을 알 수 있다. a3, b3은 0이 아니므로 Ba2+D = Bb2+D = 0이어야 한다. 그런데 0 < a < b

이므로 B, D가 모두 0이 아니라고 하면 Ba2+D < Bb2+D 이 되어 모순이다.그러므로 B, D는

둘다 0일 수밖에 없다. 따라서 P (x) = Ax6 + Cx4Ex2 + G 이므로 우함수이고 P (x) = P (¡x)이다. ¤

Baltic 1998-7Baltic 1998-7

R을 모든 실수들의 집합이라고 하자.모든 x; y 2 R에 대해 다음 방정식을 만족하는 함수 f : R! R을 모두 찾아라.

f(x) + f(y) = f(f(x)f(y))

풀이 (KAIST 수학과 03학번 김재훈)

f(2f(x)f(y)) = f((f(x) + f(x))f(y)) = f(f(f(x)2)f(y)) = f(f(x)2) + f(y) = 2f(x) + f(y)

f(2f(x)f(y)) = f((f(y) + f(y))f(x)) = f(f(f(y)2)f(x)) = f(f(y)2) + f(x) = 2f(y) + f(x)

따라서, 2f(x) + f(y) = f(x) + 2f(y), 즉 f(x) = f(y) 로 f(x) = c 의 상수함수이다. 다시 준

식에 대입하면 c+ c = c 이므로 c = 0 이 된다. 즉, f(x) = 0 의 영함수가 유일하다. }

Baltic 1998-8Baltic 1998-8

Pk(x) = 1 + x+ x2 + ¢ ¢ ¢ + xk¡1 라 하자. 모든 실수 x와 모든 양의 정수 n에 대해서 다음 식이 성

립함을 보여라.nX

k=1

Ãn

k

!Pk(x) = 2

n¡1Pn

µ1 + x

2

Page 169: Baltic Way 팀수학경시대회 1990-2005 pdf 보기

1998년 Baltic Way 풀이 169

증명 x 6= 1 일 때 Pk(x) =xk ¡ 1x¡ 1 이므로

(우변) =2n

2¢¡1+x2

¢n ¡ 11+x2 ¡ 1 =

(1 + x)n ¡ 2nx¡ 1

=

nPk=1

¡nk

¢xk ¡

nPk=1

¡nk

¢x¡ 1 =

nXk=1

Ãn

k

!xk ¡ 1x¡ 1

= (좌변)

이다. 문제의 양변은 n차 다항식이고 무한히 많은 해를 갖고 있으므로 항등정리에 의해 x = 1

일 때도 성립한다. ¤

주 x = 1 일 때 굳이 따로 본다면Pn

k=1

¡nk

¢k = n ¢ 2n¡1 의 식이 되는데 이것은 잘 알려진 식

이다. 조합적 논의로, n명 중에서 부분집합(위원회)을 하나 구성하는데, 그 중 한 명을 위원장

으로 하는 경우의 수로 생각하면 쉽게 증명된다.

Baltic 1998-9Baltic 1998-9

®, ¯는 0 < ® < ¯ < ¼=2 를 만족하고, °와 ±는 다음과 같이 정의되는 수이다.

(i) 0 < ° < ¼=2 이고, tan °는 tan®와 tan¯의 산술평균이다.

(ii) 0 < ± < ¼=2 이고,1

cos ±는

1

cos®와

1

cos¯의 산술평균이다.

° < ± 임을 보여라.

증명 (KAIST 수학과 03학번 김재훈)

아래 그림처럼 AB의 길이가 1이고 \CAB = ®, \DAB = ¯ 인 직각삼각형들을 생각해보자.

이 때 tan® = BC, tan¯ = BD 이다. 따라서 CD의 중점을 M으로 두면 BM은 CM과 DM의

산술평균이므로 °의 정의에 따라 \MAB = ° 이다. 그리고, 이 그림에서 빗변들의 길이가

AC =1

cos®; AD =

1

cos¯; AM =

1

cos °

이 된다. 그럼 ±의 정의에 따라 1cos ± =

AC+AD2 이다. 중선에 관한 성질

AM <AC +AD

2

Page 170: Baltic Way 팀수학경시대회 1990-2005 pdf 보기

170

이 성립함이 잘 알려져있는데(굳이 증명하자면, A의 M에 대한 대칭점 P를 잡으면 AC+AD =

AC + CP > AP = 2AM), 이것은1

cos °<

1

cos ±

임을 의미한다. cosx는 0 < x < ¼=2 에서 감소하므로 1= cosx는 증가하고, 따라서, ° < ± 이다.

¤

주 1cos x

=p1 + tan2 x 이므로 f(x) =

p1 + x2 로 정의하면 증명하고자 하는 것은

f(tan °) = f

µtan®+ tan¯

2

¶<

f(tan®) + f(tan¯)

2= f(tan ±)

이 된다. 즉, f가 아래로 볼록함을 확인하여 증명할 수도 있다.

Baltic 1998-10Baltic 1998-10

n ¸ 4 은 짝수이다. 정 n각형과 정 (n¡ 1)각형이 단위원에 내접한다. 이 n각형의 각각의 꼭지점에

대해서, 원주를 따라 재었을 때 그 점과 가장 가까운 (n¡ 1)각형의 꼭지점까지의 거리를 생각한다.

S를 이 n개의 거리의 합이라 하자. S가 n에 의해서만 결정되고 두 다각형의 상대적 위치와는 상관

이 없음을 보여라.

증명 (KAIST 수학과 03학번 김재훈)

n각형의 꼭지점을 차례로 A0; A1; A2; : : : 라 하고 n ¡ 1각형의 꼭지점도 B0; B1; B2; : : : , 그리

고 A0에 가장 가까운 점을 B0이라 하자. A0의 대칭점을 A00이라 하면 A00 = An2

이고, A00은

Bn2 ¡1과 Bn

2사이에 있다(굳이 증명하자면 A00의 편각이 Bn

2¡1의 편각과 Bn2의 편각 사이에 있

음을 부등식으로 보이면 되고, 이것은 쉽다).

특히 B0의 대칭점을 B00이라 하면, B00은 호 Bn2¡1Bn

2의 중점이다. 따라서, A00에 가장 가까운

점은 Bn2¡1과 Bn

2둘 중 하나이고, 그것을 B000이라 하면 그림에서

A0B0 +A00B000 = A00B

00 +A00B

000 = B00 +B000

=c

2(n¡ 1)

Page 171: Baltic Way 팀수학경시대회 1990-2005 pdf 보기

1998년 Baltic Way 풀이 171

이 됨을 알 수 있다(c는 원주의 길이, 즉 2¼). 즉, n각형의 꼭지점들은 대칭점들끼리 짝을 이뤄,

가장 가까운 n¡ 1각형의 꼭지점과의 원주거리의 합이 항상 c2(n¡1)로 일정하다. 따라서, 구하는

n개의 거리의 총합은 n2 ¢ c

2n¡2 로 역시 항상 일정하다. ¤

별증1 n각형의 꼭지점들로 나뉘어진 원주 위의 n개의 구간(같은 길이의 호)을 생각하자. 이

중 n¡ 1각형의 꼭지점을 하나도 포함하지 않는 곳이 있으므로, 그곳을 호 AA0, 그리고 이 호를

포함하는 n¡ 1각형의 호를 BB0이라 하자(B, B0은 각각 A, A0에 가장 가까운 꼭지점). 또, 원

주거리로 d = AB, d0 = A0B0 라 하자. 그 합을 k라 하면

k = d+ d0 = BB0 ¡AA0 =c

n(n¡ 1) (c = 2¼)

이다. 그럼 A를 지나는 지름 l로 원을 두 부분으로 나누었을 때, B가 있는 쪽의 근거리(n각형

의 꼭지점과 가장 가까운 n¡ 1각형의 꼭지점과의 원주거리)들은 차례로

d; d+ k; d+ 2k; : : : ; d+ (n2 ¡ 1)k

가 되고, 반대쪽의 근거리들도 차례로

d0; d0 + k; d0 + 2k; : : : ; d0 + (n2 ¡ 1)k

가 된다. 이것들을 모두 합하면

S = (d+ d0) + (d+ d0 + 2k) + (d+ d0 + 4k) + ¢ ¢ ¢+ (d+ d0 + 2(n2 ¡ 1)k)

=n

2k + 2k ¢

n2(n2¡ 1)2

=1

4n2k

로 일정한 값이 된다. ¤

별증2 두 다각형의 꼭지점으로 이루어진 호 중에서 가장 짧은 것을 AB = d 라 하자. AB > 0

이면 AB = 0 이 될 때까지 n¡ 1각형을 살짝 회전시키자. 그렇게 회전하는 동안 n각형의 꼭지

점은 가장 가까운 n¡ 1각형의 꼭지점이 다른 점으로 바뀌지 않는다. 왜냐하면,

(1) A를 지나는 지름 l로 원을 두 부분으로 나누었을 때, B쪽(L영역이라 하자)에 있는 점들

은 회전할수록 근거리가 점점 줄어들게 되므로 다른 점으로 바뀔 수 없다.

(2) 그리고, 그 반대쪽 영역(R영역이라 하자)에 있는 점들은 회전할수록 근거리가 점점 커지

는데, 그 최대값은 A에서 가장 먼 꼭지점(A의 대칭점)에서 발생하고, AB = 0 이 될 때

비로소 n¡ 1각형의 두 꼭지점의 (원주상의) 중점이 되므로, 역시 그 전에는 가장 가까운

점이 다른 점으로 바뀔 수 없다.

이렇게 보았듯이, AB = 0 이 될 때까지 회전하는 동안 L영역의 n=2개의 꼭지점은 근거리가

d씩 줄어들고 R영역의 n=2개의 꼭지점은 근거리가 d씩 늘어나므로 그 합은 변하지 않는다. 따

라서, 근거리의 합은 항상 일정하고, 그 값은 AB = 0 일 때의 것으로 계산하면 쉽게 얻을 수 있

다. ¤

Page 172: Baltic Way 팀수학경시대회 1990-2005 pdf 보기

172

Baltic 1998-11Baltic 1998-11

a, b, c는 외접원의 반지름이 R인 삼각형의 세 변의 길이이다.

R ¸ a2 + b2

2p2a2 + 2b2 ¡ c2

의 부등식을 증명하여라. 그리고 언제 등호가 성립하겠는가?

증명1 (대전과학고 1학년 고기혁)

분모를 이항하고 싸인 법칙 a = 2R sinA, b = 2R sinB, c = 2R sinC 를 대입하면 다음과 같이

동치변형된다.

2Rq4R2(2 sin2A+ 2 sin2B ¡ sin2 C) ¸ 4R2(sin2A+ sin2B)q

2(sin2A+ sin2B)¡ sin2 C ¸ sin2 A+ sin2 B2(sin2A+ sin2B)¡ sin2 C ¸ (sin2 A+ sin2 B)2

(sin2A+ sin2B)(2¡ sin2 A¡ sin2B) ¸ sin2 C(sin2A+ sin2 B)(cos2A+ cos2B) ¸ sin2(A+B)

(sin2A+ sin2 B)(cos2B + cos2A) ¸ (sinA cosB + sinB cosA)2

이것은 코시부등식으로 보거나 혹은 이항해서 (sinA cosA ¡ sinB cosB)2 ¸ 0 의 완전제곱식

으로 보면 성립함을 알 수 있다. sinA cosA = 12 sin 2A 이므로, 등호조건은 sin 2A = sin 2B 일

때, 즉 \A = \B 또는 \A+\B = ¼ 일 때이다. 다시 쓰면, a = b 또는 \C = \R 일 때이다. ¤

증명2 (서울 서초고 1학년 정명진, 수정됨)

R = c2 sinC 를 대입하고 양변의 분모를 이항한 후 다음과 같이 코싸인 제 2법칙 c2 = a2 + b2 ¡

2ab cosC 를 적용해보자.

cp2a2 + 2b2 ¡ c2 ¸ (a2 + b2) sinC

c2(2a2 + 2b2 ¡ c2) ¸ (a2 + b2)2 sin2 C

(a2 + b2 ¡ 2ab cosC)(a2 + b2 + 2ab cosC) ¸ (a2 + b2)2 sin2 C

(a2 + b2)2 ¡ 4a2b2 cos2 C ¸ (a2 + b2)2 sin2 C

(a2 + b2)2 cos2 C ¡ 4a2b2 cos2 C ¸ 0(a2 ¡ b2)2 cos2 C ¸ 0

와 같이 동치변형되고, 이것은 틀림없이 성립하는 식이다. 등호는 a = b 또는 \C = 90± 일 때

성립한다. ¤

증명3 C에서 그은 중선을 m = CM이라 하자. 파푸스의 중선정리에서 4m2+ c2 = 2a2+2b2

이므로, 문제는

4Rm ¸ a2 + b2

Page 173: Baltic Way 팀수학경시대회 1990-2005 pdf 보기

1998년 Baltic Way 풀이 173

를 증명하라는 것과 같다. 이것을 다음과 같이 계속 동치변형해보자.

8Rm ¸ 4m2 + c2

4R2 ¡ c2 ¸ 4m2 ¡ 8Rm+ 4R2

R2 ¡³ c2

´2¸ (m¡R)2

외접원의 중섬을 O라 하면 OM2 = OA2 ¡AM2 = R2 ¡ ( c2)2 이므로 이 식은

OM ¸ jCM ¡OCj

이 되고, 이것은 4OMC에서의 삼각부등식이므로 성립한다. 등호는 O, M , C가 일직선을 이

룰 때, 즉 a = b 의 이등변삼각형이거나 \C가 직각일 때(O =M 일 때) 성립한다. ¤

주 S = abc4R, 즉 R = abc

4S 를 좌변에 대입하고 S를 헤론의 공식으로 표현하면 준식은 a, b, c로

만 나타낼 수 있다. 양변 제곱하고 적절히 이항하여 열심히 소거하고 하다보면

(a2 ¡ b2)2(a2 + b2 ¡ c2)2 ¸ 0

로 정리할 수 있다. 하지만 고생스러운 작업이다.

Baltic 1998-12Baltic 1998-12

\BAC = 90± 인 삼각형 ABC가 있다. D는 변 BC 위의 점이고, \BDA = 2\BAD 를 만족한다.

다음 식을 증명하여라.1

jADj =1

2

µ1

jBDj +1

jCDj¶

증명 (KAIST 수학과 03학번 김재훈)

AD의 연장선이 4ABC의 외접원과 만나는 점을 E라 하고, 빗변 BC의 중점이자 이 외접원의

중심을 O라 하자.

그럼 \EDO = \BDA = 2\BAD = 2\BCE = \BOE 이므로(차례로 맞꼭지각, 문제의 조

건, 원주각, 중심각), 4EDO는 이등변삼각형이고 ED = EO = 12BC 이다. 원멱의 정리에 의

Page 174: Baltic Way 팀수학경시대회 1990-2005 pdf 보기

174

해서

BD ¢ CD = AD ¢DE = AD ¢ 12BC

= AD ¢ 12(BD + CD)

이고, 양변을 AD ¢ BD ¢ CD 로 나누면

1

AD=1

2

µ1

BD+

1

CD

¶가 됨을 확인할 수 있다. ¤

Baltic 1998-13Baltic 1998-13

볼록 오각형 ABCDE에서, 변 AE와 BC가 평행하고 \ADE = \BDC 이다. 대각선 AC와 BE는

점 P에서 만난다. \EAD = \BDP 이고 \CBD = \ADP 임을 보여라.

증명 (O±cial Solution)

4ADE, 4BDC의 외접원을 각각 K1, K2라 하고, DP의 연장선이 K2와 다시 만나는 점을 F라

하자.

K1에서의 AE에 대한 원주각과 K2에서의 BC에 대한 원주각이 같으므로 두 원의 반지름 r1,

r2의 비는

r1 : r2 = AE : BC

가 된다. 따라서, 4PEA를 4PBC로 대응시키는 P를 중심으로 한 닮음변환을 T라 하면, T는

K1을 K2로 대응시키고, 그럼 또 DE는 FB로 대응시킨다는 것을 알 수 있다. 이로부터

\EAD = \BCF = \BDP

임을 알 수 있고(차례로 닮음변환 T , 원주각), 마찬가지로 하여 \CBD = \ADP 도 확인된다.

¤

Baltic 1998-14Baltic 1998-14

jABj < jACj 인 삼각형 ABC가 주어져있다. B를 지나면서 AC에 평행한 직선이 각 BAC의 외각

의 이등분선과 점 D에서 만난다. C를 지나면서 AB에 평행한 직선이 같은 이등분선과 점 E에서 만

난다. F는 변 AC 위의 점이고 jFCj = jABj 를 만족한다. jDF j = jFEj 임을 보여라.

Page 175: Baltic Way 팀수학경시대회 1990-2005 pdf 보기

1998년 Baltic Way 풀이 175

증명 BD \CE = G 라 하고, F를 지나 AB에 평행한 직선이 BD와 만나는 점을 H라 하자.

\GDE = \CAE = \BAD = \GED 이므로 4GDE는 이등변삼각형이다. 또, ¤ABGC,

¤FHGC는 평행사변형이므로 FH = CG = AB = FC = HG 이고, 그럼 ¤FHGC는 마름모

이다. 따라서, FG는 이등변삼각형 GDE의 꼭지각 \G의 이등분선이므로 DE를 수직이등분하

고, FDE도 FD = FE 인 이등변삼각형이다. ¤

Baltic 1998-15Baltic 1998-15

예각 삼각형 ABC가 주어져있다. A에서 BC에 내린 수선의 발을 D라 하자. E는 선분 AD 위의 점

으로jAEjjEDj =

jCDjjDBj

를 만족한다. D에서 BE에 내린 수선의 발을 F라 하자. \AFC = 90± 임을 보여라.

증명 (대전과학고 1학년 고기혁)

ADCG가 직사각형이 되도록 점 G를 잡자.

그럼 AE : ED = CD : DB = AG : DB 가 되므로 직각삼각형 EAG와 EDB는 닮았고, 따라

서 BEG는 일직선을 이룬다. \DFG = \R = \DAG 이므로 AFDG는 DG를 지름으로 하는

원 위에 있다. ADCG가 직사각형임에서 CA도 그 원의 지름이므로 \AFC = \R 이다. ¤

별해 (KAIST 수학과 03학번 김재훈)

점열 AED와 CDB가 비례하므로 AD : CB = ED : DB 이고, 이것을 4BDE 내의 직각삼각

형의 닮음과 연관시키면 AD : CB = ED : DB = DF : FB, 즉

AD : DF = CB : BF (¤)

Page 176: Baltic Way 팀수학경시대회 1990-2005 pdf 보기

176

가 된다. AD ? CB, FD ? FB 임과 (¤)을 함께 연관하여 생각하면, F를 중심으로 한 90± 회

전변환(+확대축소)에 의해

4FAD » 4FCB

따라서, 나머지 한 쌍의 변 AF와 FC도 90±를 이룬다. ¤

주 많은 학생들이 4FAD » 4FCB 대신에 4FAE » 4FCD 를 증명한 풀이를 하였다. 좌

표로 놓고 풀 수도 있지만 막노동이다.

Baltic 1998-16Baltic 1998-16

13£ 13 체스판에서 중앙의 한 칸만을 제외하고 4£ 1 크기의 42개의 타일로 전부 까는 것이 가능하

겠는가? (각 타일은 정확히 체스판의 네 칸을 깔고, 타일들은 겹치지 않는 것으로 한다.)

풀이 (KAIST 수학과 03학번 김재훈)

다음 그림과 같이 색칠을 하자.

² ² ² ² ² ² ²± ± ± ± ± ±² ² ² ² ² ² ²± ± ± ± ± ±² ² ² ² ² ² ²± ± ± ± ± ±² ² ² ² ² ² ²± ± ± ± ± ±² ² ² ² ² ² ²± ± ± ± ± ±² ² ² ² ² ² ²± ± ± ± ± ±² ² ² ² ² ² ²± ± ± ± ± ±

± ± ± ± ± ± ±² ² ² ² ² ²± ± ± ± ± ± ±² ² ² ² ² ²

± ± ± ± ± ± ±² ² ² ² ² ²± ± ± ± ± ± ±² ² ² ² ² ²

± ± ± ± ± ± ±² ² ² ² ² ²± ± ± ± ± ±² ² ² ² ² ²£

그러면 4£ 1의 타일을 하나 놓을 때마다 ²과 ±이 2개씩 덮힌다. 만약 42개의 타일로 모두 까는

것이 가능하다면 ²과 ±가 같은 개수(84개씩)로 있어야 한다. 그러나 실제 세어보면 ²과 ±보다

2개 더 많다. 따라서, 42개의 타일로 이 체스판을 모두 까는 것은 불가능하다. }

Baltic 1998-17Baltic 1998-17

n과 k는 양의 정수이다. 같은 크기의 nk개의 물건과 각각 n개의 물건을 담을 수 있는 k개의 상자가

있다. 각 물건은 k가지 색 중 어느 하나로 칠해져 있다. 각 상자가 기껏해야 2가지 색의 물건들을 담

도록 물건들을 모두 상자에 담을 수 있음을 보여라.

증명 (KAIST 수학과 03학번 김재훈)

수학적 귀납법을 사용해보자. k = 1; 2 일 때는 자명하다. k = s¡ 1 일 때 성립한다고 가정하고,

k = s 일 경우를 생각해보자. ns개의 물건이 s가지 색으로 칠해져있으므로, 각 색깔별로 물건

의 개수를 세었을 때 n개 이하인 색이 반드시 있고, 마찬가지로 n개 이상인 색도 있다. 전자를

A색, 후자를 B색이라고 하자.그럼 상자 하나를 골라서 A색인 물건을 모두 집어넣자.그리고 빈

공간에 (n개에서 모자란 만큼) B색의 물건을 집어넣자. 그러면 남은 물건들만 볼 때 n(s¡ 1)개의 물건이 (A색은 이미 모두 넣고 없어서) s¡ 1가지 색으로 칠해져있으므로, 귀납적 가정에 의

해서 남은 s ¡ 1개의 상자도 조건에 맞게 채울 수 있다. 따라서 이미 채워놓은 상자까지 해서

k = s 일 때도 명제가 성립하고, 이렇게 수학적 귀납법에 의해 모든 자연수 k에 대해서 문제는

참이다. ¤

Page 177: Baltic Way 팀수학경시대회 1990-2005 pdf 보기

1998년 Baltic Way 풀이 177

Baltic 1998-18Baltic 1998-18

다음 성질을 갖는 집합 S가 존재하도록 하는 양의 정수 n을 모두 구하여라.

(i) S는 2n¡1보다 작은 n개의 양의 정수들로 구성되어 있다.

(ii) S의 임의의 서로 다른 부분집합 A, B에 대해서, A의 원소들의 합은 B의 원소들의 합과 같지

않다.

풀이 2n¡1보다 작은 n개의 수로 만들 수 있는 최대의 합은 n ¢ 2n¡1 ¡ 12n(n + 1) 이고, S의

부분집합은 2n가지이므로 모두 서로 다르다면 가장 큰 수는 2n ¡ 1 이상이다. 따라서,

2n ¡ 1 · n ¢ 2n¡1 ¡ n(n+ 1)

2

이다. n = 1; 2; 3 은 이 부등식을 만족하지 못하므로 불가능하다. n = 4 일 때는

S = f3; 5; 6; 7g

로 하면 가능함을 확인할 수 있다. 그리고 귀납적으로, n일 때 S = fa1; a2; : : : ; ang 이 조건을

만족한다면,

S0 = f1; 2a1; 2a2; : : : ; 2ang

은 n+1일 때 조건을 만족하는 집합이 된다. 조건 (i)이 만족됨은 쉽고, 조건 (ii)만 확인하면 되

겠다. 1의 포함여부에 따라 합이 홀수와 짝수로 나뉘므로, 1을 포함하는 것끼리, 또 그렇지 않

은 것끼리만 비교하면 되며, (1이 있다면 제외한 후) 2로 나누어주면 n일 때의 비교와 같아 합

이 서로 같을 수 없음을 알 수 있다.

¢ ¢ ¢ 답 n ¸ 4 인 모든 자연수 }

Baltic 1998-19Baltic 1998-19

각각 1000명의 선수로 이루어져 있는 두 팀 사이의 탁구 시합이 있다. 각 선수들은 상대팀의 모든

선수와 각각 정확히 한 번씩 경기를 하고, 무승무는 없다. 어느 한 팀에 속한 10명의 선수를 잘 뽑으

면, 상대팀의 모든 선수가 이 10명의 선수들 중 누군가 1명에게는 패하였음을 증명하여라.

증명 먼저 다음의 보조정리를 확인하자.

보조정리 A팀의 모든 선수가 B팀의 모든 선수와 서로 한 번씩 경기를 하고 무승부는

없었다면, 상대팀의 절반 이상을 이긴 선수가 적어도 한 명은 있다.

보조정리의 증명 A팀, B팀의 인원을 각각 m, n명이라 하자. 문제에서 찾는 선수가 한

명도 없다고 가정하자. 그럼 A팀 선수가 이긴 경기의 수는 m ¢ n2보다 작고, B팀 선수가

이긴 경기의 수도 n ¢ m2 보다 작아서, 총 경기의 수는 둘의 합인 mn보다 작다. 그러나, 양

팀의 모든 선수가 서로 한 번씩 경기해야 하므로 총 경기의 수는 mn이라야 하고, 이것은

모순. 따라서, 상대팀의 절반 이상을 이긴 선수가 한 명은 있다. ¤

Page 178: Baltic Way 팀수학경시대회 1990-2005 pdf 보기

178

이제 원래의 문제를 일반화하여 다음의 명제를 증명하자.

확장명제 A팀과 B팀에 각각 2m명 미만, 2n명 미만의 선수가 있고, 두 팀 사이의 모든

선수가 서로 한 번씩 무승부 없이 경기를 했다. 그럼 상대팀을 모두 제압한 A팀의 n명 혹

은 B팀의 m명이 존재한다. 단, 상대팀을 모두 제압했다는 것은 상대팀의 누구든 적어도

한 번은 이 멤버 중 한 명에게 졌다는 것을 뜻한다.

m = n = 1 일 때를 생각하면 각 팀은 2명 미만, 즉 1명씩이므로 자명하다. 이제 m+ n = k 일

때 성립한다고 가정하고 m+ n = k + 1 일 때를 생각하자. 보조정리에 의해 상대팀의 절반 이

상을 이긴 선수가 있다. 일반성을 잃지 않고 그 선수를 A팀의 a선수라고 하고, B팀에서 a에게

진 선수를 모두 제외하여 남은 선수들을 모아 B0팀이라고 하자. 그럼

jAj < 2m; jB0j < 2n¡1; m+ (n¡ 1) = k

이므로 A : B0의 대결을 생각할 때, 귀납법의 가정에 의해 상대팀을 모두 제압한 A팀의 n¡ 1명혹은 B0팀의 m명이 존재한다. B0팀의 m명이 존재하면 그 m명이 A : B의 대결에서도 여전히

A팀을 모두 제압한 m명이고, A팀의 n ¡ 1명이 존재하면 거기에 a를 추가하여(a가 n ¡ 1명에

포함된 경우는 이상한 경우이지만, 그런 경우에도 아무나 a 대신 포함하여 n명을 만들면 된다)

B팀을 제압한 n명이 구성된다. 따라서, m + n에 대한 귀납법으로 위의 확장명제가 증명되었

다. 원래의 문제는 m = n = 10 이고 1000 < 210 일 때이므로 역시 성립한다. ¤

Baltic 1998-20Baltic 1998-20

m의 자리수 중에 1, 9, 9, 8이 이 순서로 나타난다면, m이 1998을 덮는다고 한다. (예를 들어,

215993698은 1998을 덮지만 213326798은 덮지 않는다.) 자리수에 0을 포함하지 않는 n자리의 양의

정수 중에서 1998을 덮는 것의 개수를 k(n)이라 하자(n ¸ 5). k(n)을 8로 나누었을 때 나머지는 얼

마인가?

풀이 k가 아닌 숫자를 ¹k로 나타내자. 1998을 덮는 수는 자리수가 왼쪽부터

¹1들, 1, ¹9들, 9, ¹9들, 9, ¹8들, 8, ¤들(아무 숫자나 가능)

이 차례로 나타난다. ¹k의 위치에는 항상 8가지 숫자(0은 제외하기로 했으므로)가 올 수 있으

므로, (1, 9, 9, 8의 위치를 고정시켰다고 할 때) 개수가 8의 배수가 되지 않으려면 ¹k꼴의 자리

가 하나도 없어야 한다. 즉, 1998¤ ¢ ¢ ¢ ¤꼴만 세면 된다. 이것은 9n¡4개이므로 k(n) ´ 9n¡4 ´ 1(mod 8). ¢ ¢ ¢ 답 1 }

Page 179: Baltic Way 팀수학경시대회 1990-2005 pdf 보기

1999년 Baltic Way 풀이 179

1999년 Baltic Way 풀이

Baltic 1999-1Baltic 1999-1

다음 연립방정식을 만족하는 모든 실수 a, b, c, d를 구하여라.8>>>>>>><>>>>>>>:

abc+ ab+ bc+ ca+ a+ b+ c = 1

bdc+ bc+ cd+ db+ b+ c+ d = 9

cda+ cd+ da+ ac+ c+ d+ a = 9

dab+ da+ ab+ bd+ d+ a+ b = 9

풀이 각 식의 양변에 1을 더하면 다음과 같이 좌변이 인수분해된다.

(a+ 1)(b+ 1)(c+ 1) = 2

(b+ 1)(c+ 1)(d+ 1) = 10

(c+ 1)(d+ 1)(a+ 1) = 10

(d+ 1)(a+ 1)(b+ 1) = 10

이 네 식을 모두 변변 곱하면 (a+ 1)3(b+ 1)3(c+ 1)3(d+ 1)3 = 2 ¢ 103, 즉

(a+ 1)(b+ 1)(c+ 1)(d+ 1) = 103p2

이다. 앞에서 구한 세 항의 곱들을 이 식에 각각 대입하면 2(d + 1) = 10(a + 1) = 10(b + 1) =

10(c+1) = 10 3p2 임을 알 수 있고, 따라서 (a; b; c; d) = ( 3

p2¡ 1; 3p2¡ 1; 3p2¡ 1; 5 3p2¡ 1) ¢ ¢ ¢

답 }

Baltic 1999-2Baltic 1999-2

n을 십진법으로 쓴 후 마지막 세 자리를 지웠더니 n의 세제곱근이 되었다. 이런 성질을 갖는 양의

정수 n을 모두 찾아라.

풀이 세제곱근을 m이라 하면 문제는

m3 = 1000m+ r (0 · r < 1000)

임을 의미한다. 여기서 1000m · m3 < 1000m+ 1000, 즉

1000 · m2 < 1000 +1000

m

의 부등식을 얻는다. 왼쪽 부등식에서 m ¸ 32, 그럼 오른쪽 부등식은 m2 · 1031 이 되고 다시

m · 32. 따라서, m = 32 이고, n = m3 = 215 = 32768 ¢ ¢ ¢ 답 }

Page 180: Baltic Way 팀수학경시대회 1990-2005 pdf 보기

180

Baltic 1999-3Baltic 1999-3

a1 + a2 + ¢ ¢ ¢+ an = 0 을 만족하는 모든 실수 a1; a2; : : : ; an에 대해 다음 부등식을 만족하는 모든

양의 정수 n ¸ 3 을 찾아라.

a1a2 + a2a3 + ¢ ¢ ¢+ an¡1an + ana1 · 0

풀이 (KAIST 수학과 02학번 이은미)

(1) n = 3 일 때: a1 + a2 + a3 = 0 에서 얻은 a3 = ¡(a1 + a2) 를 대입하여 a3을 소거하면

a1a2 + a2a3 + a3a1 = ¡a21 ¡ a1a2 ¡ a22 = ¡12a21 ¡ 1

2(a1 + a2)

2 ¡ 12a22 · 0

으로 항상 성립한다.

(2) n = 4 일 때:

a1a2 + a2a3 + a3a4 + a4a1 = (a1 + a3)(a2 + a4) = ¡(a1 + a3)2 · 0

으로 역시 항상 성립한다.

(3) n ¸ 5 일 때: 1; 1;¡1;¡1; 0; : : : ; 0 의 수열을 생각하면

a1a2 + a2a3 + ¢ ¢ ¢+ ana1 = 1¡ 1 + 1 + 0 + ¢ ¢ ¢+ 0 = 1 > 0

으로 성립하지 않는다.

(1), (2), (3)에 의해 조건을 만족하는 n은 3과 4뿐이다. }

Baltic 1999-4Baltic 1999-4

모든 양의 실수 x, y에 대해

f(x; y) = min

µx;

y

x2 + y2

¶가 성립한다. 임의의 양수 x, y에 대해 f(x; y) · f(x0; y0) 을 만족하는 x0, y0이 존재함을 보여라.

그리고 f(x0; y0)을 구하여라.

증명 산술-기하평균 부등식에서

x ¢ y

x2 + y2· xy

2xy=1

2

따라서, x와 yx2+y2 둘 중 하나는 1p

2이하이고, 따라서

f(x; y) · 1p2

이다. 실제로 f( 1p2; 1p

2) = 1p

2일 때 등호가 성립한다. ¤

Page 181: Baltic Way 팀수학경시대회 1990-2005 pdf 보기

1999년 Baltic Way 풀이 181

Baltic 1999-5Baltic 1999-5

점 (a; b)는 원 x2 + y2 = 1 위에 있다. 이 점에서 원에 접하는 접선은 포물선 y = x2 + 1 과 정확히

한 점에서 만난다. 이러한 점 (a; b)를 모두 찾아라.

풀이 주어진 원의 접선의 방정식은

ax+ by = 1

이다. b = 0 일 때는 a = §1 이고, 이 때 접선 x = §1 은 모두 포물선과 딱 한 점 (§1; 2)에서

만난다. b 6= 0 일 때는 접선의 방정식을

y = ¡a

b¢ x+ 1

b

로 바꿔쓸 수 있다. 이것을 포물선의 식에 대입하면 이차식

x2 +a

bx+ 1¡ 1

b= 0

이 나오는데, 한 점에서 만나야 하므로 중근을 가져야 한다. 즉, 판별식으로 풀면

D =³ab

´2 ¡ 4 + 4b= 0

a2 ¡ 4b2 + 4b = 0

(a; b)는 원 위의 점이므로 a2 + b2 = 1 이고 a2 = 1¡ b2 을 대입하면

1¡ 5b2 + 4b = 0(1 + 5b)(1¡ b) = 0

따라서, b = 1 또는 ¡ 15 이다. b = 1 일 때 a = 0 이고, b = ¡ 15 일 때 a = §2p65 이다.

¢ ¢ ¢ 답 (§1; 0), (0; 1), (§2p65 ;¡ 15 ) }

Baltic 1999-6Baltic 1999-6

n £ n 체스판에서(n ¸ 4) 한 쪽 구석칸으로부터 대각선으로 맞은 편에 있는 구석칸으로 knight를

옮기려면 최소 몇 번 이동해야 하는가?

풀이 출발칸의 좌표를 (0; 0), 대각선 맞은 편의 구석칸을 (m;m)으로 두자(n = m + 1).

knight는 (§2;§1) 또는 (§1;§2)씩 움직일 수 있다. 따라서, 좌표의 합 x+ y 는 §1 또는 §3씩변한다. 즉, x+ y는

(1) 최대 3만큼 변할 수 있고,

(2) 매번 움직일 때마다 항상 홀짝이 변한다.즉, (0; 0)에서 (m;m)까지는 x+y가 짝수인 2m만

큼 변해야 하므로 짝수번 움직여야 한다.

Page 182: Baltic Way 팀수학경시대회 1990-2005 pdf 보기

182

따라서, m = 3k꼴이면 (1)에 의해 적어도 (3k+3k)=3 = 2k번은 움직여야 한다. m = 3k+1 또

는 3k+2꼴이면 같은 논리로 2k번으로는 모자라는데, (2)에 의해 짝수번 움직여야 하므로 적어

도 2k + 2번은 움직여야 한다. 그리고, 그 횟수만의 이동이 다음과 같이 가능하므로 이것이 실

제 최소값이다.

2

10

42

31

0

43

21

0

(A) (B) (C)

m = 3k; 3k + 1; 3k + 2 일 때 각각 (A), (B), (C)의 방법을 먼저 한 번씩 사용하고, 그 후에

는 계속 (A)의 방법을 사용하면 된다. n = m + 1 이므로, 결론을 종합하면 최소이동횟수는

n = 3k ¡ 1; 3k; 3k + 1 일 때 2k번, 즉 2bn+13 c번이다. }

Baltic 1999-7Baltic 1999-7

8£ 8 체스판 위에서 공통된 변이나 꼭지점을 가지는 두 칸을 서로 인접하다고 한다. king이 적당한

칸에서 시작해서 모든 칸을 한 번씩 지나도록 하는데, 맨처음을 제외한 모든 이동이 이미 지났던 칸

들 중 짝수 개가 인접한 칸으로 움직이도록 할 수 있는가?

풀이 (KAIST 수학과 02학번 이은미)

king이 움직이는 칸을 순서대로 a1; a2; : : : ; a64라 하고, ai에 인접한 이미 방문한 칸의 개수를

bi라고 하자(1 · i · 64). 이 때,

b1 = 0; b2 = 1; b3; b4; : : : ; b64는 모두 짝수

가 되도록 king이 움직여야 한다. 조건에 맞는 king의 이동방법이 있다고 하고,

B = b1 + b2 + ¢ ¢ ¢+ b64

에 대해 생각해보자. 임의의 칸 ai (1 · i · 64)는 B에 자신에게 인접한 칸 수만큼 기여함을

보이자. ai에 인접한 칸의 개수를 k개라 하면 k개 중 bi개는 이미 지나간 칸이고, 이 칸들의 bj

(j < i) 에는 영향을 미치지 않는다. 아직 지나지 않은 나머지 k¡ bi개의 인접한 칸에 대해서는

차후에 그 칸을 지날 때 bj (i < j) 에 1씩 기여한다. 즉, ai는 bi + (k¡ bi) = k 만큼 B에 기여한

다. 따라서, 각 ai들에 대해 그 인접한 칸의 수 ki들을 모두 합하면

B =64Xi=1

ki = 4£ 3 + 24£ 5 + 36£ 8 = 420

이다. 그런데 b0 = 0, b1 = 1 이므로 B =P64

i=1 bi 는 홀수가 되어 모순. 따라서, 주어진 조건대

로 king이 이동할 수 있는 방법은 없다. }

Page 183: Baltic Way 팀수학경시대회 1990-2005 pdf 보기

1999년 Baltic Way 풀이 183

Baltic 1999-8Baltic 1999-8

서로 다른 무게를 갖는 1999개의 동전이 있다. 임의의 세 개의 동전에 대해 그 중 중간 무게를 갖는

동전을 한 번의 작동으로 골라낼 수 있는 기계가 있다. 이 기계를 사용하여 1000번째의 무게에 해당

되는 동전을 찾아내려고 하는데, 1000000번 이상 작동시킬 필요가 없음을 증명하여라. 또한 이 동

전이 이 기계를 사용하여 무게 순서를 알아낼 수 있는 유일한 동전임을 보여라.

증명 모든 동전을 주머니 A에 넣자. A에서 세 개의 동전을 꺼내 기계를 한 번 작동시켜 알

아낸 중간 무게의 동전을 주머니 B로 옮겨넣고, 나머지 두 동전은 도로 A에 넣는다. 이 과정을

A에 단 두 개의 동전만 남을 때까지 1997번 반복하자. 그럼 B에 있는 1997개의 동전들은 어느

것도 가장 무겁거나 가장 가벼운 동전이 될 수 없으므로, A에 남은 두 개의 동전이 가장 무거운

동전과 가장 가벼운 동전이다. 이제 A에 있는 두 동전을 주머니 X로 옮겨넣고, B의 모든 동전

을 A로 옮긴 후 아까 했던 과정을 1995번 반복한다. 그럼 A에는 두 번째로 무겁거나 가벼운 두

동전이 남는다. 이런 식으로 무게순서로 양끝점에 해당하는 두 동전을 계속 X로 걸러내는 작업

을 999번 반복하여

1997 + 1995 + 1993 + ¢ ¢ ¢+ 3 + 1 = 9992 < 1000000

번 기계를 작동시키면 X에는 2 ¢ 999 = 1998개의 동전이 모이고, 마지막 남은 하나의 동전이 양

끝에서 1000번째인, 즉 중간 무게의 동전이 된다. 따라서, 1000000번 이내에 중간 무게의 동전

을 찾을 수 있다.

이제다른 무게순서의 동전은 이 기계로 찾을 수없음을보이자.만일 k 6= 1000번째로무거운동

전을 이 기계로 찾아낼 수 있다면, 그 동전을 찾아내는 작동순서 프로그램 P = (p1; p2; : : : ; pm)

이 있을 것이다. 이제 동전 전체의 무게순서를 뒤집어서(즉, r = 1; 2; : : : ; 999 에 대해 r번째 무

거운 동전과 r번째 가벼운 동전의 위치를 모두 서로 바꿔서) 다시 이 P에 적용하자. 뒤집기 전

에 각각의 작동 pi에서 세 동전 ai; bi; ci를 점검하여 bi가 중간무게라는 결과를 얻었다면 뒤집은

후에도 ai < bi < ci 의 순서와 ai > bi > ci 의 순서가 서로 뒤바뀌었을 뿐이므로 작동결과는

똑같이 bi가 중간무게라고 대답할 것이다. 즉, 모든 작동에서 뒤집기 전과 뒤집은 후의 작동결

과가 같으므로, 이 프로그램 P에 의해 k번째 무거운 동전을 찾았다고 답을 했다면, k번째 가벼

운 동전이 대신 답으로 나올 수 있다. 즉, 이 기계는 어떤 프로그램으로도 k번째 가벼운 동전과

k번째 무거운 동전을 구분하지 못한다. 따라서, k번째 가벼운 동전과 k번째 무거운 동전이 동

일한 경우(k = 1000 일 때)를 제외하고는 이 기계로 어떤 동전도 그 무게순서를 알아낼 수 없

다. ¤

Baltic 1999-9Baltic 1999-9

한 변의 길이가 3인 정육면체는 27개의 단위 정육면체로 나뉘어진다. 1, 2, 3, : : : ; 27의 수를 각 단

위 정육면체마다 하나씩 배정하여 붙였다. 정육면체의 가로, 세로, 높이 중 어느 한 방향으로 한 줄

을 이루는 세 정수의 합을 일렬합이라 하자. 그럼 각 방향마다 9개씩 모두 27개의 일렬합이 생긴다.

이 27개의 일렬합 중 홀수인 것은 최대 몇 개인가?

Page 184: Baltic Way 팀수학경시대회 1990-2005 pdf 보기

184

풀이 1 + 2+ ¢ ¢ ¢+ 27 = 27¢282 이 짝수이므로 가로, 세로, 높이 중 어느 한 방향의 9개의 일렬

합 모두를 더하면 짝수라야 한다. 따라서, 그 방향의 일렬합이 모두 홀수일 수 없고, 그럼 각 방

향마다 짝수인 일렬합이 최소한 하나씩 있다. 즉 홀수인 일렬합은 많아야 24개이다. 24개인 경

우는 다음과 같이 하면 실제로 구성할 수 있다:

(상단면)

홀 홀 홀

홀 취소 홀

홀 홀 홀

(중단면)

(하단면)

중단면의 9개칸에 모두 홀수 넣고, 하단면과 상단면에는 같은 한 대각선에만 홀수를 배정한

다(이렇게 하면 모든 일렬합이 홀수가 됨). 그런데, 이렇게 하려면 홀수가 15개 필요한데 실제

로는 14개뿐이므로, 홀수를 배정했던 곳 한 칸을 취소(짝수를 배정)한다. 그럼 그 칸을 포함하

는 가로, 세로, 높이의 일렬합이 짝수로 변하므로 홀수인 일렬합은 24개가 된다. ¢ ¢ ¢ 답 24개

}

Baltic 1999-10Baltic 1999-10

반지름이 1인 원판의 (원주를 포함한) 점들을 세 부분집합으로 분할하는데, 거리 1만큼 떨어진 두

점이 한 집합에 포함되지 않도록 할 수 있는가?

풀이 거리가p3만큼 떨어진 임의의 두 점 A, B는 항상 같은 부분집합에 속해야 한다.왜냐하

면, A, B를 중심으로 반지름 1인 두 원을 그려 만나는 두 교점을 C, D라 하면 ACD와 BCD는

모두 한 변의 길이가 1인 정삼각형이 되므로, A, C, D는 모두 서로 다른 집합에 있어야 하고 B,

C, D도 모두 서로 다른 집합에 있어야 해서, 결국 A와 B는 같은 집합(C, D가 속하지 않은 집

합)에 있어야 한다. 물론 C, D가 주어진 원판을 벗어나는 경우가 없음을 확인할 필요가 있다.

원주 위의 한 점 P를 잡고 P를 중심으로 반지름p3인 원을 그리고 그중 원판에 포함되는 부분

을 K라 하자. 그럼 호 K의 위에 있는 (원판 위의) 점들은 앞에서 증명한 사실에 의해 모두 P와

같은 집합에 포함되어야 한다. K 위의 점 중 거리가 1만큼 떨어진 두 점이 틀림없이 있는데, 이

두 점은 (P와) 같은 집합에 포함된다. 따라서, 거리 1만큼 떨어진 두 점이 모두 서로 다른 집합

에 포함되도록 할 수는 없다. }

Baltic 1999-11Baltic 1999-11

어느 세 점도 한 직선 위에 있지 않은, 평면 위의 임의의 네 점이 있다. 이 네 점 중 세 점을 지나고,

네 번째 점 역시 지나거나 혹은 내부에 포함하는 원이 존재함을 보여라.

증명1 어느 세 점 A, B, C의 외접원을 먼저 생각한다. 남은 한 점이 이 외접원의 바깥에 있

다면, 처음 세 점으로 만들어진 삼각형의 각 변을 직선으로 연장하여 삼각형 외부를 6개의 영

Page 185: Baltic Way 팀수학경시대회 1990-2005 pdf 보기

1999년 Baltic Way 풀이 185

역으로 나눈다.

원 바깥의 점 D가 꼭지점 A 뒤쪽의 영역에 있으면 BCD의 외접원이 A를 품는다(A가 삼각형

BCD 내부에 놓이므로). D가 변 AB 바깥 영역에 있으면 DBC의 외접원이 A를 품는다(원주

각 \BDC < \BAC 로 확인). 나머지 영역들도 각각 이 두 가지 중 한 경우와 마찬가지이므로

증명이 되었다. ¤

증명2 (과천고 2학년 홍성준)

4개의 점을 모두 포함하는 볼록포를 생각하자. 볼록포의 모양은 삼각형 혹은 볼록사각형이므

로, 그 다각형(볼록포)을 이루는 변 중 하나를 택하면, 나머지 두 점은 그 변에 대해 같은 쪽

에 있게 된다. 이제 변을 이루는 두 점을 A, B라 하고 나머지 두 점을 C, D라 한 뒤 \ACB,

\ADB 중 작은 쪽을 취하자(같은 경우엔 임의로 취함).

일반성을 잃지 않고 \ACB가 더 작은 쪽이라면, AB에 대한 원주각의 성질에서 D는 4ABC의

외접원 내부(경계 포함)에 포함된다. ¤

증명3 네 점을 모두 포함하는 커다란 원을 생각하고, 점을 못이라고 생각하고 적당한 방향

으로 원을 떨어뜨리면 두 점(A, B라 하자)과 만난다. 그 두 점을 지나도록 유지하면서(중심이

AB의 수직이등분선 위를 움직이도록 하면서) 원의 크기를 연속적으로 변화시키면 (이렇게 연

속적으로 변하는 원을 모두 모으면 직선 AB를 제외한 평면 전체의 점을 이루므로) 언젠가 또

다른 한 점을 처음으로 지날 때가 있고, 그 원이 우리가 찾는 원이 된다. ¤

Page 186: Baltic Way 팀수학경시대회 1990-2005 pdf 보기

186

Baltic 1999-12Baltic 1999-12

2jABj = jACj+ jBCj 인 삼각형 ABC가 있다. ABC의 내심, ABC의 외심, AC의 중점과 BC의 중

점이 모두 한 원 위에 있음을 보여라.

증명1 (제주 대기고 1학년 오재성)

BC, AC의 중점을 각각 M , N이라 하고, AK = AM 인 AB 위의 점 K를 잡자. 그리고, I를 내

심이라 하자.

4IAN ´ 4IAK, 4IBM ´ 4IBK 가 된다(SAS합동). 그럼 \INC = \IKB = \IMB 이

므로

\IMC + \INC = \IMC + \IMB = 180±

따라서, ¤IMCN은 원에 내접한다. \OMC = \R = \ONC 로 ¤OMCN도 원에 내접하고,

4CMN의 외접원은 유일하므로 I, O, M , C, N은 모두 한 원 위에 있다. ¤

증명2 (서울 중산고 1학년 최서준)

내심을 I, 외심을 O, BC와 AC의 중점을 각각 M , N이라 하자. I, O, M , N이 한 원 위에 있음

을 보이는 것이 목표이다.

¡!CI와 외접원의 교점을 D라 할 때,

DA = DB = DI (=: k)

임은 잘 알려져있다. 굳이 보이자면 \DAI = 12\A + 1

2\C = \DIA 등으로 증명할 수 있다.

Page 187: Baltic Way 팀수학경시대회 1990-2005 pdf 보기

1999년 Baltic Way 풀이 187

¤ADBC에서 톨레미의 정리에 의해

k ¢ BC + k ¢AC = AB ¢ CD

2k ¢ AB = AB ¢ (k + IC)

2k = k + IC

따라서, IC = k, 즉 I는 CD의 중점이다. 그럼 I는 O에서 현 CD에 내린 수선의 발이고, 그럼

\OIC = \OMC = \ONC = \R

이므로 I, M , N은 모두 OC를 지름으로 하는 원 위에 있다. ¤

증명3 (과천고 2학년 홍성준)

외심을 O, 내심을 I라 하고, BC와 AC의 중점을 각각 M과 N , 내접원과 각 변의 접점을 P , Q,

R이라 하자(그림 참조). 그리고, 각 꼭지점 A, B, C에서 접점까지의 거리를 x, y, z라 하자.

\OMC +\ONC = 180± 이므로 O, M , C, N은 한 원 위의 점이다. 이제 I, M , C, N이 한 원

위의 점임만 보이면 된다. 이것을 위해

4IPM ´ 4IQN (1)

을 먼저 증명하자. I가 내심이므로 IP = IQ 이고 \IPN = \IQN = 90±. 따라서,

PM = QN (2)

만 보이면 (1)이 증명된다. PM = 12jy ¡ zj, QN = 1

2jz ¡ xj 이므로 jy ¡ zj = jz ¡ xj 를 확인해

야 한다. 문제의 조건에서 2(x+ y) = (x+ z) + (y + z) 이므로

x+ y = 2z

따라서, y ¡ z = z ¡ x 가 확인되었고 (2)와 (1)이 모두 증명되었다. 그럼 (1)로부터 \IMC +

\INC = \INA+\INC = 180± 이고, 따라서 I, M , C, N은 한 원 위의 점이다. 그러므로, I,

O, M , C, N은 한 원 위에 있고 문제가 해결되었다. ¤

Page 188: Baltic Way 팀수학경시대회 1990-2005 pdf 보기

188

Baltic 1999-13Baltic 1999-13

삼각형 ABC의 각 A와 각 B의 이등분선이 변 BC, CA와 각각 점 D, E에서 만난다. jAEj+ jBDj =jABj 라고 할 때, 각 C의 크기를 구하여라.

풀이1 (충북과학고 1학년 장호철)

AB 위에 AF = AE 인 점 F를 잡자. 그리고, AD \BE = K 라 하자.

여기서

4AKF ´ 4AKE; 4BKF ´ 4BKD

\AKE = \AKF = a, \BKD = \BKF = b 라 하자. 두 직선 AD와 BE의 K에서의 맞꼭지

각으로 a = b 가 되고, AKD가 일직선이므로 3a = 180±, 즉 a = b = 60± 이다. 4KAB의 외각

에서1

2\A+ 1

2\B = a = 60±

이므로 12(180± ¡ \C) = 60±, 즉 \C = 60± 이다. }

주 위의 풀이에서처럼 F , K를 잡을 때, AD, BE가 각각 EF , FD를 수직이등분함으로부터

4DEF가 정삼각형이고 K가 그 중심임을 보일 수도 있다.

풀이2 (김규완, 서준영, 홍성준)

BC = a, CA = b, AB = c 라 하자.

각의 이등분선 정리에서

AE = b ¢ c

a+ c

BD = a ¢ c

b+ c

Page 189: Baltic Way 팀수학경시대회 1990-2005 pdf 보기

1999년 Baltic Way 풀이 189

이고, 이것을 AE +BD = AB 에 대입하면

bc

a+ c+

ac

b+ c= c

b

a+ c+

a

b+ c= 1

b(b+ c) + a(a+ c) = (a+ c)(b+ c)

a2 + b2 = ab+ c2

이 된다. cosine 제2법칙에서

cosC =a2 + b2 ¡ c2

2ab=

ab

2ab=1

2

이므로 \C = 60± 이다. }

Baltic 1999-14Baltic 1999-14

삼각형 ABC는 jABj = jACj인 이등변 삼각형이다. 점 D와 E는 각각 변 AB와 AC 위에 있다. B를

지나고 AC에 평행한 직선이 직선 DE와 F에서 만난다. 또, C를 지나고 AB에 평행한 직선은 직선

DE와 G에서 만난다. 사변형 PQRS의 넒이를 [PQRS]와 같이 나타내기로 할 때, 다음을 증명하여

라.[DBCG]

[FBCE]=jADjjAEj

증명 (서울 중산고 1학년 최서준)

이등변삼각형의 좌우대칭성에서 사다리꼴의 높이가 서로 같으므로(평행선 AB, CG의 간격과

평행선 AC, BF의 간격이 같으므로)

넓이비는 밑변과 윗변의 길이의 합과 비례한다. 즉,

[DBCG]

[FBCE]=

DB + CG

FB +CE

또, 평행선의 엇각 등으로 4EAD » 4FBD » 4ECG 이므로

AD

AE=

DB

FB=

CG

CE=

DB + CG

FB + CE(가비의 리)

이다. 따라서, 문제의 식이 성립한다. ¤

Page 190: Baltic Way 팀수학경시대회 1990-2005 pdf 보기

190

Baltic 1999-15Baltic 1999-15

\C = 60± 이고 jACj < jBCj 인 삼각형 ABC가 있다. 변 BC 위에 jBDj = jACj 가 되도록 점 D를

잡는다. 또, 변 AC의 연장선 위에 jACj = jCEj 가 되도록 점 E를 잡는다. jABj = jDEj 임을 증명

하여라.

증명 (KAIST 수학과 02학번 이은미)

BD = FC 가 되도록 변 BC 위에 점 F를 잡는다.

이 때 삼각형 ABF는 꼭지각이 60±인 이등변삼각형이므로 정삼각형이 되고, 그럼 \AFB =

120± = \ECD, AF = AC = EC, FB = BC ¡AC = CD 로

4AFB ´ 4ECD

가 된다(SAS합동). 따라서 AB = DE 이다. ¤

Baltic 1999-16Baltic 1999-16

적당한 양의 정수 m, n에 대해 k = 19n ¡ 5m 꼴로 나타내어지는 수들 중 가장 작은 양의 정수 k를

구하여라.

풀이 (과천고 2학년 홍성준)

먼저 m = n = 1일 때 k = 14이다.이제 14보다 더 작은 수가 가능한지 확인해보자. 19n¡5m ´§1 (mod 5) 이고 19n ¡ 5m 은 짝수이므로 가능한 후보는 4와 6뿐이다.

(i) 19n ¡ 5m = 4 일 때: mod 3으로 1¡ 2m ´ 1, 즉 2m ´ 0 이므로 모순.

(ii) 19n¡5m = 6일 때: mod 4로 (¡1)n¡1 ´ 2이므로 n은 홀수. 그런데 mod 5로 (¡1)n ´ 1이므로 n은 짝수이고 이것은 모순.

따라서, 14보다 작은 수는 모두 불가능하고, 14가 최소이다. }

별해 (성남 분당중앙고 1학년 임동혁)

m = n = 1 일 때 k = 14 이고, 14보다 더 작은 수는 불가능함을 보이자. 5m ´ 5; 6; 11; 17; 9; 7;16; 4; 1 (mod 19) 이므로

k ´ ¡5m ´ 14; 13; 8; 2; 10; 12; 3; 15; 18 (mod 19)

Page 191: Baltic Way 팀수학경시대회 1990-2005 pdf 보기

1999년 Baltic Way 풀이 191

이다.따라서, 14보다 작은 k는 13, 8, 2, 10, 12, 3만 가능하다.그런데 k ´ (¡1)n ´ 1; 4 (mod 5)에서 14보다 작은 k는 1, 4, 6, 9, 11만 가능하다. 겹치는 수가 없으므로 모두 불가능함을 알 수

있다. 따라서, k의 최소값은 14이다. }

Baltic 1999-17Baltic 1999-17

다음을 만족하는 유한한 정수 수열 c1; c2; : : : ; cn이 존재하는가? a+ c1; : : : ; a + cn 들이 모두 소수

가 되도록 하는 정수 a의 개수가 하나보다 많지만 무한히 많지는 않다.

풀이 존재한다. 3, 5, 11, 17, 29로 하면 a = 0 일 때와 a = 2 일 때(5, 7, 13, 19, 31)가 모두 소

수가 된다. 또한, 이 5개의 수는 5로 나눈 나머지가 모두 다르므로 a를 얼마로 택하든 5의 배수

를 항상 꼭 하나 포함하는데, a가 2보다 큰 자연수이면 그 5의 배수는 5보다 크므로 항상 합성

수가 된다. }

주 다른 수열도 얼마든지 있다. 3, 7, 13, 19, 37, 43, 67로 잡으면 a = 4 일 때 7, 11, 17, 23,

41, 47, 71도 모두 소수가 되고, 그외의 자연수 a에 대해서는 항상 7의 배수인 합성수를 포함한

다.

Baltic 1999-18Baltic 1999-18

m은 m ´ 2 (mod 4) 인 양의 정수이다. m = ab 와 0 < a¡ b <p5 + 4

p4m+ 1 을 만족하는 양의

정수쌍 (a; b)는 많아야 하나뿐임을 보여라.

증명 (서울 한영고 2학년 김민규)

제곱하면

(a¡ b)2 < 5 + 4p4m+ 1

양변에 4m = 4ab 를 더해주면

(a+ b)2 < 4 + 4p4m+ 1 + (4m+ 1) = (

p4m+ 1 + 2)2

괄호 안이 모두 양수이므로 제곱을 벗겨내면

a+ b <p4m+ 1 + 2

한편, m = ab ´ 2 (mod 4)임에서 a와 b는 홀짝이 다르므로 같을 수 없으므로, (a+b)2 ¸ 4ab =4m 에서 등호가 성립할 수 없다. 즉, (a+ b)2 ¸ 4m+ 1. 따라서,

p4m+ 1 · a+ b <

p4m+ 1 + 2

길이가 2인구간이므로홀수 a+b가될수있는것은이범위에서딱하나뿐이다.그것을 a+b = n

이라 하자. 그럼 m, n이 주어졌을 때 a + b = n, ab = m 을 만족하는 a > b 를 찾는 것이므로,

이차방정식

x2 ¡ nx+m = 0

Page 192: Baltic Way 팀수학경시대회 1990-2005 pdf 보기

192

의 해로 유일하게 결정된다(두 근 중 큰 것이 a, 작은 것이 b가 된다). 그러므로, 문제의 조건을

만족하는 (a; b)는 많아야 한 쌍이다. ¤

Baltic 1999-19Baltic 1999-19

모든 소수 p에 대해 p2 + k 가 합성수가 되는 양의 짝수 k가 무한히 많음을 보여라.

증명 3 - p 이면 p2 ´ 1 (mod 3) 임에 착안해보자. k = 6m+ 2 로 잡으면 p가 3일 때를 제외

하면 항상 합성수(3의 배수)가 된다. p = 3 일 때도 해결하기 위해 5 j 32 + k 가 되도록 m을 결

정하면 m = 5n+ 4. 즉, k = 30n+ 26꼴이면 항상 p2 + k 가 합성수가 된다. ¤

주 k = 66r + 2 등 다른 꼴도 얼마든지 생각할 수 있다.

Baltic 1999-20Baltic 1999-20

a, b, c, d는 a > 3b > 6c > 12d 와 a2¡ b2+c2¡d2 = 1749 를 만족하는 소수들이다. a2+ b2+ c2+d2

의 가능한 모든 값을 구하여라.

풀이 제곱수 4개가 모두 홀수이면 그 합차는 짝수로 1749가 될 수 없으므로 a, b, c, d 중 적어

도 1개는 짝수이고, 그럼 d = 2. 여기서 문제의 부등식 조건을 검토하면 c ¸ 5, b ¸ 11, a ¸ 37이 된다. 한편, a > 3b, c > d 임에서

1749 = a2 ¡ b2 + c2 ¡ d2 > 8b2

이 성립하고, 그럼 b · 13 이다. 즉, b는 11 아니면 13이고, 또 어느 경우나 c = 5 가 된다.

b = 13 이면 a2 = 1749 + b2 ¡ c2 + d2 = 1897 은 제곱수가 아니므로 곤란하고, b = 11 이

면 a2 = 1749 + b2 ¡ c2 + d2 = 1849 = 432 이므로 답이 될 수 있다. a2 + b2 + c2 + d2 =

432 + 112 + 52 + 22 = 1999 ¢ ¢ ¢ 답 }

Page 193: Baltic Way 팀수학경시대회 1990-2005 pdf 보기

2000년 Baltic Way 풀이 193

2000년 Baltic Way 풀이

Baltic 2000-1Baltic 2000-1

K는 삼각형 ABC의 내부의 점이다. 직선 AB에 대해 K와 반대쪽에 있는 점 M과, 직선 BC에 대

해 K와 반대쪽에 있는 점 N을 잡자.

\MAB = \MBA = \NBC = \NCB = \KAC = \KCA

일 때, MBNK가 평행사변형임을 보여라.

증명1 (대전과학고 2학년 김진현)

\KCN = \ACB 이고 KC : NC = AC : BC 임에서 4KNC는 4ABC와 KC : AC 의 비율

로 닮았다.

같은 방법으로, 4AMK도 4ABC와 AK : AC 의 비율로 닮았다. 따라서,

4AMK ´ 4KNC

그럼 MK(= NC) = BN 이고 MB(= AM) = KN 이므로 ¤MBNK는 평행사변형이다. ¤

증명2 (KAIST 수학과 03학번 김재훈)

문제의 일정한 각을 a라 하자. AC : KC = BC : NC 이고 \ACB = \KCN 이므로 4CKN은

4CAB를 C를 중심으로 a만큼 회전시키면서 축소한 것이다. 즉, KN은 AB로부터 a만큼 기울

어진 직선이고, 따라서 KN k BM 이다. 마찬가지로 KM k BN 이고, 따라서 ¤MBNK는 평

행사변형이다. ¤

Baltic 2000-2Baltic 2000-2

\A = 90± 인 이등변 삼각형 ABC가 주어져 있다. M은 AB의 중점이다. A를 지나고 CM에 수직

인 직선이 변 BC와 P에서 만난다. \AMC = \BMP 임을 증명하여라.

Page 194: Baltic Way 팀수학경시대회 1990-2005 pdf 보기

194

증명 정사각형 ABDC를 생각하자.

BD의 중점을 N이라 하면 대칭성에 의해 AN과 DM은 대각선 BC 위에서 교점 P 0을 갖는다.

그리고, CM ? AN 임은 이 그림에서 자명하므로, P = P 0 이다. 따라서, \AMC = \BMD =

\BMP 이다. ¤

Baltic 2000-3Baltic 2000-3

\A = 90± 이고 jABj 6= jACj 인 삼각형 ABC가 주어져 있다. 변 BC, CA, AB 위에 각각 점 D, E,

F를 AFDE가 정사각형이 되도록 잡는다. 직선 BC와 직선 FE, 그리고 삼각형 ABC의 외접원의

점 A에서의 접선이 한 점에서 만남을 증명하여라.

증명 X = BC \EF 라 하면 정사각형의 대각선 EF에 대한 대칭성에서 4XFD ´ 4XFA.

따라서, \XAF = \XDF = \C 이므로, 현과 접선이 이루는 각의 성질에서 XA는 4ABC의

외접원에 접한다. 그러므로, BC, EF , 그리고 A에서의 접선은 한 점 X에서 만난다. ¤

Baltic 2000-4Baltic 2000-4

\A = 120± 인 삼각형 ABC가 주어져 있다. K와 L은 각각 변 AB, AC 위의 점이다.삼각형 ABC의

바깥쪽에 정삼각형 BKP와 CLQ를 그리자. 다음을 증명하여라.

jPQj ¸p3

2¢ (jABj+ jACj)

Page 195: Baltic Way 팀수학경시대회 1990-2005 pdf 보기

2000년 Baltic Way 풀이 195

증명 (대전과학고 1학년 김두리)

A에서 각의 이등분선 AD를 그리면 엇각-엇각으로 BP k CQ (k AD) 임을 알 수 있다.

준식의 우변은 상수이므로 좌변만 살피면 되는데, PQ ? AD, 즉 PQ가 두 직선 BP와 CQ 사

이의 거리일 때 PQ의 길이가 최소이다. 그 때 PQ는 A에서 BP , CQ에 내린 두 높이를 합한

것과 같으므로 우변과 같다. 따라서, 문제의 부등식이 성립하고, 등호성립조건은 PQ ? AD 일

때이다. ¤

Baltic 2000-5Baltic 2000-5

삼각형 ABC는jBCj

jABj ¡ jBCj =jABj+ jBCj

jACj 를 만족한다. \A : \C 의 비를 구하여라.

풀이 (대전 대덕중 3학년 고기혁, 조금 수정)

문제의 식을 정리하면 c2 ¡ a2 = ab, 즉 c2 = a(a+ b). BC의 C쪽 연장선 위에 CD = AC 되는

점 D를 잡으면 BA2 = BC ¢ BD, 즉 4ABC » 4DBA.

따라서, \C = 2\D = 2\A 이다. ¢ ¢ ¢ 답 1 : 2 }

주 BA가 4ABC의 외접원에 접함을 이용해도 된다.

Baltic 2000-6Baltic 2000-6

Fredek은 호텔을 경영한다. 그는 n ¸ 3 명의 손님이 호텔을 찾았을 때면 언제나, 다른 손님들 중 아

는 사람의 수가 같고 둘이 다 알거나 둘다 모르는 손님이 있는 두 명의 손님을 고를 수 있다고 주장

하였다. Fredek의 주장이 옳은 n은 어떤 값들인가? (알고 모르는 것은 상호 대칭적인 관계이다.)

Page 196: Baltic Way 팀수학경시대회 1990-2005 pdf 보기

196

풀이 (KAIST 수학과 03학번 김재훈)

n명의 사람들 각각이 아는 사람의 수는 0; : : : ; n¡ 1명 사이이다. 그런데 0명을 아는 사람(아무

도 모르는 사람)과 n¡ 1명을 아는 사람(모두와 아는 사람)은 함께 있을 수 없으므로, 비둘기집

의 원리에 의해서 n이 어떤 값이든 아는 사람의 수가 동일한 두 사람을 항상 찾을 수 있다.

n일 때 Fredek의 주장이 거짓이라고 하고, 아는 사람의 수가 같은 두 사람 A, B를 생각하자.

A와 B를 둘다 알거나 둘다 모르는 사람이 없어야 하므로 A와 B를 제외한 n¡ 2명의 사람들은

A, B 중 정확히 한 명만을 알아야 한다. 즉, A를 아는 사람의 집합 X와 B를 아는 사람의 집합

Y로 이분되고, 그럼 n은 짝수여야 한다.

즉, n이 홀수이면 Fredek의 주장은 항상 참이다.

n = 4 일 때는 다음의 왼쪽 그림과 같이 거짓인 경우가 있다(A와 B는 아는 사람의 수가 같고,

C와 D는 각각 A와 B 둘 중 한 명만 안다). 실선의 변으로 이은 것은 서로 아는 것을 의미한다.

다음 n = 6 일 때를 보자(오른쪽 그림). A와 B가 아는 사람의 수는 모두 2명(A와 B는 서로 모

를 때) 또는 3명(A와 B가 서로 알 때)이다. 따라서,

² m > 3 또는 m < 2 일 때, m명을 아는 사람은 많아야 1명뿐다.

A, B와 아는 사람의 수가 같은 또다른 사람 C를 생각해보자. A-B, A-C, B-C의 관계에서 각각

A, B, C는 서로 모두 알거나 서로 모두 몰라야 한다. 어느 경우나 A, B를 둘다 알거나 둘다 모

르는 C가 존재하므로 곤란하다. 따라서, 이런 사람 C는 없고, 그럼

² m = 2 또는 3일 때, m명을 아는 사람은 많아야 2명이다.

위의 그림에서 아무도 모르거나 모두를 다 아는 사람은 없다. 따라서, 각각의 손님이 아는 사람

의 수는 최대

1; 2; 2; 3; 3; 4

들이 있을 수 있고, 이 수들은 정확히 6개이므로 이게 정확히 각각의 손님이 아는 사람의 수들

이 된다. 서로 아는 사람을 변으로 이을 때, 하나의 변은 두 사람과 관계되므로, 각자가 아는 사

람들의 수를 모두 합하면 변의 총개수의 2배, 즉 짝수가 된다. 그런데 1+2+ 2+3+3+4는 홀

수이므로 곤란하다. 따라서, 6명일 때는 거짓일 수 없다.

n = 2k ¸ 6 일 때 참이라 하고 n = 2k+ 2 일 때를 풀자. n = 2k+ 2 일 때 거짓이라면, 아는 사

람의 수가 같은 A와 B 두 사람을 제외시키자. 그럼 남은 2k명의 사람들은 아는 사람의 수가 정

확히 1씩 줄어들었다. 그리고, 가정에 따라 2k명일 때는 참이므로, 그 중 아는 사람의 수가 같은

C와 D가 있고, C와 D를 둘다 알거나 둘다 모르는 사람 E도 있다. A와 B를 함께 생각해도 C와

Page 197: Baltic Way 팀수학경시대회 1990-2005 pdf 보기

2000년 Baltic Way 풀이 197

D는 아는 사람의 수가 같고, E는 C와 D를 둘다 알거나 둘다 모르므로 모순. 즉, 2k+ 2명일 때

도 참이다. 그러므로, 귀납적으로 n이 6 이상의 짝수일 때 Fredek의 주장은 언제나 참이다.

¢ ¢ ¢ 답 4를 제외한 모든 수 }

Baltic 2000-7Baltic 2000-7

각각 ON과 OFF 두 가지 상태가 있는, 40£ 50 배열로 된 조정 단추들이 있다. 한 단추를 건드리면

그 단추 자신과 그 단추와 같은 행 혹은 같은 열에 있는 모든 단추의 상태가 뒤바뀐다. 처음에 모두

OFF로 되어있던 조정 단추들을 계속 건드려 언젠가 모두 ON이 되게 할 수 있음을 보여라. 그리

고, 최소 몇 번 건드리면 되는지 구하여라.

풀이 2m £ 2n 크기의 배열로 일반화하여 다루자. 같은 단추를 두 번 건드린 것은 건드리지

않은 것과 같으므로 많아야 한 번씩만 건드렸다고 생각하자. 모든 단추를 다 건드리면 각 단추

마다 홀수(2m+ 2n¡ 1)번 상태가 바뀌므로 모두 ON이 된다.

이제 모두 ON이 되도록 하려면 이 방법뿐임을 보이자. 건드리지 않은 단추 x가 있다고 하자.

x가 ON이 되기 위해서는, x를 포함한 행과 열에 건드린 단추의 수가 모두 홀수개라야 하므로,

대칭적으로 생각하여 x를 포함한 행 R에 건드린 단추가 홀수개 있고 x를 포함한 열 C에 건드

린 단추의 수가 짝수개 있다고 하자.

(1) R의 각 단추 ai에 대해 생각해보면, 이미 같은 행에서 홀수번 상태가 바뀌므로, ai를 포함

한 열 Ci(ai는 제외)에는 건드린 단추의 수가 각각 짝수개라야 한다. 그럼 배열 전체에서

건드린 단추의 수 T는

T = R+ C1 + C2 + ¢ ¢ ¢+ C2n =홀+짝+짝+ ¢ ¢ ¢+짝 =홀

(2) 이번에는 C의 각 단추 bi에 대해 생각해보자. 비슷한 논리로, 이미 같은 열에서 짝수번 상

태가 바뀌므로, bi를 포함한 열 Ri(bi는 제외)에는 건드린 단추의 수가 각각 홀수개이다.

그럼

T = C +R1 +R2 + ¢ ¢ ¢+R2m =짝+홀+홀+ ¢ ¢ ¢+홀 =짝

위에서 T의 홀짝이 서로 맞지 않으므로 모순이고, 따라서 건드리지 않은 단추 x는 하나도 없어

야 한다. }

Baltic 2000-8Baltic 2000-8

파티에서 열 네 명의 친구가 만났다. 그들 중 한 명인 Fredek은 빨리 자러 가고 싶었다. 그는 친구 중

10명에게 작별인사를 했고, 나머지 3명은 깜박 잊어버리고 침실로 갔다. 잠시 후 그가 파티에 다시

돌아와서, 친구 중 10명에게 작별인사를 했고(앞서와 같은 친구에게 할 필요는 없다) 침실로 갔다.

이 후 Fredek은 몇 번 더 파티에 돌아왔고, 그 때마다 정확히 10명에게 작별인사를 하곤 다시 침실

로 갔다. 모든 친구에게 각각 최소 한 번 이상 작별인사를 하고서야 그는 다시 돌아오지 않았다. 아

침에 Fredek은 열 세 명의 친구에게 모두 다른 횟수만큼의 작별인사를 했음을 깨달았다! Fredek이

파티에 돌아온 횟수는 최소 몇 번인가?

Page 198: Baltic Way 팀수학경시대회 1990-2005 pdf 보기

198

풀이 n번 되돌아왔다면 인사하고 침실로 간 것은 n+1번이므로 인사 횟수는 총 10(n+1)번

이다. 마지막으로 돌아와서 작별인사를 했을 때 처음 인사한 친구가 있으므로 꼭 한 번 인사한

사람이 적어도 한 명 있고, 따라서, Fredek이 인사한 횟수는 최대

(n+ 1) + n+ ¢ ¢ ¢+ (n¡ 10) + 1 = 12n¡ 53

번이다. 즉, 12n¡ 53 ¸ 10(n+ 1) 이고, 이것을 풀면 2n ¸ 63, 즉

n ¸ 32

가 된다. 실제로 32번 돌아오는 경우(33번 인사하는 경우)는 다음과 같이 구성될 수 있다(²이인사한 경우, ±이 인사하지 않은 경우).

인 사

²²²²² ²²²²² ²²²²² ²²²²² ²²²²² ²²²²² ²²²²²²²² ²²²²² ²²²²² ²²²²² ²²²²² ²²²²² ²²±²²²²² ²²²²² ²²²²² ²²²²² ²²²²² ²²²²² ±±²²²²²² ²²²²² ²²²²² ²²²²² ²²²²² ²²±±± ²²²²²²²² ²²²²² ²²²²² ²²²²² ²²²±± ±±²²² ²²²²²²²² ²²²²² ²²²²² ²²²±± ±±±²² ²²²²² ²²²²²²²² ²²²²² ²²±±± ±±±²² ²²²²² ²²²²² ²²²²²²²² ±±±±± ±±²²² ²²²²² ²²²²² ²²²²² ²²²±±±±± ²²²²² ²²²²² ²²²²² ²²²²² ²²²²² ±±±²²²²² ²²²²² ²²²²² ²²²²² ²±±±± ±±±±± ²²²²²²²² ²²²²² ²±±±± ±±±±± ±²²²² ²²²²² ²²²±±±±± ±±±±± ±²²²² ²²²²² ²²²²² ²²²²² ²²±±±±±± ±±±±± ±±±±± ±±±±± ±±±±± ±±±±± ±±²따라서, 파티에 돌아온 횟수는 최소 32번. }

Baltic 2000-9Baltic 2000-9

단위 정사각형칸으로 구성된 2k£ 2k 의 체스판 위에서 뛰어다니는 개구리가 있다. 개구리는 한 번

뛰어p1 + k2 의 거리를 이동하고, 그것은 반드시 한 칸의 중앙에서 다른 칸의 중앙까지여야 한다.

체스판의 m개의 칸에 X표를 하고, X표 된 칸으로부터 개구리가 뛰어 바로 옮겨갈 수 있는 모든 칸

에 O표를 했다(그 칸에 이미 X표가 되어있으면 O표로 바꾼다). O표가 모두 n개라 할 때, n ¸ m 임

을 보여라.

증명 가로로 k칸, 세로로 1칸 떨어진 곳으로 뛰는 것이 가능하다. 그럼 2k £ 2k 크기의 체스

판의 모든 칸을 다음과 같이 둘씩 쌍으로 묶자.

²²±±

²의 칸끼리 쌍이 되고, ±의 칸끼리 쌍이 되는 것이다. 그 아래의 두 칸끼리도 비슷하게 쌍으로

묶고, 그 오른쪽의 두 칸끼리도 비슷하게 쌍으로 계속 묶는다. 그럼 모든 칸이 둘씩 쌍이 되고,

한 쌍인 두 칸은 개구리가 서로 뛸 수 있는 칸이 된다. 각각의 쌍에 대해 X표가 1개이면 O표도

1개 이상, X표가 2개이면 O표도 2개됨을 알 수 있다. 따라서, 처음에 X표 된 칸의 수보다 나중

에 O표된 칸의 수가 더 많다. ¤

Page 199: Baltic Way 팀수학경시대회 1990-2005 pdf 보기

2000년 Baltic Way 풀이 199

Baltic 2000-10Baltic 2000-10

칠판에 두 개의 양의 정수가 쓰여져있다. 처음에는 그 중 하나는 2000이고 다른 하나는 2000보다 작

았다. 두 수의 산술평균 m이 정수이면, 다음 작업을 할 수 있다: 두 수 중 하나를 지우고 m을 대신

쓴다. 이 작업이 11번 이상 수행될 수 없음을 보여라. 그리고, 이 작업이 10번 수행되는 예를 들어

라.

증명 한 번 작업할 때마다 두 수의 차가 계속 절반으로 줄어든다. 원래 두 수의 차가 211보다

작으므로 11번 수행하고 나면 두 수의 차가 1보다 작아져서 두 수 모두 정수일 수 없다. 그리고,

두 수의 차가 정확히 210 = 1024 일 때(다른 하나가 976일 때) 10번 수행된다. ¤

Baltic 2000-11Baltic 2000-11

양의 정수들의 수열 a1; a2; : : : 은 임의의 m, n에 대해 다음을 만족한다: 만약 m이 n의 약수이고

m < n 이면, am은 an의 약수이고 am < an 이다. 이 때, a2000의 최소값을 구하여라.

풀이 a1 < a5 < a25 < a125 < a250 < a500 < a1000 < a2000 이고

a1 j a5 j a25 j a125 j a250 j a500 j a1000 j a2000

이므로 a2000 ¸ 27 이 된다. 다시 말하면,

a2000 ¸ 2a1000 ¸ 22a500 ¸ ¢ ¢ ¢ ¸ 27a1 ¸ 27 (¤)

일반적으로 gn을 n에 곱해진 소인수의 개수, 즉

n = pe11 pe22 ¢ ¢ ¢ perr 일 때 gn = e1 + e2 + ¢ ¢ ¢+ er

이라 하고 an = 2gn 으로 정의하면 문제의 조건을 잘 만족하는 수열이 된다. 이 때 a2000 = 2

7

이므로 (¤)의 식의 등호가 성립 가능하다. 따라서, a2000의 최소값은 27 이다. }

Baltic 2000-12Baltic 2000-12

양의 정수 x1; x2; : : : ; xn은 이 중 어느 것도 다른 것의 앞부분이 되지 않는 수들이다. (예를 들어,

12는 12, 125, 12405의 앞부분이다.) 다음을 증명하여라.

1

x1+1

x2+ ¢ ¢ ¢+ 1

xn< 3

Page 200: Baltic Way 팀수학경시대회 1990-2005 pdf 보기

200

증명 자연수 a과 길이(자리수)가 같고 마지막 한 자리만 다를 수 있는 자연수들을 a와 같은

계열의 수라고 부르자. x1; : : : ; xn 중 가장 자리수가 긴 것 하나를 택해 a라 하자. a의 마지막 한

자리를 제외한 수 b는 a의 앞부분이므로 x1; : : : ; xn에 있을 수 없고 b를 앞부분으로 갖는 수는

a와 같은 계열의 수들뿐이므로(a보다 자리수가 더 긴 것은 없기 때문) a와 같은 계열의 수들을

모두 제거하고 대신 b를 넣은 수열 y1; : : : ; ym은

1

10b+

1

10b+ 1+ ¢ ¢ ¢+ 1

10b+ 9<

1

10b+

1

10b+ ¢ ¢ ¢+ 1

10b| {z }10개

=1

b

에 의해 역수의 합이 더 커진다. 이런 작업을 다시 y1; : : : ; ym에 대해 반복하면 자리수의 길이

가 계속 줄어들어 모든 항이 자리수가 1인 수가 되고,

1

x1+1

x2+ ¢ ¢ ¢+ 1

xn<1

1+1

2+ ¢ ¢ ¢+ 1

9< 3

이 된다. ¤

Baltic 2000-13Baltic 2000-13

a1; a2; : : : ; an은 정수로 이루어진 등차수열이고, i = 1; 2; : : : ; n¡ 1 에 대해서는 항상 i가 ai의 약수

이지만, n은 an의 약수가 아니라고 한다. n이 어떤 소수의 거듭제곱임을 보여라.

증명 ai = a+ id 로 쓸 수 있다. 그럼

i j ai = a+ id =) i j an - an = a+ nd =) n - a

가 된다. 즉, a는 2; : : : ; n¡ 1 들의 배수이지만 n의 배수는 아니다. 만일 n이 1보다 큰 서로 소

인 두 자연수 r와 s의 곱 n = rs 로 나타낼 수 있다면 r j a, s j a 이므로 n j a 가 되어 모순. 따

라서, n은 소인수를 딱 1개만 갖는 수이다. ¤

Baltic 2000-14Baltic 2000-14

양의 약수의 개수에 100을 곱했더니 자기 자신이 되는 양의 정수 n을 모두 찾아라.

풀이 k의 양의 약수의 개수를 d(k)로 나타내기로 하자. n = 100d(n) 이므로 100의 배수이고,

따라서

n = 2a+25c+2m (a; c ¸ 0; 2; 5 - m)

이라고 둘 수 있다. d(n) = (a+ 3)(c+ 3)d(m) 이 되므로, n = 100d(n) 으로부터

2a5cm = (a+ 3)(c+ 3)d(m) (1)

이 된다. 다음 보조정리를 확인해두자.

Page 201: Baltic Way 팀수학경시대회 1990-2005 pdf 보기

2000년 Baltic Way 풀이 201

보조정리 임의의 자연수 k에 대해,

d(k) · k

이고, 등호는 k = 1; 2 일 때만 성립한다.

보조정리의 증명 k의 양의 약수가 될 수 있는 후보는 1; :::; k의 k개뿐이므로 d(k) · k

이다. 특히 k ¡ 1 은 k와 서로 소이므로 k ¡ 1 ¸ 2 이면 k ¡ 1 은 k의 약수가 아니어서 등

호가 성립하지 않는다. k = 1; 2 일 때는 등호가 성립함을 금방 확인할 수 있다. ¤

위의 보조정리에 의해 m ¸ d(m) 이므로

2a5c · (a+ 3)(c+ 3) (2)

이어야 한다. a ¸ 3, c ¸ 1 이면 2a > a+3, 5c > c+3 이므로 좌변이 커서 성립하지 않는다. 따

라서, a < 3 또는 c < 1 일 때만 살피면 된다. 즉, a = 0; 1; 2 또는 c = 0 일 때만 살피면 된다.

(i) a = 0 일 때: (1)을 다시 쓰면

5cm = 3(c+ 3)d(m)

이다. 보조정리에 의해 5c · 3(c + 3) 이어야 하는데, 이것을 만족하는 것은 c = 0; 1뿐이

다. c = 1 이면 우변이 짝수가 되어 곤란하므로 c = 0 만 가능하다. 즉,

m = 9d(m)

9 j m 이므로 m = 3b+2r (2; 3; 5 - r) 이라 하고 대입하여 정리하면

3br = (b+ 3)d(r)

역시 보조정리에 의해 3b · b+3 이어야 하는데, 이것을 만족하는 것은 b = 0; 1뿐이다. 그

런데, b = 0 이면 우변만 3의 배수라서 안 되고, b = 1 일 때도 우변만 짝수라서 안 된다.

즉, a = 0 일 때는 성립하지 않는다.

(ii) a = 1 일 때: (1)에서 좌변은 4의 배수가 아닌데 우변은 4의 배수이므로 모순.

(iii) a = 2 일 때: (1)을 다시 쓰면

4 ¢ 5cm = 5(c+ 3)d(m)

우변이 5의 배수이므로 c = 1 + d 로 쓰면

4 ¢ 5dm = (d+ 4)d(m)

보조정리에 의해 4 ¢ 5d · d + 4 여야 하는데, 그것을 만족하는 경우는 d = 0 일 때(c = 1

일 때)뿐이다.

m = d(m)

만 남으므로, 보조정리에 의해 홀수인 m은 1만 가능하다. 즉, n = 2a+25c+1m = 2453 만

가능하다.

Page 202: Baltic Way 팀수학경시대회 1990-2005 pdf 보기

202

(iv) c = 0 일 때: (1)을 다시 쓰면

2am = 3(a+ 3)d(m)

(i){(iii) 이외의 경우만 보면 되므로 a ¸ 3 일 때만 보면 되고, 그 때 2a · 3(a + 3) 이 성

립하는 경우는 a = 3; 4뿐이다.

(a) a = 3 일 때:

4m = 9d(m)

이 되는데, 9 j m 이므로 m = 3b+2r (2; 3; 5 - r) 로 두면

4 ¢ 3br = (b+ 3)d(r)

보조정리에 의해 4 ¢ 3b · b+ 3 이어야 하는데, 이것을 만족하는 b는 없다.

(b) a = 4 일 때:

16m = 21d(m)

여기서 7 j m 이므로 m = 7d+1r (2; 5; 7 - r) 로 두면

16 ¢ 7dr = 3(d+ 2)d(r)

보조정리에 의해 16 ¢ 7d · 3(d+ 2) 여야 하는데, 이것을 만족하는 d는 없다.

(i){(iv)에서 만족하는 것은 (iii)의 경우의 n = 2453 = 2000뿐이다. }

Baltic 2000-15Baltic 2000-15

n은 2나 3으로 나누어지지 않는 양의 정수이다. 모든 정수 k에 대해, (k+1)n¡ kn¡ 1 이 k2+ k+1

로 나누어짐을 보여라.

증명1 x2 + x + 1 = 0 의 두 허근을 w, ¹w 라 하고, f(x) = (x + 1)n ¡ xn ¡ 1 이라 하자.

n = 6t§ 1 꼴이고 w3 = 1 이므로

f(w) = (w + 1)n ¡ wn ¡ 1= (¡w2)n ¡ wn ¡ 1= (¡w2)§1 ¡ w§1 ¡ 1= ¡((w§1)2 + w§1 + 1)

여기서 w§1 = w or ¹w 이므로 f(w) = 0 이 된다. 마찬가지로 f( ¹w) = 0. 즉, 인수 정리에 의해

x2 + x+ 1 j f(x) 이다. ¤

Page 203: Baltic Way 팀수학경시대회 1990-2005 pdf 보기

2000년 Baltic Way 풀이 203

증명2 f(x) = (x+ 1)n ¡ xn ¡ 1 이라 하고 f(x)를 x2 + x+ 1 로 나눈 나머지를 n에 따라 구

해보자.

n (x+ 1)n xn f(x)

0 1 1

1 x+ 1 x 0

2 (x+ 1)2 ´ x x2 ´ ¡(x+ 1)3 (x+ 1)x ´ ¡1 ¡x(x+ 1) ´ 14 ¡(x+ 1) x

5 ¡x ¡(x+ 1) 0

6 1 1

와 같이 n이 6을 주기로 반복됨을 알 수 있다. 그리고 n = 6t § 1 꼴이면 항상 f(x) ´ 0

(mod x2 + x+ 1) 이다. ¤

Baltic 2000-16Baltic 2000-16

모든 양의 실수 a, b, c에 대해 다음이 성립함을 증명하여라.pa2 ¡ ab+ b2 +

pb2 ¡ bc+ c2 ¸

pa2 + ac+ c2

증명1 \AOB = \BOC = 60± 이고 OA = a, OB = b, OC = c 가 되도록 O, A, B, C를 잡

아보자.

그럼 cos 제2법칙에 의해

AB2 = a2 + b2 ¡ 2ab ¢ cos 60± = a2 ¡ ab+ b2

BC2 = b2 + c2 ¡ 2bc ¢ cos 60± = b2 ¡ bc+ c2

AC2 = a2 + c2 ¡ 2ac ¢ cos 120± = a2 + ac+ c2

이므로, 문제의 부등식은 AB +BC ¸ AC 의 삼각부등식이 되어 성립한다. ¤

증명2 (전남과학고 1학년 심민수)

준식의 양변을 제곱시킨다.

a2 ¡ ab+ b2 + b2 ¡ bc+ c2 + 2p(a2 ¡ ab+ b2)(b2 ¡ bc+ c2) ¸ c2 + ca+ a2

2p(a2 ¡ ab+ b2)(b2 ¡ bc+ c2) ¸ ab+ bc+ ca¡ 2b2

Page 204: Baltic Way 팀수학경시대회 1990-2005 pdf 보기

204

이 식의 양변을 또 제곱시키면 다음과 같다.

4(a2b2 ¡ a2bc+ c2a2 ¡ ab3 + ab2c¡ abc2 + b4 ¡ b3c+ b2c2)

¸ a2b2 + b2c2 + c2a2 + 4b4 + 2(a2bc+ ab2c+ abc2)¡ 4b2(ab+ bc+ ca)

정리하면

3a2b2 + 3b2c2 + 3c2a2 ¡ 6a2bc+ 6ab2c¡ 6abc2 ¸ 03(ab+ bc¡ ca)2 ¸ 0

이것은 참이고, 지금까지의 과정은 역으로 잘 거슬러올라갈 수 있으므로 문제의 식도 참이다.

특히, 등호조건은 ab+ bc = ca 일 때, 즉 1c+ 1

a= 1

b 일 때이다. ¤

Baltic 2000-17Baltic 2000-17

다음 연립방정식의 모든 실수해를 구하여라.8>>>>>><>>>>>>:

x + y + z + t = 5

xy + yz + zt + tx = 4

xyz + yzt + ztx + txy = 3

xyzt = ¡1

풀이 (부산 건국고 2학년 신승현)

x와 z, 그리고 y와 t를 각각 한 쌍으로 보면 다음과 같이 생각할 수 있다.

(x+ z) + (y + t) = 5

(x+ z)(y + t) = 4

xz(y + t) + yt(z + x) = 3

(xz)(yt) = ¡1

처음 두 식에서 x+ z와 y + t는 이차방정식 A2 ¡ 5A+ 4 = 0 의 두 근임을 알 수 있다. 따라서,

(x+ z; y + t) = (1; 4) or (4; 1) 이다.

(i) (x + z; y + t) = (4; 1) 일 때: 이를 세 번째식에 대입하면 xz + 4yt = 3 이고, 마지막 식

에서 (xz)(4yt) = ¡4 이므로, xz와 4yt는 이차방정식 B2 ¡ 3B ¡ 4 = 0 의 두 근이다. 즉,

(xz; 4yt) = (4;¡1) or (¡1; 4). 전자의 경우 x = z = 2, fy; tg = f 1§p2

2 g 가 되고, 후자의

경우는 y, t에 관한 이차방정식 C2 ¡ C + 1 = 0 의 판별식이 음이므로 실근을 갖지 않는

다.

(ii) (x+z; y+t) = (1; 4)일 때: (i)의 경우와 대칭적이므로 마찬가지로 하여 fx; zg = f 1§p2

2g,

y = t = 2 가 된다.

Page 205: Baltic Way 팀수학경시대회 1990-2005 pdf 보기

2000년 Baltic Way 풀이 205

(i), (ii)에서 구하는 실수해는

(x; y; z; t) =

µ2;1 +

p2

2; 2;1¡p22

¶;

µ2;1¡p22

; 2;1 +

p2

2

¶;µ

1 +p2

2; 2;1¡p22

; 2

¶;

µ1¡p22

; 2;1 +

p2

2; 2

¶들이 전부이다. }

Baltic 2000-18Baltic 2000-18

다음 방정식을 만족하는 모든 양의 실수 x, y를 구하여라.

x+ y +1

x+1

y+ 4 = 2 ¢ (p2x+ 1 +

p2y + 1 )

풀이 (광주 광덕고 2학년 임익주)

x; y > 0 이므로, A.M ¸ G.M 에 의해

x+ y +1

x+1

y+ 4 =

µx+

2x+ 1

x

¶+

µy +

2y + 1

y

¶¸ 2p2x+ 1 + 2

p2y + 1

이 된다. 문제의 식은 여기서 등호가 성립할 경우가 된다. 즉,

x =2x+ 1

x; y =

2y + 1

y

이다. 분모를 이항하여 이차방정식을 풀면 x; y = 1 § p2 이다. 그럼 x; y > 0 이므로 x =

1 +p2 = y 가 유일한 해이다. }

Baltic 2000-19Baltic 2000-19

t ¸ 1

2는 실수이고, n은 양의 정수이다. 다음을 증명하여라.

t2n ¸ (t¡ 1)2n + (2t¡ 1)n

증명 a = (t¡ 1)2, b = 2t¡ 1 이라 하면 a+ b = t2 이므로, 준식은

(a+ b)n ¸ an + bn

과 같다. t ¸ 12 일 때 a; b ¸ 0 이므로, 식의 전개를 생각하면 이것은 당연히 성립한다. ¤

Baltic 2000-20Baltic 2000-20

모든 양의 정수 n에 대해, xn을 다음과 같이 정의한다.

xn =(2n+ 1) ¢ (2n+ 3) ¢ ¢ ¢ ¢ ¢ (4n¡ 1) ¢ (4n+ 1)

2n ¢ (2n+ 2) ¢ ¢ ¢ ¢ ¢ (4n¡ 2) ¢ 4n1

4n< xn ¡

p2 <

2

n임을 증명하여라.

Page 206: Baltic Way 팀수학경시대회 1990-2005 pdf 보기

206

증명 (서울 당산서중 2학년 박민재)

부등식 A2 > (A+ 1)(A¡ 1) 을 이용한다.

x2n =(2n+ 1)[(2n+ 1)(2n+ 3)][(2n+ 3)(2n+ 5)] ¢ ¢ ¢ [(4n¡ 1)(4n+ 1)](4n+ 1)

(2n)2(2n+ 2)2 ¢ ¢ ¢ (4n)2

<(2n+ 1)(2n+ 2)2(2n+ 4)2 ¢ ¢ ¢ (4n)2(4n+ 1)

(2n)2(2n+ 2)2 ¢ ¢ ¢ (4n)2

=(2n+ 1)(4n+ 1)

(2n)2=8n2 + 6n+ 1

4n2< 2 +

2

n

x2n =(2n+ 1)2(2n+ 3)2 ¢ ¢ ¢ (4n+ 1)2

(2n)2(2n+ 2)2 ¢ ¢ ¢ (4n)2

>[(2n)(2n+ 2)][(2n+ 2)(2n+ 4)] ¢ ¢ ¢ [(4n)(4n+ 2)]

(2n)2(2n+ 2)2 ¢ ¢ ¢ (4n)2

=4n+ 2

2n= 2 +

1

n

즉, 다음과 같다.1

n< x2n ¡ 2 < 2

n(¤)

x2n < 2 + 2n· 4 이므로 xn < 2, 또 xn > 0 임은 당연하고, 그럼 1 < xn +

p2 < 4, 즉

1

4<

1

xn +p2< 1

이 성립한다. 이것을 (¤)에 변변 곱하면1

4n< xn ¡

p2 <

2

n이 됨을 알 수 있다. ¤

Page 207: Baltic Way 팀수학경시대회 1990-2005 pdf 보기

2001년 Baltic Way 풀이 207

2001년 Baltic Way 풀이

Baltic 2001-1Baltic 2001-1

어떤 시험에 8문제가 준비되어 있다. 각 학생들에게 이것들 중 3개가 주어진다. 어느 두 학생도 공

통적으로 받는 문제는 기껏해야 1개이다. 학생들은 최대 몇 명이 가능하겠는가?

풀이 두 문제의 묶음을 문제쌍이라고 부르기로 하자. 문제의 조건에 의해 어떤 문제쌍도 두

학생에게 동시에 주어지는 법이 없다. 임의의 한 문제 a에 대해 a가 네 학생에게 주어지면 각

학생마다 a를 포함하는 문제쌍이 둘씩 있으므로 a를 포함하는 문제쌍이 모두 8개. a를 제외하

면 7문제이므로, 비둘기집의 원리에 의해 이 8개의 문제쌍에는 a를 제외하고 중복하여 나타나

는 문제가 또 있고, 그럼 문제의 조건에 부합되지 않는다. 따라서,

² 한 문제는 많아야 3명의 학생에게만 주어질 수 있다.

그럼 임의의 한 문제 a를 포함하는 문제쌍은 최대 6번 나타난다. 각각의 문제에 대해 모두 그

러하므로, 각각의 문제가 문제쌍에 나타나는 횟수는 많아야 8 £ 6 = 48번이고, 그럼 문제쌍은

많아야 482= 24개 있다. 한 명의 학생으로부터 3개의 문제쌍이 구성되므로 학생수는 많아야

243= 8명이다. 그리고

123; 145; 167; 246; 258; 347; 368; 578

과 같이 나눠주면 8명이 실제 가능하다. }

Baltic 2001-2Baltic 2001-2

n ¸ 2 는 양의 정수이다. f1; 2; 3; : : : g의 서로 소이고 공집합이 아닌 n개의 부분집합을 잘 구성하

여, 모든 양의 정수가 서로 다른 부분집합에 있는 최대 n개의 정수들의 합으로 유일하게 표현되게

할 수 있는가?

풀이 n개의 부분집합 중 n¡ 1개는

f1g; f2g; f4g; f8g; : : : ; f2n¡2g

로 하고, 남은 1개의 부분집합은

2n¡1의 배수를 모두 모은 집합

으로 하면 된다. 즉, 임의의 양의 정수 m을 그 2진법 전개로 생각하여, 1의 자리수, 21의 자리

수, 22의 자리수, 2n¡2의 자리수, 그리고 2n¡1의 자리 위쪽 전부로 구분하여 합하는 것으로 생

각하면 되겠다. }

Page 208: Baltic Way 팀수학경시대회 1990-2005 pdf 보기

208

Baltic 2001-3Baltic 2001-3

1, 2, : : : ; 49의 수들이 7£ 7 로 배열되어 있고, 각 행과 각 열마다 수의 합이 계산되었다. 이 14개의

합들 중 어떤 것은 홀수이고 어떤 것은 짝수이다. 홀수인 합을 모두 합한 것을 A, 짝수인 합을 모두

합한 것을 B라 하자. A = B 가 되도록 수를 배열하는 것이 가능한가?

풀이 A와 B를 합하면 모든 행과 열을 한 번씩 합한 것이 된다. 즉

A+B = 1 + 2 + ¢ ¢ ¢+ 49 = 49 ¢ 502

= 49 ¢ 25

이것은 홀수이므로 A = B 는 불가능하다. }

Baltic 2001-4Baltic 2001-4

p와 q는 서로 다른 두 소수이다. 다음을 증명하여라.jpq

k+j2pq

k+j3pq

k+ ¢ ¢ ¢+

j (q ¡ 1)pq

k=1

2(p¡ 1)(q ¡ 1)

(bxc는 x를 넘지 않는 가장 큰 정수를 나타낸다.)

증명 아래 그림을 떠올리는 것으로 충분하다.

0 < x < b, 0 < y < a 의 직사각형 영역에는 격자점이 (a¡ 1)(b¡ 1)개 있고, 대각선 y = (a=b)x

를 기준으로 위쪽 직각삼각형과 아래쪽 직각삼각형이 (격자점 위치를 고려하여)합동이므로 이

들 격자점들이 같은 수만큼 분포한다. a, b가 서로 소이므로 대각선 위에는 격자점이 없다. 문

제의 식의 좌변은 아래쪽 직각삼각형의 격자점의 개수이다. ¤

주 p와 q는 꼭 소수일 필요는 없고, 서로 소인 두 자연수이면 충분하다. 이렇게 확장된 문제

가 hProblem-Solving Through Problemsi (Loren C. Larson 저) 라는 책의 11쪽에 실려있고, 우

리 나라에서 1990년 KMO 최종선발전에도 출제된 적이 있다. 따라서, 이 문제가 출제된 것은

출제팀의 실수라고 여겨진다.

Baltic 2001-5Baltic 2001-5

한 원 위의 2001개의 주어진 점들을 빨강이나 녹색으로 칠했다. 이제 모든 점의 색을 다음의 방법

으로 동시에 다시 결정한다: 만약 한 점 P의 바로 이웃한 두 점이 모두 P와 같은 색이었으면 P의

색을 바꾸지 않고 그대로 두지만, 만약 그렇지 않다면 P는 다른 색깔로 바꿔 칠한다. 처음에 원이

Page 209: Baltic Way 팀수학경시대회 1990-2005 pdf 보기

2001년 Baltic Way 풀이 209

채색된 꼴을 F1이라 하고, 위와 같이 다시 칠한 꼴을 F2, 작업을 계속 반복하여 F3, : : : 라 하자.

Fn0 = Fn0+2 가 되는 수 n0 · 1000 가 있음을 증명하여라. 이 얘기는 1000을 999로 바꾸어도 역시

성립하는가?

증명 모든 점의 색이 같은 때에는 색이 계속 유지되므로 F1 = F3 으로 자명하다. 그 외의 경

우만 보기로 하자. 같은 색으로 계속 이웃한 점들의 묶음을 체인이라고 하자. 길이가 3 이상인

체인의 양끝점은 다음 시점에는 색깔이 변하여 고립점(길이 1인 체인)이 되고 그외의 (체인 내

부의) 점은 색깔이 유지되어 체인의 길이가 2만큼 줄어든다. 길이가 1이거나 2인 체인은 색깔

이 변하면서도 체인의 길이를 그대로 유지한다. F1에서의 가장 긴 체인의 길이는 최대 2000이

고 각 체인의 길이는 매번 2씩 줄어들어 언젠가 길이가 1 또는 2가 되므로 999번이면 모두 길

이가 2 이하가 된다. 즉 F1000 = F1000+2 로, n0 · 1000 이 존재한다. 한편, 맨처음 길이 2000인

체인이 하나 있었을 때는 998번 해도 길이 4인 체인이 남아있으므로 n · 999 에서 Fn = Fn+2

를 만족하는 n이 없다. ¤

Baltic 2001-6Baltic 2001-6

한 원 c 위에 점 A, B, C, D, E가 차례로 있고, AB k EC, AC k ED 이다. 원 c의 점 E에서의 접선

이 직선 AB와 점 P에서 만난다. 직선 BD와 EC는 점 Q에서 만난다. jACj = jPQj 임을 증명하여

라.

증명 \PEA = \ACE = \CED = \CBQ 이고(차례로 접선과 현이 이루는 각, 엇각, 원주

각),

AE = BC (등변사다리꼴), 또 \PAE = \AEC = \BCQ 이므로(차례로 엇각, 등변사다리

꼴),

4PEA ´ 4QBC

이다(ASA합동). 따라서, AP = QC. 그럼 APQC는 평행사변형이고, AC = PQ 임이 확인된

다. ¤

Page 210: Baltic Way 팀수학경시대회 1990-2005 pdf 보기

210

Baltic 2001-7Baltic 2001-7

평행사변형 ABCD가 주어져있다. A를 지나는 원이 선분 AB, AC, AD와 각각 선분 내부의 점 M ,

K, N에서 만난다. 다음을 증명하여라.

jABj ¢ jAM j+ jADj ¢ jAN j = jAKj ¢ jACj

증명 ¤AMKN을 A를 중심으로 확대하거나 축소해도 AM , AN , AK에 같은 비례상수가 곱

해지므로 식은 그대로 유지된다. 따라서, K가 C에 일치하도록 확대해놓고 생각해도 된다.

¤AMCG가 평행사변형이 되도록 점 G를 잡고, 4NDC의 외접원이 AC와 다시 만나는 점을

H라 하자. 그럼 \AGD = \CMB = 180± ¡ \ANC = \DHC 이므로 ¤AHDG도 원에 내접

하는 사각형이다. 이로부터

AD ¢ AN = AH ¢ ACAB ¢AM = CD ¢ CG = CH ¢ CA

이고, 이것을 변변 더하면 AD ¢AN +AB ¢AM = (AH +CH)AC = AC2 으로 원하던 식이 된

다. ¤

주 이것은 익히 잘 알려져있던 문제로,츨제 실수인 듯하다.예를 들어, 1918년 헝가리 EÄotvÄos

경시대회에도 비슷한 문제가 나왔었다.

Baltic 2001-8Baltic 2001-8

ABCD는 볼록사변형이고, N은 BC의 중점이다. 또, \AND = 135± 라 하자. 다음을 증명하여라.

jABj+ jCDj+ 1p2¢ jBCj ¸ jADj

Page 211: Baltic Way 팀수학경시대회 1990-2005 pdf 보기

2001년 Baltic Way 풀이 211

증명 AN , ND에 대한 B, C의 대칭점을 각각 P , Q라 하자.

\AND = 180± ¡ 45± 임에서 \PNQ = 180± ¡ 90± = 90± 임을 알 수 있고, 그럼

AB +BCp2+ CD = AP + PQ+QD ¸ AD

의 최단거리 부등식이 된다. ¤

Baltic 2001-9Baltic 2001-9

마름모 ABCD가 주어져있다. \APD + \BPC = 180± 를 만족하며 이 마름모 안을 움직이는 점

P의 자취를 구하여라.

풀이 4APD를 평행이동하여 4BQC가 되도록 하자. 그럼 \BQC +\BPC = 180± 이므로

¤BQCP는 원에 내접하는 사각형이 된다.

그럼 \PBC = \PQC = \PDC 이다(차례로 원주각, 평행사변형의 대각). B, P , D가 일

직선을 이루지 않는다면, \PBD = \PDB 임에서 CBD와 PBD가 모두 이등변삼각형이므

로 P는 마름모의 대각선 AC 위에 있는 점이 된다. 역으로, P가 AC나 BD 위에 있으면 조건

\APD + \BPC = 180± 를 잘 만족한다는 것도 금방 확인할 수 있다(P가 AC 위에 있으면

\PBC = \PDC = \PQC 이므로 PBQC는 원에 내접하는 사각형이 됨). 따라서, 구하는 자

취는 두 대각선이다. }

Baltic 2001-10Baltic 2001-10

삼각형 ABC에서, \BAC의 이등분선이 변 BC와 점 D에서 만난다. jBDj ¢ jCDj = jADj2 과

\ADB = 45± 임을 알 때, 삼각형 ABC의 모든 각을 구하여라.

Page 212: Baltic Way 팀수학경시대회 1990-2005 pdf 보기

212

풀이 D가 AE의 중점이 되는 점 E를 잡으면 AD ¢DE = BD ¢DC 이므로 ¤ABEC는 원에

내접하는 사각형이다.

AD가 각의 이등분선이므로 BE = EC. E에서 BC에 내린 수선의 발을 M이라 하고 외접원

의 중심을 O라 하면, \ADB = 45± 라 했으므로 4OAE와 4MDE는 닮음비 2 : 1의 직각이등

변삼각형이다. 즉, M은 OE의 중점이고 BO = BE 로 4OBE는 정삼각형이다. 이로부터 각을

구하면

\A = 1

2\BOC = 60±

\B = 180± ¡ 45± ¡ 12\A = 105±

\C = 180± ¡ \A¡\B = 15±

가 된다. ¢ ¢ ¢ 답 (\A;\B;\C) = (60±; 105±; 15±) }

Baltic 2001-11Baltic 2001-11

함수 f는 모든 양의 정수에서 정의되고 실수값을 갖는다. 임의의 정수 a > 1, b > 1 에 대해,

d = gcd(a; b) 라 할 때,

f(ab) = f(d) ¢µf³ad

´+ f³ bd

´¶의 식이 성립한다. f(2001)의 모든 가능한 값을 구하여라.

풀이 임의의 자연수 n > 1 에 대해

f(n2) = f(n ¢ n) = f(n)(f(1) + f(1)) = 2f(1)f(n) (1)

f(n4) = f(n3 ¢ n) = f(n)(f(n2) + f(1)) = f(1)f(n)(2f(n) + 1) (2)

f(n4) = 2f(1)f(n2) = 4f(1)2f(n) (3)

이다. (2)와 (3)으로부터

f(1) = 0; or f(n) = 0; or f(n) = 2f(1)¡ 12

(¤)

임을 알 수 있다. 상수함수 f(n) ´ 0 은 문제의 조건을 잘 만족하므로 0은 f(2001)로 가능한 값

임을 알 수 있다. 이제 다른 값이 있나 찾아보기 위해 f(2001) 6= 0 이라 하자. 만일 f(1) = 0 이

Page 213: Baltic Way 팀수학경시대회 1990-2005 pdf 보기

2001년 Baltic Way 풀이 213

면 f(2001) = f(3 ¢ 667) = f(1)(f(3)+ f(667)) = 0 이므로 곤란하다. 따라서, f(1); f(2001) 6= 0이므로 (¤)에서 f(2001) = 2f(1)¡ 1

2 이다. 또, (1)에서 n = 2001 일 때를 생각하면 f(20012)도

0이 아님을 알 수 있고, 그럼 다시 (¤)에서 f(20012) = 2f(1)¡ 12 이다. 즉,

f(2001) = f(20012) = 2f(1)f(2001)

이므로 f(2001) = 12 이다. 실제로 상수함수 f(n) ´ 1

2 도 문제의 조건을 잘 만족하므로 12도 틀

림없이 가능한 값이다. ¢ ¢ ¢ 답 0, 12 }

Baltic 2001-12Baltic 2001-12

a1; a2; : : : ; an은

nXi=1

a3i = 3 과

nXi=1

a5i = 5 를 만족하는 양의 실수들이다.nXi=1

ai >3

2임을 증명하여

라.

증명 ai들이 모두 양수이므로 Cauchy-Schwarz 부등식에서

(a51 + a52 + ¢ ¢ ¢+ a5n)(a1 + a2 + ¢ ¢ ¢+ an) ¸ (a31 + a32 + ¢ ¢ ¢+ a3n)2

이 성립한다. A = a1 + ¢ ¢ ¢+ an 이라 하면 이것은 5X ¸ 32 과 같고, 따라서 X ¸ 95 > 3

2 이 된

다. ¤

Baltic 2001-13Baltic 2001-13

a0; a1; a2; : : : 는 a0 = 1 과 an = ab7n=9c + abn=9c (n = 1; 2; : : : ) 을 만족하는 실수 수열이다.

ak <k

2001!가 되는 양의 정수 k가 존재함을 보여라. (bxc는 x를 넘지 않는 가장 큰 정수를 나타낸

다.)

증명 (O±cial Solution)

다음 방정식을 생각하자. µ7

9

¶x+

µ1

9

¶x= 1

q79 +

p19 =

p7+13 > 1, 79 +

19 < 1 이므로 이 방정식은 1

2 < ® < 1 인 근을 갖는다. 이제 적당

히 큰 M > 0 에 대해

an ·M ¢ n® (1)

가 성립함을 보이자. 그럼 n®

n 의 차수가 음수이므로, n이 충분히 크면 얼마든지 작아질 수 있

어akk

< 12001! 이 되는 k가 존재함이 자명해진다. 우선 1 · n · 8 에 대해 (1)을 만족하는 M을

택하자. n ¸ 9 이면 1 · ¥n9

¦;¥7n9

¦< n 이므로, n보다 작은 자연수에 대해 (1)이 항상 성립한

Page 214: Baltic Way 팀수학경시대회 1990-2005 pdf 보기

214

다고 가정하면

an = ab7n=9c + abn=9c

·M ¢¹7n

9

º®+M ¢

jn9

k®·M ¢

µ7n

9

¶®+M ¢

³n9

´®=M ¢ n®

µµ7

9

¶®+

µ1

9

¶®¶=M ¢ n®

로 n일 때도 성립한다. 따라서, 수학적 귀납법으로 (1)이 증명되었고, 문제도 확인되었다. ¤

Baltic 2001-14Baltic 2001-14

2n개의 카드가 있다. 각각의 카드에 구간 [1; 2]의 실수를 하나씩 적었다. 카드를 두 더미로 나누는

데, 두 더미 각각의 합 s1, s2가n

n+ 1· s1

s2· 1 을 만족하도록 할 수 있음을 증명하여라.

증명 2n개의 실수를 크기 순서로 a1 · a2 · ¢ ¢ ¢ · a2n 이라 하고,

s1 = a1 + a3 + ¢ ¢ ¢+ a2n¡1

s2 = a2 + a4 + ¢ ¢ ¢+ a2n

이라 하자. 그럼 s2 ¡ s1 은 구간 [a1; a2n] 의 일부 조각들의 길이를 모은 것이므로 구간 전체의

길이 a2n ¡ a1 보다 작다. 즉,

s2 ¡ s1 = (a2n ¡ a2n¡1) + (a2n¡2 ¡ a2n¡3) + ¢ ¢ ¢+ (a4 ¡ a3) + (a2 ¡ a1)

¸ (a2n ¡ a2n¡1) + (a2n¡1 ¡ a2n¡2) + ¢ ¢ ¢+ (a3 ¡ a2) + (a2 ¡ a1)

= a2n ¡ a1

이로부터, 0 · s2 ¡ s1 · 1 이고, 또 s1 ¸ n 이므로,

1 ¸ s1s2¸ s1

s1 + 1¸ n

n+ 1

임을 확인할 수 있다. ¤

별증 (대전 어은중 1학년 임준혁, 수정됨)

귀류법으로,그런 s1, s2를 찾을 수 없다고 하자. 2n개의 수를 임의로 n개씩 두 더미로 나누어 큰

쪽은 s2, 작은 쪽을 s1이라 할 때, s2¡ s1 이 최소가 되도록 나누었다고 하자. 그리고 s = s1+s2

라 하자. s1s2· 1 이므로, 그럼 귀류법의 가정에 의해 s1

s2< n

n+1 이다. (n + 1)s1 < ns2 의 양변

에 (n+ 1)s2 혹은 ns1 을 더하면

s2 >n+ 1

2n+ 1s

s1 <n

2n+ 1s

) s2 ¡ s1 >1

2n+ 1s ¸ 2n

2n+ 1(= d1)

Page 215: Baltic Way 팀수학경시대회 1990-2005 pdf 보기

2001년 Baltic Way 풀이 215

이 된다. s2에 합해진 항들의 평균이 s1의 평균보다 크므로, a · b 인 s1의 항 a와 s2의 항 b가

있다. 만일 b ¡ a · d1 이면 a와 b를 s2와 s1에 서로 바꿔넣으면 js2 ¡ s1j 의 값이 더 작아지므

로 최소성에 모순. 따라서, b > a+ d1 이어야 한다. 이제

b > a+ dk (dk =(2n+ 1)k ¡ 1(2n+ 1)k

) (¤)

임을 수학적 귀납법으로 증명하겠다. k = 1 일 때는 보였고, k일 때 성립한다고 가정하자.

s = s1 + s2 ¸ 2n+ dk = (2n+ 1)dk+1 이므로

s2 ¡ s1 >s

2n+ 1¸ dk+1

만일 b¡ a · dk+1 이면 a와 b를 바꿔넣으면 js2 ¡ s1j 이 더 작아지므로 최소성에 모순. 따라서,

b > a + dk+1 이고, k + 1 일 때도 증명되었다. 즉, (¤)이 항상 성립함을 보였다. k ! 1 이면

dk ! 1 이므로 (¤)이 모든 k에 대해 성립하면

b ¸ a+ 1

이어야 하고, a; b 2 [1; 2] 임에서 a = 1, b = 2 일 수밖에 없다. 그럼, s2의 모든 항이 s1의 항인

1보다 크거나 같고 모두 b가 될 수 있고, 따라서 s2의 모든 항이 2이다. 마찬가지로 s1의 모든 항

이 1이다.즉, 카드는 n개의 1과 n개의 2로 구성되어야 하는데, 그 때 먼저 2를 s1과 s2에 고르게

나누어넣고 다음 1을 고르게 나누어넣으면 s2 ¡ s1 · 1 로 만들 수 있으므로 역시 s2 ¡ s1 = n

의 최소성에 모순이다(n = 1 일 때는 예외이지만, 그 때는 문제가 성립함이 자명하므로 상관없

다). 따라서, 귀류법의 가정이 틀린 것이고, 문제의 조건을 만족하는 s1, s2를 항상 찾을 수 있

다. ¤

Baltic 2001-15Baltic 2001-15

a0; a1; a2; : : : 는 i ¢ a2i ¸ (i + 1) ¢ ai¡1ai+1 (i = 1; 2; : : : ) 을 만족하는 양의 실수들의 수열이다. 또,

x, y는 양의 실수이고, bi = xai + yai¡1 (i = 1; 2; : : : ) 이라 하자. 모든 정수 i ¸ 2 에 대해 부등식

i ¢ b2i > (i+ 1) ¢ bi¡1bi+1 이 성립함을 보여라.

증명 (KAIST 수학과 03학번 윤태동)

보이고자 하는 식의 bj들을 모두 aj들로 대입하면 다음과 같다.

i(xai + yai¡1)2 > (i+ 1)(xai¡1 + yai¡2)(xai+1 + yai)

x, y는 양수이므로 이 부등식은 양변의 x2, y2, xy의 계수를 각각 비교하여 다음을 보이면 충분

하다.

ia2i ¸ (i+ 1)ai¡1ai+1ia2i¡1 > (i+ 1)ai¡2ai

2iaiai¡1 ¸ (i+ 1)(ai¡1ai + ai¡2ai+1)

Page 216: Baltic Way 팀수학경시대회 1990-2005 pdf 보기

216

첫 번째 식은 문제에 주어진 조건이므로 당연히 성립하고, 두 번째 식은 양변에 i¡1i 를 곱하면

역시 문제의 조건에서 금방 확인된다.마지막 식은 우변의 ai¡1항을 좌변으로 넘기면 다음과 같

고,

(i¡ 1)aiai¡1 ¸ (i+ 1)ai¡2ai+1

이것은 다음 두 식을 변변 곱하여 양변을 iaiai¡1로 나눠준 것과 같으므로 성립한다.

ia2i ¸ (i+ 1)ai¡1ai+1(i¡ 1)a2i¡1 ¸ iai¡2ai

따라서, 원하는 부등식이 증명되었다. ¤

Baltic 2001-16Baltic 2001-16

f는 양의 정수들에서 정의되고 실수값을 가지는 함수로, 다음 조건을 만족한다: 모든 n > 1 에 대해

f(n) = f

µn

p

¶¡ f(p)

인 n의 소인수 p가 존재한다. f(2001) = 1 이라 할 때, f(2002)의 값은 얼마인가?

풀이 임의의 소수 p에 대해 p의 소인수는 p 자신뿐이므로

f(p) = f(1)¡ f(p)

즉, f(p) = 12f(1)로일정한상수이다.그상수를 f(p) = c라하자.그럼 f(pq) = f(p)¡f(q) = 0,

다시 f(pqr) = f(pq)¡ f(r) = ¡c, 귀납적으로

f(p1 ¢ ¢ ¢ pk) = (2¡ k)c

가 된다(p, q, r, p1; : : : ; pk들은 소수). 2001 = 3 ¢ 23 ¢ 29 이므로 f(2001) = ¡c, 즉 c = ¡1.2002 = 2 ¢ 7 ¢ 11 ¢ 13 이므로 f(2002) = ¡2c = 2 ¢ ¢ ¢ 답 }

Baltic 2001-17Baltic 2001-17

n은 양의 정수이다. 집합 f1; 2; 3; : : : ; 2ng에서 다음의 성질을 갖는 2n¡1 + n 개 이상의 수들을 찾을

수 있음을 증명하여라: 임의의 서로 다른 두 수 x, y에 대해, x+ y 는 x ¢ y 의 약수가 아니다.

증명 (서울 경희고 1학년 이승후, 수정됨)

위 집합의 모든 홀수들(2n¡1개)과 2의 거듭제곱수들(n개)로 찾으면 된다.

(1) x, y가 둘 다 홀수일 때: x+ y 는 짝수이고 xy는 홀수이므로 x+ y j xy 일 수 없다.

(2) x, y가 둘 다 짝수일 때: 두 수를 2a, 2b (a < b) 라 하면 x + y = 2a(1 + 2b¡a) 에서

1 + 2b¡a는 1보다 큰 홀수가 되어 xy = 2a+b 의 약수가 될 수 없다.

Page 217: Baltic Way 팀수학경시대회 1990-2005 pdf 보기

2001년 Baltic Way 풀이 217

(3) x, y 중 홀수와 짝수가 하나씩일 때: x를 홀수, y를 2의 거듭제곱수라 하자. x + y는 홀수

이므로 y는 서로 소이다. 따라서, x + y j xy 이려면 x+ y j x 여야 하는데, x + y · x 일

수 없으므로 이것은 불가능하다.

따라서, 위에서 찾은 것이 문제의 조건을 만족하는 2n¡1 + n개의 수이다. ¤

Baltic 2001-18Baltic 2001-18

a는 홀수이다. n 6= m 인 모든 양의 정수 n, m에 대해, a2n

+ 22n

과 a2m

+ 22m

은 서로 소임을 증

명하여라.

증명 tm = a2m

+ 22m

로 정의하자. x2 ¡ y2 = (x¡ y)(x+ y) 의 공식에 의해 다음과 같이 식

을 쓸 수 있다.

tm = a2m ¡ 22m + 2 ¢ 22m

= (a2m¡1 ¡ 22m¡1)tm¡1 + 2 ¢ 22m

= (a2m¡2 ¡ 22m¡2)tm¡2tm¡1 + 2 ¢ 22m

= ¢ ¢ ¢= (a2

0 ¡ 220)t0t1 ¢ ¢ ¢ tm¡1 + 22m+1

따라서, 일반성을 잃지 않고 n < m 이라 할 때 tm과 tn이 공통소인수 p를 갖는다면, 위의 식

에 의해 p j 22m+1 이어야 한다. 그런데, tm은 홀수이므로 p = 2 일 수 없어서 모순. 그러므로,

tm과 tn은 서로 소이다. ¤

Baltic 2001-19Baltic 2001-19

360의 양의 약수의 개수와 같은 개수의 양의 약수를 갖는 가장 작은 양의 홀수는 무엇인가?

풀이 360 = 233251 이므로 양의 약수의 개수는 (3 + 1)(2 + 1)(1 + 1) = 24개. 24를 2 이상의

수들의 곱으로 분해하는 방법은

24 = 12 ¢ 2 = 8 ¢ 3 = 6 ¢ 4 = 6 ¢ 2 ¢ 2 = 4 ¢ 3 ¢ 2 = 3 ¢ 2 ¢ 2 ¢ 2

들이 전부인데, 각각의 경우의 최소의 홀수 31151, 3752, 3553, 355171, 335271, 325171111 들을

모두 비교하면

31151 > 3752 > 3553 > 355171 > 335271 > 325171111

로 가장 작은 것은 325171111 = 3465 ¢ ¢ ¢ 답 }

Page 218: Baltic Way 팀수학경시대회 1990-2005 pdf 보기

218

Baltic 2001-20Baltic 2001-20

정수 순서쌍 (a; b; c; d)로부터, 임의의 정수 n에 대해

(c; d; a; b); (b; a; d; c); (a+ nc; b+ nd; c; d); (a+ nb; b; c+ nd; d)

등의 순서쌍들을 바로 얻을 수 있다고 하자. 이 과정을 몇 번 반복하여 (1; 2; 3; 4)로부터 (3; 4; 5; 7)을

얻을 수 있겠는가?

풀이 문제에 주어진 변형에서 jad¡ bcj의 값이 언제나 불변임을 확인할 수 있다. (1; 2; 3; 4)와

(3; 4; 5; 7)에 대해 각각의 jad¡ bcj 의 값은 2와 1로 서로 다르므로 서로 변형하여 얻을 수 없는

순서쌍이다. }

주 (a; b; c; d)를 행렬

0@a b

c d

1A로 생각하면 문제의 변형은 elementary operation 들로서, 행렬

식의 값의 크기를 변화시키지 않는다. 이것이 큰 힌트가 될 수 있다.

Page 219: Baltic Way 팀수학경시대회 1990-2005 pdf 보기

2002년 Baltic Way 풀이 219

2002년 Baltic Way 풀이

Baltic 2002-1Baltic 2002-1

다음 연립방정식을 실수 범위에서 풀어라.

a3 + 3ab2 + 3ac2 ¡ 6abc = 1b3 + 3ba2 + 3bc2 ¡ 6abc = 1c3 + 3ca2 + 3cb2 ¡ 6abc = 1

풀이 (대전과학고 1학년 송지용)

세 식을 차례로 ①, ②, ③이라 하자. ①¡②에서

a3 ¡ b3 + 3ab(b¡ a) + 3c2(a¡ b) = 0

(a¡ b)f(a¡ b)2 + 3c2g = 0

따라서, 어느 항이 0이든 여기서 확실하게 a = b 라는 것을 알 수 있다. 마찬가지로 ②¡③에서

b = c 이다.

) a = b = c

①식의 b, c에 a를 대입하면, a3 + 3a3 + 3a3 ¡ 6a3 = 1, 즉 a3 = 1 이므로 a = 1. 따라서

a = b = c = 1 이다. }

Baltic 2002-2Baltic 2002-2

a, b, c, d는 다음을 만족하는 실수들이다.

a+ b+ c+ d = ¡2ab+ ac+ ad+ bc+ bd+ cd = 0

네 수 a, b, c, d 중 적어도 하나는 ¡1 이하임을 증명하여라.

증명 (부산중앙고 2학년 김규현)

a; b; c; d > ¡1 이라고 가정하면 b + c + d > ¡3 이므로 a < 1 이다. 같은 방법으로 ¡1 <

a; b; c; d < 1 이고, 그럼 0 · a2; b2; c2; d2 < 1 이므로

0 · a2 + b2 + c2 + d2 < 4

이다. 그런데

(a+ b+ c+ d)2 = a2 + b2 + c2 + d2 + 2(ab+ ac+ ad+ bc+ bd+ cd)

= a2 + b2 + c2 + d2 = 4

이므로 모순. 따라서 a, b, c, d 중 적어도 하나는 ¡1이하이다. ¤

Page 220: Baltic Way 팀수학경시대회 1990-2005 pdf 보기

220

별증 (O±cial Solution)

a; b; c; d > ¡1 이라 하고, A = a + 1, B = b + 1, C = c + 1, D = d + 1 로 치환하자. 그럼

A;B;C;D > 0 이고,

A+B +C +D = 2

그리고, ab = (A¡1)(B¡ 1) = AB¡ (A+B)+1 등으로부터 ab+ac+ad+ bc+ bd+ cd = 0 은

(AB +AC +AD +BC +BD + CD)¡ 3(A+B +C +D) + 6 = 0

AB +AC +AD +BC +BD +CD = 0

이 된다. 이것은 A;B;C;D > 0 임에 모순. 따라서 a, b, c, d 중 적어도 하나는 ¡1이하이다. ¤

Baltic 2002-3Baltic 2002-3

임의의 정수 m;n ¸ 0 에 대해 다음 관계식

am2+n2 = a2m + a2n

을 만족하는 실수 수열 a0 · a1 · a2 · ¢ ¢ ¢ 을 모두 찾아라.

풀이 (O±cial Solution)

a0 = a02+02 = 2a20. 따라서, a0 = 0 또는 1

2 .

(1) a0 =12 일 경우:

a1 = a12+02 = a21 +14 에서 (a1 ¡ 1

2)2 = 0. 따라서 a1 =

12.

a2 = a12+12 = 2a21 =

12.

ak =12 일 때 ak2 = ak2+02 =

14 +

14 =

12 .

즉, 귀납적으로 12 = a4 = a16 = ¢ ¢ ¢ . 그럼 a0 · a1 · a2 · ¢ ¢ ¢ 에 따라

a0 = a1 = a2 = ¢ ¢ ¢ = 1

2

(2) a0 = 0 일 경우:

a1 = a12+02 = a21. 따라서, a1 = 0 또는 1.

(2i) a1 = 0 일 경우:

a2 = a12+12 = 0.

ak = 0 일 때 ak2 = ak2+02 = 0.

그럼 (1)에서와 마찬가지로, 귀납적으로

a0 = a1 = a2 = ¢ ¢ ¢ = 0

(2ii) a1 = 1 일 경우:

a2 = a12+12 = 2.

Page 221: Baltic Way 팀수학경시대회 1990-2005 pdf 보기

2002년 Baltic Way 풀이 221

a4 = a22+02 = 4.

a5 = a22+12 = 5.

a52 = a32+42 = a23 + a24 으로부터 a3 = 3.

a8 = a22+22 = 8.

a9 = a32+02 = 9.

a10 = a32+12 = 10.

62 + 82 = 102 + 02

72 + 12 = 52 + 52

으로부터 a6 = 6, a7 = 7 을 얻는다. 이제

(2k + 1)2 + (k ¡ 2)2 = (2k ¡ 1)2 + (k + 2)2

(2k + 2)2 + (k ¡ 4)2 = (2k ¡ 2)2 + (k + 4)2

이 두 항등식에 의해 귀납적으로 항상 an = n 임이 확인된다.

위에 의해 구하는 수열은 an ´ 0, 12 , n 의 세 가지뿐이다. }

Baltic 2002-4Baltic 2002-4

n은 양의 정수이다. x1 + x2 + ¢ ¢ ¢+ xn = 1 인 임의의 음아닌 실수 x1; x2; : : : ; xn에 대해

nXi=1

xi(1¡ xi)2 ·

µ1¡ 1

n

¶2이 성립함을 증명하여라.

증명 (민족사관고 1학년 남율희)

f(x) = x(1¡ x)2 로 두면

f(x) = x3 ¡ 2x2 + x

f 0(x) = 3x2 ¡ 4x+ 1f 00(x) = 6x¡ 4

따라서, f(x)는 x 2 [0; 23 ] 에서 위로 볼록, x 2 [ 23 ; 1] 에서 아래로 볼록이다. x1 · x2 · ¢ ¢ ¢ · xn

이라 해도 일반성을 잃지 않는다.

(i) xn · 2

3일 때: 젠센 부등식에 의해

nXi=1

xi(1¡ xi)2 = f(x1) + f(x2) + ¢ ¢ ¢+ f(xn)

· nf³x1 + ¢ ¢ ¢+ xn

n

´= nf

µ1

n

¶=

µ1¡ 1

n

¶2(등호 : x1 = x2 = ¢ ¢ ¢ = xn =

1n 일 때)

Page 222: Baltic Way 팀수학경시대회 1990-2005 pdf 보기

222

(ii)2

3· xn · 1 일 때: (n ¸ 2 일 때만 생각, n = 1 일 때는 자명)

f 0(x) = (3x¡ 1)(x¡ 1) 임으로부터 f(x)는 x 2 [0; 13 ] 에서 증가, x 2 [ 13 ; 1] 에서 감소하므

로 (x1; xn) 대신 (x1 + xn ¡ 23 ;23 ) 를 대입하면 더 커진다.

nXi=1

f(xi) · f

µx1 + xn ¡ 2

3

¶+

n¡1Xi=2

f(xi) + f

µ2

3

¶그럼 이 식은 다시 (i)의 경우에 포함되므로 (1¡ 1

n)2 이하가 된다.

(i), (ii)에 의해 문제의 부등식이 항상 성립한다. ¤

별증 식을 모두 전개하여 다시 정리해보자.nXi=1

x3i ¡ 2nXi=1

x2i +

nXi=1

xi ¡ 1 + 2

n¡ 1

n2· 0

여기서nXi=1

xi = 1; 혹은

nXi=1

µxi ¡ 1

n

¶= 0

이므로 다음과 같이 된다(c는 임의의 실수).nXi=1

µx3i ¡ 1

n3

¶¡ 2

nXi=1

µx2i ¡ 1

n2

¶+ c

nXi=1

µxi ¡ 1

n

¶· 0

nXi=1

µxi ¡ 1

n

¶µx2i +

1¡ 2nn

xi + c0¶· 0

nXi=1

µxi ¡ 1

n

¶2µxi +

2¡ 2nn

¶· 0

n ¸ 2 일 때 xi +2n ¡ 2 · 0 이므로 이 식은 잘 성립한다. n = 1 일 때는 원래 문제의 식을 바로

검토하면 자명하다. ¤

Baltic 2002-5Baltic 2002-5

pa+

pb =p2 +

p3 을 만족하는 모든 양의 유리수해 (a; b)를 찾아라.

풀이 양변을 제곱하면

a+ b+ 2pab = 2 +

p3

2¡ a¡ b = k 라 하면,

2pab = k +

p3

양변을 제곱하면,

4ab = k2 + 3 + 2kp3

a, b는 유리수이므로 좌변은 유리수, k도 유리수. 따라서, 우변도 유리수여야 하므로 k = 0.

a+ b = 2; ab =3

4

따라서, a; b는 x2 ¡ 2x + 34 = 0 의 두 근. (x ¡ 1

2 )(x ¡ 32 ) = 0. 그러므로, (a; b) =

¡12 ;32

¢또는¡

32 ;12

¢. }

Page 223: Baltic Way 팀수학경시대회 1990-2005 pdf 보기

2002년 Baltic Way 풀이 223

Baltic 2002-6Baltic 2002-6

단위 정사각형들로 구성된 m £ n 직사각형 판(m;n ¸ 2)에서 다음과 같은 혼자서 하는 게임을 진

행한다. 우선, `차' 하나를 적당한 칸에 둔다. 매 회의 이동에서 이 `차'는 수직이나 수평 방향으로

임의의 칸을 전진한다. 단, 전에 움직여온 방향에서 시계방향으로 90±를 꺾은 방향으로 가야한다는

조건을 추가한다. (즉, 왼쪽으로 이동한 후에는 위쪽으로, 그 다음에는 오른쪽으로, ... 이런 식으로

움직여야 한다.) `차'가 모든 칸을 꼭 한 번씩 들러 원래의 칸으로 돌아올 수 있는 m과 n은 어떤 값

이어야 하는가? (단, `차'가 어떤 칸에 들렀다는 것은 그 칸 위를 스쳐 지나간 경우는 아니고 거기서

멈춘 경우만을 고려하기로 한다.)

풀이 (부산 데레사여고 1학년 방윤희)

m, n은 모두 짝수여야 한다. 가장자리 칸들에 주목한다.

왼쪽 가장자리 세로줄을 생각해 보자. `차'의 움직임에 따라 ①, ②, ③, ④의 번호를 각 칸에 붙

인다고 하자. 왼쪽 가장자리 세로줄 칸들에는 ① 또는 ④의 번호만이 붙여질 수 있다. 어떤 경

우에도 ①과 ④의 번호가 붙여진 판의 개수는 서로 같다. 번호가 붙여진 순서대로 쓰면 ④ 다음

에는 반드시 ①이, ① 다음에는 반드시 ④가 오며, 처음으로 붙여진 번호가 ①이라면(차의 첫칸

이 이 줄에 있는 경우) 마지막으로 붙여진 번호가 ④이고, 처음으로 붙여진 번호가 ④라면(차가

다른 줄에서 출발한 경우) 마지막으로 붙여진 번호는 반드시 ①이기 때문이다.

따라서 왼쪽 가장자리 세로줄의 칸수는 짝수이다. 같은 방법으로 맨위 가로줄의 칸수가 짝수임

을 역시 보일 수 있으며 따라서 m, n은 모두 짝수.

이제 m, n이 모두 짝수이면 다음과 같이 움직여 원래 칸으로 돌아올 수 있다.

칸에 쓰인 숫자는 차가 움직이는 순서이다. 두 가장자리줄부터 모든 칸을 한 번씩 들른다. 왼

쪽 맨위칸을 처음칸, 오른쪽 맨위칸을 2m번째 칸이 되도록 가운데 대칭축을 중심으로 마주보

① ②

① ②

③④

2m-2

2

1 2n

3

5

6

2m-1

4

7

2m+1

2n

2m

Page 224: Baltic Way 팀수학경시대회 1990-2005 pdf 보기

224

는 두 세로줄의 모든 칸을 순서대로 들르면 마지막 4mn번째에는 모든 칸을 다 들렀다가 처음

칸에 도착할 수 있다. }

Baltic 2002-7Baltic 2002-7

평면 위에 n개의 볼록사각형을 그려 평면을 몇 개의 영역으로 나눈다(한 영역은 무한 영역이다). 이

렇게 나뉘는 영역의 개수의 최대값을 구하여라.

풀이 (고양 백석중 2학년 김재윤, 수정됨)

문제에서 구하는 최대 영역수를 f(n)이라 하자. 우선 f(1) = 2 이다. 이미 n개의 볼록사각형이

그려져 있을 때 f(n)에서 f(n+ 1)로 볼록사각형을 하나 더 추가하면서 영역이 얼마나 늘어나

는지 생각해보자. 새로 그려지는 볼록사각형의 네 변 각각은 이미 그려진 n개의 볼록사각형과

각각 최대 2개의 점에서 만나므로 새로 그려지는 볼록사각형은 최대 4 ¢ 2 ¢ n = 8n개의 교점을

갖는다. 즉, 새로 그려지는 볼록사각형은 이 교점들에 의해 잘린 8n개의 선 조각으로 이루어지

고, 이 각각의 선 조각은 n개의 볼록사각형에 의해 나뉘어진 어떤 영역을 둘로 나누게 된다. 따

라서, 새로 그려지는 n + 1번째 볼록사각형에 의해 최대 8n개의 영역이 추가된다. 특히, 한 원

위에 4n개의 점을 찍고 n개씩 건너뛴 점들 이어 n개의 볼록사각형을 그리는 것을 생각하면, 정

확히 이만큼 추가되는 경우가 항상 있다는 것을 알 수 있다. 따라서,

f(n+ 1) = f(n) + 8n

이 되고, 이 점화식을 풀면

f(n) = 8((n¡ 1) + ¢ ¢ ¢+ 1) + f(1)

= 4n(n¡ 1) + 2= 4n2 ¡ 4n+ 2

이다. }

주 이 문제를 볼록k각형에 대한 문제로 확장할 수 있다. n + 1번째 볼록k각형을 그릴 때 새

로 최대 2kn개의 영역이 생기므로,

g(n) = g(n¡ 1) + 2k(n¡ 1)= 2k((n¡ 1) + ¢ ¢ ¢+ 1) + g(1)

= kn(n¡ 1) + 2 = kn2 ¡ kn+ 2

이 된다.

Page 225: Baltic Way 팀수학경시대회 1990-2005 pdf 보기

2002년 Baltic Way 풀이 225

Baltic 2002-8Baltic 2002-8

P는 어느 세 점도 한 직선 위에 있지 않은 평면 위의 n ¸ 3 개의 점들의 집합이다. P의 점들을 꼭지

점으로 하는

Ãn¡ 12

!개의 삼각형들을 골라 집합 T라 하는데, T의 각 삼각형은 T의 다른 어떤 삼

각형과도 공유하지 않는 변을 적어도 하나씩 가져야 한다. 이렇게 T를 고르는 방법의 수가 얼마나

될까?

풀이 (부산 건국고 1학년 신승현, 수정됨)

n = 3 일 때는 삼각형이 하나밖에 구성되지 않으므로 자명하게 1가지 방법뿐임을 알 수 있다.

n ¸ 4 일 때만 보자. T의 삼각형 각각에 대해 그 삼각형에만 있는 변을 하나씩 택해(둘 이상 있

어도 하나만 택한다) `독립변'이라고 부르자. P의 두 점을 잇는 선분은 모두¡n2

¢개이고 독립변

은¡n¡12

¢개이므로 두 개 이상의 삼각형이 공유해도 되는 변은

Ãn

2

!¡Ãn¡ 12

!= n¡ 1(개)

뿐이다. 이들을 `공유변'이라고 부르자. n¡ 1개의 공유변들이 T의 각 삼각형에서 독립변을 제

외하고 남은 2¡n¡12

¢= (n¡ 1)(n¡ 2)개의 변을 중복하여 구성해야 하므로, 평균적으로 한 공유

변은 n¡ 2개의 삼각형에 의해 공유된다. 그런데, P의 임의의 한 선분은 그 선분의 양끝점을 제

외한 나머지 n¡ 2개의 점을 고를 때마다 삼각형이 구성되기 때문에 최대 n¡ 2개의 삼각형에

속할 수 있고, 따라서 공유변은 각각 정확히 n¡ 2개의 삼각형에 의해 공유된다. 지금까지 알게

된 사실을 정리하면 다음과 같다.

(1) P의 임의의 변은 독립변 아니면 공유변이다.

(2) 공유변은 n¡ 1개, 독립변은¡n¡12

¢개이다.

(3) 공유변을 갖는 삼각형은 모두 T의 삼각형이다.

(4) T의 임의의 삼각형은 2개의 공유변과 1개의 독립변을 갖는다.

T의 삼각형 ABC를 하나 택하고, AB와 AC는 공유변, BC는 독립변이라고 하자. 그리고, 또

하나의 점 X를 생각하자. 만일 AX가 독립변이라면 ABX;ACX 2 T 임에서 BX, CX는 공유

변이다. 그럼 BCX 2 T 이고, 독립변 BC가 두 삼각형 ABC와 BCX에 의해 공유되므로 모순.

따라서, AX는 공유변임을 알 수 있다. X는 임의의 점이었으므로, A를 포함하는 n¡ 1개의 변

이 모두 공유변이고, 나머지는 모두 독립변이다. 따라서, T는 P의 한 점 A를 포함하는 모든 삼

각형들만 모은 집합이다. 또한, 이런 집합 T는¡n¡12

¢개(A를 제외한 다른 두 점을 택하는 경우

의 수)의 삼각형을 갖고 T의 임의의 삼각형 ABC는 독립변 BC를 유일하게 가지므로 문제의

조건을 잘 만족한다. P에서 한 점 A를 골라낼 때마다 서로 다른 T가 구성되므로, T를 고르는

방법은 모두 n가지가 된다(n ¸ 4). }

Page 226: Baltic Way 팀수학경시대회 1990-2005 pdf 보기

226

Baltic 2002-9Baltic 2002-9

두 명의마술사가다음과 같은 마술을 보인다.마술사 A가 방에서나가고,마술사 B가 1; 2; : : : ; 100의

번호가 각각 적힌 100장의 카드를 들고 세 명의 관객에게 차례로 각각 한 장씩 고르게 한다. 마술사

B는 관객들이 고른 카드를 알고 있고, 자기가 한 장을 더 골라 4장의 카드를 만든다. 이 4장의 카드

를 관객들이 섞은 후, 마술사 A를 불러 그에게 준다. 마술사 A는 이 4장의 카드를 살펴본 후, 첫 번

째 관객이 고른 카드, 두 번째 관객이 고른 카드, 세 번째 관객이 고른 카드를 모두 알아맞춘다. 이

런 마술이 가능함을 증명하여라.

증명 (부산 데레사여고 1학년 방윤희, 수정됨)

첫 번째, 두 번째, 세 번째 관객이 고른카드와 B가 고른카드를 각각 x, y, z, b라 하자. B는 A와

약속한 대로 p, q를 계산하여(0 · p < 6, 0 · q < 4)

x+ y + z + b ´ 4p+ q (mod 24)

가 되도록 b를 선택한다. 이 약속은 다음과 같이 이루어진다:

1부터 96까지를 길이가 24인 4개의 구간

Iq = (24q; 24(q + 1)] (q = 0; 1; 2; 3)

으로 나눌 수 있고, Iq 중에는 x; y; z 중 어느 것도 포함하지 않는 것이 있다. 그 q를 택하고 그

구간에서 b를 뽑는다. x; y; z의 크기 순서는 x < y < z, x < z < y, : : : ; z < y < x 등 6가지가

가능하다. 이들 각각에 0부터 5까지의 고유번호를 약속하고 그것을 p로 택한다.

마술사 A는 q = (x+ y + z + b) mod 4 를 계산하여 구간 Iq에 속한 유일한 카드 b를 찾아낼 수

있고, 다시 p = (x+ y+ z+ b¡ q)=4 mod 6 을 계산하여 x, y, z의 뽑힌 순서를 알아낼 수 있다.

¤

Baltic 2002-10Baltic 2002-10

N은 양의 정수이다. 두 사람이 다음과 같은 게임을 한다. 첫 번째 사람이 먼저 25 이하의 양의 정수

들|반드시 서로 다를 필요는 없는|의 목록을 적는데 그 합은 최소 200 이상이다. 두 번째 사람이

여기서 몇 개의 수를 고르는데, 그 합 S가 200¡N · S · 200 +N 의 범위에 있도록 하면 이긴다.

두 번째 사람이 최선을 다하면 항상 이길 수 있는 N의 최소값은 얼마인가?

풀이 (O±cial Solution)

N · 10이면 두 번째 사람이 어떻게 해도 이길 수 없는 경우가 있다.첫 번째 사람이 5개의 24와

4개의 23을 목록에 적으면 그 합은 212이고, 이 중 어느 하나의 수를 빼더라도 189 이하가 되므

로 190 · S · 210 이 되도록 할 수 없다.

N = 11 이면 두 번째 사람이 항상 이길 수 있음을 보이자. 189 · S · 211 이 되도록 하면 된다.

목록의 모든 수의 합에서 가장 작은 수부터 차례로 삭제하여 처음으로 211 이하가 될 때 멈추

자. 만일 마지막으로 삭제된 수가 23 이하이면 212¡ 23 = 189 이므로 저 구간에 걸리게 된다.

Page 227: Baltic Way 팀수학경시대회 1990-2005 pdf 보기

2002년 Baltic Way 풀이 227

따라서, 이제 211 이하의 합에 24와 25들만 남은 경우만 살피면 된다. 처음으로 211 이하가 되

었을 때이므로 남은 수의 합은 212¡ 25 = 187 · S · 211 의 범위에 있고, 그럼 7개로는 모자라

고(175 이하) 9개로는 넘치므로(216 이상) 꼭 8개의 f24; 25g가 필요하다. 그럼 192 · S · 200이 되어 성립한다.

따라서, 구하는 N의 최소값은 11이다. }

Baltic 2002-11Baltic 2002-11

n은 양의 정수이다. 평면 위에 n개의 점이 있는데, 어느 세 점도 한 직선 위에 있지 않고 각 두 점

사이의 거리가 모두 다르다고 한다. 각 점에서부터 가장 가까운 두 점을 찾아 그 두 점까지 선분을

그어 연결한다. 이 선분들은 중복될 수도 있고, 서로 엇갈릴 수도 있으며, 어느 한 쪽 점의 입장에서

만 가장 가까운 두 점에 해당되고 반대쪽 점의 입장에서는 해당되지 않더라도 선분으로 연결해야

한다. 11개보다 더 많은 선분이 연결된 점이 존재하지 않음을 증명하여라.

증명 (창원남고 2학년 조지환)

12개 이상의 선분이 연결된 점 O가 존재한다고 가정하자.

12개의 선분에 차례로 번호를 붙여 짝수 번째의 6개의 선분만을 생각하면 그 중 사잇각이 60±

이하인 것이 있다. 따라서,

\AOB;\AOC;\BOC · 60±

인 세 점 A, B, C가 있다. OA;OB;OC 중 OA가 가장 길다고 하자. 그럼 A는 O에서 가장 가

까운 두 점에 해당되지 않는다. 따라서, OA가 이어지려면, O는 A에서 가장 가까운 두 점에 해

당되어야 한다. 그런데(a = OA, b = OB)

AB2 = a2 + b2 ¡ 2ab cos\AOB

< a2 + b2 ¡ 2b2 cos 60± = a2

따라서, AB < a. 마찬가지로 AC < a. 즉, A에서 O보다는 B, C가 더 가까우므로 OA는 생길

수 없다. 이것은 모순. 그러므로, 11개보다 더 많은 선분이 연결된 점은 존재하지 않는다. ¤

O

A

B

C 30

60

Page 228: Baltic Way 팀수학경시대회 1990-2005 pdf 보기

228

Baltic 2002-12Baltic 2002-12

평면 위에 서로 다른 네 점이 주어져 있다. 이 중 어느 점을 X로 택해도

XY = XZ +XW

가 성립하도록 나머지 점들을 Y , Z, W로 이름붙일 수 있다고 한다. 이 네 점은 한 직선 위에 있음

을 증명하여라.

증명 (서울 봉원중 3학년 안충현)

네 점을 A, B, C, D라고 명명하자. 이 중 AC가 가장 길다고 하자. 그럼

AC = AB +AD; CA = CB + CD

가 성립한다. 이 두 식을 변변 합하면

2AC = (AB +BC) + (AD +DC) ¸ AC +AC

와 같은 삼각부등식을 얻을 수 있다. 등호가 성립하는 경우이므로, B와 D는 직선 AC 위에 있

다. 따라서, A, B, C, D는 한 직선 AC 위에 있다. ¤

Baltic 2002-13Baltic 2002-13

4ABC는 \BAC > \BCA 를 만족하는 삼각형이고, D는 AB = BD 를 만족하는 변 AC 위의 점

이라 하자. 또, F는 4ABC의 외접원 위에 있는 점으로, 직선 FD가 변 BC에 수직이고 두 점 F ,

B는 직선 AC에 대해 반대편 영역에 놓인다고 하자. 직선 FB가 변 AC에 수직임을 증명하여라.

증명 (고양 백신고 1학년 신동주, 조금 수정됨)

B에서 AC로의 수선이 외접원과 만나는 점을 F 0이라 하자. 그리고, F 0D가 BC와 만나는 점을

G, 외접원과 만나는 점을 E라 하자.

A;B;C; F 0은 한 원 위의 네 점이고 4BF 0A ´ 4BF 0D 이므로

\CEF 0 = \CAF 0 = \F 0AD = \CDE

\BCA = \BF 0A = \BF 0D = \BCE

따라서, 4CDE는 CD = CE 인 이등변삼각형이고 BC는 꼭지각의 이등분선이므로 \BGD =

90±. F의 유일성에 의해 F = F 0 이므로, FB와 AC는 수직이다. ¤

B

A

F'

D

G E

C

Page 229: Baltic Way 팀수학경시대회 1990-2005 pdf 보기

2002년 Baltic Way 풀이 229

Baltic 2002-14Baltic 2002-14

4ABC에서 세 변 AC, AB, BC 위에 각각 점 L, M , N이 있는데, BL은 \ABC의 이등분선이고

세 선분 AN , BL, CM이 한 점에서 만난다. \ALB = \MNB 이면 \LNM = 90± 임을 증명하여

라.

증명 (대전과학고 1학년 김진현, 조금 수정됨)

MN k AK 가 되도록 BC 위에 점 K를 잡자.

\AKB = \MNB = \ALB 이므로 ALKB는 원에 내접하는 사각형이다. \LBK = \LBA

이므로 AL = LK. AL = LK = LT 가 되도록 AC 위에 점 T를 잡자. 그럼 L은 4AKT의 외

심이고 \AKT = 90±. 체바의 정리에 의해

MB

AM¢ NC

BN¢ LACL

= 1

이고, MN k AK 로부터 MBAM

= NBKN 이므로 NC

KN¢ LACL

= 1. 따라서,

CN : NK = CL : LT

가되고,그럼 LN k TK 이다.따라서,각을평행이동한것을생각하면 \LNM = \TKA = 90±.

¤

Baltic 2002-15Baltic 2002-15

한 정육면체의 표면에 거미 한 마리와 파리 한 마리가 앉아있다. 파리는 거미가 이 정육면체의 표면

을 따라 자신에게 다가올 수 있는 최단거리를 가장 멀게 하고 싶다. 정육면체의 중심에 대해 대칭이

되는 반대편에 앉는 게 언제나 최선이 되는가?

풀이 (대구 남산고 1학년 유상훈)

거미의 위치를 P , 파리의 위치를 P 0이라 하자. 일단 아래 그림과 같이 한 면을 중심으로 잡고

점 A, B, C, D를 잡아서 중점 대칭을 시키면 각각 점 A0, B0, C 0, D0이 된다. (길이를 비교하기

Page 230: Baltic Way 팀수학경시대회 1990-2005 pdf 보기

230

위해 사각형A0B0C 0D0를 4개로 나타냄)

AB 위의 점 P를 AP = 2t, PB = 3t 라 두고 P의 중점대칭점을 P 0이라 두면 A0P 0 = 2t,

P 0B0 = 3t, 그리고 한 변의 길이는 5t가 된다. 이 때, PP 0의 길이는 왼쪽(혹은 오른쪽)의 접면

으로 구하면p(10t)2 + (5t)2 =

p125t2. 아래쪽(혹은 위쪽)으로는

p(1t)2 + (10t)2 =

p101t2.

그러므로 PP 0의 최소 거리는p101t2이 된다. 그런데 P 0을 A0쪽으로 t만큼 옮긴 점을 P 00이라

하고 PP 00의 최단거리를 비슷하게 구하면, 왼쪽으로는p(11t)2 + (5t)2 =

p146t2, 오른쪽은p

(9t)2 + (5t)2 =p106t2, 상하로는

p(10t)2 + (2t)2 =

p104t2이 된다. 그러므로 PP 00의 최단

거리는p104t2이 되어 PP 0의 최단거리보다 더 멀어 진다.

그러므로 정육면체의 중심에 대해 대칭이 되는 반대편에 앉는 게 항상 최선이 되지는 않는다.

}

Baltic 2002-16Baltic 2002-16

다음을 만족하는 음이 아닌 정수 m을 모두 찾아라: am = (22m+1)2 + 1 을 나누는 서로 다른 소수

가 두 개 이하이다.

풀이 (O±cial Solution)

a0 = 4 + 1 = 5, a1 = 65 = 5 ¢ 13, a2 = 1025 = 52 £ 41 로 m = 0; 1; 2 일 때는 성립한다.

m ¸ 3 에서 성립하는 경우가 있다고 가정하자. 우선 5 j am = (42m+1 + 1) 임을 쉽게 알 수 있

다.

am = (22m+1)2 + 1

= (22m+1 + 2m+1 + 1)(22m+1 ¡ 2m+1 + 1)

로 인수분해되는데, 두 인수는 모두 홀수이고 차가 2의 거듭제곱이므로 서로 소이다. 두 인수는

모두 1보다 크므로, 소인수가 2개뿐이려면 각각 어떤 소수의 거듭제곱꼴일 수밖에 없다. 즉, 둘

중 하나는 5의 거듭제곱이다.

2m+1(2m § 1) = 5t ¡ 1 = (5¡ 1)(5t¡1 + ¢ ¢ ¢ 5 + 1)

B' A'

P'

C' D'

A B

C D

P D'

D'

D'C' C'

C'

B'B'

B'

A' A'

A'

P'

P'P'

P'

Page 231: Baltic Way 팀수학경시대회 1990-2005 pdf 보기

2002년 Baltic Way 풀이 231

m ¸ 3 이므로 좌변은 23의 배수이고, 따라서 (5t¡1 + ¢ ¢ ¢ 5+ 1)은 짝수가 되어야 한다. 그럼 t도

짝수이다.

2m+1(2m § 1) = 5t ¡ 1 = (5 t2 + 1)(5 t2 ¡ 1)

5t2 + 1 ´ 2 (mod 4) 이므로 2 k 5 t2 + 1, 따라서 2m k 5 t2 ¡ 1 여야 한다. 적당한 홀수 k에 대해

5t2 ¡ 1 = 2m ¢ k 라 두면 5

t2 + 1 = 2m ¢ k + 2 는 2(2m § 1)의 약수이므로

2m¡1 ¢ k + 1 j 2m § 1

k > 2 이면 약수가 더 크므로 여기서 k = 1 일 수밖에 없고, 그럼 2m¡1 + 1 j 2m § 1. 그러나

2m¡1+1 < 2m§ 1 < 2(2m¡1+1) 이어서 이것은 모순이다. 즉, m ¸ 3 에서는 해가 없다. 결국,

조건을 만족하는 m = 0; 1; 2. }

Baltic 2002-17Baltic 2002-17

수열

Ã2002

2002

!;

Ã2003

2002

!;

Ã2004

2002

!; : : : 의 각 항을 2002로 나눈 나머지는 주기적으로 반복됨을 보여

라.

증명 (광주 대광여고 1학년 김유진, 수정됨)

an =

Ã2002 + n

2002

!=(n+ 1)(n+ 2) ¢ ¢ ¢ (n+ 2002)

2002!

이라 하고, an mod 2002가 c = 2002! 2002를 주기로 반복됨을 보이자.

2002! an+c = (n+ c+ 1)(n+ c+ 2) ¢ ¢ ¢ (n+ c+ 2002)

´ (n+ 1)(n+ 2) ¢ ¢ ¢ (n+ 2002) (mod c)

´ 2002! an (mod c)

이것은 2002! 2002 j 2002!(an+c ¡ an) 와 같으므로, 2002 j an+c ¡ an, 즉

an+c ´ an (mod 2002)

가 성립한다. ¤

Baltic 2002-18Baltic 2002-18

다음을 만족하는 정수 n > 1 을 모두 찾아라: n6 ¡ 1 의 임의의 소인수는 (n3 ¡ 1)(n2 ¡ 1)의 약수

이다.

Page 232: Baltic Way 팀수학경시대회 1990-2005 pdf 보기

232

풀이 (전남과학고 2학년 정도옥)

n6 ¡ 1 = (n+ 1)(n2 ¡ n+ 1)(n3 ¡ 1)(n3 ¡ 1)(n2 ¡ 1) = (n3 ¡ 1)(n+ 1)(n¡ 1)

이므로 n2 ¡ n + 1 의 소인수 q에 대해서만 확인하면 된다. 이것은 n ¡ 1 과 서로 소이므로

n2 + n+ 1 혹은 n+ 1 의 소인수가 되어야 한다. 또, n2 ¡ n+ 1 은 홀수이므로 q도 홀수이다.

(1) q j n2 + n+ 1 이라고 하면 q j (n2 + n+ 1)¡ (n2 ¡ n+ 1) = 2n 을 만족해야 한다. q는 홀

수이므로 q j n, 그럼 q j 1 이 되어 모순이 생긴다.

(2) q j n + 1 이라면 q j (n2 ¡ n + 1) ¡ (n + 1)(n ¡ 2) = 3 이 성립해야 한다. 즉, q = 3 이

고 n2 ¡ n + 1 의 소인수는 3뿐이어야 한다. 그런데 n = 3k + 2꼴이므로, n2 ¡ n + 1 =

9k2 + 9k + 3 = 3(3k2 + 3k + 1) 로 3 이외의 또 다른 소인수를 가지지 않으려면 k = 0.

n = 2 가 유일하게 문제의 조건을 만족한다. }

Baltic 2002-19Baltic 2002-19

n은 양의 정수이다. 방정식

x+ y +1

x+1

y= 3n

은 양의 유리해를 가지지 않음을 증명하여라.

증명 (창원남고 2학년 조지환)

위 방정식이 양의 유리해 x = qp , y =

sr (gcd(q; p) = gcd(r; s) = 1) 를 가진다고 가정하자.

q

p+

p

q+

s

r+

r

s= 3n

q2rs+ p2rs+ s2pq + r2pq = 3n ¢ pqrsrs(q2 + p2) + pq(s2 + r2) = 3npqrs

(q2 + p2) +pq

rs(s2 + r2) = 3npq

따라서, rs j pq(s2 + r2). 여기서 gcd(sr; s2 + r2) = 1 이므로 rs j pq. 대칭적으로 pq j rs 도 성

립. ) pq = rs. 따라서

p2 + q2 + r2 + s2 = 3npq

그런데 아래의 (¤)에 의해 p2+q2+r2+s2은 3의 배수가 될 수 없다.따라서, x+y+1

x+1

y= 3n

은 양의 유리해를 가지지 않는다.

(¤) p2 + q2 + r2 + s2 이 3의 배수가 될 수 없는 이유:

p와 q는 서로 소이므로 둘 중에 하나만 3의 배수이거나 둘다 3의 배수가 아니다. 또 pq = rs

이므로 p, q, r, s 중 정확히 2개만 3의 배수이거나 넷 모두 3의 배수가 아니다. 전자의 경우

p2 + q2 + r2 + s2 이 mod 3 으로 2이고, 후자의 경우는 1이다. 따라서, p2 + q2 + r2 + s2 은 어

떤 경우에도 3의 배수가 될 수 없다. ¤

Page 233: Baltic Way 팀수학경시대회 1990-2005 pdf 보기

2002년 Baltic Way 풀이 233

Baltic 2002-20Baltic 2002-20

모든 항이 ab꼴(a와 b는 양의 정수, b ¸ 2)이고 공차가 0이 아닌 무한등차수열이 존재하는가?

풀이1 (O±cial Solution 1)

임의의 무한등차수열을 an = a+ nd (d > 0) 이라고 하자.

an ´ p (mod p2)

인 n과 소수 p를 찾아보자. d보다 큰 소수 p를 하나 택하면 d는 p와 서로 소이므로 p2과도 서로

소이고, 따라서 dd¡1 ´ 1 (mod p2) 인 (잉여역수) d¡1이 존재한다. n0 ´ (p¡ a)d¡1 (mod p2)

으로 잡으면 an0 ´ p (mod p2) 이 되므로 an0은 거듭제곱수가 아니다. }

풀이2 (O±cial Solution 2)

n 이하의 제곱수는pn개 이하이다. n 이하의 세제곱수는 3

pn개 이하이다. 2m > n 이면 m번 이

상의 거듭제곱수(1은 제외)는 n 이하의 수 중에 없으므로, n 이하의 거듭제곱수의 개수를 Xn이

라 하면

Xn ·pn+ 3

pn+ ¢ ¢ ¢+ blog2 nc

pn

· pn+pn+ ¢ ¢ ¢+pn

< (log2 n)pn

한편, 임의의 무한등차수열 ak = a+ kd (d > 0) 에서 n 이하의 항의 개수를 Yn이라 하면

Yn =jn¡ a

d

k¼ cn

따라서, n!1 이면 Xn ¿ Yn 이므로, 무한등차수열의 항이 모두 거듭제곱수일 수는 없다. }

풀이3 (O±cial Solution 3)

임의의 무한등차수열 an = a+ (n¡ 1)d (d > 0) 에서 모든 항의 역수의 합을 구하면

S =1

a+

1

a+ d+

1

a+ 2d+ ¢ ¢ ¢

¸ 1

M+

1

M +M+

1

M + 2M+ ¢ ¢ ¢

=1

M

µ1 +

1

2+1

3+ ¢ ¢ ¢

¶=1

단, M ¸ a; d. 그런데, 모든 거듭제곱수들의 역수의 합을 T라 하면

T · 1 +1Xn=2

µ1

n2+1

n3+1

n4+ ¢ ¢ ¢

= 1 +1Xn=2

1

n2¢ 1

1¡ 1n

= 1 +1Xn=2

1

n(n¡ 1)

= 1 +1Xn=2

µ1

n¡ 1 ¡1

n

¶= 2

즉, S > T 이므로, 모든 항이 거듭제곱수인 무한등차수열은 없다. }

Page 234: Baltic Way 팀수학경시대회 1990-2005 pdf 보기

234

2003년 Baltic Way 풀이

Baltic 2003-1Baltic 2003-1

Q+를 양의 유리수 전체의 집합이라고 하자. x 2 Q+에 속하는 모든 x에 대해서 다음을 만족하는 함

수 f : Q+ ! Q+ 를 모두 구하여라.

(1) : f( 1x ) = f(x)

(2) : (1 + 1x)f(x) = f(x+ 1)

풀이 (대전과학고 1학년 김진현, 수정됨)

g(x) =f(x)

f(1)이라 하면 g(1) = 1 이고 g도 f처럼 (1), (2)를 만족한다. (2)에서

g(x)

x=

g(x+ 1)

x+ 1(¤)

이므로 임의의 자연수 n에 대해g(n)

n은 상수이고, 그럼 그 값은

g(1)

1= 1, 즉 g(n) = n 이다.

다시 (1)에서 임의의 자연수 n에 대해 g( 1n) = n 임을 바로 알 수 있다.

g

µb

a

¶=

ab

gcd(a; b)2즉 g

µq

p

¶= pq (gcd(p; q) = 1)

임을 보이자. p + q 에 대한 수학적 귀납법으로 보이자. p + q = 2 일 때는 g(1) = 1 임을 말하

므로 자명. p+ q > 2 일 때는 p 6= q 이고, (1)에 의해 일반성을 잃지 않고 p < q 라 할 수 있다.

p+ q보다 작을 때 항상 성립한다고 가정하고, (2)에서 x+ 1 = qp 를 대입하면,

g

µq

p

¶=

µ1 +

p

q ¡ p

¶g

µq ¡ p

p

¶=

q

q ¡ p¢ (q ¡ p)p = pq

로 p+ q 일 때도 성립함을 알 수 있다. 따라서 증명되었고,

f

µb

a

¶=

cab

gcd(a; b)2

이다(c는 임의의 양의 유리상수). }

Baltic 2003-2Baltic 2003-2

x3 + px+ q = 0 의 모든 실수해가 부등식 4qx · p2 을 만족함을 보여라.

증명1 (광주 우산중 2학년 김세영)

x3 + px+ q = 0 의 임의의 실수해를 a라 하자. a3 + pa+ q = 0 이므로, a는 이차방정식

ax2 + px+ q = 0

의 실수해이기도 하다. 이 이차방정식이 실수해를 가지므로, 판별식 p2 ¡ 4aq ¸ 0, 즉 4aq · p2

이 성립한다. 위에서 a는 임의로 택한 실수해이므로, 4qx · p2 이다. ¤

Page 235: Baltic Way 팀수학경시대회 1990-2005 pdf 보기

2003년 Baltic Way 풀이 235

증명2 (대전 어은중 1학년 김영식)

¡q = x3 + px 를 우리가 확인하고자 하는 식 p2 ¡ 4qx ¸ 0 에 대입하자.

p2 + 4x(x3 + px) ¸ 04x4 + 4px+ p2 ¸ 0

(2x2 + p)2 ¸ 0

으로, x, p는 실수이므로 항상 성립하는 식임이 확인되었다. ¤

Baltic 2003-3Baltic 2003-3

x, y, z는 xyz = 1 을 만족하는 양의 실수이다. 다음이 성립함을 보여라.

(1 + x)(1 + y)(1 + z) ¸ 2µ1 + 3

ry

x+ 3

rz

y+ 3

rx

z

증명 (대원외국어고 1학년 최일규)

좌변을 전개하고 xyz = 1 이므로 양변에서 2를 빼자.

x+ y + z + xy + yz + zx ¸ 2µ3

ry

x+ 3

rz

y+ 3

rx

z

¶x = a3, y = b3, z = c3 으로 치환하자. a3b3c3 = 1 이므로 abc = 1 이고, 위 식은 다음과 같이

된다.

a3b3 + b3c3 + c3a3 + a3 + b3 + c3 ¸ 2µb

a+

c

b+

a

c

¶= 2(b2c+ c2a+ a2b)

산술-기하 평균 부등식에 의해

a3b3 + b3c3 + b3c3

3¸ ab3c2 = b2c

b3c3 + c3a3 + c3a3

3¸ bc3a2 = c2a

c3a3 + a3b3 + a3b3

3¸ ca3b2 = a2b

b3 + b3 + c3

3¸ b2c

c3 + c3 + a3

3¸ c2a

a3 + a3 + b3

3¸ a2b

이 6개의 식을 모두 변변 더하면 문제의 식을 얻는다. 등호는 a3 = b3 = c3, 즉 x = y = z = 1

일 때 성립. ¤

Baltic 2003-4Baltic 2003-4

a, b, c를 양의 실수라 하자. 다음이 성립함을 보여라.

2a

a2 + bc+

2b

b2 + ca+

2c

c2 + ab· a

bc+

b

ca+

c

ab

Page 236: Baltic Way 팀수학경시대회 1990-2005 pdf 보기

236

증명 (서산 서령고 1학년 김영제)

양변에 abc(> 0)를 곱하여 생각하자.

2a2bc

a2 + bc+2ab2c

b2 + ca+2abc2

c2 + ab· a2 + b2 + c2

좌변에 조화-산술평균 부등식을 사용하면

2a2bc

a2 + bc+2ab2c

b2 + ca+2abc2

c2 + ab· a2 + bc

2+

b2 + ca

2+

c2 + ab

2

a2 + b2 + c2 ¸ ab+ bc+ ca 는 잘 알려진 부등식이므로 이 우변이 a2 + b2 + c2 보다 크지 않다

는 것은 바로 나온다. 따라서 문제의 부등식은 성립하고, 등호는 a = b = c 일 때 성립한다. ¤

Baltic 2003-5Baltic 2003-5

수열 (an)은 다음과 같이 정의된다: a1 =p2, a2 = 2, an+1 = ana

2n¡1 (n ¸ 2). 1보다 작지 않은 모

든 n에 대해서 다음이 성립함을 보여라.

(1 + a1)(1 + a2) ¢ ¢ ¢ (1 + an) · (2 +p2 )a1a2 ¢ ¢ ¢ an

증명 (서산 서령고 1학년 김영제)

귀납적으로, an =p22n¡1

이 된다(수열의 점화식에 대입해보면 금방 확인됨).p2의 지수부를

이진법으로 전개하여 생각하면(예를 들어 x1011(2) = x1000(2) + x10(2) +x1(2)) 문제의 좌변은 다

음과 같이 전개됨을 알 수 있다.

(좌변) = (1 + a1)(1 + a2) ¢ ¢ ¢ (1 + an)

= 1 +p2 +

p22+p23+ ¢ ¢ ¢+p22

n¡1

=

p22n ¡ 1p2¡ 1 = (

p2 + 1)(

p22n ¡ 1)

그리고 우변은 다음과 같다.

(우변) = (2 +p2 )a1a2 ¢ ¢ ¢ an

= (2 +p2 )p21+2+4+¢¢¢+2n¡1

= (2 +p2 )p22n¡1

= (p2 + 1)

p22n

따라서, 우변이 항상p2 + 1 만큼 더 크다는 것을 알 수 있다. ¤

Baltic 2003-6Baltic 2003-6

n ¸ 2 과 d ¸ 1 는 d j n 을 만족하는 정수들이고, x1; x2; : : : ; xn 은 x1 + x2 + ¢ ¢ ¢+ xn = 0 을 만족

하는 실수들이다. xi1 + xi2 + ¢ ¢ ¢+ xid ¸ 0 을 만족하는 1 · i1 < i2 < ¢ ¢ ¢ < id · n 인 d개의 첨자

를 고르는 방법이 최소한¡n¡1d¡1¢가지가 있음을 보여라.

Page 237: Baltic Way 팀수학경시대회 1990-2005 pdf 보기

2003년 Baltic Way 풀이 237

증명 n = de 라고 하자. n개의 수들의 집합 X = fx1; : : : ; xng 에서 d개의 수를 골라 만든 부

분집합들 A1; A2; : : : ; Am을 생각하자(m =¡nd

¢). 집합 X를 서로 소인 것 e개의 Ai들로 분할하

는 방법의 수를 w라 하고, 그 방법들의 집합을 W라 하자. W에는 각각의 Ai들이 고르게 포함

되어 있으므로, 각각의 Ai들이 나타나는 횟수는 똑같이 k라고 할 수 있다. 한편으로는 W에는

w개의 원소가 있고 각각의 원소는 e개의 Ai를 포함하며, 또 한편으로는 W에는 m개의 Ai들이

각각 k번씩 나타나므로 다음의 등식이 성립한다.

we = mk

W의 방법 각각에는 (x1; : : : ; xn의 총합이 0이므로) 원소의 합이 0 이상인 Ai가 최소한 하나씩

은 나타난다. 따라서, W에는 원소의 합이 0 이상인 Ai가 모두 최소한 w번은 나타난다. 그중

최대 k개씩만이 같은 Ai가 중복된 것일 수 있으므로, W에는 원소의 합이 0 이상인 서로 다른

Ai들이 최소한

w

k=

m

e=

¡nd

¢nd

=

Ãn¡ 1d¡ 1

!개가 있다. 문제에서의 d개의 첨자를 고르는 방법이란 Ai를 택하는 방법과 같은 것이므로 이로

써 증명이 되었다. ¤

Baltic 2003-7Baltic 2003-7

X는 다음 성질을 갖는 f1; 2; : : : ; 10000g의 부분집합이다: `a; b 2 X 이고 a 6= b 이면 a ¢ b 62 X 이 된

다.' X의 원소의 개수는 최대 몇 개인가?

풀이 다음의 X가 원소의 개수가 최대인 것임을 보이자.

X0 = f100; 101; : : : ; 10000g

a; b 2 X0 이고 a 6= b 이면 a ¢ b > 10000 이므로 문제의 조건을 만족한다. 따라서, X는 9901개

의 원소를 가질 수 있다.

그보다 더 많은 원소를 갖는다면 X에는 100보다 작은 원소가 반드시 있다. 100보다 작은 X의

모든 원소들을 x1 < ¢ ¢ ¢ < xk라 하자. 1 2 X 인 경우에는 X에 다른 수가 더 포함될 수 없으므

로 1 62 X 인 경우만 보아도 된다.

1 < x1 < ¢ ¢ ¢ < xk < 100

x와 합하여 200이 되는 수를 x0으로 나타내기로 하자. 다음의 각각의 집합은 두 수가 모두 X에

있을 수 없다.

fx01; x1x01g; fx02; x2x02g; : : : ; fx0k; xkx0kg

또한,

100 < x0k < ¢ ¢ ¢ < x01 < 200 < xkx0k < ¢ ¢ ¢ < x1x

01 < 10000

Page 238: Baltic Way 팀수학경시대회 1990-2005 pdf 보기

238

으로 모두 100에서 10000 사이의 서로 다른 수들이다. 따라서, X는 100보다 큰 수들 중 가질 수

없는 수가 적어도 k개가 있고, 그럼 X0보다 더 많은 원소를 갖지 못한다. 즉, X0가 최대의 경우

이다. }

Baltic 2003-8Baltic 2003-8

테이블 위에 2003개의 캔디가 놓여있다. 두 명이 번갈아가며 사탕을 먹는데, 자기 차례에 하나의 캔

디를 먹든가 테이블 위에 있는 캔디의 절반(캔디가 홀수 개라면 하나를 뺀 절반)을 먹는다. 각 차례

마다 최소한 한 개의 캔디는 먹어야한다. 가장 마지막 캔디를 먹는 사람이 이 게임에서 지게 된다.

누구|첫 번째, 혹은 두 번째 사람|에게 필승의 전략이 있는가?

풀이 (전주 상산고 1학년 윤소훈)

두 번째 사람에게 필승의 전략이 있다.

거꾸로 살펴보자. 1부터 차례대로 `지게 되는 숫자', `이기게 되는 숫자'를 따지면서 보자. 내가

하나를 먹거나 반을 먹어서 상대편에게 `지게 되는 숫자'를 만들어 넘겨줄 수 있으면 그 수는

`이기게 되는 숫자'라는 간단한 원리이다.

자기가 캔디를 먹어야 하는 순간 캔디가 하나 남아있으면 그 사람은 지게 된다. 따라서, 1은 `지

게 되는 숫자'이다. 그리고 캔디가 2개가 남아 있으면 그 사람은 절반을 먹어 상대편에게 `지게

되는 숫자' 1을 넘겨줄 수 있으므로 2는 `이기게 되는 숫자'이다. 여기서 일반적으로 `지게 되는

숫자' + 1 은 `이기게 되는 숫자'임을 알 수 있다(1). 또 3개가 남으면 반을 먹든 하나를 먹든 상

대편에게 무조건 `이기게 되는 숫자' 2를 넘겨주게 되므로 3은 `지게 되는 숫자'이다. 그럼 (1)에

의해 4는 `이기게 되는 숫자'이다. 5는 반을 먹어 `지게 되는 숫자' 3을 상대편에게 넘겨줄 수 있

으므로 5는 `이기게 되는 숫자'이다.

다음 설명의 이해를 돕기 위해 직접 위와 같은 방식으로 30까지 계산한 결과를 인용하겠다. O는

`이기는 수', X는 `지는 수'이고, 앞에서부터 차례로 1, 2, 3, 4, : : :에 대한 표시이다.

XOXOOOXOXOXOOOXOOOXOOOXOXOXOOO

여기서 짝수는 모두 O임을 관찰할 수 있다(2). 이것을 증명하기로 하자.

처음에 O는 짝수이다. 짝수인 O 다음에 X가 온다면 (1)에 의해 그 다음 짝수도 O가 된다. 짝수

인 O 다음에 또 O가 오는 경우를 생각해보자. 앞의 O를 2n, 뒤의 O를 2n+1 이라 하자. 2n+1

에서는 캔디를 하나만 먹으면 상대편에게 O인 2n을 넘겨주어 지게 되는데, 2n+ 1은 O이므로

이렇게 되어선 안 된다. 즉, 2n+ 1 에서는 캔디를 절반을 먹어 상대편에게 X인 n + 1 을 넘겨

주는 상황이 된다. 그럼 2n+ 2 에서도 캔디를 절반을 먹어 상대편에게 X인 n+ 1 을 넘겨줄 수

있으므로, 2n+ 2 도 역시 O가 된다. 따라서, O인 짝수 다음의 짝수도 항상 O임을 알 수 있고

그럼 (2)가 성립한다는 것을 알게 된다.

원래 문제로 돌아가자. 2003개에서 첫 번째 사람이 하나를 먹으면 2002개가 되고 절반을 먹으

면 1002개가 된다. 그런데 2002, 1002는 모두 짝수이므로 `이기게 되는 숫자'이다.따라서, 두 번

째 사람에게 필승의 전략이 있다. }

Page 239: Baltic Way 팀수학경시대회 1990-2005 pdf 보기

2003년 Baltic Way 풀이 239

Baltic 2003-9Baltic 2003-9

n은 144 이하의 양의 정수이다. `n이 a보다 작은가?'와 같은 식으로 10번의 질문을 할 수 있다. 답변

은 뜸을 들이고 나온다:즉 i번째 질문의 답은 i+1번째 질문을 한 후에 들을 수 있다(i = 1; 2; : : : ; 9).

10번째 질문의 답은 물어본 즉시 나온다. n을 알아낼 수 있는 질문 방법을 찾아보아라.

풀이 (부산 데레사여고 1학년 방윤희, 수정됨)

처음 질문을 한 후 답을 듣지 못하고 바로 두 번째 질문을 해야 한다. 한 번의 질문으로는 길이

2인 범위 a · n · a+ 1 내에서 n을 찾을 수 있다.

\n은 a보다 큰가?"

두 번의 질문으로는 길이 3인 범위 a · n · a+ 2 내에서 n을 찾을 수 있다.

① \n은 a+ 1보다 큰가?"

② \n은 a보다 큰가?"

두 질문을 모두 끝내고 나서 ①의 대답이 `그렇다'이면 n = a+ 2, `아니다'이면 바로 뒤에 나오

는 ②의 대답에 따라 `그렇다'이면 n = a+ 1, `아니다'이면 n = a 이다.

k번의 질문으로 알아낼 수 있는 n의 범위길이를 xk로 나타내기로 하자.

xk+2 ¸ xk + xk+1 (¤)

임을 보이자. a < n · a+ xk + xk+1 일 때 처음 질문에서

\n은 a+ xk+1 보다 큰가?"

를 물어본다. 이 질문의 결과에 따라 범위는 다음 둘 중 하나가 된다.

(1) a+ xk+1 < n · a+ xk + xk+1 (2) a < n · a+ xk+1

두 번째 질문은 처음 질문의 답이 (2)일 것이라 생각하고 질문한다. 두 번째 질문을 한 후 얻은

처음 질문의 답이 `그렇다'이면 (1)의 경우가 되는데, 그 범위길이는 xk이므로 남은 k번의 질문

으로 n을 구할 수 있다. 두 번째 질문을 한 후 얻은 처음 질문의 답이 `아니다'이면 (2)의 경우

가 되는데, 그 범위길이는 xk+1이므로 k + 1의 질문으로 n을 구할 수 있다. 그런데, 이미 제대

로된 질문 하나(두 번째 질문)를 던졌으므로 k번의 질문만 더 해서 n을 구할 수 있다. 즉, 길이

xk + xk+1 인 범위에서는 항상 k+ 2번의 질문으로 n을 알아낼 수 있으므로, (¤)이 증명되었다.

F1 = 1, F2 = 2, Fk+2 = Fk+Fk+1 로 정의되는 피보나치 수열을 생각하자.그럼 (¤)과 x1 = F1,

x2 = F2 에 의해 xk ¸ Fk 가 항상 성립한다. 즉, x10 ¸ F10 = 144 로 10번의 질문이면 문제에서

의 144 이하의 양의 정수 n을 알아낼 수 있다. 이 때, k번째 질문 방법은 다음과 같다: 현재 알

고 있는(답을 들은) 범위가 a < x · b 일 때

\n은 a+ F10¡k 보다 큰가?"

}

Page 240: Baltic Way 팀수학경시대회 1990-2005 pdf 보기

240

Baltic 2003-10Baltic 2003-10

평면 위에서 각 좌표가 모두 정수인 점을 `격자점(lattice point)'이라고 한다. 점³x1 + x2 + x3 + x44

;y1 + y2 + y3 + y4

4

´을 네 점 (xi; yi) (i = 1; 2; 3; 4) 의 `중심(centroid)'이라고 한다. n은 다음 성질을 만족하는 가장 큰

자연수이다: `임의로 택한 4개의 점의 중심이 항상 격자점이 아닌 점이 되도록 평면 위에 n개의 서

로 다른 격자점을 잡을 수 있다.' n = 12 임을 증명하여라.

증명 (마산 해운중 3학년 신훈)

먼저 n = 12 일 때 가능함을 보이자. 12개의 점을 네 묶음으로 나누어, 한 묶음에 3개의 점이 들

어가게 한다. 한 묶음에 있는 3개의 점의 x, y좌표는 mod 4로(4로 나눈 나머지로) 다음과 같이

모두 같게 만든다.

x mod 4 y mod 4

묶음1 0 1

묶음2 1 2

묶음3 2 0

묶음4 3 3

여기서 네 점을 골라 그 중심의 x좌표를 격자점으로 하는 방법은 다음의 2가지뿐이다.

(1) 한 묶음에서 두 점을 고르고, 거기서 §2한 번호를 가진 묶음에서 두 점을 고르는 경우

(2) 한 묶음에서 두 점을 고르고, 거기서 §1한 번호를 가진 묶음에서 각각 한 점씩 고르는 경

이 두 경우에서 y좌표를 따져보면 정수가 되는 경우가 없음을 확인할 수 있다.

다음 n = 13 일 때는 항상 중심이 격자점인 네 점이 있음을 보이자. 다음의 보조정리를 이용하

자.

보조정리 평면 위의 임의의 5개의 격자점 중에는 그 중점이 격자점인 두 점이 항상 있

다.

보조정리의 증명 x, y좌표를 홀짝으로 분류하면 (홀,홀), (홀,짝), (짝,홀), (짝,짝) 의 네

가지가 있으므로, 5개의 격자점 중에는 비둘기집의 원리에 의해 같은 분류에 속하는 두 점

이 항상 있다. 이 두 점의 중점이 격자점이 된다. ¤

13개의 격자점 중 5개를 고르면 위의 보조정리에 의해 그 중 중점이 격자점인 점 2개가 있다.

이 2점을 분리하고, 나머지 11개의 격자점에서 같은 방법으로 2개의 점을 더 분리한다. 9개의

점, 7개의 점, 5개의 점이 남았을 때 2개씩을 더 분리한다. 이렇게 두 점씩 5묶음의 점이 분리되

었고, 각 묶음의 중점은 격자점이므로 위의 보조정리에 의해 다시 그 중점이 격자점이 되는 두

중점이 있다.Ãx1+x22

+ x3+x42

2;y1+y22

+ y3+y42

2

!=³x1 + x2 + x3 + x4

4;y1 + y2 + y3 + y4

4

´

Page 241: Baltic Way 팀수학경시대회 1990-2005 pdf 보기

2003년 Baltic Way 풀이 241

와 같이 중점의 중점은 네 점의 중심이므로 증명이 되었다. ¤

주 O±cial Solution에서는 n = 12 일 때 다음과 같은 예를 제시하고 있다.

x mod 4 y mod 4

묶음1 0 0

묶음2 0 1

묶음3 1 1

묶음4 1 0

이 예가 앞의 신훈 학생의 예보다는 좀더 설명이 간결하게 되는 듯하다.

Baltic 2003-11Baltic 2003-11

평면 위에서 1000개의 점을 골라, 최소 6000개의 거리(두 점 사이의 거리)가 모두 같도록 할 수 있

을까?

풀이 (광주 대광여고 1학년 이고운)

다음 그림과 같이 점 4개를 잡는 것으로 시작하자. 5개의 선분은 같은 거리 d이다.

그리고 jAj = jBj = d 이고 A와 B의 편각이 60± 가 되는 두 개의 벡터 A, B를 잡자. 단, A나

B의 방향이 위의 그림의 선분에 평행하지 않도록 한다. 위의 그림을 벡터 A와 B만큼 각각 평

행이동하여 두 개의 복사본을 만들자.

그럼 원본과 복사본들의 대응되는 꼭지점들은 한 변의 길이가 d인 정삼각형을 이루게 된다. 다

시 이렇게 얻어진 12개의 점을 또 정삼각형법으로 3배로 불리는 과정을 계속 반복하면 다음과

같이 된다.

점의 개수 d인 거리의 개수

(앞단계£ 3) (앞단계£ 3 +현단계 점의 수)

1단계 4 5

2단계 3£ 4 = 12 3£ 5 + 12 = 273단계 3£ 12 = 36 3£ 27 + 36 = 1174단계 3£ 36 = 108 3£ 117 + 108 = 4595단계 3£ 108 = 324 3£ 459 + 324 = 17016단계 3£ 324 = 972 3£ 1701 + 972 = 6075

따라서, 972개 이하의 점만으로도 가능하다. }

Page 242: Baltic Way 팀수학경시대회 1990-2005 pdf 보기

242

Baltic 2003-12Baltic 2003-12

정사각형 ABCD가 있다. M은 변 BC 내부(선분의 끝점이 아닌 선분의 위)에, N은 변 CD 내부에

있고, \MAN = 45±이다. AMN의 외심이 변 AC 위에 있음을 증명하여라.

증명 (고양 백석고 2학년 조상영)

동일법을 써서 증명하자.

만약에 X1, X2가 동일한 점이라면 Y1, Y2도 동일한 점이 된다(비례식을 써서 쉽게 구할 수 있

음). 먼저 AB를 1로 잡자. 그리고 AM = a, AN = b 로 잡자.

4AND » 4AX2M 이므로 AD : AN = AM : AX2

이다. 즉, 1 : b = a : AX2 이므로

AX2 = ab

두 번째로 AX1의 경우를 보아도 마찬가지 계산을 할 수 있고

AX1 = ab

가 된다. 따라서, X1과 X2는 일치하고, Y1과 Y2도 일치한다. Y1 = Y2 는 4AMN의 중점에서

수선을 올려 만난 점이므로 이 점이 외심이고, AC 위에 존재한다. ¤

Baltic 2003-13Baltic 2003-13

ABCD는 BC = 2 ¢AB 인 직사각형이다. BC의 중점을 E라 하고 P를 변 AD 내부의 임의의 점이

라 하자. A에서 변 BP 위에, D에서 변 CP 위에 내린 수선의 발을 각각 F와 G라 한다. 이 때, 점

E, F , P , G가 모두 한 원 위에 있음을 보여라.

증명 (고양 백석고 2학년 조상영)

AP = a, PD = b, AB = 1, KE ? BP , EL ? PC 가 되도록 하자.

Page 243: Baltic Way 팀수학경시대회 1990-2005 pdf 보기

2003년 Baltic Way 풀이 243

먼저 ¤PKEL을 보면, \PKE + \PLE = 180± 이므로 원에 내접함을 알 수 있다. 따라서

\KPL+ \KEL = 180±

여기서

4KFE » 4LGE (1)

임을 밝히면 \KEF = \LEG 가 되어 \KEL = \FEG, 즉

\KPL+ \FEG = 180±

로 ¤FPGE가 원에 내접함을 보일 수 있다.

먼저, 피타고라스 정리에 의해 p = BP =pa2 + 1 이다. 그럼 비례식으로

BF =1

p; AF =

a

p

이다. 그런데 4ABF ´ 4BEK (ASA합동) 이므로

FK =1

p¡ a

p=1¡ a

p; KE = FB =

1

p

가 되고, 따라서

FK : KE = 1¡ a : 1

마찬가지 방법으로 피타고라스 정리와 비례식을 써서

GL : LE = b¡ 1 : 1

을 얻을 수 있다. 그런데 a+ b = 2 이므로

GL : LE = 1¡ a : 1

이 된다. 따라서, (1)이 증명되었다. ¤

Baltic 2003-14Baltic 2003-14

임의로 주어진 삼각형 ABC의 바깥에 정삼각형 AMB, BNC, CKA를 그리자. MN의 중점에서

AC 위로 수선을 내리고, 마찬가지로 NK와 KM의 중점에서 각각 AB와 BC 위로 수선을 내린다.

이 세 개의 수선이 모두 한 점에서 만남을 증명하여라.

증명 MN , NK, KM의 중점을 각각 D, E, F라 하자. 4DEF는 4KMN를 무게중심 G에

대해 180± 회전하여 12로 축소한 것(혹은 G가 2 : 1 의 내분점이 되도록 축소한 것)이다. 이 축

Page 244: Baltic Way 팀수학경시대회 1990-2005 pdf 보기

244

소변환을 T라 하고 하자.

K, M , N에서 AC, AB, BC에 내린 수선은 이 변들을 수직이등분하므로 4ABC의 외심 O에

서 모두 만난다. P = T (O) 라 하면 DP , EP , FP는 각각 KO, MO, NO와 평행하므로 AC,

AB, BC와는 각각 수직이다. 즉, D, E, F에서 AC, AB, BC로 내린 수선은 모두 한 점 P에서

만난다. ¤

Baltic 2003-15Baltic 2003-15

원에 내접하는 사각형의 대각선 AC와 BD의 교점을 P라 하자. P를 통과하는 원이 변 CD의 중점

M에서 그 변과 접하고, 대각선 BD, AC와 각각 Q, R에서 만난다. BS = DQ 가 되도록 BD 위에

점 S를 잡자. S를 지나 AB에 평행한 선이 AC와 T에서 만난다. AT = RC 임을 증명하여라.

증명 (대전 노은중 3학년 김희주)

¤ABCD가 한 원 위에 있으므로

AP ¢ CP = BP ¢DP (1)

P , Q, R, M을 지나는 원이 M에서 접하므로

CM2 = CR ¢ CP

DM2 = DQ ¢DP

Page 245: Baltic Way 팀수학경시대회 1990-2005 pdf 보기

2003년 Baltic Way 풀이 245

M이 CD의 중점이므로 CM2 = DM2, 따라서

CR ¢ CP = DQ ¢DP (2)

T는 S를 지나면서 AB k ST 인 점이므로 AP : BP = AT : BS, 따라서

BP ¢ AT = AP ¢BS (3)

(1)£ (2)£ (3) 을 하면

DQ ¢AT = BS ¢ CR

BS = DQ 라 하였으므로 AT = CR 이다. ¤

Baltic 2003-16Baltic 2003-16

a¡ b는 소수, ab는 완전제곱수(어떤 정수의 제곱이 되는 수)가 되는 양의 정수쌍 (a; b)를 모두 구하

여라.

풀이 gcd(a; b) j a¡ b 이고 a¡ b = p(소수) 이므로, gcd(a; b) = 1 또는 gcd(a; b) = p 이다.

(i) gcd(a; b) = 1 일 때: ab가 완전제곱수이므로 a = m2, b = n2 (m, n은 서로 소) 꼴이다.

이 때 a ¡ b = m2 ¡ n2 = (m + n)(m ¡ n) = p(소수)이므로 m + n = p, m ¡ n = 1. 즉,

p = 2n+ 1(홀수인 소수) 에 대해 a = (n+ 1)2, b = n2 이다.

(ii) gcd(a; b) = p 일 때: ab가 완전제곱수이므로 a = pm2, b = pn2 (m, n은 서로 소) 꼴이다.

이 때 a¡ b = p(m+ n)(m¡ n) 이 소수가 될 수 없으므로 이런 경우는 없다.

따라서 구하는 해는, 임의의 홀수인 소수 p = 2n+ 1 에 대해 (a; b) = ((n+ 1)2; n2) 들이다. }

Baltic 2003-17Baltic 2003-17

양의 정수 n의 모든 (양의)약수들이 오름차순으로 배열에 저장되어있다. Mary는 임의로 선택된 약

수 d > 1 에 대해 그것이 소수인지를 판정하는 프로그램을 만들어야한다. n이 d보다 크지 않은 k개

의 약수를 갖는다고 하자. Mary는 n의 약수들 중 처음 dk=2e개에 대해서만 d의 약수가 되는지 확

인하면 충분하다고 주장하였다: \만약 그들 중에서 1보다 큰 d의 약수가 발견된다면 d는 합성수이

고, 그렇지 않으면 d는 소수이다." Mary의 주장이 옳은가?

풀이 (전남과학고 1학년 김주성)

Mary의 주장이 옳다. d가 n의 약수라 하면 모든 d의 약수는 n의 약수이다. 만약 d가 합성수라

고 가정하고 d의 약수 중 가장 작은 소수를 i라고 하자. Mary의 의견이 옳다는 것을 밝히기 위

해서는 이 i보다 작은 n의 약수가 k=2개 미만임을 밝히면 된다.

i보다 작은 n의 임의의 약수를 j라고 하자.그러면 i ¢j 또한 n의 약수이고, d > i ¢ i 이므로 d > ij

임을 알 수 있다. 그러면 i보다 크고 d보다 작은 n의 약수의 개수는 i보다 작은 약수의 개수보

다 많거나 같다는 것을 알 수 있다. 따라서, i보다 작은 약수의 개수는 d보다 작거나 같은 약수

의 개수 k의 절반에 미치지 못한다. }

Page 246: Baltic Way 팀수학경시대회 1990-2005 pdf 보기

246

Baltic 2003-18Baltic 2003-18

모든 정수를 파랑, 녹색, 빨강, 노랑 중 하나의 색깔로 칠한다. 만약 a, b, c, d가 모두 0이 아니고 같

은 색깔이면 항상 3a¡ 2b 6= 2c¡ 3d 이 되도록 색칠하는 것이 가능할까?

풀이1 (부산 건국고 2학년 신승현)

가능하다. 0이 아닌 임의의 정수 x를 x = 3m® (®는 3의 배수가 아님. 음수일 수 있음) 꼴로 유

일하게 나타낼 수 있다. 즉, m과 ®를 x에 대한 함수 m(x), ®(x)로 볼 수 있다. 아래와 같이 색

칠하자.

² m이 짝수이고 ® ´ 1 (mod 3) 이면 x는 파랑

² m이 짝수이고 ® ´ 2 (mod 3) 이면 x는 녹색

² m이 홀수이고 ® ´ 1 (mod 3) 이면 x는 빨강

² m이 홀수이고 ® ´ 2 (mod 3) 이면 x는 노랑

즉, x와 y가 같은 색이면

m(x) ´ m(y) (mod 2); ®(x) ´ ®(y) (mod 3)

이다. 0은 색이 없다고 봐도 무방하다. 이 때, a, b, c, d가 같은 색이면

3(a+ d) 6= 2(b+ c)

임을 보이자. a, d가 같은 색이므로, 일반성을 잃지 않고 m(a) ¸ m(d) 라 하자.

a+ d = 3m(a)®(a) + 3m(d)®(d)

= 3m(d)(3m(a)¡m(d)®(a) + ®(d))

´ 3m(d)®(d)(32k + 1) (mod 3)

에서

m(3(a+ d)) = m(a+ d) + 1 = m(d) + 1

임을 알 수 있다. 마찬가지로, b, c가 같은 색이므로, 일반성을 잃지 않고 m(b) ¸ m(c) 라 하면,

m(2(b+ c)) = m(b+ c) = m(c)

이다. c, d도 같은 색이므로 m(c) ´ m(d) (mod 2), 즉 m(c) 6= m(d) + 1 이고, 따라서

m(3(a+ d)) 6= m(2(b+ c))

이므로 3(a+ d) = 2(b+ c) 일 수 없다. }

Page 247: Baltic Way 팀수학경시대회 1990-2005 pdf 보기

2003년 Baltic Way 풀이 247

풀이2 (O±cial Solution)

답은 `yes'이다. 0은 제외하고 생각해도 되고, 다음과 같이 색칠하면 된다. 0이 아닌 정수 k에 대

해 k = 5mk¤ (5 - k¤) 인 정수 k¤와 음 아닌 정수 m이 유일하게 정의된다. 두 정수 k1, k2에 대

k¤1 ´ k¤2 (mod 5)

일 때 같은 색으로 색칠한다. 3a¡ 2b = 2c¡ 3d 인 같은 색의 네 정수 a, b, c, d가 존재한다고 가

정하자. 3a¡ 2b¡ 2c+ 3d = 0 의 양변을 m(gcd(a; b; c; d))로 나누어 다음의 식을 얻을 수 있다.

3 ¢ 5A ¢ a¤ ¡ 2 ¢ 5B ¢ b¤ ¡ 2 ¢ 5C ¢ c¤ + 3 ¢ 5D ¢ d¤ = 0

단, A, B, C, D는 음 아닌 정수들이고, 이 중 적어도 하나는 0이다. mod 5로 보면

3(5A ¢ a¤ + 5B ¢ b¤ + 5C ¢ c¤ + 5D ¢ d¤) ´ 0 (mod 5)

이다. 그런데, a¤ ´ b¤ ´ c¤ ´ d¤ 6́ 0 (mod 5) 이므로, 위 식은

5A + 5B + 5C + 5D ´ 0 (mod 5)

가 된다. 이 식은 A, B, C, D 중에 0인 것이 적어도 하나 있으면 성립하지 않으므로 모순이다.

}

Baltic 2003-19Baltic 2003-19

양의 정수 a, b가 있다. 만약 a3 + b3 이 어떤 정수의 제곱이라면, a+ b 는 서로 다른 두 개의 소수의

곱으로 표현될 수 없음을 보여라.

증명 (부산 건국고 1학년 신승현, 수정됨)

귀류법으로 증명하자. a3+ b3 = z2, a+ b = pq (p 6= q 는 소수) 라 하자. (a+ b)3 = (a+ b)((a+

b)2 ¡ 3ab), 즉z2 = pq(p2q2 ¡ 3ab)

pq j z2 이므로 pq j z 여야 하고, 그럼 pq j (p2q2 ¡ 3ab), 즉

pq j 3ab 혹은 (a+ b) j 3ab

(i) p, q 중에 3이 있을 때: 일반성을 잃지 않고 p = 3, q j a 라 하자. a < a + b = 3q 이므로

a = q 또는 2q 이고, 따라서 fa; bg = fq; 2qg. 이 때 a3 + b3 = 9q3 = (3q)2 ¢ q 로 완전제곱

수가 아니므로 모순.

(ii) p, q 모두 3이 아닐 때: pq j ab. 일반성을 잃지 않고 p j a 라 하자. p j pq = a + b 이므로

p j b 이기도 하다. q j ab 이므로 a, b 중에 pq의 배수인 것이 있다. 그럼 pq = a+ b ¸ pq+1

로 모순.

따라서, 항상 모순이 되고, 문제의 명제는 성립한다. ¤

Page 248: Baltic Way 팀수학경시대회 1990-2005 pdf 보기

248

Baltic 2003-20Baltic 2003-20

n은 자신을 제외한 모든 (양의) 약수들의 합과 이 약수들의 개수를 더한 값이 n 자신과 같은 양의

정수이다. n = 2m2꼴임을 증명하여라.

증명 (광주과학고 2학년 김대영)

n의 소인수분해를 pa11 pa22 ¢ ¢ ¢ pakk 라 하자. 그럼 주어진 조건대로 적어보면

2n+ 1 = (모든 약수의 합) + (모든 약수의 개수)

= (1 + p1 + ¢ ¢ ¢+ pa11 ) ¢ ¢ ¢ (1 + pk + ¢ ¢ ¢+ pakk ) + (1 + a1) ¢ ¢ ¢ (1 + ak)

(i) n이 홀수일 경우; pi가 모두 홀수이므로, 위의 식의 홀짝을 따지면

1 ´ (1 + 1 + ¢ ¢ ¢+ 1a1) ¢ ¢ ¢ (1 + 1 + ¢ ¢ ¢+ 1ak) + (1 + a1) ¢ ¢ ¢ (1 + ak)

´ (1 + a1) ¢ ¢ ¢ (1 + ak) + (1 + a1) ¢ ¢ ¢ (1 + ak)

´ 2X ´ 0 (mod 2)

따라서 모순.

(ii) n이 짝수일 경우; p1 = 2 이고 나머지 pi들은 홀수이므로, 역시 홀짝을 따지면

1 ´ (2a1+1 ¡ 1)(1 + a2) ¢ ¢ ¢ (1 + ak) + (1 + a1) ¢ ¢ ¢ (1 + ak)

´ (¡1 + 1 + a1)(1 + a2) ¢ ¢ ¢ (1 + ak)

´ a1(1 + a2) ¢ ¢ ¢ (1 + ak)

이므로, a1는 홀수이고 나머지 ai들은 모두 짝수.

따라서, (ii)의 경우라야 하고 이 때 n = 22b1+1p2b22 ¢ ¢ ¢ p2bkk = 2m2꼴이다. ¤

Page 249: Baltic Way 팀수학경시대회 1990-2005 pdf 보기

2004년 Baltic Way 풀이 249

2004년 Baltic Way 풀이

Baltic 2004-1Baltic 2004-1

임의의 첨자 n = 1; 2; : : : 에 대해 다음 조건

(1) an + a2n ¸ 3n

(2) an+1 + n · 2pan ¢ (n+ 1)

을 만족하는 음이 아닌 실수들의 수열 a1; a2; a3; : : : 가 주어져 있다.

(a) 임의의 n 2 N 에 대해 부등식 an ¸ n 이 만족됨을 증명하여라.

(b) 이런 수열의 예를 하나 찾아라.

풀이 (서울 노일중 3학년 이대훈)

어떤 ak가 존재하여 ak < k 라고 가정하자. 그러면 (2)의 조건에 의하여

ak+1 + k · 2p

ak(k + 1) · ak + k + 1 (기하평균 ·산술평균)

) ak+1 · ak + 1

마찬가지로 ak+2 · ak+1 + 1 · ak + 2 가 성립하므로, 귀납적으로

ak+n · ak + n

따라서,

ak + a2k = ak + ak+k · ak + ak + k < 3k

이므로 (1)에 모순. 따라서 ak ¸ n 이 성립하고, 이러한 수열의 예로는 an = n+ 1이 있다. }

Baltic 2004-2Baltic 2004-2

P (x)는 음이 아닌 계수들로 된 다항식이다. 부등식 P ( 1x)P (x) ¸ 1 이 x = 1 에서 성립하면 모든 양

수 x에서도 성립함을 증명하여라.

증명 P (x) = anxn + ¢ ¢ ¢ a1x+ a0 이라 하면, 계수가 모두 양수임에서 임의의 양수 x에 대해

코시 부등식으로

P

µ1

x

¶P (x) =

³anxn+ ¢ ¢ ¢+ a1

x+ a0

´(anx

n + ¢ ¢ ¢+ a1x+ a0)

¸ (an + ¢ ¢ ¢+ a0)2 = P (1)2

= P

µ1

1

¶P (1)

P (1)2 ¸ 1 이면 P (1) ¸ 1 이므로(* P (1) ¸ 0), 그럼 모든 x에 대해 P (x) ¸ 1 이다. ¤

Page 250: Baltic Way 팀수학경시대회 1990-2005 pdf 보기

250

Baltic 2004-3Baltic 2004-3

p; q; r은 양의 실수들이고 n 2 N 이다. pqr = 1 일 때, 다음을 보여라.

1

pn + qn + 1+

1

qn + rn + 1+

1

rn + pn + 1· 1

증명 pnqnrn = 1 이므로 pn, qn, rn을 P , Q, R로 치환하는 것으로 생각하면 n = 1 일 때와

모두 동치이다. 식을 쉽게 쓰기 위해, n = 3 일 때로 풀기로 하자.

(좌변) =1

p3 + q3 + 1+

1

q3 + r3 + 1+

1

r3 + p3 + 1

=r

p3r + q3r + r+ ¢ ¢ ¢

=r

(p+ q)(p2 ¡ pq + q2)r + r+ ¢ ¢ ¢

=r

(p+ q)( pq¡ 1 + q

p) + r

+ ¢ ¢ ¢

· r

p+ q + r+ ¢ ¢ ¢ = p+ q + r

p+ q + r= 1

여기서 부등식은 A.M ¸ G.M 에서 qp+ p

q¸ 2 임을 이용한 것이다. ¤

Baltic 2004-4Baltic 2004-4

x1; x2; : : : ; xn은 산술평균이 X인 실수들이다. 다음 부분집합들 각각의 산술평균이 모두 X를 넘지

않도록하는양의정수K가존재함을증명하여라: fx1; x2; : : : ; xKg, fx2; x3; : : : ; xKg, fx3; : : : ; xKg,: : : ; fxK¡1; xKg, fxKg

증명1 (대전과학고 1학년 김두리)

x1; x2; : : : ; xn이 있으면 x1부터 살핀다.

(i) x1 · X 이면 K = 1.

(ii) x1 > X 이면 x1 = X + a1 라 놓자. 만약 x2 · X ¡ a1 이라면 K = 2.

(iii) x2 > X ¡ a1 이라면 다시 x2 = X ¡ a1+ a2 라 놓자. 즉, x1+x2 = 2X + a2. x3 · X ¡ a2

이면 K = 3.

(iv) x3 > X ¡ a2 이면 다시 x3 = X ¡ a2 + a3 이라 놓자. 즉, x1 + x2 + x3 = 3X + a3.

...

이렇게 앞에서부터 하나하나 살핀다. 이렇게 하면 일반적으로 xk을 살폈을 때

x1 + ¢ ¢ ¢+ xk¡1 = (k ¡ 1)X + ak¡1

x2 + ¢ ¢ ¢+ xk¡1 = (k ¡ 2)X + ak¡1 ¡ a1

x3 + ¢ ¢ ¢+ xk¡1 = (k ¡ 3)X + ak¡1 ¡ a2

...

Page 251: Baltic Way 팀수학경시대회 1990-2005 pdf 보기

2004년 Baltic Way 풀이 251

이고, xk · X ¡ ak¡1 이면 K = k 가 되어 끝나고, xk > X ¡ ak¡1 이면 xk = X ¡ ak¡1 + ak

로 두어 다음 단계를 계속 살피게 된다. 이 과정이 마지막까지 간다면

x1 = X + a1; x2 = X ¡ a1 + a2; : : : ; xn¡1 = X ¡ an¡2 + an¡1

이고, 이 때 반드시 xn = X ¡ an¡1 이기 때문에 K = n 이 된다. 따라서, K · n인 K는 반드시

존재한다. ¤

증명2 (대전 서원초 6학년 강성경)

보조정리 X · Y 에 대해(a+ b)X ¡ bY

a· X. (단, a; b > 0)

보조정리의 증명(a+ b)X ¡ bY

a=

aX ¡ b(Y ¡X)

a· aX

a= X. ¤

1 » n 사이의 수 ®에 대해

P®i=1 xi

®가 최소가 되는 ®를 특별히 k라고 하자. 그러면 조건에 의

해서 k > l 인 l에 대해 Pli=1 xi

l¸Pk

i=1 xi

k

가 성립. 그렇다면 [보조정리]에서

Y =

Pli=1 xi

l; X =

Pki=1 xi

k

a = k ¡ l, b = l이면

k ¢Pk

i=1 x

k¡ l ¢

Pli=1 xi

lk ¡ l

·Pk

i=1 x

k

즉, Pki=l+1 xi

k ¡ l·Pk

i=1 xi

k;

xl+1 + ¢ ¢ ¢+ xkk ¡ l

· x1 + ¢ ¢ ¢+ xkk

따라서, fx1; : : : ; xkg의 산술평균 · X 이면 k ¸ l 인 모든 l에 대해 fxl; : : : ; xkg의 산술평균 · X

이다. 그런데 fx1; : : : ; xng의 산술평균 = X 이므로, 조건에서 fx1; : : : ; xkg의 산술평균 · X.

따라서 k는 조건을 만족하는 K가 된다. ¤

증명3 (전남과학고 1학년 이홍민)

K가 존재하지 않는다고 가정하자. 그러면 1부터 n까지의 모든 k에 대해,

fx1; x2; : : : ; xkg; fx2; x3; : : : ; xkg; : : : ; fxkg

들중 적어도 하나의 산술평균이 X를 넘어야 한다.

² k = 1 일 때도 성립해야 한다. x1은 X보다 크다.

Page 252: Baltic Way 팀수학경시대회 1990-2005 pdf 보기

252

² k = 2 일 때, fx1; x2g와 fx2g 중 하나는 평균이 X보다 커야 한다. x1이 X보다 크므

로, 만약 fx2g가 X보다 크면 fx1; x2g도 평균이 X보다 크고, fx2g가 X보다 크지 않아도

fx1; x2g는 평균이 X보다 커야 한다(적어도 하나는 X보다 커야 하므로).

² k = 3 일 때, fx1; x2; x3g와 fx2; x3g와 fx3g 중 하나는 평균이 X보다 커야 한다. x1이

X보다 크므로 만약 fx2; x3g의 평균이 X보다 크면 fx1; x2; x3g의 평균도 X보다 크다. 또,

fx1; x2g의 평균이 X보다 크므로 fx3g의 평균이 X보다 크면 fx1; x2; x3g의 평균도 X보

다 크다. 결국, fx1; x2; x3g의 평균은 어느 경우에나 X보다 크다.

² 같은 방법으로, fx1; x2; : : : ; xkg에서

fx1g; fx1; x2g; fx1; x2; x3g; : : : ; fx1; x2; : : : ; xk¡1g

은 전 단계에서 평균이 X보다 크다고 증명되어 있고, 그에 의해 fx1; x2; : : : ; xkg는

fx2; x3; : : : ; xkg; fx3; x4; : : : ; xkg; : : : ; fxkg

중 어느 하나의 평균이 X보다 크면 fx1; x2; : : : ; xkg의 평균도 X보다 커야 한다.

² 이렇게 계속하여 k = n 이 되면 fx1; x2; : : : ; xng의 평균이 X보다 크게 되어 모순이 된다.

그러므로 가정이 잘못되었고 K는 존재한다. ¤

증명4 (성남 수내중 3학년 임동혁)

귀류법. 만약 이런 K가 존재하지 않는다고 하자. 그렇다면, 어떤 K에 대해서든 주어진 집합

중에서 적어도 하나는 x1; x2; : : : ; xn의 산술평균 X보다 크다. K = n 일 때를 보자. 가정에 의

해 이것보다 평균이 큰 집합 존재. 그 집합을 고르자. 그리고 그 집합의 가장 앞의(i가 가장 작

은) 원소를 xi라 하자. i > 1 이면 K를 다시 i¡ 1로 정하자. 그리고 또 그 때 가정의 산술평균

보다 큰 집합을 선택하자. 이것을 가능할 때까지 반복하면, 결국 i = 1 이 되어 마지막 집합은

x1을 포함하게 된다. 지금까지 선택된 집합들의 산술평균들은,모두 x1; x2; : : : ; xn의 산술평균

보다 크다. 그런데 지금까지 선택된 집합들의 원소를 모두 나열해보면 x1; x2; : : : ; xn이다. 따라

서, 모순. ¤

Baltic 2004-5Baltic 2004-5

정수 k에 대해 정의되는 다음 함수의 치역을 구하여라.

f(k) = (k)3 + (2k)5 + (3k)7 ¡ 6k

단, (k)2n+1은 2n+ 1의 배수 중에서 k에 가장 가까운 수를 나타낸다.

Page 253: Baltic Way 팀수학경시대회 1990-2005 pdf 보기

2004년 Baltic Way 풀이 253

풀이 (k)2n+1과 k의 차는 최대 n이다. (k)3 = k + a, (2k)5 = 2k + b, (3k)7 = 3k + c 라 하면

f(k) = a+ b+ c

jaj · 1, jbj · 2, jcj · 3 이므로

jf(k)j = ja+ b+ cj · jaj+ jbj+ jcj · 6

이다. 3, 5, 7이 서로 소이므로 중국의 나머지 정리에 의해 임의의 a, b, c에 대해 항상 정수해

k가 존재하므로 f(k)의 치역은 f¡6;¡5; : : : ; 6g 전체이다. }

Baltic 2004-6Baltic 2004-6

정육면체의 6개의 면에 양의 정수가 하나씩 쓰여져 있다.이 정육면체의 각각의 꼭지점에서는 그 점

에 접하는 세 면의 수들의 곱을 계산한다. 이 곱들의 합이 1001이다. 각 면에 쓰인 6개의 정수의 합

은 얼마인가?

풀이 마주보는 면에 쓰인 수끼리 짝을 지어 a, a0; b, b0; c, c0이라 하자. 마주보는 두 면 중 한

면씩 골라 세 면을 결정하면 그것과 각 꼭지점이 일대일대응하므로, 꼭지점에 쓰인 수들의 합

(a+ a0)(b+ b0)(c+ c0) = 1001 = 7 ¢ 11 ¢ 13

이 된다. 따라서, a+ a0 + b+ b0 + c+ c0 = 7 + 11 + 13 = 31. }

Baltic 2004-7Baltic 2004-7

양의 정수들의 집합 X는 최소한 2개의 원소를 갖고, n > m 인 임의의 m;n 2 X 에 대해 n = mk2

을 만족하게 하는 k 2 X 가 항상 존재한다. 이런 집합 X를 모두 찾아라.

풀이 (서울 당산서중 2학년 박민재, 서울 중앙중 1학년 이수홍)

X의 원소를 작은 것부터 차례로

a < b < c < ¢ ¢ ¢

라 하자. 문제의 조건에서 a, b는 항상 존재하고, b = am2 인 m 2 X 이 존재해야 한다. b ¸ 2이므로 m ¸ b 이면 b < am2 으로 우변이 더 커지므로, m < b 가 성립한다. 그럼 a, b의 최소성

에서 m = a 일 수밖에 없고, 따라서

b = a3

이 된다. 만일 세 번째 원소 c가 존재한다면, 조건에 의해서 어떤 n 2 X 에 대해서 c = an2 이

어야 한다. 여기서도 부등식 n < c 가 성립하고, b < c 이므로 m < n, 즉 n = b 일 수밖에 없다.

c = a7

Page 254: Baltic Way 팀수학경시대회 1990-2005 pdf 보기

254

그러면 c = bk2 이 되는 k = a2 이 X의 원소라야 하는데, 이것은 a < k < b 가 되어 모순이다.

따라서, c는 존재할 수 없고, 즉

X = fa; a3g

이다. a > 1 일 때 이런 X는 항상 문제의 조건을 만족한다. }

Baltic 2004-8Baltic 2004-8

f(x)는 상수함수가 아닌 정수계수 다항식이다. f(n)이 2004개 이상의 소인수를 갖도록 하는 정수

n이 존재함을 증명하여라.

증명 (서울 당산서중 2학년 박민재)

f의 치역에 포함되는 어떤 정수 m이 존재하여 m이 f의 치역의 모든 원소들 중에서 소인수를

가장 많이 가지고 있다고 가정해보자. 이러한 함수를 평행이동 시켜서 f(0) = m 이 되도록 할

수 있다(f(k) = m 이면 g(x) := f(x + k) 로 치환). 그런데 f의 다항식 전개를 생각하면, 임의

의 상수 p에 대해

f(pm2) = am2 + f(0) = am2 +m = m(am+ 1)

의꼴이된다(이렇게되는적당한수 a가존재한다). gcd(am+1; a) = 1이므로,만일 jam+1j > 1이라면 f(pm2)은 m보다 더 많은 소인수를 가지게 되어 모순이 된다. 즉 jam+ 1j = 1 일 수밖

에 없다. 그런데 이것은 f가 같은 함수값을 무한히 여러 번 갖는다는 것이고, 이것은 항등정리

에 의해 f가 상수함수가 되므로 역시 모순이다. 결국 처음에 가정했던 m은 존재하지 않고, 그

러므로 f(n)은 2004개 이상은 물론, 얼마든지 많은 소인수를 가질 수 있다. ¤

주 중국의 나머지 정리를 이용할 수도 있다.

Baltic 2004-9Baltic 2004-9

n¡ 1개의 자연수들의 집합 S가 주어져 있다(n ¸ 3). 이 집합의 어떤 두 원소의 차는 n으로 나누어

떨어지지 않는다. S의 공집합이 아닌 부분집합을 잘 골라 그 원소들의 합이 n의 배수가 되게 할 수

있음을 증명하여라.

증명 (대전과학고 1학년 김두리, 수정됨)

문제의 조건에 의해 일반성을 잃지 않고 n - a1 ¡ a2 인 두 원소 a1, a2가 존재한다.

a1; a2; a1 + a2; a1 + a2 + a3; : : : ; a1 + a2 + ¢ ¢ ¢+ an¡1

이렇게 n개의 그룹을 생각한다. 만약 이 중 n으로 나누어지는 게 있다면 당연히 명제 성립. 만

약 이 중 n으로 나누어지는 게 없다면, n으로 나눈 나머지가 n ¡ 1가지만 가능하므로, 비둘기

집의 원리에 따라 나머지 같은 두 그룹 반드시 존재. n - a1 ¡ a2 의 가정에 의해 a1과 a2가 그

두 그룹일 수는 없으므로, 그 두 그룹의 차 역시 몇 개의 ai들의 합임을 위의 꼴에서 알 수 있고,

그것이 n으로 나누어진다. ¤

Page 255: Baltic Way 팀수학경시대회 1990-2005 pdf 보기

2004년 Baltic Way 풀이 255

Baltic 2004-10Baltic 2004-10

임의의 n 2 N 에 대해 jpn+1 ¡ 2pnj = 1 인 소수들의 무한 수열 p1; p2; : : : ; pn; pn+1; : : : 이 존재하는

가?

풀이 pn+1 = 2pn § 1 이다. pn ¸ 2 이므로 pn+1 ¸ 2pn ¡ 1 > pn 으로 증가수열이다. 따라서,

초항 몇 개를 제외하고 시작하는 것으로 생각하면, 이 수열의 모든 항이 5 이상인 것으로 보아

도 상관없다. 5 이상의 소수는 모두 6k § 1꼴임으로부터,

(i) pn = 6k+1꼴이면, pn+1 = 2pn § 1 = 12k+3 or 12k+1 이고, 12k+ 3 은 소수가 아니므

로 무연근. 즉, pn+1도 6k + 1꼴이다.

(ii) pn = 6k ¡ 1꼴이면, pn+1 = 2pn § 1 = 12k ¡ 1 or 12k ¡ 3 이고, 역시 소수이려면 pn+1도

6k ¡ 1꼴임을 알 수 있다.

즉, 초항 몇 개를 제외하면 이 수열은

pn+1 = 2pn + 1 과 pn+1 = 2pn ¡ 1

의 둘 중 한 쪽으로 점화식이 완전히 고정된다. 이런 규칙적인 점화식으로 계속 소수만을 얻을

수 있다는 것은 소수의 분포가 불규칙함을 생각하면 곤란한 얘기가 될 것이다. 일반적으로 다

음과 같은 보조정리를 증명할 수 있다.

보조정리 임의의 자연수 a, b에 대해, pn+1 = apn + b 의 점화식으로 정의되는 수열의

모든 항이 소수일 수는 없다.

보조정리의 증명 귀류법으로 모두 소수라고 가정하자. 만일 a와 b가 서로 소가 아니

면 pn+1 = apn + b 가 a와 b의 최대공약수의 배수가 되므로 소수가 아니다. 따라서, a와

b는 서로 소이다. 그럼 유클리드 호제법의 원리에 의해 a와 p2 = ap1 + b 도 서로 소이

다. p = p2 라 하고 이 수열을 mod p로 생각하자. p로 나눈 나머지는 유한 개뿐이므로

mod p로 같은 두 항이 나타나게 되고, pn mod p 가 결정되면 pn+1 = apn + b mod p 도

결정되므로 이 수열은 어느 시점부터는 순환하게 된다. 한편, pn+1 mod p 가 결정되면

pn = a¡1(pn+1 ¡ b) mod p 도 결정되므로(a¡1은 mod p에 대한 a의 잉여역수) 이 수열은

역방향으로도 순환하고, 따라서 이 수열은 mod p에 대해 처음부터 완전순환한다. p2 ´ 0(mod p) 이므로 이것은 p의 배수인 항들이 주기적으로 나타난다는 것이 되고, 이 수열은

증가수열이므로 그 항들은 모두 p의 배수인 합성수가 되어 모순이다. ¤

위의 보조정리에 의해 pn이 모두 소수일 수 없음을 알 수 있다. }

Baltic 2004-11Baltic 2004-11

각 칸에 +1 혹은 ¡1이 쓰여져 있는 m£ n 크기의 표가 주어져 있다. 맨처음에는 ¡1이 하나뿐이고

나머지는 모두 +1이다. 한 번의 움직임은, ¡1인 칸을 하나 택해 그 ¡1을 0으로 바꾸고 그 칸과 이

Page 256: Baltic Way 팀수학경시대회 1990-2005 pdf 보기

256

웃한 칸의 숫자들에 동시에 ¡1을 곱한다(두 칸이 한 변을 공유하고 있으면 서로 이웃한다고 한다).

처음 ¡1이 어느 칸에 있든 상관없이 항상 이런 움직임만을 반복하여 모든 칸을 0으로 만들 수 있는

(m;n)을 모두 찾아라.

풀이 m과 n 중에 홀수가 있을 때는 가능하고, 둘다 짝수이면 불가능함을 보이겠다.

(1) m과 n 중에 홀수가 있을 때: 행의 폭이 홀수라고 하자(열의 경우에도 대칭적으로 마찬

가지이다). 처음의 ¡1인 칸 x와 같은 행에 있는 칸들만을 x로부터 시작하여 모두 조작한

다(그림에서 (²; ¤; ±) = (1;¡1; 0)).

²²²²²²²²²²²²²¤²²²²²²!

²²²²²²²²¤²²²¤±¤²²²¤²!

²²²²²²²¤¤¤²¤±±±²²¤¤¤! ¢ ¢ ¢ !

²²²²²¤¤¤¤¤±±±±±¤¤¤¤¤

그럼 그 행은 모두 0이 되고, 그 바로 윗행 R과 바로 아랫행 R0은 모두 ¡1이 되며, 나머

지 칸들은 그대로 1이다. 이제 R에서 홀수번째 칸을 먼저 모두 조작한 후 R의 짝수번째

칸을 모두 조작하면 R이 모두 0이 되고 R의 바로 윗행은 모두 ¡1이 되며, 나머지 칸들은

그대로이다.

²²²²²¤¤¤¤¤±±±±±¤¤¤¤¤!

¤²¤²¤±¤±¤±±±±±±¤¤¤¤¤!

¤¤¤¤¤±±±±±±±±±±¤¤¤¤¤

따라서, 이런 과정을 반복하면 한 행씩 모두 0으로 바뀌여, 결국 모두 0이 된다.

(2) m, n이 모두 짝수일 때: 가로 혹은 세로로 인접한 두 칸(이것을 짝이라고 부르자)의 사이

마다 점을 하나씩 찍자.² ² ² ² ² ²² ² ² ² ² ²² ² ² ¤ ² ²² ² ² ² ² ²¢¢¢¢

¢¢¢¢

¢¢¢¢

¢¢¢¢

¢¢¢¢

¢¢¢¢¢¢¢¢¢¢¢¢¢¢¢¢¢¢

그럼 점은 모두

(m¡ 1)n+m(n¡ 1) =짝수개

가 찍힌다. 그리고, 각 짝의 어느 한 칸의 조작에 의해 나머지 한 칸의 부호가 바뀔 때마다

그 짝 사이의 점을 제거하자. 일단 점이 제거되면 그 짝의 어느 한 칸이 0이 되므로 이미

제거한 점을 다시 제거해야 하는 상황은 생기지 않고, 또 모든 칸이 0이 되려면 각 짝마

다 둘 중 한 칸이 먼저 0이 되어야 하는 시점이 있으므로 모든 점이 제거되어야 한다. 따

라서, 각 칸의 부호가 바뀌는 총 횟수 X는 이 점들의 개수와 같으므로 짝수이다. 그런데,

1이었던 칸이 0이 되려면 1! ¡1! 1! ¢ ¢ ¢ ! ¡1! 0, 즉 ¡1이 된 후 0이 되어야 하므

로 부호가 홀수번 바뀌고(X에 홀수를 기여함), ¡1이었던 칸이 0이 되려면 부호가 짝수번

바뀐다(X에 짝수를 기여함). 즉, 처음에 1이었던 칸이 홀수개이므로 X는 홀수이다. 이것

은 조금 전에 X가 짝수라고 했던 것과 모순이므로 불가능하다.

따라서, m과 n 중 적어도 하나가 홀수인 모든 (m;n)이 답이 된다. }

Page 257: Baltic Way 팀수학경시대회 1990-2005 pdf 보기

2004년 Baltic Way 풀이 257

주 (과천고 2학년 홍성준)

이것은 1998년 IMO short-list에 있는 문제의 변형이다. 그 문제에서는 ¡1인 칸 대신에 1인 칸

을 조작하도록 되어 있고, 맨 처음에 항상 구석에만 1이 주어지도록 되어 있지만, 간단한 변

형을 통하면 결국 같은 문제이다. 이 문제는 Titu Andreescu의 책 Mathematical Olympiad

Challenge에도 그래프 이론과 연결해서 서술한 풀이가 수록되어 있다.

Baltic 2004-12Baltic 2004-12

서로 다른 2n개의 수들이 일렬로 늘어놓여 있다. 한 번의 움직임에서 우리는 두 수의 위치를 바꾸거

나 혹은 세 수를 순환적으로 바꿀 수 있다(세 수 a; b; c를 골라 a를 b의 자리로, b를 c의 자리로, c를

a의 자리로 옮길 수 있다). 항상 이 수들을 증가하는 순서로 배열할 수 있기에 충분한 회수는 최소

몇 번인가?

풀이 (대전과학고 1학년 진태진, 수정됨)

최소 n번이면 충분하다. 자기 위치에 있지 않은 수 a를 하나 택하자. a가 있어야 할 위치에는

b가 있다고 하자. 만일 b가 있어야 할 위치에 a가 있다면, a와 b의 위치를 서로 바꾼다. b가 있어

야 할 위치에 또다른 수 c가 있다면, a ! b ! c ! a 와 같이 순환적으로 위치를 바꾼다. 그럼

언제나 2개 이상의 수가 한 번의 시행으로 제자리에 가도록 할 수 있다. 따라서, n번의 시행이

면 2n개의 수가 모두 제자리에 가도록 할 수 있다.

n번 미만으로는 안 되는 경우가 있음을 보이자. 각 수가 옮겨져야할 위치를 꼬리를 물어 나타

내면(a가 b의 위치로 옮겨져야 한다면 a! b! ¢ ¢ ¢ ), 전체 수들이 몇 개의 cycle로 분할되어 구

성된다(이미 자기 자리에 있는 수는 길이 1인 cycle a! a 로 생각하자). 문제의 두 종류의 시행

을 어떻게 적용해도 이 cycle의 수는 최대 2개까지만 늘어날 수 있다(자세한 과정은 지루하므로

생략). 따라서, 처음에 2n개의 수들이 모두 하나의 큰 cycle로 순환하고 있다고 하면, 모두 제자

리에 놓이도록 하려면 cycle의 개수가 2n개가 되어야 하므로 적어도 n번의 시행이 필요하다. }

Baltic 2004-13Baltic 2004-13

유럽 연합의 25개 회원국이 다음 규칙에 따라 한 위원회를 구성하였다: (1) 위원회는 매일 회의를

가져야 한다. (2) 각 회의마다 최소한 한 회원국이 참석해야 한다. (3) 임의의 서로 다른 두 회의에

는, 참석하는 회원국의 집합이 달라야 한다. (4) n번째 회의에는, 각각의 k < n 에 대해, k번째 회의

에 참석했던 회원국 중 적어도 한 나라가 참석해야 한다. 이 위원회는 회의를 최장 며칠 동안 가질

수 있는가?

풀이 (대전과학고 1학년 김두리, 수정됨)

224일이 최대이다.

전체 25개 회원국의 집합을 M이라 하면 각각의 회의는 M의 부분집합들이다. 조건 (4)에 의해,

임의의 두 회의 A와 B에 대해 A와 B는 서로 소일 수 없다. 따라서, 어떤 부분집합 A가 회의로

열렸다면, 그 여집합 Ac는 회의로 열릴 수 없다. M의 모든 부분집합들을 서로 여집합이 되는

Page 258: Baltic Way 팀수학경시대회 1990-2005 pdf 보기

258

것끼리 둘씩 묶어줄 수 있고(모두 224묶음), 각 묶음에서 실제 회의가 될 수 있는 것은 둘 중 하

나뿐이므로, 회의는 길어야 224일이다.

한편, 한 회원국 m 2M 을 택해 m을 포함하는 모든 부분집합들을 회의로 한다면 문제의 조건

을 모두 만족함을 알 수 있다. 이런 부분집합이 정확히 224개이므로 이것이 답임이 확인되었다.

}

Baltic 2004-14Baltic 2004-14

땅콩 한 더미란 4개 이상의 땅콩을 말한다. 두 명이 n ¸ 4개의 땅콩 한 더미로 시작하여 다음과 같

은 게임을 한다. 각 차례에서 한 명이 현재 놓인 땅콩 더미들 중에서 한 더미를 택해 그것을 두 집합

으로 나눈다(한 쪽이 공집합이 되면 안 되고, 이 두 집합은 꼭 더미가 될 필요는 없으며, 땅콩 개수

는 임의로 나누면 된다). 둘이 번갈아 차례를 갖다가, 더 이상 자기 차례를 실행할 수 없는 사람이

지게 된다. 먼저 하는 사람이 필승의 전략을 가질 수 있는 n은 어떤 값들인가?

풀이 (KAIST 수학과 05학번 이승진)

답은 n = 4k, 4k + 1, 4k + 2꼴일 때이다. 즉, n = 4k + 3꼴일 때만 나중에 하는 사람이 이긴다.

이 문제를 보다 일반화하여, 여러 개의 더미로 게임을 시작할 수 있는 것으로 보고, 각 더미의

개수 중 4k꼴인 것이 a개, 4k + 1꼴인 것이 b개, 4k + 2꼴인 것이 c개일 때,

(a; b; c) ´ (0; 0; 0); (1; 1; 1) (mod 2) () (a; b; c) 2 L (¤)

임을 증명하기로 하겠다. 단, L은 먼저 하는 사람이 지는 경우(losing state)들의 집합이다. 먼

저 하는 사람이 이기는 경우들의 집합은 W = Lc라 하자. 1개나 2개나 3개짜리 집합은 게임에

더이상 관여되지 않으므로 버려진다고 말하기로 하자.

(1) L의 원소는 어떻게 실행해도 한 번의 차례가 지나면 항상 W의 원소임을 확인하자. L의

원소가 L의 원소로 바뀌었다고 하고 모순을 찾으면 된다.

(1a) (0; 0; 0) ! (0; 0; 0) 또는 (1; 1; 1) ! (1; 1; 1) 일 때: 4k + 1꼴의 더미 하나를 깼다면,

b의 홀짝이 바뀌면 안 되므로 쪼개진 집합 중 하나는 4k+1꼴의 더미여야 한다. 그럼

다른 한 집합은 4k꼴의 더미가 되어 a의 홀짝이 바뀌므로 모순. 4k나 4k + 2꼴의 더

미를 깼을 때도 마찬가지이다. 4k + 3꼴의 더미를 깼을 때는 a, b, c 중 1개 혹은 2개

의 홀짝이 바뀌므로 역시 모순이다.

(1b) (0; 0; 0) ! (1; 1; 1) 또는 (1; 1; 1) ! (0; 0; 0) 일 때: a, b, c 중 하나를 깼다면 다른 두

종류의 더미가 하나씩 생겨나야 한다(예를 들어 a를 하나 줄였다면, b, c가 하나씩 늘

어나야 한다). 그것은 4k, 4k+ 1, 4k + 2꼴을 하나씩 가지고 mod 4의 덧셈등식(예를

들어 0 ´ 1 + 2 (mod 4))을 하나 만들어야 한다는 것인데, 셋의 합이 홀수라 양변의

홀짝이 맞을 수 없어 모순이다. 4k+3꼴의 더미를 깼다면 (1a)에서처럼 a, b, c 중 1개

혹은 2개의 홀짝이 바뀌므로 역시 모순이다.

Page 259: Baltic Way 팀수학경시대회 1990-2005 pdf 보기

2004년 Baltic Way 풀이 259

(2) W의 원소는 잘 실행하면 항상 L의 원소로 만들 수 있음을 보이자. L이 아닌 각 경우에

대해 다음과 같이 작업하면 항상 (0; 0; 0)의 꼴로 만들어줄 수 있다.

(1; 0; 0) : 4k ! f1; 4(k ¡ 1) + 3g(0; 1; 0) : 4k + 1 ! f2; 4(k ¡ 1) + 3g(0; 0; 1) : 4k + 2 ! f3; 4(k ¡ 1) + 3g(1; 1; 0) : 4k + 1 ! f1; 4kg(1; 0; 1) : 4k + 2 ! f2; 4kg(0; 1; 1) : 4k + 2 ! f1; 4k + 1g

(1), (2)에 의해 (¤)이 사실임이 증명되었다. 단, 전체 땅콩의 개수 n에 대해 수학적 귀납법을 적

용하는 것이므로, n = 4; 5; 6; 7 등 작을 때 성립한다는 것은 미리 확인해둘 필요가 있다. }

주 n이 짝수일 때는 먼저 하는 사람이 이긴다는 것을 다음과 같이 간단히 말할 수도 있다: 먼

저 하는 사람이 같은 개수의 두 집합으로 나누고, 그 다음부터는 다음 하는 사람의 플레이를 똑

같이 따라 하여 두 집합이 계속 대칭이 되도록 하면 된다. 그러나, n이 홀수일 때를 해결하려면

역시 꽤 많은 얘기를 해야 한다.

한편, 위의 (2)를 잘 관찰하면, 필승의 권리를 가진 자가 (1; 1; 1)이 게임 중에 한 번도 나오지 않

도록 하면서 승리할 수 있음을 알 수 있다.

Baltic 2004-15Baltic 2004-15

한 원을 13개의 호로 쪼개어 순서대로 1부터 13까지의 번호를 매겼다. 다섯 마리의 벼룩 A, B, C,

D, E가 각각 호 1, 2, 3, 4, 5에 앉아있다. 벼룩은 원주의 양쪽 방향으로 다섯 위치 떨어져있는 비어

있는 호로 뛸 수 있다. 한 번에 한 마리의 벼룩만 뛸 수 있고, 두 마리의 벼룩이 같은 호에 있을 수

는 없다. 몇 번의 뜀 후에 벼룩들은 다시 처음의 1, 2, 3, 4, 5의 호로 돌아왔는데, 처음과는 앉은 순

서가 바뀌었다고 한다. 어떤 순서가 가능한가?

풀이 벼룩이 바로 뛸 수 있는 위치들을 이웃한 위치라 하고, 이웃한 위치끼리 계속 연결해보

1¡ 6¡ 11¡ 3¡ 8¡ 13¡ 5¡ 10¡ 2¡ 7¡ 12¡ 4¡ 9¡ 1과 같은 원형의 연결을 얻는다. 벼룩은 이런 위치 순서로만 옮겨가므로 이 원형의 연결에서 다

섯 벼룩의 순서는 바뀌지 않는다. 즉, 1¡ 3¡ 5¡ 2¡ 4¡ 1 이 회전되는 관계의 위치만 가능하

고, 그럼 처음의 순서 외에 가능한 순서는 DEABC, BCDEA, EABCD, CDEAB뿐이다. }

Baltic 2004-16Baltic 2004-16

주어진 원의 한 할선과 한 접선이 원밖의 점 P에서 만난다. 할선은 원과 두 점 A와 B에서 만나고,

접선은 점 C에서 원과 접한다. 그리고, P를 지나는 지름에 대해 이 세 점은 같은 쪽에 있다. C에서

이 지름에 내린 수선의 발을 Q라 하자. QC가 \AQB를 이등분함을 증명하여라.

Page 260: Baltic Way 팀수학경시대회 1990-2005 pdf 보기

260

증명 (대전과학고 1학년 김두리)

할선과 접선에 대한 비례성질과 직각삼각형의 닮음으로부터

PC2 = PA£ PB = PO £ PQ

따라서, A, B, Q, O는 한 원 위에 있다. 그럼

\AQO = \ABO = \BAO = \BQP

으로 같고, 이것을 90±에서 뺀 각도 서로 같으므로

\AQC = \BQC

따라서, QC는 \AQB의 이등분선이다. ¤

Baltic 2004-17Baltic 2004-17

두 변의 길이가 3과 4인 직사각형을 생각하자. 각 변 위에서 꼭지점이 아닌 한 점씩을 임의로 택해,

이 네 점으로 이어 만든 사각형의 네 변의 길이를 각각 x, y, z, u라 하자. 25 · x2+y2+z2+u2 · 50임을 증명하여라.

주 1970년 캐나다 수학올림피아드에 아래와 같은 문제가 출제된 적이 있다.

CanMO 1970-5 한 변의 길이가 1인 정사각형의 각 변 위에 꼭지점을 하나씩 잡아

새로운 사각형을 만들었다. 이 사각형의 네 변의 길이 a, b, c, d가 다음 부등식을 만

족함을 보여라.

2 · a2 + b2 + c2 + d2 · 4

이 문제와 위의 문제를 보다 일반화하여 증명하기로 하자.

증명 일반적으로 직사각형의 두 변의 길이를 m과 n, 내접하는 사각형의 네 변의 길이를 a,

b, c, d로 하면 다음이 성립한다.

m2 + n2 · x2 + y2 + z2 + u2 · 2(m2 + n2)

Page 261: Baltic Way 팀수학경시대회 1990-2005 pdf 보기

2004년 Baltic Way 풀이 261

이것을 증명하자. 원래의 직사각형을 ABCD라 하고, 내접하는 사각형의 꼭지점은 이 직사각

형의 둘레에 APBQCRDS의 순서로 있다고 하자. PQ = a, QR = b, RS = c, SP = d 라 하

고, AP , PB, BQ, QC, CR, RD, DS, SA를 각각 a1, a2, b1, b2, c1, c2, d1, d2라 하자.

피타고라스의 정리에 의해

a2 + b2 + c2 + d2 = a21 + a22 + b21 + b22 + c21 + c22 + d21 + d22 (¤)

a1 + a2 = c1 + c2 = m, b1 + b2 = d1 + d2 = n 이고 모두 음이 아닌 수이므로, 여기에 잘 알려

진 부등식(전개하여 정리하면 양쪽 부등식 모두 (x¡ y)2 ¸ 0 의 완전제곱식으로 확인된다)

(x+ y)2

2· x2 + y2 · (x+ y)2 (x; y ¸ 0)

을 적용하면, (¤)은 다음과 같이 된다.

m2

2+

n2

2+

m2

2+

n2

2· a2 + b2 + c2 + d2 · m2 + n2 +m2 + n2

따라서, 원하던 부등식이 증명되었다. ¤

Baltic 2004-18Baltic 2004-18

삼각형 ABC의 꼭지점 A에서 나온 광선이 변 BC와 X에서 만나고 4ABC의 외접원과 Y에서 만

난다. 1AX

+ 1XY

¸ 4BC 임을 증명하여라.

증명 (공주사대부고 2학년 윤혜림)

준식을 동치변형하면

XY +AX

AX ¢XY¸ 4

BC

즉, (XY +AX)(BX +XC) ¸ 4 ¢ AX ¢XY

Page 262: Baltic Way 팀수학경시대회 1990-2005 pdf 보기

262

를 증명하면 된다. 산술기하평균에 의해

XY +XA ¸ 2pXY ¢XA

BX +XC ¸ 2pBX ¢XC

또한, AY와 BC는 서로 교차하는 현이므로 위의 우변은 일치한다. 즉, 변변 곱하면

(XY +AX)(BX +XC) ¸ 2pXY ¢XA ¢ 2

pBX ¢XC = 4 ¢AX ¢XY

로 증명된다. ¤

Baltic 2004-19Baltic 2004-19

주어진 삼각형 ABC의 변 BC의 중점을 D라 하자. \BAM = \DAC 를 만족하는 변 BC 위의 점

을 M이라 하자. 삼각형 CAM의 외접원이 변 BC와 만나는 또다른 점을 L이라 하자. 또, 삼각형

BAM의 외접원이 변 AC와 만나는 또다른 점을 K라 하자. KL k BC 임을 증명하여라.

증명 (성남 수내중 3학년 임동혁)

동일법을 이용하자. BC k KL 이 되도록 K를 변 AC 위에 잡았을 때, 점 K, M , B, A가 한 원

위에 있음을 증명하면 된다.

BC k KL 이므로 BK와 LC의 교점 X는 중선 AD 위에 있다. 이 때, 문제의 조건, 원주각, 평

행임을 차례로 이용하면

\KAX = \BAM = \LCM = \KLX

이 된다. 따라서, 원주각에 의해 점 A, L, X, K는 한 원 위에 있다. 다시 비슷하게

\KAM = \LAX = \LKX = \KBM

이 되므로, 점 B, M , K, A는 한 원 위에 있다. ¤

Baltic 2004-20Baltic 2004-20

공통의 양끝점 A, B를 갖는 세 원호 w1; w2; w3이 직선 AB의 한 쪽에 놓여 있다. w2는 w1과 w3 사

이에 있다. B에서 나온 두 줄기의 광선이 이 호들과 각각 M1;M2;M3와 K1;K2;K3에서 만난다.

M1M2M2M3

= K1K2K2K3

임을 증명하여라.

Page 263: Baltic Way 팀수학경시대회 1990-2005 pdf 보기

2004년 Baltic Way 풀이 263

증명 (전남과학고 1학년 이홍민)

그림에서 원주각으로

\AK1B = \AM1B; \AK2B = \AM2B; \AK3B = \AM3B

이다. 그러므로

4AK1K2 » 4AM1M2; 4AK2K3 » 4AM2M3

이다(AA닮음). 이로부터

K1K2 :M1M2 = AK2 : AM2 = K2K3 :M2M3

임을 알 수 있고, 이것이 문제에서 증명하고자 하는 바이다. ¤

Page 264: Baltic Way 팀수학경시대회 1990-2005 pdf 보기

264

2005년 Baltic Way 풀이

Baltic 2005-1Baltic 2005-1

a0은 양의 정수이다. 수열 fangn¸0 을 다음과 같이 정의하자: 0 · ci · 9 인 정수 ci들에 대해

an =

jXi=0

ci10i

로 나타날 때,

an+1 = c20050 + c20051 + ¢ ¢ ¢+ c2005j

이다. 즉, an+1은 an을 십진법으로 전개했을 때의 각 자리수의 2005제곱의 합이다. 이 수열의 모든

항이 서로 다르도록 a0을 정하는 것이 가능한가?

풀이 (서울 경희중 3학년 이승후)

보조정리 1 k £ 92005 < 10k¡1 를 만족하는 자연수 k가 존재한다.

증명 한 예로, k = 92005 가 있다. ¤

k는 자연수이므로 최소원의 법칙에 의해 최소의 자연수 m이 존재한다.

보조정리 2 m 이상의 모든 정수는 위의 보조정리를 만족한다.

증명 k에 대해 성립한다면

k £ 92005 < 10k¡1

k £ 92005 + 92005 < 10k¡1 + 92005 < 10k¡1 + 9£ 10k¡1

(k + 1)92005 < 10k

으로 k + 1 일 때도 성립한다. 귀납적으로 m 이상은 항상 성립한다. ¤

이 두 보조정리가 의미하는 바는 fang의 자리수는 계속 증가하지 못한다는 것이다. 왜냐하면,

ai가 k자리일 때 k ¸ m 이면

ai+1 · k ¢ 92005 < 10k¡1 · ai

로 감소하고, k · m 이면

ai+1 · m ¢ 92005 < 10m¡1

로 계속 m자리 이하를 유지하기 때문이다. 즉, a0이 m자리 이하면 보조정리 1과 2에 의하여 수

열의 어느 항도 m자리를 넘지 못하고, a0이 m자리 이상이면 보조정리 2에 의하여 계속 감소하

다가 m자리 아래로 떨어진다. 따라서, 이 수열에 나타나는 수의 종류는 유한 개뿐이므로 비둘

기집의 원리에 의해 유한 번 안에 같은 두 항이 존재한다. 따라서 그런 a0은 존재하지 않는다.

}

Page 265: Baltic Way 팀수학경시대회 1990-2005 pdf 보기

2005년 Baltic Way 풀이 265

Baltic 2005-2Baltic 2005-2

0 · ®; ¯; ° < 90± 이고 sin®+ sin ¯ + sin ° = 1 을 만족하는 세 각 ®, ¯, °가 있다. 다음을 증명하여

라.

tan2 ®+ tan2 ¯ + tan2 ° ¸ 3

8

증명 (서울 대왕중 3학년 최서준)

우선, 0± · ®; ¯; ° < 90± 이므로, 0 · sin®; sin¯; sin ° < 1 이다.

tan2 ®+ tan2 ¯ + tan2 ° =sin2 ®

cos2 ®+sin2 ¯

cos2 ¯+sin2 °

cos2 °

=sin2 ®

1¡ sin2 ® +sin2 ¯

1¡ sin2 ¯ +sin2 °

1¡ sin2 °= ¡3 + 1

1¡ sin2 ® +1

1¡ sin2 ¯ +1

1¡ sin2 °¸ ¡3 + 9

(1¡ sin2 ®) + (1¡ sin2 ¯) + (1¡ sin2 °) (By A.M ¸ H.M)

= ¡3 + 9

3¡ (sin2 ®+ sin2 ¯ + sin2 °)

한편,

(12 + 12 + 12)(sin2 ®+ sin2 ¯ + sin2 °) ¸ (sin®+ sin ¯ + sin °)2 = 1

sin2 ®+ sin2 ¯ + sin2 ° ¸ 1

3

3¡ (sin2 ®+ sin2 ¯ + sin2 °) · 8

3

¡3 + 9

3¡ (sin2 ®+ sin2 ¯ + sin2 °) ¸ ¡3 + 9£3

8=3

8

단, 등호조건은 sin® = sin ¯ = sin ° =1

3. ¤

별증 (과천고 1학년 홍성준)

0± · A < 90± 에서 sin 함수는 위로 볼록하다. 따라서, Jensen 부등식에 의해

sin®+ sin ¯ + sin °

3· sin ®+ ¯ + °

3

sin®+ ¯ + °

3¸ 1

3에서 sin2

®+ ¯ + °

3¸ 1

9, cos2

®+ ¯ + °

3· 8

9. 그럼

tan2®+ ¯ + °

3=sin2

®+ ¯ + °

3

cos2®+ ¯ + °

3

¸ 1

8

이제1

3(tan2 ®+ tan2 ¯ + tan2 °) ¸ tan2 ®+ ¯ + °

3를 증명하면 된다.

(tan2 x)00 = (2 tan x sec2 x)0 = 2(tanx)0 sec2 x+ 2 tanx ¢ 2 secx ¢ secx ¢ tan x= 2 sec4 x+ 4 sec2 x ¢ tan2 x ¸ 0

Page 266: Baltic Way 팀수학경시대회 1990-2005 pdf 보기

266

따라서, y = tan2 x 는 아래로 볼록. 그러므로, 다시 Jensen 부등식에 의해

tan2 ®+ tan2 ¯ + tan2 ° ¸ 3 tan2 ®+ ¯ + °

3¸ 3

8

이 된다. (Q.E.D) ¤

주 두 함수 f , g가 아래로 볼록이고 증가함수이면 f ± g도 아래로 볼록이고 증가함수이다. 증

명은 다음과 같다:

(f ± g)(x) + (f ± g)(y)2

=f(g(x)) + f(g(y))

2

¸ f

µg(x) + g(y)

2

¶¸ f

³g³x+ y

2

´´= (f ± g)

³x+ y

2

´f(y) = y2, g(x) = tanx 로 두고 0 · x < 90±, 0 · y < 1 에서 생각하면 이 문제의 상황이 된

다.

Baltic 2005-3Baltic 2005-3

수열 fakgk¸1 은 다음과 같이 정의된다: a1 = 1, a2 =12,

ak+2 = ak +1

2ak+1 +

1

4akak+1(k ¸ 1)

다음을 증명하여라.1

a1a3+

1

a2a4+

1

a3a5+ ¢ ¢ ¢+ 1

a98a100< 4

증명 (서울 대왕중 3학년 최서준)

ak+2 = ak +12ak+1 +

14akak+1

이고 14akak+1

은 양수이므로 분명

ak+2 > ak +1

2ak+1

이 성립할 것이다. 따라서,

ak+1 < 2ak+2 ¡ 2ak이고, 양변을 akak+1ak+2로 나누면

1

akak+2<

2

akak+1¡ 2

ak+1ak+2

이 된다. 이로부터 telescoping에 의해

1

a1a3+

1

a2a4+ ¢ ¢ ¢+ 1

a98a100<

2

a1a2¡ 2

a2a3+

2

a2a3¡ 2

a3a4+ ¢ ¢ ¢+ 2

a98a99¡ 2

a99a100

=2

a1a2¡ 2

a99a100<

2

a1a2= 4

임을 알 수 있다. ¤

Page 267: Baltic Way 팀수학경시대회 1990-2005 pdf 보기

2005년 Baltic Way 풀이 267

Baltic 2005-4Baltic 2005-4

모든 실수 x에 대해 P (x2 + 1) = P (x)2 + 1 을 만족하는 실계수 다항식 P (x)를 3개만 찾아라.

풀이 (서울 당산서중 1학년 박민재)

f(x) := x2 + 1 이라고 하자. 그러면 주어진 식은 P (f(x)) = f(P (x)) 가 된다. 즉

P ± f = f ± P

이다. 그러므로

P = fk = f ± f ± ¢ ¢ ¢ ± f (k번 합성)

라면 위의 식이 성립함은 자명하다. P (x)를 몇 개 구해보면, k = 0; 1; 2; 3 인 경우

² k = 0: P (x) = x

² k = 1: P (x) = f(x) = x2 + 1

² k = 2: P (x) = f(f(x)) = (x2 + 1)2 + 1 = x4 + 2x2 + 2

² k = 3: P (x) = f(f(f(x))) = ((x2 + 1)2 + 1)2 + 1 = x8 + 4x6 + 8x4 + 8x2 + 5

위의 경우 모두 대입해보면 성립함을 알 수 있다. }

Baltic 2005-5Baltic 2005-5

a, b, c는 abc = 1 인 양의 실수들이다. 다음을 증명하여라.

a

a2 + 2+

b

b2 + 2+

c

c2 + 2· 1

증명1 (부산 장전중 2학년 안현태)

a2 + 1 ¸ 2a 이므로 aa2+2

· a2a+1 , 따라서

a

2a+ 1+

b

2b+ 1+

c

2c+ 1· 1

을 보이면 충분하다. 양변에 (2a+ 1)(2b+ 1)(2c+ 1)을 곱하여 전개하면

a(2b+ 1)(2c+ 1) + b(2c+ 1)(2a+ 1) + c(2a+ 1)(2b+ 1) · (2a+ 1)(2b+ 1)(2c+ 1)12abc+ 4(ab+ bc+ ca) + (a+ b+ c) · 8abc+ 4(ab+ bc+ ca) + 2(a+ b+ c) + 1

4abc · (a+ b+ c) + 1

abc = 1 이므로

3 · a+ b+ c

과 동치가 되고, 이것은 산술-기하평균 부등식 a + b + c ¸ 3 3pabc 로부터 잘 성립하는 식이다.

¤

Page 268: Baltic Way 팀수학경시대회 1990-2005 pdf 보기

268

증명2 (과천고 1학년 홍성준)

a = eX , b = eY , c = eZ 로 놓자. abc = 1 () X + Y + Z = 0.

f(t) =et

e2t + 2

라 하면 원 부등식은f(X) + f(Y ) + f(Z)

3· f

µX + Y + Z

3

¶꼴이다. Jensen부등식을 적용하기 위해 f(t)의 이계도함수를 구하자.

f 0(t) =et(e2t + 2)¡ et(2e2t)

(e2t + 2)2=¡e3t + 2et(e2t + 2)2

f 00(t) =(¡3e3t + 2et)(e2t + 2)2 + (e3t ¡ 2et)2 ¢ (e2t + 2)(2e2t)

e2t + 2)4

=(¡3e3t + 2et)(e2t + 2) + 4e2t(e3t ¡ 2et)

e2t + 2)3

=e5t ¡ 12e3t + 4et(e2t + 2)3

=et(e4t ¡ 12e2t + 4)

(e2t + 2)3

et > 0 이므로 e4t ¡ 12e2t + 4 < 0 을 풀면 6¡ 2p8 < e2t < 6 + 2p8, 즉

2¡p2 < et < 2 +

p2

이 구간에서는 Jensen 부등식에 의해 성립한다①. 일계도함수의 해를 구해보면 et =p2 에서

극값을 가지므로,

² 0 < et · 2¡p2 에서 최대값 = f(ln(2¡p2)) = 1

4.

² et ¸ 2 +p2 에서 최대값 = f(ln(2 +p2)) =

1

4.

² et =p2 일 때 최대값 = f(ln

p2) =

p2

4.

따라서, Jensen 구간에 속하지 않는 et가 있을 경우에는

a

a2 + 2+

b

b2 + 2+

c

c2 + 2· 2 ¢

p2

4+1

4=2p2 + 1

4< 1 ¢ ¢ ¢ ②

이 된다. ①, ②에 의해 원 부등식이 증명되었다. ¤

주 2003년 KMO 1차시험에 다음과 같은 문제가 있었다.

참고: KMO 2003년 1차시험 고등부 17번참고: KMO 2003년 1차시험 고등부 17번

양수 a, b, c에 대하여 식a

7a+ b+

b

7b+ c+

c

7c+ a

이 취할 수 있는 값의 범위를 구하여라.

Page 269: Baltic Way 팀수학경시대회 1990-2005 pdf 보기

2005년 Baltic Way 풀이 269

풀이1 (KAIST 03학번 김린기)b

a= x,

c

b= y,

a

c= z 로 치환해주면 식이 간단해진다. xyz = 1, x; y; z > 0 의 제한식에서 범위

가 다음과 같음을 보이자.1

7<

1

7 + x+

1

7 + y+

1

7 + z· 3

8

(오른쪽 부등식) 양변에 8(7 + x)(7 + y)(7 + z)를 곱하고 정리하면

3 ¢ 8 ¢ 72 + 2 ¢ 8 ¢ 7(x+ y + z) + 8(yz + zx+ xy)

· 3 ¢ 73 + 3 ¢ 72(x+ y + z) + 3 ¢ 7(yz + zx+ xy) + 3xyz(= 1)

다시 정리하면 다음의 식과 동치가 된다.

144 · 35(x+ y + z) + 13(yz + zx+ xy)

산술-기하평균 부등식에서

x+ y + z ¸ 3 3pxyz = 3; yz + zx+ xy ¸ 3 3p

x2y2z2 = 3

이므로 이로부터 위 부등식이 성립함을 확인할 수 있고, 등호는 x = y = z(= 1) 일 때, 즉

a = b = c 일 때 성립한다.

(왼쪽 부등식) 일반성을 잃지 않고 x ¸ y ¸ z 라 하자. 그럼 x ¸ 1, z · 1 임은 당연하다. 만일

y · 1 이면1

7 + y+

1

7 + z¸ 1

8+1

8=1

4

이므로 OK. y > 1 이면 x > 1, zx < 1 이기도 하므로

1

7 + x+

1

7 + z>

1

7 + x+

x

7x+ zx>

1

1 + 7x+

x

7x+ 1=1 + x

1 + 7x>1

7

로 항상 부등식은 성립한다. 또한, x; y ! 1 이면 z ! 0 이 되고, 이 때 준식은1

7로 수렴한다

는 것도 알 수 있다. }

풀이2 (오른쪽부등식)준식을 A라 하고, x = eX , y = eY , z = eZ 로 다시 치환하자. xyz = 1

이므로 X + Y + Z = 0 이 된다. 이제

F (T ) =1

7 + eT

라 하면 오른쪽 부등식은

A

3=

F (X) + F (Y ) + F (Z)

3· F

µX + Y + Z

3

¶(¤)

의 Jensen 부등식 꼴이 됨을 볼 수 있다. F를 미분하면

F 0(T ) =¡(7 + eT )0

(7 + eT )2=

¡eT(7 + eT )2

F 00(T ) =(¡eT )0(7 + eT )2 ¡ (¡eT )[(7 + eT )2]0

(7 + eT )4=¡eT (7 + eT ) + eT ¢ 2eT

(7 + eT )3

=eT (eT ¡ 7)(7 + eT )3

Page 270: Baltic Way 팀수학경시대회 1990-2005 pdf 보기

270

따라서, eT < 7 이면 F 00(T ) < 0 이고, 그 범위에서 위로 볼록한 함수이다. 즉, x; y; z < 7 이면

(¤)의 Jensen 부등식에 의해 오른쪽 부등식이 성립한다. 그렇지 않은 경우라면, 일반성을 잃지

않고 x ¸ 7 이라 하면,

A =1

7 + x+

1

7 + y+

1

7 + z· 1

14+1

7+1

7=5

14<3

8

으로 역시 성립한다.

(왼쪽 부등식) 먼저 x; y !1 이면 A! 17 임을 알 수 있다. 귀류법으로 A · 1

7 인 경우가 있다

고 가정해보자. 일반성을 잃지 않고 z가 제일 작다고 하자. 그럼 z · 70.1

7 + x+

1

7 + y= A¡ 1

7 + z· 1

7¡ 1

7 + z· 1

7¡ 1

7 + 1=

1

7 + 72

이므로, x도 y도 72보다 작아서는 안 된다. 그럼 z =1

xy· 1

74이어야 한다. 이 방법을 확장하

자.1

72n+1< z · 1

72n(¤¤)

인 자연수 n이 존재한다.1

7¡ 1

7 + 1a

=1

7 + 72a를 이용하면

1

7 + x;

1

7 + y· 1

7¡ 1

7 + z· 1

7¡ 1

7 + 172n

=1

7 + 72n+2

따라서, x; y ¸ 72n+2 ¸ 72n 이어야 하고, 그럼 z =1

xy· 1

72n+1이어야 한다. 이것은 (¤¤)에 모

순. 따라서, 가정 A · 17 이 틀렸고, 항상 A > 1

7 이다. }

풀이3 (대전 대덕중 3학년 이태희)

오른쪽 부등식만 확인하자. A < 17 +

17 +

17 =

37 은 쉬우므로 B = 3

7 ¡A(> 0) 로 두자.

7B = 3¡ 7A =µ1¡ 7a

7a+ b

¶+ ¢ ¢ ¢ = b

7a+ b+

c

7b+ c+

a

7c+ a

이 된다. 이제 B ¸ 356 을 보이면 되겠다. Cauchy 부등식으로(분모를 이항하여 생각)

7B ¸ (a+ b+ c)2

(7a+ b)b+ (7b+ c)c+ (7c+ a)a

=a2 + b2 + c2 + 2bc+ 2ca+ 2ab

x2 + y2 + z2 + 7yz + 7zx+ 7xy

=3

8+5

8¢ a2 + b2 + c2 ¡ bc¡ ca¡ ab

a2 + b2 + c2 + 7bc+ 7ca+ 7ab¸ 3

8

따라서, A = 37¡B · 3

7¡ 356= 38 이 성립한다. }

Baltic 2005-6Baltic 2005-6

K와 N은 1 · K · N 인 양의 정수들이다. 서로 다른 N장의 카드 묶음을 섞는데, 묶음의 제일 위

에 있는 K장의 카드를 들어낸 후 그 K장의 순서를 뒤집어서 원래 묶음의 제일 아래로 옮겨넣는 시

행을 반복하여 섞는다. 이 카드 묶음은 4 ¢ N2=K2 번 이하의 시행으로 다시 맨처음의 순서로 돌아

옴을 증명하여라.

Page 271: Baltic Way 팀수학경시대회 1990-2005 pdf 보기

2005년 Baltic Way 풀이 271

증명 (KAIST 수학과 05학번 이승진)

N = Kq + r (0 · r < K) 라 하고, K장의 카드를 한 층이라 부르자. q번 시행하여 q개의 층을

차례로 내려보내면 아래 그림과 같은 꼴이 된다(그림에서는 q = 4 인 경우).

q번¡!

여기까지에서 1층의 r번째 카드부터 1층의 마지막 카드까지(이 영역을 1층의 하단이라고 부

르자)가 제 위치에서 순서가 뒤바뀌어있음을 알 수 있다. 따라서, q번 더(총 2q번) 시행하면

1층의 하단은 모두 원래의 자기 자리로 돌아온다. 또, 위의 그림에서 1번 더(총 q + 1번) 시

행하면 1층의 상단이 제 위치에서 순서가 뒤바뀌게 됨을 관찰할 수 있다. 즉, q + 1번 더(총

2(q + 1)번) 시행하면 1층의 상단도 모두 원래 자리로 돌아온다. 따라서, 공배수인 2q(q + 1)번

시행하면 1층은 모두 원래 자리로 돌아옴을 알 수 있다. 나머지 층도 마찬가지로 같은 횟수에

원래 자리로 돌아옴을 관찰할 수 있다. 예를 들어, 2층에 대해서 생각하면, 처음에 1번 시행하

면 2층이 1층의 위치로 가고, 거기서 2q(q + 1)번 더 시행하면 다시 그 1층의 위치에 똑같은 순

서로 있다. 거기서 마지막 시행을 되돌리면 2층의 원래 자리에 있게 된다. 즉, 일반적으로 m층

은 (m ¡ 1) + 2q(q + 1) ¡ (m ¡ 1) = 2q(q + 1)번 시행하면 자기 자리에 있음을 알 수 있다.

2q(q + 1) · 4q2 · 4 ¢N2=K2 이므로 문제가 증명되었다. ¤

Baltic 2005-7Baltic 2005-7

각 칸에 0또는 1이 적혀 있는, n행 6열의 숫자배열판이 있다.단, n > 2이다.이 배열판의 각 행은 서

로 다르다. 또, 어떤 두 행이 (x1; x2; x3; x4; x5; x6)과 (y1; y2; y3; y4; y5; y6) 이라면 (x1y1; x2y2; x3y3;

x4y4; x5y5; x6y6)인 행도 항상 존재한다. 절반 이상이 0으로 되어 있는 열이 있음을 증명하여라.

증명 (대구 지산중 2학년 김규완, 과천고 2학년 홍성준)

서로 다른 행이 둘 이상 있으므로 0을 포함하는 행이 반드시 있다.

(1) 0을 딱 1개 포함하는 행이 있을 때: 그 행이 011111이라고 해도 일반성을 잃지 않는다. 그

럼 (1; x2; x3; x4; x5; x6)의 행이 있을 때마다 (0; x2; x3; x4; x5; x6)의 행도 존재한다. 즉, 일

대일(단사) 함수를 만들 수 있으므로, 첫 번째 열이 0인 행이 1인 행보다 많거나 같다.

(2) 0을 딱 2개 포함하는 행이 있을 때: 그 행이 001111이라고 해도 된다. (1)에서와 비슷한 논

리로, 00 ¢ ¢ ¢ 으로 시작하는 행이 11 ¢ ¢ ¢ 로 시작하는 행보다 많거나 같다. 즉, 처음 두 수

가 ij인 행의 개수를 nij라고 하면 n00 ¸ n11 이다. 또, n01 ¸ n10 이라 하자(반대의 경우

는 처음 두 열을 바꾸면 똑같음). 그럼 n00 + n01 ¸ n10 + n11 이고, 이것은 첫 번째 열에

0이 1보다 많거나 같음을 뜻한다.

Page 272: Baltic Way 팀수학경시대회 1990-2005 pdf 보기

272

(3) 이제 111111을 제외한 모든 행이 0을 3개 이상 포함할 때만 살피면 된다. 먼저 각 행을 살

피면 111111을 제외하면 모든 행이 0을 1의 개수 이상 포함하므로 a · b + 6 이다. 단, a,

b는 각각 전체 1의 개수와 0의 개수. 이번엔 각 열을 살피면(귀류법으로 절반 이상이 0인

열이 없다고 할 때) 모든 열에서 1의 개수가 0의 개수보다 최소 1개 더 많으므로 a ¸ b+6

이다. 따라서, 등호가 성립해야 하고, 그럼 111111을 제외한 모든 행에 0이 정확히 3개씩

있다. 문제에서 n ¸ 3 이라 했으므로 0을 3개 포함하는 서로 다른 두 행이 있고, 그럼 그

두 행의 곱도 있어야 하는데 그것은 0을 4개 이상 포함하므로 모순이다.

(1){(3)에 의해 절반 이상이 0인 열이 반드시 있다. ¤

Baltic 2005-8Baltic 2005-8

25£ 25 개의 단위정사각형칸으로 된 격자판이 있다. 이 격자판의 격자선을 따라 적당한 크기의 정

사각형을 빨간 펜으로 덧그리자. 모든 격자선들이 빨간 색으로 덧칠되기 위해 그려야 하는 빨간 정

사각형은 최소 몇 개인가?

풀이 (서울 당산서중 1학년 박민재, 조금 수정)

먼저 정사각형은 최소 48개가 필요함을 보이자. 격자판의 둘레를 제외한 나머지 24개의 수평선

과 24개의 수직선을 생각하자. 그런데 둘레를 제외한 수평선이나 수직선들은 한 개의 정사각형

으로 한 개가 완전히 빨간색으로 칠해질 수 없다. 즉 수평선이나 수직선 하나 당 각각 양끝 부분

을 포함하는 최소한 2개의 정사각형이 그려져야 한다. 그리고, 정사각형을 어떻게 그려도 둘레

를 제외한 각 수평선의 양끝 부분을 최대 2개 포함한다.결국 정사각형은 최소한(24+24)£2

2= 48

개가 필요하다.

이제 정사각형 48개로 격자판의 모든 격자선을 칠할 수 있음을 보이자. 먼저 격자판의 한 대각

선을 생각하자. 이 대각선 위에는 맨 끝의 점 두 개를 제외하고 24개의 점이 있다. 24개의 점마

다 각 점에서 가장 먼 맨 끝의 점, 이렇게 두 점을 포함하는 정사각형을 만들자. 같은 방법으로

두 번째 대각선에서도 24개의 정사각형을 만들면 총 48개의 정사각형을 그렸고, 이 정사각형들

이 25£ 25 격자판의 모든 격자선들을 빨간색으로 덧칠함은 자명하다. }

Baltic 2005-9Baltic 2005-9

어떤 직사각형을 200 £ 3 개의 단위정사각형칸으로 구분하였다. 이 직사각형을 1 £ 2 크기의 작은

직사각형으로 쪼개는 방법의 수는 3으로 나누어떨어짐을 증명하여라.

증명 (서울 중동고 1학년 이준석)

겨울캠프에서 지수함수에 주어진 점화식(문제는 n = 100)을 연립하면

an+1 = 4an ¡ an¡1

이 된다. 여기서

an+1 ´ an ¡ an¡1 (mod 3)

Page 273: Baltic Way 팀수학경시대회 1990-2005 pdf 보기

2005년 Baltic Way 풀이 273

이다. 이로부터 조금 전개하면

an+1 ´ an ¡ an¡1 (mod 3)

´ (an¡1 ¡ an¡2)¡ an¡1 (mod 3)

´ ¡an¡2 (mod 3)

´ an¡5 (mod 3)

를 얻는다. 따라서

an ´ an¡6 (mod 3)

임을 얻고 a100 ´ a4 (mod 4) 이다. 그런데 a4 = 153 으로 3의 배수이다. 따라서 a100은 3의 배

수가 된다. ¤

참고: KAIST Cyber영재교육 2005년 겨울캠프 교재 중에서참고: KAIST Cyber영재교육 2005년 겨울캠프 교재 중에서

다음 그림과 같은 (2n)£ 3 의 크기를 갖는 놀이판은 2£ 1 의 도미노를 3n개 사용하면 겹치지 않게

완전히 깔 수 있다.

그럼 그렇게 완전히 까는 방법은 몇 가지나 될까? 단, 놀이판은 그 테두리가 서로 다른 색으로 칠이

되어 있거나 해서 대칭적이지 않은 것으로 본다. 즉, 180± 빙 돌려서 같은 모양이면 하나로 본다거

나 하지 않고, 그냥 지금 위치 그대로 고정된 채로만 방법을 따지는 것으로 한다.

풀이 이 방법의 수를 an이라 하자. 이 놀이판의 맨 왼쪽 윗칸에 `H' 표시를 하고,

그 칸을 도미노로 어떻게 까느냐에 따라 경우를 나누기로 하자. 그럼 다음과 같은 경우의 나

HMB

Page 274: Baltic Way 팀수학경시대회 1990-2005 pdf 보기

274

무(tree, 수형도) 그림이 나온다.

즉, H칸을 세로로 깔 때에는 맨 아래의 B칸이 가로로 깔릴 수 밖에 없고, H칸을 가로로 깔 때

에는 그 밑의 M칸이 역시 가로로 깔리느냐 혹은 세로로 깔리느냐에 따라 위의 나무와 같이 경

우가 더 나뉘게 된다. 각각의 경우에 해당하는 경우의 수를 그림에서처럼 pn, qn, rn이라고 하

면, an = pn + qn + rn (¤) 으로 계산하면 된다. qn은 an¡1과 같고, 만일 pn = rn도 ak들로 세어

질 수 있도록 경우를 잘 따질 수 있으면, 우리는 (¤)에서 an의 점화식을 얻을 수 있게 된다.

꼼꼼히 잘 생각하며 경우를 따지면 pn을 ak꼴들과 연관시킬 수도 있다. 그러나 여기서는 다음

과 같이 (2n)£ 3에 두 칸을 더 덧붙인 놀이판을 도입하여,

이 놀이판에 도미노로 완전히 까는 방법의 수를 bn이라 하고, an과 bn 사이의 연립점화식을 찾

아보기로 하자. 먼저 앞에서의 pn과 rn이 bn¡1과 같으므로, an = an¡1+2bn¡1 의 식이 하나 만

들어졌다.

bn에 대한 경우의 나무도 마찬가지로 찾아보면,

와 같이 되어, bn = an + bn¡1 의 식이 또 만들어졌다. 지금까지 우리는 연립점화식

a1 = 3; an = an¡1 + 2bn¡1 (n ¸ 2)b0 = 1; bn = an + bn¡1 (n ¸ 1)

MB

rn

qn

pn

an

bn-1

Page 275: Baltic Way 팀수학경시대회 1990-2005 pdf 보기

2005년 Baltic Way 풀이 275

을 얻었다. 여기서 bk의 항들이 모두 소거될 수 있도록 잘 변형하여 ak의 항들만 남기면 an에

관한 점화식이 나올 것이다. (후략) }

Baltic 2005-10Baltic 2005-10

m = 30030 = 2 ¢ 3 ¢ 5 ¢ 7 ¢ 11 ¢ 13 이고, M은 m의 양의 약수들 중 딱 2개의 소인수를 갖는 것들의

집합이다. 다음 성질을 갖는 최소의 정수 n을 구하여라: M에서 n개의 수를 어떻게 택해도 그 중

abc = m 이 되는 세 수 a, b, c가 존재한다.

풀이 (부천 상도중 2학년 김성민)

S(M) =¡62

¢= 15개.

먼저, 6개의 수 중 1개를 제외한 5개에서 모두 다 뽑는 경우의 수:¡52

¢= 10. ) n ¸ 11.

n이 11일 때, M의 원소들을 다음과 같이 5개의 집합으로 나누자.

(2 ¢ 3; 5 ¢ 7; 11 ¢ 13), (3 ¢ 5; 7 ¢ 11; 13 ¢ 2), (2 ¢ 5; 3 ¢ 11; 7 ¢ 13),(3 ¢ 7; 2 ¢ 11; 5 ¢ 13), (2 ¢ 7; 3 ¢ 13; 5 ¢ 11)

여기서 11개를 뽑으면 비둘기집의 원리에 의해 적어도 한 집합에서는 3개의 수가 모두 뽑히게

된다. 그 세 수를 곱하면 m이 되므로 조건을 만족하는 세 수가 존재하게 된다. }

Baltic 2005-11Baltic 2005-11

삼각형 ABC에서 변 BC와 AC에서 각각 점 D와 E를 잡아 BD = AE 가 되도록 하자. 두 삼각형

ADC와 BEC의 외심을 잇는 직선이 두 직선 AC, BC와 각각 K, L에서 만난다. KC = LC 임을

증명하여라.

증명 (서울 양정중 1학년 고건)

두 삼각형 ADC와 BEC의 외심을 각각 F , G라 하고, BC, CD, AC, EC의 중점을 각각 M ,

N , O, P라 하자.

그림에서 C로부터의 2배 확대/축소를 생각하면 MN = 12BD, OP = 1

2AE. 문제에서 BD =

AE 이므로 MN = OP 이다. MNFQ, POFR이 직사각형이 되도록 Q, R을 잡자. 그러면,

Page 276: Baltic Way 팀수학경시대회 1990-2005 pdf 보기

276

\FQG = \FRG = \R, FG는 공통, QF = FR 이므로(RHS합동)

4GQF ´ 4FGR

이로부터

\KLC = \MLG = \QFG = \GFR = \LKC

따라서, 4KLC는 이등변 삼각형이다. 그러므로, KC = LC. ¤

Baltic 2005-12Baltic 2005-12

ABCD는 BC = AD 인 볼록사각형이다. AB와 CD의 중점을 각각 M과 N이라 하자. 두 직선

AD와 BC는 직선 MN과 각각 P와 Q에서 만난다. CQ = DP 임을 증명하여라.

증명1 MN을 x-축으로 놓고 AD와 BC의 x-폭과 y-폭을 생각하자. M과 N이 중점임에서 두

선분의 y-폭이 같음을 쉽게 알 수 있고, 문제에서 두 선분의 길이가 같다고 했으므로, 그럼 두

선분의 x-폭도 같다(RHS합동). 따라서, 두 선분 AD와 BC는 직선 MN으로부터 같은 폭으로

떨어져있고 기울기도 (부호를 무시하면) 같으므로, 두 선분을 연장했을 때 MN에 이르는 거리

인 CQ와 DP도 같다. ¤

증명2 (부천 상도중 2학년 김성민)

4QCN와 4PND를 N을 중심으로 180± 회전시킨다. 그리고, P , Q, A, B와 대응하는 점을 각

각 A0, B0, P 0, Q0이라 하자.

그러면 사각형 ABA0B0이 평행사변형이 되고, QQ0 k A0B k B0A 이 된다. 따라서,

\CA0B = \CP 0Q; \CBA0 = \CQP 0

그런데4CA0B가이등변삼각형이므로(* BC = AD = CA0) \CA0B = \CBA0,즉 \CP 0Q =

\CQP 0. 따라서, CQ = CP 0 = DP 이다. ¤

Page 277: Baltic Way 팀수학경시대회 1990-2005 pdf 보기

2005년 Baltic Way 풀이 277

증명3 (서울 정곡초 6학년 이수홍)

AC의 중점을 O라 하고, C, D에서 MN에 내린 수선의 발을 각각 H1, H2라 하자.

중점연결정리에의해BC kMO. 따라서 \CQH1 = \OMN . 마찬가지로 \DPH2 = \ONM .

그런데 MO = 12BC = 1

2AD = NO 이므로 4OMN은 이등변삼각형이고, 따라서

\CQH1 = \DPH2 ¢ ¢ ¢ ①

그리고 CH1 k DH2, CN = ND 이므로 4CNH1 ´ 4DNH2 가 되어서

CH1 = DH2 ¢ ¢ ¢ ②

가 된다. ①, ②에 의해 4CQH1 ´ 4DPH2 가 되어서 CQ = DP . ¤

증명4 (성남 은행중 2학년 최한음)

DA와 CB의 교점을 F라 하고 E를 AC의 중점이라 하자. 이 때, 중점연결정리에 의해 ME =

12BC 이고, EN = 1

2AD 이다. 이 때 AD = BC 이므로 ME = EN 이다. ME k CQ 이므

로 \NME = \PQF 이고, EN k AD 이므로 \DPN = \ENM 인데, ME = EN 이므로

\ENM = \EMN 이 되어

\FPQ = \DPN = \FQP

이다. 즉, FP = FQ 이다. 4DFC와 점 N , P , Q에서 메넬라우스 정리를 사용하면

DN £ CQ£ FP

NC £ FQ£ PD= 1

이다. 여기서 DN = NC, FP = FQ 이므로, CQ = PD 이다. ¤

증명5 (한국과학영재학교 1학년 박지현)

Page 278: Baltic Way 팀수학경시대회 1990-2005 pdf 보기

278

ABCD와 합동인 사각형 A0B0C0D0을 점 B = A0, C = D0 이 되도록 붙인다.

직선 MN과 M 0N 0의 교점을 E라 하면 \EMB + \EM 0B = 180± 에서 사각형 EMBM 0 은

원의 내접사각형이다. 즉 \MEM 0의 이등분선은 호 MM 0을 이등분하므로 점 B를 지난다(*jMBj = jM 0Bj). 사각형 ENCN 0에 대해서도 마찬가지로 하면 \NEN 0의 이등분선은 점 C를

지난다. 따라서 세 점 B, C, E는 한 직선 위에 있으므로 E와 Q는 같은 점이고,

jDP j = jD0Ej = jCEj = jCQj

임을 알 수 있다. ¤

주 좌표로 놓고 풀어도 된다.

Baltic 2005-13Baltic 2005-13

다음 크기의 직사각형을 완전히 덮기 위해 필요한 반지름p2의 원판은 최소 몇 개인가?

(a) 6£ 3 크기의 직사각형

(b) 5£ 3 크기의 직사각형

풀이 (서울 당산서중 1학년 박민재)

(a) 일단 아래와 같이 점 A, B, C, D, E, F를 잡자.

Page 279: Baltic Way 팀수학경시대회 1990-2005 pdf 보기

2005년 Baltic Way 풀이 279

그러면 이 6개의 점들은 거리가 각각 3 이상이 된다. 그런데 원의 반지름이p2 ; 1:4 이므로 지

름은 3보다 작고 결국 저 6개의 점 중 2개를 포함하는 원은 존재하지 않는다. 즉 원은 최소한

6개 이상 필요하다. 6개로 덮을 수 있다는 것은 3 £ 6 직사각형을 아래 그림과 같이 원에 완전

히 들어가는 6개의 직사각형으로 분할하면 된다.

즉 3£ 6 직사각형은 최소 6개의 원판이 필요하다.

(b) 비슷한 방법으로 아래와 같이 점 A, B, C, D, E를 잡자. 단 여기서 E는 직사각형의 중심이

다.

그러면 이 5개의 점들은 거리가 각각 2p2보다 크다(AE =

q344

>q324=p8 = 2

p2). 즉 원

하나가 저 5개의 점 중 2개를 포함할 수 없고 필요한 원판은 최소 5개이다. 5개로 덮을 수 있다

는 것은 3£ 5 직사각형을 아래 그림과 같이 원에 완전히 들어가는 5개의 직사각형으로 분할하

Page 280: Baltic Way 팀수학경시대회 1990-2005 pdf 보기

280

면 된다.

즉 3£ 5 직사각형은 최소 5개의 원판이 필요하다. }

Baltic 2005-14Baltic 2005-14

삼각형 ABC의 중선들이 M에서 만난다. D와 E는 DC = CE = AB 를 만족하는 직선 BC 위의

서로 다른 두 점이고, P와 Q는 각각 선분 BD와 BE 위의 점으로, 2BP = PD 와 2BQ = QE 를

만족한다. \PMQ 의 크기를 구하여라.

풀이 (대전 충남고 1학년 한규형)

A에서 BC와 평행, C에서 AB와 평행하게 보조선을 그어 만나는 점을 B0이라 하자.

즉, ¤AB0CB는 평행사변형이다. 그리고, AB = B0C = CE = DC 이다. R을 AC의 중점이라

하자. M은 무게중심이므로

BM :MB0 =2

3BR :

1

3BR+BR = 1 : 2

BP : PD = BQ : QE = 1 : 2 이므로 4BPM » 4BDB0, 4BQM » 4BEB0, 즉

PM k DB0; QM k EB0

이로부터 \PMQ = \DB0E = \R 이다. }

Page 281: Baltic Way 팀수학경시대회 1990-2005 pdf 보기

2005년 Baltic Way 풀이 281

별해 (제주과학고 1학년 강진호)

좌표로 풀자. A(x; y), AB = C, C(a; 0)이라 할 때 P³a¡ c

3; 0´, Q³a+ c

2; 0´가 된다. M은 무

게중심이므로 M³x+ a

3;y

3

´. 따라서,

MP 2 =³x+ c

3

´2+³y3

´2MQ2 =

³x¡ c

3

´2+³y3

´2변변 합하면

MP 2 +MQ2 =2

9(x2 + y2 + c2)

=4

9c2 (* x2 + y2 = c2)

=³a+ c

3¡ a¡ c

3

´2= PQ2

따라서, 피타고라스의 정리에 의해 \PMQ = 90±. }

Baltic 2005-15Baltic 2005-15

두 직선 e와 f가 점 H에서 수직으로 만난다. A와 B는 e 위의 점, C와 D는 f 위의 점이고, A, B,

C, D, H는 모두 서로 다른 점이다. 각각 점 B와 D를 지나고 AC에 수직인 두 직선 b와 d, 그리고

각각 점 A와 C를 지나고 BD에 수직인 두 직선 a와 c를 생각하자. a와 b의 교점을 X, c와 d의 교점

을 Y라 하자. XY가 H를 지남을 증명하여라.

증명 (서울 정곡초 6학년 이수홍)

e를 y-축, f를 x-축, H를 원점으로 놓고 A = (0; a), B = (0; b), C = (c; 0), D = (d; 0) 으로 하

자. 그러면

b : y =c

ax+ b

d : y =c

a(x¡ d)

a : y =d

bx+ a

c : y =d

b(x¡ c)

이다. 그러면

X =

µa¡ bca ¡ a

b

;c¡ aca ¡ d

b

¶; Y =

µ cdab(b¡ a)ca ¡ d

b

;cdab(d¡ c)ca ¡ d

b

¶따라서 XY의 기울기는

c¡ d¡ cdab (d¡ c)

a¡ b¡ cdab(b¡ a)

=(c¡ d)(1 + cd

ab )

(a¡ b)(1 + cdab)=

c¡ d

a¡ b

이다. 그러므로 XY의 방정식은

y =c¡ d

a¡ b

µx¡ a¡ b

ca ¡ d

b

¶+

c¡ dca ¡ d

b

=c¡ d

a¡ bx

가 되어서 (0; 0), 즉 H를 지난다. ¤

Page 282: Baltic Way 팀수학경시대회 1990-2005 pdf 보기

282

Baltic 2005-16Baltic 2005-16

p는 소수이고 n은 양의 정수, 그리고 q는 (n+1)p ¡ np 의 한 양의 약수이다. q¡ 1 이 p로 나누어떨

어짐을 보여라.

증명 (서울 당산서중 1학년 박민재)

q가 소수(prime number)인 경우만을 생각하자. 왜냐하면 만약 q가 합성수라면 그것은 (n +

1)p ¡ np 의 약수인 소인수들을 갖는다. 그 소인수들이 모두 각각 p로 나눈 나머지가 1이라면

그것들을 다 곱한 q 역시 p로 나눈 나머지가 1이므로 q ¡ 1은 p로 나누어지기 때문이다. 이제

q가 소수인 경우에 대해서 q ¡ 1이 p로 나누어 떨어짐을 보이면 된다.

q가 (n+ 1)p ¡ np 의 약수이므로 (n+ 1)p ¡ np ´ 0 (mod q), 즉

(n+ 1)p ´ np (mod q)

이다. 여기서 n+1과 q, 그리고 n과 q는 서로 소라고 할 수 있는데, 왜냐하면 만약 q가 둘 중 하

나를 나눈다면 나머지 하나는 나눌 수 없고(n+ 1과 n은 서로 소이므로) 그러면 (n+ 1)p ¡ np

이 q의 배수가 될 수 없기 때문이다. 이제 p의 기약잉여계의 한 원소 i에 대해서

(n+ 1)i ´ ni (mod q) (1)

가 성립한다고 가정하자. i가 p의 기약잉여계의 원소이므로 ki ´ ¡1 (mod p) 인 p와 서로 소인

어떤 양의 정수 k가 존재한다. 즉,

mp¡ ki = 1

을 만족하는 양의 정수 m, k가 존재한다.

(n+ 1)mp ´ nmp (mod q)

(n+ 1)ki ´ nki (mod q)

이고, n+ 1과 n은 q와 서로 소이므로, 변변 나눠주면

n+ 1 ´ n (mod q)

그런데 이것은 q = 1 이어야 하므로 모순이다. 즉, (1)을 만족하는 i는 p의 기약잉여계의 원소

일 수 없다. 그러므로 (1)을 만족하는 i는 p의 배수여야 한다. 그런데 q가 소수이고 n+ 1과 n은

q와 서로 소이므로 Fermat의 작은 정리에 의해서

(n+ 1)q¡1 ´ nq¡1 ´ 1 (mod q)

가 성립한다. 결국 위에서 증명한대로 q ¡ 1은 p의 배수여야 한다. ¤

별증 (서울 정곡초 6학년 이수홍)

(n+ 1)p ¡ np 의 모든 소인수가 pk + 1꼴임을 보이겠다. 어떤 소인수 q가 있다면

(n+ 1)p ´ np (mod q)

Page 283: Baltic Way 팀수학경시대회 1990-2005 pdf 보기

2005년 Baltic Way 풀이 283

양변에 이산로그를 씌우면

p ¢ indy(n+ 1) ´ p ¢ indy(n) (mod q ¡ 1)

그런데 gcd(p; q ¡ 1) = 1 이면

indy(n+ 1) ´ indy(n) (mod q ¡ 1)n+ 1 ´ n (mod q)

즉, q j 1 이 되어 모순. 따라서 gcd(p; q ¡ 1) = p 이고, q = pk + 1꼴이다. 따라서 (n+ 1)p ¡ np

의 모든 약수는 pk + 1꼴이 되어서 p j q ¡ 1 이다. ¤

Baltic 2005-17Baltic 2005-17

수열 fxngn¸0 이 다음과 같이 정의된다: x0 = a, x1 = 2 이고, n > 1 에 대해 xn = 2xn¡1xn¡2 ¡xn¡1 ¡ xn¡2 + 1 이다. 모든 n ¸ 1 에 대해 2x3n ¡ 1 이 항상 완전제곱수가 되는 정수 a를 모두 찾

아라.

풀이 (성남 은행중 2학년 최한음)

1보다 큰 임의의 자연수 c에 대해 2xc ¡ 1 = 4xc¡1xc¡2 ¡ 2xc¡1 ¡ 2xc¡2 + 1 이므로,

2xc ¡ 1 = (2xc¡1 ¡ 1)(2xc¡2 ¡ 1)

으로 나타낼 수 있다. yk = 2xk ¡ 1 으로 치환하면

yc = yc¡1yc¡2

이 성립한다. 이제 1보다 큰 임의의 b에 대해

y3b = y3b¡1y3b¡2 = y23b¡2y3b¡3

이므로 y3b가 완전제곱수이기 위한 필요충분조건은 y3b¡3이 완전제곱수인 것이다. 즉 귀납적으

로, y3b가 완전제곱수이기 위한 필요충분조건은 y3이 완전제곱수인 것이다.

y3 = 18a¡ 9 = 9(2a¡ 1)

임은 계산을 통해 쉽게 알 수 있고, 9도 완전제곱수이므로 2a ¡ 1 = (2n¡ 1)2꼴이다. 즉, 임의

의 자연수 n에 대해 a = 2n2 ¡ 2n+ 1꼴의 수이다. }

Baltic 2005-18Baltic 2005-18

x와 y는 양의 정수이고 z = 4xy=(x + y) 는 홀수라 하자. z는 4n ¡ 1꼴의 약수를 가짐을 증명하여

라. 단, n은 양의 정수이다.

Page 284: Baltic Way 팀수학경시대회 1990-2005 pdf 보기

284

증명 (과천고 1학년 홍성준)

4xy = xz + yz 의 양변에 4를 곱하여 다음과 같이 정리할 수 있다.

(4x¡ z)(4y ¡ z) = z2

z가 홀수이므로 4n§ 1꼴만 약수로 가진다. z가 4n+ 1꼴의 인수로만 구성된다면 z2 또한 그러

하다. 그러나 이 때, 4x ¡ z, 4y ¡ z는 4n ¡ 1꼴의 수로서 각각 4n + 1꼴의 곱만으로는 표현불

가(모순). 따라서, z는 4n¡ 1꼴의 약수를 갖는다. ¤

주 불가리아 1981년 4차시험에도 출제되었던 문제.

Baltic 2005-19Baltic 2005-19

2005개의 서로 다른 양의 완전제곱수들의 합이 다시 완전제곱수가 되는 경우를 찾을 수 있는가?

풀이 (서울 경희중 3학년 이승후)

1 = a0 < a1 < ¢ ¢ ¢ < a2003 이고, a1; : : : ; a2003은 모두 짝수라 하자. a20 + a21 + a22 + ¢ ¢ ¢+ a22003 =

1 + 2A 라 하면, a2003 < A 이므로 a20; a21; : : : ; a

22003; A

2은 모두 서로 다른 제곱수이다. 그리고

이들을 다 더하면

1 + 2A+A2 = (A+ 1)2

즉, 서로 다른 2005개의 제곱수의 합으로 다른 제곱수를 만들었다. }

Baltic 2005-20Baltic 2005-20

n = p1p2 ¢ ¢ ¢ pk 가 (p1 + 1)(p2 + 1) ¢ ¢ ¢ (pk + 1) 을 나누는 양의 정수 n을 모두 구하여라. 단,

p1p2 ¢ ¢ ¢ pk는 n을 소수들의 곱으로 나타낸 것이고, 이 소수들은 서로 다를 필요는 없다.

풀이 (제주 제일중 3학년 오재성)

p1; p2; p3; : : : ; pk 중 가장 큰 소인수를 pn이라 하자. (p1 + 1); (p2 + 1); (p3 + 1); : : : ; (pk + 1) 중

에서 pn의 배수가 있어야 한다. 여기서 pn + 1 은 pn과 서로 소이므로 pn + 1 을 제외한 수 중

에서 pn의 배수가 있어야 한다. 그런데 여기서 pn은 가장 큰 소수이므로 px + 1이 pn의 배수라

면 px +1 = pn 이 되어야 한다. 따라서 pn = 3이어야만 한다(그리고 px = 2). 이제 n = 2a3b꼴

만 생각하면 된다.

2a3b j 3a4b

이어야 하므로, b · a · 2b 이면 된다. 즉, 구하는 n은 2x+y3x꼴의 수들이다(x, y는 y · x 인 음

아닌 정수). }

Page 285: Baltic Way 팀수학경시대회 1990-2005 pdf 보기

제 III 부

부록

Page 286: Baltic Way 팀수학경시대회 1990-2005 pdf 보기
Page 287: Baltic Way 팀수학경시대회 1990-2005 pdf 보기

대회결과 287

대회결과

1990{1991년의 대회결과는 확인하지 못했다.

1992년 Baltic Way 대회결과

수론 대수 조합 기하 계

1 덴마크 5 5 5 5 5 5 5 4 3 5 5 5 4 5 5 5 1 5 82

2 상트 페테르부르크 5 5 4 3 5 5 5 5 3 3 5 5 5 5 5 5 5 78

3 폴란드 5 5 5 5 5 5 5 5 4 4 5 3 1 5 2 5 5 73

4 라트비아 5 5 2 5 5 5 5 5 5 5 5 5 4 5 5 71

5 아이슬란드 5 0 5 5 5 5 3 1 1 5 5 4 1 0 5 5 55

6 리투아니아 5 5 5 1 5 5 5 5 1 1 2 5 5 50

7 스웨덴 5 5 5 5 5 5 1 5 5 5 46

7 에스토니아 2 5 5 0 5 5 5 5 3 5 1 5 46

1993년 Baltic Way 대회결과

수론 대수 조합 기하 계

1 폴란드 5 5 5 5 5 5 3 0 5 4 2 0 0 0 5 1 0 5 0 5 60

2 라트비아 4 0 5 3 5 5 4 0 5 5 5 0 0 0 5 4 5 0 3 0 58

3 에스토니아 5 0 5 5 5 0 5 0 5 0 5 5 0 1 4 3 5 0 2 1 56

4 스웨덴 4 5 5 5 5 2 3 2 2 0 4 4 0 2 5 4 0 0 1 1 54

5 리투아니아 5 5 5 5 3 2 3 3 5 0 1 0 0 1 5 0 0 5 2 1 51

6 핀란드 3 0 5 4 5 1 5 0 5 0 4 0 0 1 5 4 4 0 2 0 48

7 아이슬란드 5 5 5 4 5 0 3 0 5 0 2 0 0 1 5 0 2 0 0 0 42

8 덴마크 4 0 5 5 5 0 0 3 5 1 5 0 0 1 5 0 0 0 2 0 41

1994년 Baltic Way 대회결과

대수 수론 기하 조합 계

1 상트 페테르부르크 5 5 5 5 5 5 5 5 5 5 3 5 5 5 2 5 5 4 5 5 94

2 라트비아 5 3 5 5 5 5 5 5 4 5 5 5 1 5 3 5 4 5 80

3 폴란드 5 3 5 5 5 5 5 5 5 5 5 5 3 5 3 5 1 75

4 스웨덴 5 3 0 5 4 5 5 5 5 5 5 1 5 5 5 5 1 69

5 덴마크 5 3 4 5 0 5 5 5 1 5 5 5 1 5 4 5 63

6 에스토니아 5 5 1 5 0 4 5 5 5 5 5 5 1 5 0 5 61

7 핀란드 1 3 5 5 0 5 5 5 5 4 5 5 1 3 5 1 58

8 리투아니아 5 3 4 0 5 5 2 4 1 5 0 5 1 40

9 아이슬란드 5 0 0 5 0 1 0 2 0 5 1 0 4 4 1 0 1 29

Page 288: Baltic Way 팀수학경시대회 1990-2005 pdf 보기

288

1995년 Baltic Way 대회결과

수론 대수 조합 기하 계

1 폴란드 5 5 5 5 5 5 5 5 5 4 5 5 5 5 5 2 5 4 5 90

2 라트비아 0 5 5 5 5 5 5 5 2 5 5 5 3 5 5 0 5 5 3 78

3 스웨덴 0 5 5 5 1 5 5 5 0 5 5 5 5 5 5 5 66

4 리투아니아 5 5 4 5 3 5 5 5 0 1 3 3 0 5 5 5 4 2 0 65

5 덴마크 5 5 5 5 5 5 5 4 2 5 3 5 5 4 63

6 핀란드 5 4 5 5 5 5 5 5 5 5 5 5 2 61

7 상트 페테르부르크 0 4 5 5 2 3 5 5 2 3 1 2 5 2 1 45

8 에스토니아 0 5 5 3 5 5 4 4 5 2 1 39

9 아이슬란드 0 1 0 5 2 2 1 5 3 0 2 1 0 0 2 24

1996년 Baltic Way 대회결과

기하 수론 대수 조합 계

1 폴란드 5 4 5 5 5 5 5 5 5 5 5 5 5 5 5 0 5 0 5 5 89

2 라트비아 5 3 5 5 0 5 5 5 5 2 5 5 4 5 3 5 5 0 5 5 82

3 스웨덴 5 4 5 5 5 5 5 2 5 5 5 5 5 5 5 5 5 81

4 덴마크 4 5 5 5 5 5 5 5 5 5 3 5 5 3 5 70

5 상트 페테르부르크 5 5 3 5 5 5 5 2 5 3 4 4 0 5 5 5 0 66

6 핀란드 5 5 5 5 5 5 2 5 2 5 4 5 0 5 5 63

7 노르웨이 5 5 0 1 4 2 5 5 5 3 5 0 2 5 5 5 4 61

8 리투아니아 5 1 5 0 0 4 5 5 2 5 2 5 1 5 3 3 5 0 56

9 에스토니아 5 1 5 1 2 5 5 0 5 2 4 0 5 5 5 0 50

10 아이슬란드 3 5 5 1 0 5 0 2 5 0 0 2 5 5 5 43

1997년 Baltic Way 대회결과

대수 수론 기하 조합 계

1 폴란드 5 5 4 5 5 5 5 5 1 5 5 4 5 5 5 5 5 3 5 87

2 독일 5 5 4 5 5 3 4 2 5 5 5 4 5 1 5 5 0 3 5 76

3 에스토니아 1 5 1 5 5 5 5 3 5 3 4 5 5 5 5 3 1 3 5 74

3 스웨덴 5 5 5 2 5 5 5 0 5 5 4 5 5 5 5 0 3 5 74

5 덴마크 5 5 5 5 5 5 5 5 4 5 0 5 5 5 1 3 5 73

6 라트비아 5 5 3 5 5 5 2 5 4 5 4 0 1 3 5 2 1 1 5 66

7 핀란드 5 5 5 5 5 5 3 5 0 4 0 0 5 5 5 2 5 64

8 노르웨이 2 5 1 5 5 4 5 5 5 5 0 0 0 0 5 4 0 3 5 59

9 상트 페테르부르크 5 0 5 5 4 5 5 4 5 5 5 5 1 2 56

10 아이슬란드 5 5 5 0 3 4 1 5 4 5 5 42

11 리투아니아 1 0 4 5 5 0 2 1 2 5 0 2 2 5 5 39

Page 289: Baltic Way 팀수학경시대회 1990-2005 pdf 보기

대회결과 289

1998년 Baltic Way 대회결과

수론 대수 기하 조합 계

1 라트비아 5 3 5 5 5 5 5 1 5 5 0 3 0 5 5 5 5 5 0 0 72

2 에스토니아 2 5 5 0 5 5 1 5 4 4 4 5 0 5 5 5 5 0 0 5 70

3 폴란드 5 4 5 5 5 4 5 1 5 4 0 5 0 5 5 0 5 0 0 5 68

4 핀란드 5 5 0 5 5 5 5 5 2 5 0 5 0 5 0 5 5 5 0 0 67

5 상트 페테르부르크 0 4 5 5 5 5 1 5 5 5 0 0 0 5 4 5 5 0 0 3 62

6 스웨덴 2 5 5 0 4 5 5 1 1 4 4 0 0 0 1 5 5 1 5 3 56

7 덴마크 2 2 5 0 0 5 1 1 5 5 0 5 0 4 0 5 4 0 0 5 49

7 아이슬란드 4 1 5 5 5 5 5 4 1 0 0 0 0 5 5 0 3 0 1 0 49

7 노르웨이 5 5 1 0 5 5 1 0 0 5 1 0 0 5 1 5 5 0 0 5 49

10 독일 1 0 5 0 0 5 2 3 0 5 0 5 0 5 5 5 0 5 0 2 48

11 리투아니아 4 0 2 0 5 5 0 4 2 0 0 5 0 5 5 0 5 0 0 5 47

1999년 Baltic Way 대회결과

대수 조합 기하 수론 계

1 에스토니아 5 5 5 5 5 5 1 4 5 5 5 5 5 5 5 5 5 5 5 90

2 스웨덴 5 5 5 5 5 5 5 4 5 5 4 3 5 5 5 5 5 0 3 5 89

3 노르웨이 5 5 2 5 5 4 5 5 5 2 5 5 5 4 5 5 5 0 5 5 87

4 폴란드 5 5 5 5 2 5 5 2 5 4 5 5 1 5 5 5 5 5 79

5 핀란드 5 5 5 5 5 3 4 5 3 5 0 5 5 5 4 2 5 5 76

6 라트비아 5 5 1 5 5 5 5 2 5 0 5 1 3 5 5 5 0 5 5 72

7 독일 5 3 0 5 5 5 5 5 5 4 1 5 5 5 0 5 5 68

8 아이슬란드 5 5 2 1 5 3 0 5 5 2 1 5 0 5 5 5 0 5 5 64

9 덴마크 5 5 4 5 5 4 4 3 5 0 5 0 5 1 5 5 61

10 리투아니아 5 5 0 5 4 5 4 5 2 1 0 3 0 5 5 1 5 5 60

2000년 Baltic Way 대회결과

기하 조합 수론 대수 계

1 폴란드 5 5 5 5 5 5 4 5 5 5 5 5 5 5 5 5 0 5 5 0 89

2 라트비아 5 5 5 5 5 5 5 2 5 5 5 5 5 1 4 5 5 0 5 0 82

3 에스토니아 5 5 5 5 5 5 5 2 0 5 5 5 5 0 5 5 5 0 0 0 72

4 독일 5 5 5 5 5 2 1 3 5 5 5 4 5 1 5 1 0 0 5 1 68

4 노르웨이 5 5 5 5 3 3 1 2 5 4 3 5 3 3 5 5 0 5 1 0 68

6 핀란드 5 5 5 5 1 2 1 3 0 5 4 3 5 3 5 0 5 1 5 0 63

7 스웨덴 5 0 0 5 5 3 1 2 0 5 2 4 5 0 5 5 2 0 5 0 54

8 덴마크 0 5 0 5 5 0 5 0 5 5 0 5 0 1 0 0 0 5 5 0 46

9 리투아니아 0 5 4 0 5 3 1 2 0 5 5 0 0 0 0 0 5 5 5 0 45

10 아이슬란드 1 5 1 0 0 4 0 1 0 4 2 2 0 3 0 0 4 0 1 0 28

Page 290: Baltic Way 팀수학경시대회 1990-2005 pdf 보기

290

2001년 Baltic Way 대회결과

조합 기하 대수 수론 계

1 이스라엘 5 5 5 5 5 5 5 5 5 5 5 5 5 5 5 5 5 5 5 5 100

2 에스토니아 4 5 0 5 5 5 5 1 5 5 4 5 5 5 0 5 5 3 5 5 82

3 라트비아 5 5 5 5 5 5 5 1 5 5 2 0 1 5 0 5 5 3 5 5 77

4 노르웨이 5 0 5 5 5 1 5 0 4 5 3 5 0 5 4 5 5 5 5 5 77

5 폴란드 5 5 5 5 5 4 5 4 5 2 5 1 5 5 5 0 0 5 5 76

6 독일 4 5 5 5 5 5 5 1 4 0 2 5 0 5 5 5 3 0 5 5 74

7 리투아니아 5 5 0 5 1 5 5 5 5 5 5 0 5 0 5 5 0 5 2 68

8 덴마크 5 5 0 5 5 0 5 3 1 5 0 5 5 5 0 0 5 5 59

9 핀란드 5 0 5 5 5 0 0 0 3 3 0 0 5 0 5 5 5 5 0 51

10 스웨덴 5 5 5 5 0 0 2 0 1 0 0 5 0 4 5 0 5 5 47

11 아이슬란드 4 5 2 5 1 5 2 1 5 1 0 5 5 0 0 4 1 46

2002년 Baltic Way 대회결과

대수 조합 기하 수론 계

1 상트 페테르부르크 5 5 5 5 5 5 1 5 0 5 5 5 5 5 0 5 5 5 5 5 86

2 노르웨이 4 5 1 4 4 5 5 2 1 5 5 5 5 0 5 0 5 5 0 5 71

3 리투아니아 5 5 5 4 5 5 0 0 5 5 5 1 5 1 0 2 0 5 5 5 68

4 폴란드 5 5 0 5 5 5 0 0 5 4 0 5 5 0 0 4 3 5 5 5 66

5 독일 5 5 1 4 5 3 0 5 0 0 5 5 5 1 5 0 5 2 4 5 65

6 핀란드 5 5 2 5 5 5 5 0 5 5 4 1 1 0 5 1 2 0 0 1 57

7 에스토니아 5 5 3 0 5 4 4 0 5 0 0 4 5 0 2 2 0 4 5 0 53

8 스웨덴 0 5 1 0 5 5 0 5 0 5 1 5 2 0 5 1 4 2 0 1 47

9 라트비아 5 5 2 0 5 5 5 0 0 5 0 0 1 1 2 1 1 4 0 1 43

10 덴마크 5 5 0 1 4 5 0 0 5 5 0 5 0 0 2 0 0 2 0 0 39

11 아이슬란드 5 2 2 0 3 5 2 0 0 2 5 0 1 0 0 3 0 0 0 0 30

2003년 Baltic Way 대회결과

대수 조합 기하 수론 계

1 상트 페테르부르크 5 5 5 5 5 5 5 5 0 5 5 5 5 5 5 5 0 5 5 85

2 폴란드 5 5 5 5 5 0 5 2 4 0 0 5 5 5 5 5 0 5 2 68

3 에스토니아 3 5 5 5 4 0 1 5 5 1 2 5 5 5 5 0 5 0 61

4 라트비아 5 5 5 5 2 5 4 0 2 0 0 5 5 0 5 5 53

5 리투아니아 1 5 3 5 5 0 5 0 1 0 5 5 5 3 0 4 2 49

6 덴마크 5 5 0 0 1 0 1 5 5 0 0 5 5 0 2 0 1 3 5 43

7 노르웨이 1 5 0 0 5 0 3 2 5 1 0 5 0 5 5 0 2 0 39

8 핀란드 1 5 5 5 5 0 1 0 5 0 0 0 5 0 1 5 0 38

9 스웨덴 0 0 5 5 5 0 4 0 0 0 2 1 5 2 0 5 3 37

10 독일 1 0 0 4 2 0 1 1 5 0 1 5 0 0 0 4 5 0 3 0 32

11 아이슬란드 1 0 5 2 1 0 1 0 0 0 0 5 0 0 5 5 0 0 5 1 31

Page 291: Baltic Way 팀수학경시대회 1990-2005 pdf 보기

대회결과 291

2004년 Baltic Way 대회결과

대수 수론 조합 기하 계

1 상트 페테르부르크 5 4 5 4 5 5 5 5 5 5 5 5 5 1 5 5 5 5 5 0 89

2 폴란드 5 4 5 0 5 5 5 3 5 5 0 4 5 1 5 5 5 5 5 5 82

3 벨로루시 5 4 5 5 5 5 5 5 5 5 2 4 5 1 3 5 5 5 0 0 79

4 에스토니아 1 1 5 0 5 5 5 0 5 5 5 5 5 1 5 5 5 5 5 5 78

5 핀란드 0 5 5 5 3 5 2 0 5 4 1 4 5 0 5 5 5 5 4 5 73

6 노르웨이 0 5 0 5 5 5 4 3 5 4 5 3 0 5 5 0 3 5 0 5 67

7 리투아니아 5 4 4 5 5 5 5 0 0 2 3 0 2 0 5 0 5 5 5 5 65

8 독일 4 5 0 5 5 5 5 5 5 2 2 4 5 1 1 0 5 0 0 0 59

8 라트비아 4 5 5 3 5 5 0 0 5 4 2 1 5 0 5 0 5 5 0 0 59

10 덴마크 0 5 0 5 5 5 5 0 0 0 2 5 5 2 5 1 5 5 0 0 55

11 아이슬란드 4 5 0 5 0 5 3 1 2 0 0 0 5 0 3 1 4 5 1 5 49

12 스웨덴 1 4 0 3 0 5 5 0 0 2 2 0 0 0 5 0 5 0 0 0 32

2005년 Baltic Way 대회결과

대수 조합 기하 수론 계

1 폴란드 5 5 5 5 5 5 5 5 5 5 5 5 5 5 5 5 5 5 5 95

2 핀란드 5 5 5 0 5 5 5 5 5 5 5 5 5 0 0 5 5 5 75

3 상트 페테르부르크 5 5 5 0 0 5 5 5 5 5 2 5 5 5 5 5 1 5 73

4 독일 1 5 5 5 1 1 4 5 5 5 2 5 3 5 5 5 62

5 라트비아 4 0 5 1 0 4 0 1 5 5 5 5 5 0 0 5 5 5 5 60

6 리투아니아 5 5 4 0 0 5 5 5 0 5 5 4 5 5 53

7 노르웨이 3 5 4 0 1 2 4 4 5 3 2 0 5 5 5 48

8 스웨덴 5 5 0 0 0 5 5 5 5 2 0 0 5 5 5 47

9 벨기에 1 5 1 0 1 3 4 0 5 4 2 3 0 0 0 0 5 5 39

10 에스토니아 0 2 5 5 1 5 5 1 3 5 0 5 37

11 아이슬란드 1 0 0 0 5 0 0 1 1 5 5 0 4 0 0 0 0 3 0 5 30

12 덴마크 0 5 0 0 1 0 5 5 2 0 0 1 5 24